368
COMPENDIUM OMEFL Fase local de la Olimpiada Matemática Española 1998 2020 Gerard Romo Garrido

COMPENDIUM OMEFL - ToomatesXXXV Olimpiada Matemática Española Primera Fase Soluciones de la propuesta de problemas Problema 1 ¿Qué dígitos se han omitido en la siguiente multiplicación?

  • Upload
    others

  • View
    3

  • Download
    0

Embed Size (px)

Citation preview

Page 1: COMPENDIUM OMEFL - ToomatesXXXV Olimpiada Matemática Española Primera Fase Soluciones de la propuesta de problemas Problema 1 ¿Qué dígitos se han omitido en la siguiente multiplicación?

COMPENDIUM OMEFL

Fase local de la Olimpiada Matemática Española

1998 – 2020

Gerard Romo Garrido

Page 2: COMPENDIUM OMEFL - ToomatesXXXV Olimpiada Matemática Española Primera Fase Soluciones de la propuesta de problemas Problema 1 ¿Qué dígitos se han omitido en la siguiente multiplicación?

Toomates Coolección

Los documentos de Toomates son materiales digitales y gratuitos. Son digitales porque están pensados para ser consultados

mediante un ordenador, tablet o móvil. Son gratuitos porque se ofrecen a la comunidad educativa sin coste alguno. Los libros de

texto pueden ser digitales o en papel, gratuitos o en venta, y ninguna de estas opciones es necesariamente mejor o peor que las otras. Es más: Suele suceder que los mejores docentes son los que piden a sus alumnos la compra de un libro de texto en papel, esto es un

hecho. Lo que no es aceptable, por inmoral y mezquino, es el modelo de las llamadas "licencias digitales" con las que las editoriales

pretenden cobrar a los estudiantes, una y otra vez, por acceder a los mismos contenidos (unos contenidos que, además, son de una bajísima calidad). Este modelo de negocio es miserable, pues impide el compartir un mismo libro, incluso entre dos hermanos,

pretende convertir a los estudiantes en un mercado cautivo, exige a los estudiantes y a las escuelas costosísimas líneas de Internet,

pretende pervertir el conocimiento, que es algo social, público, convirtiéndolo en un producto de propiedad privada, accesible solo a aquellos que se lo puedan permitir, y solo de una manera encapsulada, fragmentada, impidiendo el derecho del alumno de poseer

todo el libro, de acceder a todo el libro, de moverse libremente por todo el libro.

Nadie puede pretender ser neutral ante esto: Mirar para otro lado y aceptar el modelo de licencias digitales es admitir un mundo más injusto, es participar en la denegación del acceso al conocimiento a aquellos que no disponen de medios económicos, en un mundo

en el que las modernas tecnologías actuales permiten, por primera vez en la historia de la Humanidad, poder compartir el

conocimiento sin coste alguno, con algo tan simple como es un archivo "pdf". El conocimiento no es una mercancía. El proyecto Toomates tiene como objetivo la promoción y difusión entre el profesorado y el colectivo de estudiantes de unos

materiales didácticos libres, gratuitos y de calidad, que fuerce a las editoriales a competir ofreciendo alternativas de pago atractivas

aumentando la calidad de unos libros de texto que actualmente son muy mediocres, y no mediante retorcidas técnicas comerciales. Este documento se comparte bajo una licencia “Creative Commons”: Se permite, se promueve y se fomenta cualquier uso,

reproducción y edición de todos estos materiales siempre que sea sin ánimo de lucro y se cite su procedencia. Todos los documentos

se ofrecen en dos versiones: En formato “pdf” para una cómoda lectura y en el formato “doc” de MSWord para permitir y facilitar su edición y generar versiones parcial o totalmente modificadas. Se agradecerá cualquier observación, comentario o colaboración a

[email protected]

La biblioteca Toomates Coolección consta de los siguientes libros:

Bloques temáticos: Problem-solving Libros de texto (en catalán)

Geometría Axiomática pdf 1 2 ... 23

Problemas de Geometría pdf 1 2 3 4 5 6 7 8 9

Introducción a la Geometría pdf doc

Teoría de números

pdf 1 2 3

Trigonometría pdf doc pdf doc

Desigualdades pdf doc

Números complejos pdf doc pdf doc

Álgebra pdf doc pdf 1 2 3 4

Combinatoria

pdf doc

Probabilidad

pdf doc

Combinatòria i Probabilitat pdf doc

Estadística pdf doc

Funcions pdf doc

Geometria analítica pdf 1 2

Àlgebra Lineal 2n batxillerat pdf doc

Geometria Lineal 2n batxillerat pdf doc

Càlcul Infinitesimal 2n batxillerat pdf 1 2

Programació Lineal 2n batxillerat pdf doc

Guía del estudiante de Olimpiadas Matemáticas pdf

Recopilaciones de pruebas PAU:

Catalunya TEC , Catalunya CCSS , Galicia , Portugal A , Portugal B

Recopilaciones de problemas olímpicos y preolímpicos (España):

OME , Cangur , Canguro , OMEFL , OMC

Recopilaciones de problemas olímpicos y preolímpicos (Internacional):

IMO , OMI , AIME , Kangourou , AMC12 (2008-2020) , SMT

Versión de este documento: 04/02/2021

Todos estos documentos se actualizan constantemente. ¡No utilices una versión anticuada! Descarga

totalmente gratis la última versión de los documentos en los enlaces superiores.

www.toomates.net

Page 3: COMPENDIUM OMEFL - ToomatesXXXV Olimpiada Matemática Española Primera Fase Soluciones de la propuesta de problemas Problema 1 ¿Qué dígitos se han omitido en la siguiente multiplicación?

Índice.

Enunciados Soluciones

35 - XXXV - (1998-99) 5 7

36 - XXXVI - (1999-00) 12 14

37 - XXXVII - (2000-01) 18 21

38 - XXXVIII - (2001-02) 27 30

39 - XXXIX - (2002-03) 34 37

40 - XL - (2003-04) 50 54

41 - XLI - (2004-05) 60 63

42 - XLII - (2005-06) 71 76

43 - XLIII - (2006-07) 87 90

44 - XLIV - (2007-08) 98 104

45 - XLV - (2008-09) 113 119

46 - XLVI - (2009-10) 136 142

47 - XLVII - (2010-11) 156 162

48 - XLVIII - (2011-12) 173 179

49 - XLIX - (2012-13) 203 209

50 - L - (2013-14) 226 232

51 - LI - (2014-15) 257 263

52 - LII - (2015-16) 279 285

53 - LIII - (2016-17) 301 305

54 - LIV - (2017-18) 313 317

55 - LV - (2018-19) 327 331

56 - LVI - (2019-20) 340 344

57 - LVII - (2020-21) 353 355

Page 4: COMPENDIUM OMEFL - ToomatesXXXV Olimpiada Matemática Española Primera Fase Soluciones de la propuesta de problemas Problema 1 ¿Qué dígitos se han omitido en la siguiente multiplicación?

Fuentes.

http://www.ugr.es/~olimpiada/historico.html

http://www.olimpiadamatematica.es/platea.pntic.mec.es/_csanchez/olimprab.htm

Todo este material ha sido agrupado en un único archivo "pdf" mediante la

aplicación online

https://www.ilovepdf.com/

Page 5: COMPENDIUM OMEFL - ToomatesXXXV Olimpiada Matemática Española Primera Fase Soluciones de la propuesta de problemas Problema 1 ¿Qué dígitos se han omitido en la siguiente multiplicación?

4/2/2021 Fase local 1999

www.olimpiadamatematica.es/platea.pntic.mec.es/_csanchez/local99.htm 1/2

OLIMPIADA MATEMÁTICA ESPAÑOLA

Fase local 1999

Primera sesión

1¿Qué dígitos se han omitido en la siguiente multiplicación?

2.- Una empresa produce semanalmente 300 bicicletas de montaña que vende íntegramente al precio de 600euros cada una. Tras un análisis de mercados observa que si varía el precio, también varían sus ventas (de formacontinua) según la siguiente proporción: por cada 7 euros que aumente o disminuya el precio de sus bicicletas,disminuye o aumenta la venta en 3 unidades.

a. ¿Puede aumentar el precio y obtener mayores ingresos?b. ¿A qué precio los ingresos serán máximos?

3.- Dado un triángulo ABC, con baricentro G.

Page 6: COMPENDIUM OMEFL - ToomatesXXXV Olimpiada Matemática Española Primera Fase Soluciones de la propuesta de problemas Problema 1 ¿Qué dígitos se han omitido en la siguiente multiplicación?

4/2/2021 Fase local 1999

www.olimpiadamatematica.es/platea.pntic.mec.es/_csanchez/local99.htm 2/2

a. Prueba que para cualquier punto del plano M se verifica:

, obteniéndose la igualdad si y solamente si M = G

b. Fijado un número

, halla el lugar geométrico de los puntos M tales que

Segunda sesión

4.- Halla todos los pares de números naturales x, y (x < y) tales que la suma de todos los números naturalescomprendidos estrictamente entre ambos es igual a 1999.

5.- Prueba que la longitud de los catetos de un triángulo rectángulo isósceles es siempre igual a la suma delos radios de sus circunferencias inscrita y circunscrita.

6.- Sean a, b y c números reales no nulos (con suma no nula) tales que:

.

Prueba que también se verifica:

Soluciones en formato Microsoft Word 97 comprimido .zip (50 Kb)

|Página principal| |Info Alumnos| |Info Profesores| |Otros sitios de interés| |Problemas|

Copyright © C. Sánchez-Rubio

Actualizado 12 Febrero 1999

Page 7: COMPENDIUM OMEFL - ToomatesXXXV Olimpiada Matemática Española Primera Fase Soluciones de la propuesta de problemas Problema 1 ¿Qué dígitos se han omitido en la siguiente multiplicación?

XXXV Olimpiada Matemática Española

Primera Fase

Soluciones de la propuesta de problemas

Problema 1

¿Qué dígitos se han omitido en la siguiente multiplicación?

2 * *

* *

* 6 1

* * 4

* * 0 1

Solución al problema 1:

Se sustituyen los nueve dígitos omitidos por letras (por ejemplo: a,b,c,d,e,f,g,h,i) y se

hace un análisis de posibilidades a partir del enunciado. La solución es: 287 x 23 = 6601

Problema 2

Una empresa produce semanalmente 300 bicicletas de montaña que vende íntegramente

al precio de 600 euros cada una. Tras un análisis de mercados observa que si varía el precio,

también varían sus ventas (de forma continua) según la siguiente proporción: por cada 7 euros

que aumente o disminuya el precio de sus bicicletas, disminuye o aumenta la venta en 3

unidades.

a) ¿Puede aumentar el precio y obtener mayores ingresos?

b) ¿A qué precio los ingresos serán máximos?

Solución al problema 2:

Al precio actual, los ingresos semanales son 180.000 003600 euros.

(a) Si incrementa el precio en 7 euros, entonces vende 297 bicicletas, obteniendo en este

caso 180.279 297607 euros. Luego la respuesta a la primera pregunta es: Sí.

(b) Llamamos x a la cantidad de euros en que incrementa el precio de cada bicicleta (x

puede ser cualquier número real positivo o negativo, según aumente o disminuya el precio),

entonces el nuevo precio de cada bicicleta será 600 + x . Según la proporción del enunciado, las

ventas las ventas variarán hasta x7

3300 bicicletas.

Se considera la función de ingresos, donde la variable es el incremento de precio:

))33000002601(7

1)

7

3300()600()( 2xx-.. xxxf .

Los puntos críticos de f(x) se obtienen al anular la derivada: )6300)(71()(' x/xf .

El punto único crítico es x = 50, que efectivamente da un máximo de la función f(x).

Entonces la solución a la pregunta es: 650.

Problema 3

Dado un triángulo ABC, con baricentro G.

a) Prueba que para cualquier punto del plano M se verifica: 222222

GCGBGAMCMBMA , obteniéndose la igualdad si y solamente si M = G

Page 8: COMPENDIUM OMEFL - ToomatesXXXV Olimpiada Matemática Española Primera Fase Soluciones de la propuesta de problemas Problema 1 ¿Qué dígitos se han omitido en la siguiente multiplicación?

b) Fijado un número 222

GCGBGAk , halla el lugar geométrico de los puntos M tales que

kMCMBMA 222

1ª solución al problema 3 (vectorial):

a) Del baricentro G de ABC sabemos que cumple:

0GCGBGA (1)

se tiene:

GCMGGCMGGCMGMCMC

GBMGGBMGGBMGMBMB

GAMGGAMGGAMGMAMA

2

2

2

222

22

222

22

222

22

y sumando,

GCGBGAMGGCGBGAMGMCMBMA 232222222

y por (1) queda:

gMGGCGBGAMGMCMBMA 22222222

33 (2)

siendo 0222

GCGBGAg una constante independiente de M.

La expresión (2) muestra de una parte que el baricentro es el punto que hace mínima la

suma de cuadrados de distancias a los vértices.

De otra nos resuelve el apartado b) ya que si imponemos la condición del enunciado,

queda:

rgk

MGkgMG

3

32

expresión que muestra que el lugar pedido es una circunferencia de centro G y radio r.

2ª solución al problema 3 (analítica):

a) Se trata de minimizar la expresión: 222

)( MCMBMAMf .

Analíticamente, sean A = (a 1 , a 2 ), B = (b 1 , b 2 ) y C = (c 1 , c 2 ) los vértices del triángulo

dado. Para un punto genérico M = (x,y) se obtiene

2

2

2

1

2

2

2

1

2

2

2

1 )()()()()()(),( cycxbybxayaxyxf

2

2

2

1

2

2

2

1

2

2

2

1321321

22 2233 ccbbaa)bbb(y)aaa(xyx

gbbb

yaaa

x

23212321 )

3()

3(3 (*)

(donde g es una determinada constante real).

Por lo tanto, esta cantidad resultará mínima si y sólo si los cuadrados que aparecen en la

expresión son cero; es decir cuando )33

()( 321321 bbb,

aaay,xM

, que corresponde

precisamente al baricentro G.

Page 9: COMPENDIUM OMEFL - ToomatesXXXV Olimpiada Matemática Española Primera Fase Soluciones de la propuesta de problemas Problema 1 ¿Qué dígitos se han omitido en la siguiente multiplicación?

b) Se trata de encontrar el lugar geométrico de los puntos M tales que kMf )( . Para ello

determinaremos en primer lugar la constante g aparecida anteriormente. En efecto, tomando

M=G en ( *) se obtiene 222

)( GCGBGAGfg . Por lo tanto:

kgbbb

yaaa

xkMf

23212321 )

3()

3(3)(

3)

3()

3( 23212321 gkbbb

yaaa

x

que es exactamente la ecuación de la circunferencia de centro G y radio 3

gkr

.

Problema 4

Halla todos los pares de números naturales x, y (x < y) tales que la suma de todos los números

naturales comprendidos estrictamente entre ambos es igual a 1999.

Solución al problema 4:

Tenemos que sumar del número x+1 hasta el número y – 1 y obtener el número 1999.

Esta suma: 19991)-( ... 21)( y)x(x corresponde a la de una progresión aritmética de

diferencia 1 y con 1 xy términos, por tanto es:

1999)1(2

)1()1(

xy

y-x

De donde se deduce la descomposición: 1999.2)1)(( xyyx

Por ser x>0 e xyxy 1 , y teniendo en cuenta que 2 y 1999 son números primos.

Solamente pueden ocurrir los siguientes casos:

Caso 1: Si 19992 xy e 11 xy . La solución de este sistema es 1998x e 2000y .

Caso 2: Si 1999 xy e 21 xy . La solución de este sistema es 998x e 1001y .

Los pares buscados son: 2000,1998 yx , que verifica 1999 = 1999,

y 1001,998 yx , que verifica 999 + 1000 = 1999.

Problema 5

Prueba que la longitud de los catetos de un triángulo rectángulo isósceles es siempre igual

a la suma de los radios de sus circunferencias inscrita y circunscrita.

Solución al problema 5:

Sea ABC un triángulo rectángulo e isósceles con ángulo

recto en el vértice C. Sea O el centro de la circunferencia

inscrita, la cual consideramos tangente a la hipotenusa AB en su

punto medio M y al lado AC en el punto P. El radio de esta

circunferencia es OMOPr .

Por ser esta circunferencia tangente a los lados del

triángulo, los radios OM y OP son perpendiculares a los lados

AB y AC en M y P respectivamente.

Por ser el triángulo isósceles, CO (y CM) es bisectriz del

ángulo recto en C, por lo que el triángulo OCP es, también,

rectángulo e isósceles (ángulo OCP de 45º); es decir,

PCOP .

Page 10: COMPENDIUM OMEFL - ToomatesXXXV Olimpiada Matemática Española Primera Fase Soluciones de la propuesta de problemas Problema 1 ¿Qué dígitos se han omitido en la siguiente multiplicación?

Por estar O en la bisectriz del ángulo CAB, los triángulos rectángulos AMO y APO son

iguales (simétricos respecto a la hipotenusa), resultando que APAM .

Por otra parte, como el triángulo es rectángulo, el punto M, punto medio de la hipotenusa,

es también el centro de la circunferencia circunscrita a ABC. Por lo que AMR , es el radio de

esta circunferencia.

En conclusión, la longitud del cateto AC la podemos escribir como:

rROPAMPCAPAC .

Problema 6

Sean a, b y c números reales no nulos (con suma no nula) tales que:

cbacba

1111.

Prueba que también se verifica:

199919991999199919991999

1111

cbacba

Solución al problema 6:

Del enunciado, deducimos: cbaabc

abacbc

cbacba

11111

Operando en la última ecuación, tenemos:

0

0

0

2

2

22

)ca)(cb)(ba(

)cabacbc)(ba(

c)ba()abacbc)(ba(

abcabcacbc)abacbc)(ba(

abc)abacbc(c)abacbc)(ba(

abc)abacbc)(cba(

Por lo tanto, ha de verificarse una de las siguientes igualdades:

ca

cb

ba

Si ba entonces 199919991999 )( bba y así

1999199919991999199919991999

1111111

ccbbcba

Por otra parte: 1999199919991999199919991999

111

ccbbcba

.

Con lo que se tiene el resultado pedido.

En los casos cb o ca se procede de manera análoga.

Solución 2.

Una alternativa al primer cálculo es despejar un número, por ejemplo c, en función de los

otros dos (a y b); para ello, escribimos la hipótesis en la forma: cbacab

ba

11y poniendo

Page 11: COMPENDIUM OMEFL - ToomatesXXXV Olimpiada Matemática Española Primera Fase Soluciones de la propuesta de problemas Problema 1 ¿Qué dígitos se han omitido en la siguiente multiplicación?

abp

bas, queda

cspc

psc

1, que una vez operada, resulta 022 spcssc y suponiendo

0s , queda en la forma 02 pscc .

Ecuación de segundo grado cuyas raíces son:

bc

ac

2

1

En cada uno de los casos el resto del análisis es el mismo que antes.

Si fuese s = a + b = 0, resulta a = - b y estaríamos en la misma situación.

Hay que señalar finalmente, que el resultado subsiste para cualquier valor de n natural,

impar.

Page 12: COMPENDIUM OMEFL - ToomatesXXXV Olimpiada Matemática Española Primera Fase Soluciones de la propuesta de problemas Problema 1 ¿Qué dígitos se han omitido en la siguiente multiplicación?

4/2/2021 Fase local 1999

www.olimpiadamatematica.es/platea.pntic.mec.es/_csanchez/loc2000.html 1/2

OLIMPIADA MATEMÁTICA ESPAÑOLA

Fase local 2000

Primera sesión

1.- Considérese la sucesión definida como a1 = 3, y an+1 = an + an2. Determínense las dos últimas cifras

de a2000.

2.- Sea P un punto del lado BC de un triángulo ABC. La paralela por P a AB corta al lado AC en elpunto Q y la paralela por P a AC corta al lado AB en el punto R. La razón entre las áreas de los triángulos RBP y QPC es k2. Determínese la razón entre las áreas de los triángulos ARQ y ABC.

3.- ¿Cuántos números, comprendidos entre 1.000 y 9.999, verifican que la suma de sus cuatro dígitos esmayor o igual que el producto de los mismos?. ¿Para cuántos de ellos se verifica la igualdad?

Segunda sesión

4.- Se consideran las funciones reales de variable real f(x) de la forma: f(x) = ax + b, siendo a y b números reales. ¿Para qué valores de a y b se verifica f2000(x) = x para todo número real x.

Page 13: COMPENDIUM OMEFL - ToomatesXXXV Olimpiada Matemática Española Primera Fase Soluciones de la propuesta de problemas Problema 1 ¿Qué dígitos se han omitido en la siguiente multiplicación?

4/2/2021 Fase local 1999

www.olimpiadamatematica.es/platea.pntic.mec.es/_csanchez/loc2000.html 2/2

[Nota: Se define f2(x) = f(f(x)), f3(x) = f(f(f(x))), y en general, fn(x) = f(fn-1(x)) = f(f(...f(x))...)) n veces]

5.- En la orilla de un río de 100 metros de ancho está situada una planta eléctrica y en la orilla opuesta, y a500 metros río arriba, se está construyendo una fábrica. Sabiendo que el río es rectilíneo entre la planta y lafábrica, que el tendido de cables a lo largo de la orilla cuesta a 9 € cada metro y que el tendido de cables sobreel agua cuesta a 15 € cada metro, ¿cuál es la longitud del tendido más económico posible entre la plantaeléctrica y la fábrica?.

6.- Se sabe que el polinomio p(x) = x3 – x + k tiene tres raíces que son números enteros. Determínese elnúmero k.

Soluciones en formato Microsoft Word 97 comprimido .zip (42 Kb)

|Página principal| |Info Alumnos| |Info Profesores| |Otros sitios de interés| |Problemas|

Copyright © C. Sánchez-Rubio

Actualizado 07 Abril 2000

Page 14: COMPENDIUM OMEFL - ToomatesXXXV Olimpiada Matemática Española Primera Fase Soluciones de la propuesta de problemas Problema 1 ¿Qué dígitos se han omitido en la siguiente multiplicación?

Problema 1.

Considérese la sucesión definida como a1 = 3, y an+1 = an + an2.

Determínense las dos últimas cifras de a2000.

Solución:

Se tiene a1 = 3 y an+1 = an + an2 = an (1 + an).

Escribimos los primeros términos de la sucesión:

3 , 12 , 156 , 156.157 = 24492 , 24492.24493 = .....56 , .....

Supongamos que an termina en 56. Entonces, an = 100 a + 56, y tenemos

an+1 = (100 a +56) (100 a +57) = 100 b +56.57 = 100b + 100c + 92 = 100d + 92,

es decir, las últimas cifras de an+1 son 92.

Análogamente, si an termina en 92, se prueba que an+1 termina en 56.

Como 2000 es par, entonces a2000 termina en 92.

Problema 2.

Sea P un punto del lado BC de un triángulo ABC. La paralela por P a AB corta al lado AC

en el punto Q y la paralela por P a AC corta al lado AB en el punto R. La razón entre las

áreas de los triángulos RBP y QPC es k2.

Determínese la razón entre las áreas de los triángulos ARQ y ABC.

Solución:

Los triángulo RBP y QPC son semejantes, de razón

k. El cuadrilátero ARPQ es un paralelogramo, y PQ =

RA.

Si BR = x, entonces

PQ = RA = kx; BA = (1 + k)x.

Área RBP = S = 2

BR.

2

xhPX

CY = k . PX = kh; CZ = CY + YZ = CY + PX = (1 + k)h

Área ABC = (1+k)2S

QT = YZ = PX = h

Área ARQ =2

AR. kS

2

kxhYZ

y

22 )k1(

k

S)k1(

kS

áreaABC

áreaARQ

Problema 3.

¿Cuántos números, comprendidos entre 1.000 y 9.999, verifican que la suma de sus cuatro

dígitos es mayor o igual que el producto de los mismos?

¿Para cuántos de ellos se verifica la igualdad?

CB

A

P

Z

X

R

Q

Y

T

Page 15: COMPENDIUM OMEFL - ToomatesXXXV Olimpiada Matemática Española Primera Fase Soluciones de la propuesta de problemas Problema 1 ¿Qué dígitos se han omitido en la siguiente multiplicación?

Si el número tuviera algún cero entre sus cifras, entonces tendríamos la desigualdad estricta. Hay

exactamente 243999000 4 números de este tipo, esto es, con una cifra igual a cero.

Consideremos el número “abcd” escrito en su expresión decimal, y supondremos que no

contiene ninguna cifra cero. Entonces la desigualdad

dcbadcba

es equivalente (dividiendo por dcba ) a

1cba

1

dba

1

dca

1

dcb

1

. (1)

Por lo tanto si tres o cuatro de estos dígitos fueran unos, entonces uno de los cuatro anteriores

sumandos serían 1 y se obtendría la desigualdad estricta. Hay exactamente 33184 números

de este tipo.

Por otra parte, demostremos que una condición necesaria para que se verifique la desigualdad es

que al menos el número debe tener dos unos entre sus cifras.

Efectivamente, supongamos por contradicción, y sin pérdida de generalidad que, 2,, dcb .

Entonces

8dcb ; 4dca ; 4dba ; 4cba ;

y así, por (1), tenemos:

8

7

4

1

4

1

4

1

8

11 ,

lo cual es una contradicción.

Resta, por lo tanto, considerar el caso en que el número tiene exactamente dos cifras iguales a

uno. Supongamos por ejemplo, que 1ba y 1, dc . En este caso, la desigualdad en cuestión

se traduce en

1d

1

c

1

dc

2

. (2)

Demostremos en primer lugar que, al menos, una de las cifras c ó d, debe ser un dos.

Efectivamente, si por el contrario 3d,c , entonces

9dc ; 3c ; 3d ,

y así, por (2), tenemos: 9

8

3

1

3

1

9

21 , lo cual es una contradicción.

Supongamos, por lo tanto que c = 2. Se obtiene entonces que

12

1

d

2 ,

lo cual es equivalente a decir que 4d .

Resumiendo:

Si d = 4, entonces se obtiene la igualdad inicial (las cifras son 1,1,2,4; y existen 12 números de

este tipo).

Si d = 3, entonces se obtiene la desigualdad estricta inicial (las cifras son 1,1,2,3; y existen 12

números de este tipo).

Si d=2, entonces se obtiene la desigualdad estricta inicial (las cifras son 1,1,2,2; y existen 6

números de este tipo).

Por lo tanto, y a modo de resumen global, la desigualdad se da en

2439 + 33 + 12 + 12 + 6 = 2502 números

y la igualdad en 12 de ellos.

Page 16: COMPENDIUM OMEFL - ToomatesXXXV Olimpiada Matemática Española Primera Fase Soluciones de la propuesta de problemas Problema 1 ¿Qué dígitos se han omitido en la siguiente multiplicación?

Problema 4.

Se consideran las funciones reales de variable real f(x) de la forma: f(x) = ax + b, siendo a y

b números reales.

¿Para qué valores de a y b se verifica f2000

(x) = x para todo número real x.

[Nota: Se define f2(x) = f(f(x)), f

3(x) = f(f(f(x))), y en general,

fn(x) = f(f

n-1(x)) = f(f(...f(x))...)) n veces]

Solución:

En primer lugar, observemos que si componemos dos funciones (lineales) del tipo ax+b,

obtenemos una función de este tipo, cuyo coeficiente en la variable x es el producto de los

respectivos coeficientes de las dos funciones.

Por lo tanto si f(x) = ax + b, entonces f2000

(x) es una función del tipo a2000

x + c; donde c es un

número que depende de a y b.

Se obtiene por lo tanto el sistema

0c

1a 2000

De donde resulta que 1a ó 1a . Analicemos ambos casos por separado.

Si 1a , entonces

f(x) = x + b, f(f(x)) = x + 2b, ..., f2000

(x) = x + 2000b,

en cuyo caso 2000b = 0, es decir, b = 0, y se obtiene la solución f(x) = x.

Si 1a , entonces

f(x) = x + b, f(f(x)) = x, f(f(f(x))) = x + b, f(f(f(f(x)))) = x, ...,

f2000

(x) = x (por ser 2000 un número par).

Por lo tanto, cualquier función del tipo f(x) = x + b (con b un número real arbitrario) es una

solución del problema.

Problema 5.

En la orilla de un río de 100 metros de ancho está situada una planta eléctrica y en la orilla

opuesta, y a 500 metros río arriba, se está construyendo una fábrica. Sabiendo que el río es

rectilíneo entre la planta y la fábrica, que el tendido de cables a lo largo de la orilla cuesta a 9 €

cada metro y que el tendido de cables sobre el agua cuesta a 15 € cada metro, ¿cuál es la

longitud del tendido más económico posible entre la planta eléctrica y la fábrica?.

Solución:

Cada trayecto tendrá un recorrido formado por un tramo sobre el río, en el que se avanzará una

distancia de b metros y uno o dos tramos a lo largo de la orilla que recorrerán los restantes

500 b metros. El recorrido de tal trayecto será L(b) y el gasto g(b).

100

a

F

E

b500 - b

Page 17: COMPENDIUM OMEFL - ToomatesXXXV Olimpiada Matemática Española Primera Fase Soluciones de la propuesta de problemas Problema 1 ¿Qué dígitos se han omitido en la siguiente multiplicación?

5501254257510075500L

;751004

3b;100

16

9b;b

9

16100;b100)b

3

5(;b1003b5

;03b100

b5;09

b1002

b2.15)b(g

);b500(9b10015)b(g);b500(b100)b(L

22

222222222

2222

2222

La longitud del recorrido más económico posible entre la planta eléctrica y la fabrica es de 550

metros.

Problema 6.

Se sabe que el polinomio p(x) = x3 – x + k tiene tres raíces que son números enteros.

Determínese el número k.

Para k = 0 tenemos )1x)(1x(xxx)x(p 3 , que tiene raíces 0, 1 y 1.

Se demuestra que este es el único valor de k para el cual p(x) tiene tras raíces enteras.

En efecto, si a, b, c son enteros, y )cx)(bx)(ax()x(p , resulta:

kabc

1bcacab

0cba

Entonces,

2cba)bcacab(2cba0)cba( 2222222

Es decir, 2cba 222 , siendo a2, b

2, c

2 son enteros no negativos). Necesariamente uno de los

valores a, b ó c deberá ser nulo, con lo que 0abck .

También pueden representar q(x) = xx3 , y observar que q(x) + k no puede tener tres raíces

enteras, pues no hay enteros ni en (-1,0) ni en (0,1).

Page 18: COMPENDIUM OMEFL - ToomatesXXXV Olimpiada Matemática Española Primera Fase Soluciones de la propuesta de problemas Problema 1 ¿Qué dígitos se han omitido en la siguiente multiplicación?

4/2/2021 Fase local 1999

www.olimpiadamatematica.es/platea.pntic.mec.es/_csanchez/loc2001.html 1/3

OLIMPIADA MATEMÁTICA ESPAÑOLA

Fase local 2001

Viernes 19 de enero de 2001

Sesión de Mañana

1.- Halla el número natural n que es el producto de los primos p, q y r, sabiendo que:

r - q = 2p y rq + p2 = 676

2. El encargado del faro de Finisterre ha recibido la comunicación de que va a haber un corte del suministroeléctrico y debe hacer funcionar el faro con ayuda del generador alimentado con gasóleo. Ese generadorconsume 6 litros de gasóleo cada hora y medio litro más cada vez que hay que ponerlo en marcha (inicialmente,está parado). En las 10 horas exactas que durará la noche, el faro no puede dejar de funcionar durante más de 10minutos seguidos. Y cuando funciona tiene que hacerlo durante al menos 15 minutos seguidos. ¿Cuántos litrosde gasóleo necesita, como mínimo, para cumplir con las normas de funcionamiento del faro?

3.- Las longitudes de los lados de un triángulo están en progresión geométrica de razón r. Halla los valoresde r para los que el triángulo es, respectivamente, acutángulo, rectángulo u obtusángulo

Page 19: COMPENDIUM OMEFL - ToomatesXXXV Olimpiada Matemática Española Primera Fase Soluciones de la propuesta de problemas Problema 1 ¿Qué dígitos se han omitido en la siguiente multiplicación?

4/2/2021 Fase local 1999

www.olimpiadamatematica.es/platea.pntic.mec.es/_csanchez/loc2001.html 2/3

Viernes 19 de enero de 2001

Sesión de Tarde

4.- Sean a, b, y c números reales. Prueba que si x3 + ax2 + bx + c tiene tres raíces reales, entonces 3b£ a2.

5.- Un cristalero dispone de una pieza de vidrio de forma triangular. Usando sus conocimientos degeometría, sabe que podría cortar de ella un círculo de radio r. Demuestra que, para cualquier número natural n,de la pieza triangular puede obtener n2 círculos de radio r/n (suponiendo que se puedan hacer siempre los cortesperfectos).

6.- Nueve personas han celebrado cuatro reuniones diferentes sentados alrededor de una mesa circular. ¿Hanpodido hacerlo sin que existan dos de esas personas que se hayan sentado una junto a la otra en más de unareunión? Razona la respuesta.

Sábado 20 de enero de 2001

Sesión de Mañana

7.- Consideramos el conjunto N = {l, 2,3,...} de los números naturales y la aplicación f : N ® N que cumplelas dos siguientes condiciones: a) f (f(n)) = n para todo nÎ N.

b) Determina el valor de f (n) para cada n Î N observando previamente que f es biyectiva y que, al no ser nunca f (f (n) + 1) = 2, tiene que ser f (1) = 2.

8.- Consideramos los siguientes 27 puntos de un cubo: el centro (1), los centros de las caras (6), los vértices(8) y los centros de las aristas (12). Coloreamos cada uno de esos puntos de azul o de rojo. ¿Puede hacerse demodo que no haya tres puntos del mismo color alineados? Demuéstralo.

9- . Un condenado queda en libertad cuando alcance el final de una escalera de 100 escalones. Pero nopuede avanzar a su antojo, puesto que está obligado a subir un solo escalón cada día de los meses impares y abajar un escalón cada día de los meses pares. Comienza el 1 de enero de 2001. ¿Qué día quedará en libertad?¿Qué día quedaría en libertad si la escalera tuviera 99 escalones?

Page 20: COMPENDIUM OMEFL - ToomatesXXXV Olimpiada Matemática Española Primera Fase Soluciones de la propuesta de problemas Problema 1 ¿Qué dígitos se han omitido en la siguiente multiplicación?

4/2/2021 Fase local 1999

www.olimpiadamatematica.es/platea.pntic.mec.es/_csanchez/loc2001.html 3/3

Soluciones en formato Microsoft Word 2000 comprimido .zip (27 Kb)

|Página principal| |Info Alumnos| |Info Profesores| |Otros sitios de interés| |Problemas|

Copyright © C. Sánchez-Rubio

Actualizado 22 Enero 2001

Page 21: COMPENDIUM OMEFL - ToomatesXXXV Olimpiada Matemática Española Primera Fase Soluciones de la propuesta de problemas Problema 1 ¿Qué dígitos se han omitido en la siguiente multiplicación?

Viernes 19 de enero de 2001

Sesión de Mañana

Problema 1. Halla el número natural n que es el producto de los primos p, q y r, sabiendo que

r - q = 2p y rq + p2 = 676.

Solución:

Tomamos x = r - p = q + p. Entonces,

x2 = (r - p) (q + p) = rq + (r - q)p - p

2 = rq + 2p

2 - p

2 = rq + p

2 = 276

luego x = 26. Y p es un primo tal que 26 - p y 26 + p son primos. Probamos con los posibles primos

p menores que 26 y se ve que eso sólo se cumple para p = 3.

Así pues, p = 3, q = 23, r = 29 y n = p.q.r = 2001.

Problema 2. El encargado del faro de Finisterre ha recibido la comunicación de que va a haber un

corte del suministro eléctrico y debe hacer funcionar el faro con ayuda del generador alimentado

con gasóleo. Ese generador consume 6 litros de gasóleo cada hora y medio litro más cada vez que

hay que ponerlo en marcha (inicialmente, está parado). En las 10 horas exactas que durará la noche,

el faro no puede dejar de funcionar durante más de 10 minutos seguidos. Y cuando funciona tiene

que hacerlo durante al menos 15 minutos seguidos. ¿Cuántos litros de gasóleo necesita, como

mínimo, para cumplir con las normas de funcionamiento del faro?

Solución:

Las mejor manera de ahorrar combustible es hacer paradas de 10 minutos (más largas no están

permitidas). Con cada parada ahorramos 1 litro de gasóleo, pero gastamos medio litro en volver a

poner en marcha el generador. Es obvio que, en principio, la mejor estrategia de ahorro consiste en

intercalar periodos de 15 minutos de funcionamiento (más breves no están permitidos) con periodos

de 10 minutos de parada. En cada proceso "arranque-15 minutos funcionando-10 minutos de

parada" se consumen 26·4

1

2

1 litros de gasóleo. Para completar las 10 horas de noche

necesitamos 2425

600 de esos procesos, luego parece que la cantidad mínima de combustible

necesaria es 48 litros. Sin embargo, hay una manera de ahorrar medio litro más. Cuando comienzan

las 10 horas de noche, se puede estar 10 minutos sin encender el faro. Si después encadenamos 23

procesos "arranque-15 minutos funcionando-10 minutos de parada", completamos 10 horas menos

15 minutos. Tendríamos que volver a arrancar el generador y hacerlo funcionar otros 15 minutos,

consumiendo de nuevo 48 litros. Ahora bien, si, por ejemplo, uno de los periodos de

funcionamiento lo hacemos de 30 minutos y "guardamos" los 10 minutos de parada intermedia para

los últimos 10 minutos de noche, el tiempo de funcionamiento es el mismo pero nos ahorramos un

arranque, es decir, medio litro de gasóleo. Basta con 47,5 litros.

Problema 3. Las longitudes de los lados de un triángulo están en progresión geométrica de razón r.

Halla los valores de r para los que el triángulo es, respectivamente, acutángulo, rectángulo u

obtusángulo.

Solución;

Page 22: COMPENDIUM OMEFL - ToomatesXXXV Olimpiada Matemática Española Primera Fase Soluciones de la propuesta de problemas Problema 1 ¿Qué dígitos se han omitido en la siguiente multiplicación?

Podemos suponer r 1, puesto que si no, basta con invertir el orden en que se consideran los lados.

Si tomamos como unidad de longitud la del lado más corto, entonces las longitudes son 1, r y r2.

Llamemos a al ángulo opuesto a r2 , que es el mayor. Por el teorema del coseno, se tiene:

r4 = 1 + r

2 - 2r cos a.

El triángulo es rectángulo si y sólo si cos a = 0, o sea, r 4 - r

2 - 1 = 0.

O lo que es lo mismo: .2

512 r Y, en definitiva,

2

51r .

Ahora es inmediato deducir que el triángulo es acutángulo (cos a > 0) si y sólo sí

2

511

r .

Análogamente, el triángulo es obtusángulo (cos a < 0) sí y sólo si 2

51

2

51

r

La última desigualdad viene forzada por el hecho de que para que exista el triángulo debe ser:

r2 < 1 + r.

Page 23: COMPENDIUM OMEFL - ToomatesXXXV Olimpiada Matemática Española Primera Fase Soluciones de la propuesta de problemas Problema 1 ¿Qué dígitos se han omitido en la siguiente multiplicación?

Viernes 19 de enero de 2001

Sesión de tarde

Problema 1. Sean a, b, y c números reales. Prueba que si x3 + ax

2 + bx + c tiene tres raíces reales,

entonces 3b a2.

Solución:

Sean las raíces y supongamos

xxxxxxcbxaxx 2323

Así pues, a y b .

Ahora se tiene:

02

33

2222

22222

ba

Nota: El recíproco también es cierto en el sentido de que si 3b a2, entonces existe algún número

real c de forma que el polinomio tiene tres raíces reales. Se puede probar fácilmente usando

derivadas, máximos y mínimos, etc.

Problema 2. Un cristalero dispone de una pieza de vidrio de forma triangular. Usando sus

conocimientos de geometría, sabe que podría cortar de ella un círculo de radio r. Demuestra que,

para cualquier número natural n, de la pieza triangular puede obtener n2 círculos de radio

n

r

(suponiendo que se puedan hacer siempre los cortes perfectos).

Solución:

El circulo de mayor radio que se puede cortar de un triángulo viene determinado por la

circunferencia inscrita. Del enunciado se deduce que el radio de la circunferencia inscrita del

triángulo dado es mayor o igual que r.

Dividimos cada lado del triángulo en n partes iguales. Por cada uno de esos puntos trazamos las

rectas paralelas a los otros dos lados. Se forman así n2 triángulos iguales, semejantes al triángulo de

partida y con razón de semejanza n

1. Obviamente, de cada uno de esos triangulítos podría cortarse

un circulo de radio n

r, puesto que el radio de su circunferencia inscrita es mayor o igual que ese

valor.

La afirmación de que aparecen exactamente n2 triangulitos se puede probar, por ejemplo, de la

siguiente forma:

Se construye un paralelogramo con dos triángulos como el inicial, se divide cada lado en n partes

iguales y se obtienen n2 paralelogramos iguales mediante paralelas a los lados. Ahora, se divide

cada uno de esos paralelogramos en dos triángulos iguales (y semejantes al inicial) mediante una

diagonal. Véase el dibujo.

Page 24: COMPENDIUM OMEFL - ToomatesXXXV Olimpiada Matemática Española Primera Fase Soluciones de la propuesta de problemas Problema 1 ¿Qué dígitos se han omitido en la siguiente multiplicación?

Problema 3. Nueve personas han celebrado cuatro reuniones diferentes sentados alrededor de una

mesa circular. ¿Han podido hacerlo sin que existan dos de esas personas que se hayan sentado una

junto a la otra en más de una reunión?. Razona la respuesta.

Solución:

La respuesta es sí, pueden celebrar las cuatro reuniones de modo que al final cada persona haya

estado sentada junto a otras dos diferentes cada vez. Para demostrarlo, consideramos las siguientes

cuatro formas de ordenar los números del 1 al 9, que representan cuatro maneras de sentarse

alrededor de la mesa comenzando en un lugar y siguiendo el giro de las agujas del reloj:

Primera reunión: 1, 2, 3, 4, 5, 6, 7, 8, 9

Segunda reunión: 1, 3, 5, 7, 9, 4, 6, 2, 8

Tercera reunión: 1, 4, 7, 3, 8, 5, 2, 9, 6

Cuarta reunión: 1, 5, 9, 3, 6, 8, 4, 2, 7

Page 25: COMPENDIUM OMEFL - ToomatesXXXV Olimpiada Matemática Española Primera Fase Soluciones de la propuesta de problemas Problema 1 ¿Qué dígitos se han omitido en la siguiente multiplicación?

Sábado 20 de enero de 2001

Sesión de Mañana

Problema 1. Consideramos el conjunto N = {l, 2,3,...} de los números naturales y la aplicación f :

N N que cumple las dos siguientes condiciones:

a) f (f(n)) = n para todo n N.

b)

.im ,3

; ,11

paresnsin

paresnsinnff

Determina el valor de f (n) para cada n N observando previamente que f es biyectiva y que, al no

ser nunca f (f (n) + 1) = 2, tiene que ser f (1) = 2.

Solución:

Por la condición a), es obvio que f es biyectiva. Y, como por la condición b) se observa que nunca

es f (f (n) + 1) = 2, forzosamente tendrá que ser 2 la imagen del único elemento que no es de la

forma f (n) + 1, o sea de 1: f (1) = 2. Y de nuevo por a), f (2) = 1.

Vamos a probar, por inducción sobre n, que

par es si1

impar es si 1)(

nn

nnnf

Para n = 1 y para n = 2 ya está visto. Supongamos n > 2.

Por hipótesis de inducción, se tiene

impar es si2

par es si )1(

nn

nnnf

y también

par es si3

impar es si 1)2(

nn

nnnf

Ahora,

par es si1

impar es si ,12)(

nnff

nnffnf

par es si1

impar es si 132

nn

nnn

Problema 2. Consideramos los siguientes 27 puntos de un cubo: el centro (1), los centros de las

caras (6), los vértices (8) y los centros de las aristas (12). Coloreamos cada uno de esos puntos de

azul o de rojo. ¿Puede hacerse de modo que no haya tres puntos del mismo color alineados?

Demuéstralo.

Solución:

La respuesta es no. Para demostrarlo, llamaremos a los puntos centrales de las caras de la siguiente

forma: F-frontal, P-posterior, I-izquierda, D-derecha, T-tapa y S-suelo; en donde el nombre

representa la situación respecto a un observador que mira el cubo desde un punto exterior situado

Page 26: COMPENDIUM OMEFL - ToomatesXXXV Olimpiada Matemática Española Primera Fase Soluciones de la propuesta de problemas Problema 1 ¿Qué dígitos se han omitido en la siguiente multiplicación?

frente a la cara frontal. El punto medio de cada arista lo denotaremos con las dos letras de los

centros de las dos caras a las que limita esa arista: FT, FI, FD, FS, IT, IP, IS, DT, DP, DS, PT, PS.

Cada vértice lo denotamos con las tres letras de los centros de las tres caras que concurren en ese

vértice: FTD, FTI, DTP, ITP, FSD, FSI, DSP, ISP. El centro del cubo lo denotamos C. Para

indicar que un punto lo hemos coloreado de azul, respectivamente de rojo, escribiremos el nombre

del punto seguido de (a), respectivamente (r).

Supondremos que no hay tres puntos alineados del mismo color y

llegaremos a una contradicción. No hay problema en suponer que

el centro es azul: C(a). (De manera análoga se razonaría si fuera

rojo.) Eso obliga a que de cada dos caras opuestas al menos una

tenga el centro rojo. En consecuencia, habrá tres caras con centro

rojo y concurrentes en un vértice. Podemos suponer, por tanto, que

tenemos F(r), T(r) y D(r). Ahora, distinguimos dos posibilidades:

Caso 1: FTD(r). Lo cual implica ITP(a) y por tanto, alineando con el centro, FSD(r). De la misma

forma, FTD(r) implica DSP(a) y por tanto FTI(r). Hemos llegado a la contradicción FSD(r), F(r),

FTI(r).

Caso 2: FTD(a). Si FT(r), entonces PT(a) y FS(a); se tiene, por tanto, la contradicción FS(a), C(a),

PT(a). Así pues, ha de ser FT(a). De la misma forma, DT(a). Ahora, FTD(a) y FT(a) implica FTI(r);

análogamente, FTD(a) y DT(a) implica DTP(r). Y hemos llegado a la contradicción FTI(r), T(r),

DTP(r).

Problema 3. Un condenado queda en libertad cuando alcance el final de una escalera de 100

escalones. Pero no puede avanzar a su antojo, puesto que está obligado a subir un solo escalón cada

día de los meses impares y a bajar un escalón cada día de los meses pares. Comienza el 1 de enero

de 2001. ¿Qué día quedará en libertad?. ¿Qué día quedaría en libertad si la escalera tuviera 99

escalones?.

Es fácil observar que el primer año va a moverse entre los escalones 1 y 36. Este, el 36, lo alcanza

el día 31 de julio. El 31 de diciembre de ese año, llegará al escalón 3. En general, si un 31 de

diciembre está en el escalón n, el año siguiente:

se mueve entre los escalones n + 1 y n + 36 y termina en el n + 3, si ese año siguiente no es bisiesto,

o bien:

se mueve entre los escalones n + 1 y n + 35 y termina en el n + 2 si ese año siguiente es bisiesto.

Con unas cuentas sencillas, vemos que el 31 de diciembre de 2024 llegará al escalón 66, tras haber

pasado el 31 de julio de ese mismo año por el escalón 99 como punto más elevado. A partir de ahí,

se observa lo siguiente respecto al año 2025: el 31 de enero termina en el escalón 97, el 28 de

febrero en el 69, y el 31 de marzo, por fin en el 100. Si la escalera hubiera tenido un peldaño menos,

habría quedado en libertad 8 meses antes, el 31 de julio de 2024.

T

S

F

P

D I

Page 27: COMPENDIUM OMEFL - ToomatesXXXV Olimpiada Matemática Española Primera Fase Soluciones de la propuesta de problemas Problema 1 ¿Qué dígitos se han omitido en la siguiente multiplicación?

4/2/2021 Fase local 1999

www.olimpiadamatematica.es/platea.pntic.mec.es/_csanchez/loc2002.html 1/3

OLIMPIADA MATEMÁTICA ESPAÑOLA

Fase local 2002

Viernes 18 de enero de 2002

Sesión de Mañana

1. Si p es un número real y las raíces de x3 + 2px2 - px + 10 = 0 están en progresión aritmética, halla dichasraíces.

2. En el triángulo ABC, la bisectriz trazada desde A divide al lado opuesto en dos segmentos, de los queconocemos uno: BT = 572 m. Si dicha bisectriz corta a la mediana BM en los segmentos BD = 200 m y DM =350 m, calcula el lado a de dicho triángulo y plantea una ecuación con incógnita c para obtener el lado c (nohace falta que lo calcules explícitamente).

3. Encuentra todos los enteros positivos m y n tales que n! + 1 = (m! - 1)2.

Viernes 18 de enero de 2002

Sesión de Tarde

Page 28: COMPENDIUM OMEFL - ToomatesXXXV Olimpiada Matemática Española Primera Fase Soluciones de la propuesta de problemas Problema 1 ¿Qué dígitos se han omitido en la siguiente multiplicación?

4/2/2021 Fase local 1999

www.olimpiadamatematica.es/platea.pntic.mec.es/_csanchez/loc2002.html 2/3

4. En un equipo de fútbol tenemos 11 jugadores, cuyas camisetas están numeradas del 1 al 11. Elegimos alazar 6 de ellos. ¿Cuál es la probabilidad de que la suma de los números de sus camisetas sea impar?

5. La suma de las edades de los 120 estudiantes que participaron el año pasado en la fase final de laOlimpiada Matemática fue de 2002 años. Demuestra que podrías haber elegido 3 de ellos tales que la suma desus edades no fuera menor de 51 años.

6. Escribo en la pizarra 14 números enteros, no necesariamente distintos, que verifican la propiedad de queal borrar cualquiera de ellos, puedo agrupar los trece restantes en tres montones de igual suma.

a) Demuestra que cada uno de los catorce es múltiplo de 3. b) ¿Es posible que alguno de los catorce que he escrito no sea el 0?

Sábado 19 de enero de 2002

Sesión de Mañana

7. En el triángulo acutángulo ABC, AH, AD y AM son, respectivamente, la altura, la bisectriz y la medianaque parten desde A, estando H, D y M en el lado BC. Si las longitudes de AB, AC y MD son, respectivamente,11, 8 y 1, calcula la longitud del segmento DH.

8. Se sabe que el número de soluciones reales del sistema

(y2 + 6)(x -1) = y (x2 + 1) (x2 + 6)(y -1) = x (y2 + 1)

es finito. Prueba que este sistema tiene un número par de soluciones reales. (Nota: Decimos que la solución (x0, y0) es real cuando x0 e y0 son números reales)

9. Considera 7 puntos arbitrarios del plano y los 21 segmentos que los conectan entre sí. Demuestra que almenos 3 de estos 21 segmentos son de distinta longitud.

Soluciones en formato Microsoft Word 2000 comprimido .zip (31 Kb)

|Página principal| |Info Alumnos| |Info Profesores| |Otros sitios de interés| |Problemas|

Copyright © C. Sánchez-Rubio

Page 29: COMPENDIUM OMEFL - ToomatesXXXV Olimpiada Matemática Española Primera Fase Soluciones de la propuesta de problemas Problema 1 ¿Qué dígitos se han omitido en la siguiente multiplicación?

4/2/2021 Fase local 1999

www.olimpiadamatematica.es/platea.pntic.mec.es/_csanchez/loc2002.html 3/3

Actualizado 27 Enero 2002

Page 30: COMPENDIUM OMEFL - ToomatesXXXV Olimpiada Matemática Española Primera Fase Soluciones de la propuesta de problemas Problema 1 ¿Qué dígitos se han omitido en la siguiente multiplicación?

Soluciones viernes 18 enero 2002 sesión de mañana

1. Sean a, 2

ca y c dichas raíces.

Así pues, cxca

xaxpxpxx

2102 23 , de donde, identificando coeficientes llegamos

a:

pca 22

3 (1);

pac

ca

2

2

(2); 102

ac

ca (3).

De (1) y (3) sigue que p

ac 5

3 y como

3

4 pca , llevando estos valores de a + c y ac a (2)

podemos concluir que 16p3 + 18p

2 + 270 0, es decir, 8p

3 + 9p

2 + 135 0. Una raíz real de este

polinomio es p 3 y como 8p3 + 9p

2 + 135 (p + 3) ( 8p

2 15p + 45), sigue que p 3 es la única

raíz real de dicho polinomio. Esto nos lleva a ac 5, a + c 4, de donde a y c son 5 y 1 y las raíces

que nos piden son 1, 2 y 5.

2.- Como BD es la bisectriz de A y BD 200 y DM 350, sigue que 4

7

DB

DM

AB

AM, de donde

AM 7k y AB 4k.

Así pues, AC 14k y, volviendo a aplicar la relación

anterior, sigue que 4

14

572

TC TC 2002

Así pues a BT + TC 2574 m.

Sea c AB 4

7cAM y

2

7cAC .

Aplicando ahora el teorema del coseno a los triángulos ABC y ABM, podemos escribir:

Cccc

cos74

492574 22

22 (1)

Cc

cc

cos2

7

16

49550

22

22 (2)

En (1), 22

2 25744

53cos7

cCc y en (2), 2

22 5502

8

65cos7

cCc

Así pues, 2

22

2

55028

652574

4

53

cc

2. Si m y n son enteros positivos y n! + 1 (m! 1)2, sigue que m 3.

La ecuación dada se transforma en n! + 1 (m!)2 2m! + 1, o sea n! m! (m! 2). Dividiendo por m!

(obviamente n > m), tenemos que n(n 1) (n 2) ... (m + 1) m! 2 y al ser m! divisible por 3 (m 3),

A

B

D

M

C T

Page 31: COMPENDIUM OMEFL - ToomatesXXXV Olimpiada Matemática Española Primera Fase Soluciones de la propuesta de problemas Problema 1 ¿Qué dígitos se han omitido en la siguiente multiplicación?

sigue que m! 2 no es divisible por 3, por lo que el término de la izquierda, n(n 1) ... (m + 1), debe

tener a lo sumo dos factores. Así pues, tenemos:

1 factor: n m + 1 m + 1 m! 2 m m! 3. Como m divide a m! 3 y divide a m! sigue que

m divide a 3 m 3 y n 4. Compruebo y es solución.

2 factor: n m + 2 (m + 2) (m + 1) m! 2 m2 + 3m + 4 m! .

Así pues 3m m! m2 4 con lo que m divide a m! m

2 4, de lo que sigue que m divide a

4 y, por tanto, m 4. Pero m 4 no es solución de m2 + 3m + 4 4! pues 16 + 12 + 4 24.

La única solución es, entonces, m 3, n 4

Soluciones viernes 18 enero 2002 sesión de tarde

4.- Hay

6

11 elecciones posibles.

La suma de los números de las camisetas de los elegidos será impar si hay entre ellos una cantidad

impar de números impares.

Escribamos ahora los casos favorables. Hay 6 números impares y 5 pares.

Una camiseta impar y 5 pares: 1·65

1

6

Tres camisetas impares y 3 pares: 10·203

3

6

Cinco camisetas impares y 1 par: 5·61

5

6

Así pues, la probabilidad pedida será 231

118

462

236

6

11

56102016

5.- Calculemos en primer lugar las ternas posibles que podríamos haber elegido:

280840118119203

120

.

Veámoslo por contradicción:

Si no hubiera ninguna terna de suma de edades mayor o igual a 51 años, es que cada una sumaba un

número de años menor o igual a 50.

Así pues, la suma de todas las ternas sería menor o igual a 50 · 280840 14042000.

Pero calculemos la suma de todas las ternas:

Cada alumno aparecerá en

2

119 ternas, o sea, en 119 · 59 7021 ternas, luego la suma de las edades

de todas las ternas sería 7021 · 2002 14056042, lo que contradice que la tal suma era menor o igual a

14042000.

6.- Sean a1, a2, a3, ..., a14 los números que he escrito y

14

1i

iaS

a) Me dicen que para cada i, S ai 3bi; siendo bi la suma de cada montón obtenido al quitar ai.

Así pues:

1 1

2 2

14 14

3

3

3

S a b

S a b

S a b

- =

- =

- =

L

Page 32: COMPENDIUM OMEFL - ToomatesXXXV Olimpiada Matemática Española Primera Fase Soluciones de la propuesta de problemas Problema 1 ¿Qué dígitos se han omitido en la siguiente multiplicación?

Sumando estas igualdades, llegamos a 14S S 3(b1 + b2 +...+ b14) 13S 3T T/13S y como 13

es primo, T/S, así que S 3c con c entero.

Escribiendo ahora S ai 3bi; como 3c ai 3bi, sigue que cada ai es múltiplo de 3.

b) Hemos probado que cada 3

ia es un número entero, llamémosle di. Trabajemos ahora con estos

nuevos catorce enteros.

14

13

i

i

Sd . Para cada i, .

3

3

3333

iiii

baSaSd

S

Así pues, quitando cada di, puedo agrupar,

los restantes en 3 montones de igual suma, con lo que cada di es múltiplo de 3 (siguiendo el

argumento de a). Esto nos lleva a que 23

ia es múltiplo de 3. Reiterando este proceso, llegamos a que

para cada k , k

ia

3 es múltiplo de 3, con lo que la única salida es que 0

3

k

ia ai 0 para cada

i, de donde no es posible que algún ai no sea 0.

Soluciones sábado 19 enero 2002 sesión de mañana

7.- Sabemos que al ser D de la bisectriz, 118

BDDC ; así pues BD 11k, DC 8k.

Por otra parte BM MC, es decir, 11k 1 8k +1 3

2k ,

3

22BD y

3

16DC .

Llamando ahora x a DH y aplicando el teorema de Pitágoras a los triángulos rectángulos ABH y AHC,

podemos escribir que 4

5

3

168

3

2211

2

2

2

2

xxx .

8.- Observando la simetría del sistema, es decir, que podemos cambiar x por y, llegamos a que si (x0, y0) es

solución, también lo es (y0, x0). Así pues, el número de soluciones -que es finito- será la suma del número de

soluciones (a, b) con a b más el número de soluciones de la forma (a, a).

Acabamos de ver que el número de solución de la forma (a, b) con a b es par, luego lo único que nos

queda probar es que si ponemos x en lugar de y, obtenemos un número par de soluciones.

Veamos: (x2 + 6) (x + 1) x (x

2 1). Esta ecuación es equivalente a x

2 + 6x 6 x, es decir, x

2 5x + 6 0

cuyas soluciones son 2 y 3, de donde las soluciones de la forma (a, a) son (2, 2) y (3, 3), es decir, un número

par como queríamos demostrar.

A

B C M D H

h 8 11

1

Page 33: COMPENDIUM OMEFL - ToomatesXXXV Olimpiada Matemática Española Primera Fase Soluciones de la propuesta de problemas Problema 1 ¿Qué dígitos se han omitido en la siguiente multiplicación?

9.- Escojamos dos de los 7 puntos A y B, de forma que la recta AB deje todos los puntos en un mismo

semiplano.

Supongamos que solo hubiera 2 longitudes distintas:

a y b. Llegaremos a una contradicción.

Tomando AB como base, que supondremos mide b,

puedo formar 5 triángulos distintos.

(Estos triángulos podrían degenerar en segmentos). Para los otros dos lados, tengo longitudes a y b, es

decir que los lados de los triángulos, en orden, sería:

baa, bab, bba, bbb, con lo que el quinto debería ser uno de éstos, es decir, al menos dos de estos

triángulos serían coincidentes y no tendríamos 7 puntos distintos.

A B

Page 34: COMPENDIUM OMEFL - ToomatesXXXV Olimpiada Matemática Española Primera Fase Soluciones de la propuesta de problemas Problema 1 ¿Qué dígitos se han omitido en la siguiente multiplicación?

4/2/2021 Fase local 1999

www.olimpiadamatematica.es/platea.pntic.mec.es/_csanchez/loc2003.html 1/3

OLIMPIADA MATEMÁTICA ESPAÑOLA

Fase local 2003

Viernes 17 de enero de 2003 Sesión de Mañana

1. ¿Cuántas ternas ordenadas de números enteros y positivos (a, b, c) distintos de la unidad hay tales que

a.b.c = 739 ?

2. Dibuja una semicircunferencia con centro en O y diámetro AB y, en su interior, otra, con diámetro OA.Traza por un punto C de OA una recta perpendicular a dicho segmento OA, que cortará a la semicircunferenciapequeña en D y a la grande en E y, finalmente, la recta AD que cortará al semicírculo grande en F. Demuestra que la circunferencia circunscrita al triángulo DEF es tangente a la cuerda AE en E

3. ¿Cuál es el número máximo de vértices que podemos elegir de un polígono regular de 21 lados para que,al trazar los segmentos que los unen entre sí, no haya dos con la misma longitud?

Viernes 17 de enero de 2003 Sesión de Tarde

Page 35: COMPENDIUM OMEFL - ToomatesXXXV Olimpiada Matemática Española Primera Fase Soluciones de la propuesta de problemas Problema 1 ¿Qué dígitos se han omitido en la siguiente multiplicación?

4/2/2021 Fase local 1999

www.olimpiadamatematica.es/platea.pntic.mec.es/_csanchez/loc2003.html 2/3

4. Determina los dos valores de x más próximos (por defecto y por exceso) a 2003º que cumplen lasiguiente ecuación trigonométrica:

5. Un cuadrado de papel ABCD se dobla de modo que el vértice A toque en un punto arbitrario E del ladoCD. Así, se obtienen tres triángulos rectos formados por una sola capa de papel. Determinar la longitud de sus lados en función de x = DE para demostrar que el perímetro del triángulo mayores la suma de los perímetros de los otros dos, y vale la mitad que el perímetro del cuadrado. (Teorema de Haga) .

6. Dado el polinomio p(x) = x3 + Bx2 + Cx + D, prueba que si el cuadrado de una de sus raíces es igual alproducto de las otras dos, entonces B3D = C3.

Sábado 18 de enero de 2003 Sesión de Mañana

1. Se dispone de pequeñas piezas de madera de tamaño 4 x 5 x 10. Decide si es posible o no apilarlas, sindejar huecos y apoyándolas siempre sobre cualquiera de sus caras, para formar un ortoedro de dimensiones 22003 x 32003 x52003.

2. Dado un triángulo de vértices A, B y C,y con lados de longitud a = BC, b = AC y c = AB, llamemos D alpunto de intersección del lado AB con la bisectriz del ángulo C. Demuestra que:

3. ¿Existirán 16 números naturales distintos y menores de 100 tales que al colocarlos en las casillas de untablero 4 x 4 el

Page 36: COMPENDIUM OMEFL - ToomatesXXXV Olimpiada Matemática Española Primera Fase Soluciones de la propuesta de problemas Problema 1 ¿Qué dígitos se han omitido en la siguiente multiplicación?

4/2/2021 Fase local 1999

www.olimpiadamatematica.es/platea.pntic.mec.es/_csanchez/loc2003.html 3/3

producto de los situados en cada fila sea el mismo y, a su vez, coincida con el de los colocados en cada columnay en las dos diagonales principales.? Si la respuesta es afirmativa, indica cuáles son. Si la respuesta es negativa, justifícalo.

Sábado 18 de enero de 2003 Sesión de tarde

4. Prueba que si los números loga x, logb x y logc x (con x distinto de 1) están en progresión aritmética,entonces

5. ¿Qué condición han de cumplir las longitudes de los lados de un triángulo cualquiera para que la líneaque une el baricentro (centro de gravedad del triángulo o punto donde coinciden las medianas) y el incentro (punto común alas tres bisectrices) sea paralela a uno de los lados?

6. Por turno, en orden alfabético, tres amigos lanzan un dado. Quien saque un 6 en primer lugar gana loapostado. Por cada euro que apueste Carlos, ¿qué cantidad han de poner Ana y Blas para equilibrar el juego y lograr quesea equitativo, es decir, para que las expectativas de ganancia sean las mismas para los tres colegas y no se veanafectadas por el orden de actuación al lanzar el dado?

Soluciones en formato Microsoft Word 2000 comprimido .zip (180 Kb)

|Página principal| |Info Alumnos| |Info Profesores| |Otros sitios de interés| |Problemas|

Copyright © C. Sánchez-Rubio

Actualizado 26 Enero 2003

Page 37: COMPENDIUM OMEFL - ToomatesXXXV Olimpiada Matemática Española Primera Fase Soluciones de la propuesta de problemas Problema 1 ¿Qué dígitos se han omitido en la siguiente multiplicación?

Fase Local.

Viernes Mañana.

Problema -1. ¿Cuántas ternas ordenadas de números naturales (a, b, c) distintos de la unidad hay

tales que

a.b.c = 739

?

Solución:

Como 7 es primo y 1a , 1b y 1c , 39. . 7 .7 .7 7p q ra b c con , ,p q r N

Por tanto, el número de ternas ordenadas (a, b, c) será el mismo que el de ternas (p,

q, r) con la condición p + q + r = 39

Tabulemos y contemos:

P q r Nº de ternas

1 1 37

37 ... 2 36

... ... ...

... 37 1

2 1 36

36 ... 2 35

... ... ...

... 36 1

3 1 35

35 ... 2 34

... ... ...

... 35 1

... ... ... ...

... ... ... ...

36 1 2 2

... 2 1

37 1 1 1

El total de ternas será:

37 1

37 36 35 ... ... 2 1 . 37 19 . 37 7032

Page 38: COMPENDIUM OMEFL - ToomatesXXXV Olimpiada Matemática Española Primera Fase Soluciones de la propuesta de problemas Problema 1 ¿Qué dígitos se han omitido en la siguiente multiplicación?

Problema – 2.

Dibuja un semicírculo con centro en O y diámetro AB y, en su interior, otro, con

diámetro OA. Traza por un punto C de OA una recta perpendicular a dicho radio OA,

que cortará al semicírculo pequeño en D y al grande en E y, finalmente, la recta AD

que cortará al semicírculo grande en F.

Demuestra que el círculo circunscrito al triángulo DEF es tangente a la cuerda AE

en E

Solución:

-El triángulo AEB es rectángulo.

Por el Teorema del Cateto: AE2 = AC.AB

-El cuadrilátero BCDF es inscriptible, pues sus ángu-

los opuestos C y F son rectos.

Así, las rectas ACB y ADF son secantes a la cir-

cunferencia que lo circunscribe.

- La potencia del punto A respecto de esa circunferencia nos da:

AC.AB = AD.AF

- Por tanto: AE2 = AD.AF.

Y esto quiere decir, por potencia de A respecto a la circunferencia que circunscribe

al triángulo DEF, que la recta AE es tangente a dicha circunferencia en E.

Problema – 3.

¿Cuál es el número máximo de vértices de un polígono regular de 21 lados que po-

demos elegir para que, al trazar los segmentos que los unen entre sí, no haya dos con

la misma longitud?

Solución:

AB

OC

D

EF

1

3

4

6

7

8

9

1213

14

21

16

18

19

20

2

5

17

15

11

10

12

3

4

5

6

7

8

9

1213

14

21

15

16

17

18

19

20

11

10

Page 39: COMPENDIUM OMEFL - ToomatesXXXV Olimpiada Matemática Española Primera Fase Soluciones de la propuesta de problemas Problema 1 ¿Qué dígitos se han omitido en la siguiente multiplicación?

Por la simetría de la figura, sólo hay 10 distancias distintas. -Como mucho, podremos elegir 5 vértices. Pues, entre cinco puntos no alineados

se pueden trazar 5,2 10C segmentos.

- Nos faltará constatar si con 5, y con qué 5, vértices se puede.

La figura de la derecha muestra una posibilidad.

Page 40: COMPENDIUM OMEFL - ToomatesXXXV Olimpiada Matemática Española Primera Fase Soluciones de la propuesta de problemas Problema 1 ¿Qué dígitos se han omitido en la siguiente multiplicación?

Fase Local.

Viernes tarde.

Problema – 4.

Determina los dos valores de x más próximos (por defecto y por exceso) a 2003º

que cumplen la siguiente ecuación trigonométrica:

2 2 2 2 2 2

1 1 1 1 1 1- - - - - -3

cos tg cotg sec cosecsen x x x x x x

Solución:

La expresión se puede escribir así

2 2 2 2 2 2cos - sec - cot - tg - cos - sen -3ec x x x x x x 2 2 2 2(1 cot ) - (1 ) - cot - tg - 1 -3x tg x x x

y se reduce a la sencilla ecuación trigonométrica 2 1tg x

que tiene por soluciones: 45º 90ºx k con k Z

Los valores pedidos se obtienen para 1 21k y 2 22k

y son 1 1935ºx y 2 2025ºx

Problema – 5.

Un cuadrado de papel ABCD, de lado unidad, se dobla de

modo que el vértice A toque en un punto arbitrario E del la-

do CD. Así, se obtienen tres triángulos rectos formados por

una sola capa de papel.

Determinar la longitud de sus lados en función de x = DE

y demostrar que el perímetro del triángulo mayor es la suma

de los perímetros de los otros dos, y vale la mitad que el pe-

rímetro del cuadrado. (Teorema de Haga)

D C

A B

E x

Page 41: COMPENDIUM OMEFL - ToomatesXXXV Olimpiada Matemática Española Primera Fase Soluciones de la propuesta de problemas Problema 1 ¿Qué dígitos se han omitido en la siguiente multiplicación?

Solución:

Denominamos, con letras mayúsculas, los puntos

característicos que produce el plegado y, con minús-

culas, los lados de los triángulos

Los lados del triángulo DEF se obtienen, resol-

viendo el sistema:

2 2 2

1

x z y

z y

222 1

1

yyx

yz

021

12 yx

yz

2

1

2

1 22 xz

xy

y su perímetro es 12

1

2

1 22

xxx

xzyxPDEF

Los triángulos rectángulos de una capa de papel son semejantes, pues, por un lado,

FED y IEC son complementarios y, por otro, EIC = GIH.

Por semejanza de los triángulos EDF y ECI.

x w

z u

. (1 )x u x xw

z z

;

y v

z u

. (1 )y u y xv

z z

Los lados del triángulo ECI son: 1u x 21

1

xv

x

2

1

xw

x

y su perímetro 2 21 1 2

21 1 1

ECI

x x xP u v w

x x x

Por semejanza de los triángulos ECI y IHG.

w r

v s

. (1 )v r v vs

w w

Los lados del triángulo IHG son:

(1 )1

1

x xr v

x

2(1 )(1 )

2(1 )

x xs

x

2(1 )1

2

xt w s

y su perímetro es

2 2

2 2

2 (1 ) (1 )(1 ) (1 ) (1 )

2(1 ) 2(1 ) 2(1 )

(1 ) 2 (1 ) (1 ) (1 ) 2 21

2(1 ) 2(1 )

IHG

x x x x x xP r s t

x x x

x x x x x xx

x x

Queda probado lo que se pedía: ( 1) (1 ) 2EDF IHG ECIP P x x P y que

2ECIP , es la mitad del perímetro del cuadrado.

y

D x

z

F

E u

v

A

r

C

w

I

t

s

G

H

B

Page 42: COMPENDIUM OMEFL - ToomatesXXXV Olimpiada Matemática Española Primera Fase Soluciones de la propuesta de problemas Problema 1 ¿Qué dígitos se han omitido en la siguiente multiplicación?

Problema – 6.

Dado el polinomio 3 2( )p x x Bx Cx D , probar que si el cuadrado de una

de sus raíces es igual al producto de las otras dos, entonces 3 3B D C

Solución:

Llamemos r , s y t a las tres raíces.

El polinomio lo podemos escribir así: ( ) ( )( )( )p x x r x s x t .

Si operamos 3 2( ) ( ) ( )p x x r s t x rs st tr x rst

e igualamos coeficientes, obtenemos las conocidas relaciones de Cardano-Vieta:

r s t B

rs st tr C

rst D

Y como 2r st , quedan así: 2

3

( )C rs r tr r s r t rB

D rst r

Finalmente, elevando al cubo esta primera expresión conseguimos lo pedido:

3 3 3 3 3( )C rB r B B D

Page 43: COMPENDIUM OMEFL - ToomatesXXXV Olimpiada Matemática Española Primera Fase Soluciones de la propuesta de problemas Problema 1 ¿Qué dígitos se han omitido en la siguiente multiplicación?

Fase Local.

Sábado Mañana.

Problema – 1.

Se dispone de pequeñas piezas de madera de tamaño 4 x 5 x 10. Decidir si es posi-

ble o no apilarlas, sin dejar huecos y apoyándolas siempre sobre cualquiera de sus

caras, para formar un ortoedro de dimensiones 22003

x 32003

x 52003

.

Solución:

La superficie de cada una de las caras del ortoedro es:

C1 = 22003

x 32003

= 62003

, C2 = 22003

x 52003

= 102003

y C3 = 32003

x 52003

= 152003

Y, de ser posible el apilamiento, debería ser combinación lineal (con coeficientes na-

turales) de superficies de las caras de las piezas de madera:

de 4 x 5 = 20, de 4 x 10 = 40 y de 5 x 10 = 50; esto es, múltiplo de 10.

Pero como 62003

no lo es, el apilamiento ortoédrico es imposible.

Problema – 2.

Dado un triángulo de vértices A, B y C, y con lados de longitud a BC , b AC y

c AB , llamemos D al punto de intersección del lado AB con la bisectriz del ángulo

C.

Demuestra que : 2 cos

2

Cab

CDa b

Solución:

- A partir del vértice B trazamos una paralela

a la bisectriz CD y prolongamos el lado AC

hasta obtener el punto E.

Y, también, CF perpendicular a BE

- Así, CB = CE = a

- Por ángulos alternos-internos, en el triángulo BCF tenemos: cos2 2

C FB EB

a a

A

BE F

C

b

a

q

D

p

Page 44: COMPENDIUM OMEFL - ToomatesXXXV Olimpiada Matemática Española Primera Fase Soluciones de la propuesta de problemas Problema 1 ¿Qué dígitos se han omitido en la siguiente multiplicación?

- Los triángulos ACD y AEB son semejantes: AC CD

AE EB

2 cos. 2

Cab

AC EBCD

AE a b

Solución – 2.

- Por el Teorema de la Bisectriz: a q

b p

- Y aplicando en los dos triangulitos el Teorema del Coseno:

2 22 2

2 22 2

- 2. . .cos2

- 2. . .cos2

CCD a CD a

a q

Cb pCD b CD b

- Distinguimos dos casos:

a b Multiplicando en cruz 2 2 2( ). 2 ( ). .cos2

Cb a CD ab b a CD

y simplificando queda: ( ). 2 cos2

Cb a CD ab

a = b En este caso el triángulo es isósceles.

Y, directamente, se tiene: 2 cos

2cos2

Cab

CCD a

a b

Solución – 3.

Por áreas de triángulos: 1

sin2 2

BCD

CS aCD

1

sin2 2

ACD

CS bCD

1sin sin .cos

2 2 2ABC

C CS ab C ab

BCD ACD ABCS S S 1

( ) sin sin cos2 2 2 2

C C Ca b CD ab

y simplificando se tiene la relación pedida

Problema – 3.

¿Existirán 16 números naturales distintos y menores de 100 tales que al colocarlos

en las casillas de un tablero 4 x 4 el producto de los situados en cada fila sea el mis-

mo y, a su vez, coincida con el de los colocados en cada columna y en las dos diago-

nales principales.?

Si la respuesta es afirmativa, indica cuáles son.

Si la respuesta es negativa, justifícalo.

Page 45: COMPENDIUM OMEFL - ToomatesXXXV Olimpiada Matemática Española Primera Fase Soluciones de la propuesta de problemas Problema 1 ¿Qué dígitos se han omitido en la siguiente multiplicación?

Solución:

Veamos que sí existen.

Consideremos estos dos conjuntos: 1, 2, 3, 5A y 7, 11, 13, 17B

Los 16 productos que se obtienen al multiplicar un número de A por uno de B son

todos distintos y menores de 100.

Los colocaremos de forma que, en cada fila, en cada columna y cada diagonal

principal, los números de A y B aparezcan como factores exactamente una vez.

Procederemos así:

1º) En una diagonal ponemos: 1.17, 2.13, 3.11 y 5.7.

2º) En las esquinas restantes: 2.11 y 3.13.

3º) Y completamos las demás casillas.

El producto de cada fila, cada columna y cada diagonal es siempre el mismo:

1.2.3.5.7.11.13.17 = 510510

1.133.172.11 5.7

2.133.7

5.13 1.7

1.17 5.11

5.17 1.11

2.173.11

3.132.7

13 3522 51

33

85

14

21 26

65 7

17 55

11

34

39

Page 46: COMPENDIUM OMEFL - ToomatesXXXV Olimpiada Matemática Española Primera Fase Soluciones de la propuesta de problemas Problema 1 ¿Qué dígitos se han omitido en la siguiente multiplicación?

Fase Local.

Sábado tarde.

Problema – 4.

Prueba que si los números loga x , logb x y logc x con ( 1)x están en progresión

aritmética, entonces

log2 . a b

c a c

Solución:

Por ser tres números en progresión aritmética 2log log logb a cx x x

log log log2

log log log

x x x

b a c y como log 0x

log log2 log log

log loga c

b bb b

a c

Uniformando la base log log2log log log log ( . )a ab b

c c c cc c b b c

Por tanto: log log log log2 . . ( . )a a a ab b b bc b c a c a c como queríamos probar.

Problema – 5.

¿Qué condición han de cumplir las longitudes de los lados de un triángulo cual-

quiera para que la línea que une el baricentro (centro de gravedad del triángulo o

punto donde coinciden las medianas) y el incentro (punto común a las tres bisectri-

ces) sea paralela a uno de los lados?

Solución:

- Llamemos I al incentro.

Por el Teorema de la Bisectriz:

En el triángulo ADC: AC CI

AD ID

En el triángulo BDC: BC CI

BD ID

Luego CI AC BC AC BC AC BC b a

ID AD BD AD BD AB c

AD

I

C

B

Page 47: COMPENDIUM OMEFL - ToomatesXXXV Olimpiada Matemática Española Primera Fase Soluciones de la propuesta de problemas Problema 1 ¿Qué dígitos se han omitido en la siguiente multiplicación?

-Dibujemos la situación completa.

Denominemos M al punto medio del lado AB y G al baricentro.

Los triángulos CIG y CDM han de ser semejantes. Así, por el Teorema de Tales:

CI CG

ID GM

Y por la propiedad del baricentro: 2CG

GM

Luego 2CI

ID

-Conclusión: 2CI a b

ID c

La línea que une baricentro e incentro de

un triángulo será paralela al lado AB si su

longitud es media aritmética de los otros

dos: 2

a bc

Problema – 6.

Por turno, en orden alfabético, tres amigos lanzan un dado. Quien saque un 6 en

primer lugar gana lo apostado.

Por cada euro que apueste Carlos, ¿qué cantidad han de poner Ana y Blas para

equilibrar el juego y lograr que sea equitativo, es decir, para que las expectativas de

ganancia sean las mismas para los tres colegas y no se vean afectadas por el orden de

actuación al lanzar el dado?

Solución:

El esquema en árbol nos ayuda-

rá a determinar las probabilida-

des que tienen cada uno de los

amigos de ganar en este juego:

MA D

I G

C

B

5/6

A

1/6

5/6

5/6

5/6

1/6

C

1/6

1/6

B A

......5/6

5/6

1/6

1/6

B C

Page 48: COMPENDIUM OMEFL - ToomatesXXXV Olimpiada Matemática Española Primera Fase Soluciones de la propuesta de problemas Problema 1 ¿Qué dígitos se han omitido en la siguiente multiplicación?

3 6 3 6

3

3 3 3

1 5 1 5 1 1 5 5( ) ... ... 1 ... ...

6 6 6 6 6 6 6 6

1 1 1 6 36

6 6 6 5 9151

6

p A

4 7 3 6

3

32 2 3 3

5 1 5 1 5 1 5 1 5 5( ) ... ... 1 ... ...

6 6 6 6 6 6 6 6 6 6

5 1 5 6 30

6 6 6 5 9151

6

p B

2 5 8 2 3 6

2 2 3

33 3 3 3

5 1 5 1 5 1 5 1 5 5( ) ... ... 1 ... ...

6 6 6 6 6 6 6 6 6 6

5 1 5 6 25

6 6 6 5 9151

6

p B

Por cada 91 € en litigio, 36 los debe poner Ana, 30 Blas y 25 Carlos.

Luego, si Carlos apuesta 1 €, Ana debe poner 1’44€ y Blas 1’20 €.

Obviamente, así, el juego es justo, pues la esperanza matemática de ganar de cada

jugador es cero. De todas formas, veámoslo:

Definimos las siguientes variables aleatorias:

XA = ganancia de Ana

1x Gana 2x Pierde

AX +2’20 -1’44

( )A ip X x 3691

5591

36 55( ) 2'20 1'44 0

91 91AE X

XB = ganancia de Blas

1x Gana 2x Pierde

BX +2’44 -1’20

( )B ip X x 3091

6191

30 61( ) 2'44 1'20 0

91 91BE X

Page 49: COMPENDIUM OMEFL - ToomatesXXXV Olimpiada Matemática Española Primera Fase Soluciones de la propuesta de problemas Problema 1 ¿Qué dígitos se han omitido en la siguiente multiplicación?

XC = ganancia de Carlos

1x Gana 2x Pierde

CX +2’64 -1

( )C ip X x 2591

6691

25 66( ) 2'64 1. 0

91 91CE X

El juego es equitativo: las expectativas de ganancia para los tres amigos es la misma.

Page 50: COMPENDIUM OMEFL - ToomatesXXXV Olimpiada Matemática Española Primera Fase Soluciones de la propuesta de problemas Problema 1 ¿Qué dígitos se han omitido en la siguiente multiplicación?

4/2/2021 XL Olimpiada Matemática Española

www.olimpiadamatematica.es/platea.pntic.mec.es/_csanchez/loc2004.html 1/4

OLIMPIADA MATEMÁTICA ESPAÑOLA

Fase local 2004

Primera sesiónMañana del viernes 15 de enero de 2004

Problema 1.

Consideremos los polinomios , (x es

la variable, a, b, c, A, B, C son parámetros). Sabemos que las tres raíces de P son positivasy que las raíces de Q son los números inversos de las raíces de P. Probad que ,

.

Problema 2.

En un tablero de damas (8 x 8), colocamos las 24 fichas del juego de modo quellenen las 3 filas de arriba. Podemos cambiar la posición de las fichas según el siguientecriterio: una ficha puede saltar por encima de otra a un hueco libre, ya sea horizontal (aizquierda o derecha), vertical (hacia arriba o hacia abajo) o diagonalmente. ¿Podemoslograr colocar todas las fichas en las 3 filas de abajo?

Problema 3.

¿Podemos trazar 2003 segmentos en el plano de forma que cada uno de ellos corteexactamente a otros tres?.

Segunda sesiónTarde del viernes 15 de enero de 2004

Problema 4.

Page 51: COMPENDIUM OMEFL - ToomatesXXXV Olimpiada Matemática Española Primera Fase Soluciones de la propuesta de problemas Problema 1 ¿Qué dígitos se han omitido en la siguiente multiplicación?

4/2/2021 XL Olimpiada Matemática Española

www.olimpiadamatematica.es/platea.pntic.mec.es/_csanchez/loc2004.html 2/4

Encontrad todas las funciones tales que para todo

número natural n.

Problema 5.

Un triángulo tiene sus vértices en cada uno de los ejes de un sistema de coordenadascartesianas en el espacio; ninguno está en el origen, ni dos de ellos coinciden el mismoeje, Demostrad que el triángulo es acutángulo.

Problema 6.

Hallad el número mínimo de apuestas de quiniela que debemos rellenar para

asegurar que obtenemos, al menos, 5 aciertos en una de ellas. (Una apuesta de quinielaconsiste en un pronóstico de resultado para 14 partidos, en cada partido hay 3 posiblesresultados).

Primera sesiónMañana del sábado 16 de enero de 2004

Problema 1.

Demostrad que si , ,

Problema 2.

Consideramos los polinomios (x es la

variable, A, B, C son parámetros). Supongamos que, si a, b, c son las tres raíces de P, lasde Q son . Determinad todos los posibles polinomios P, Q.

Problema 3.

Hallad todas las posibles formas de escribir 2003 como suma de dos cuadrados de

números enteros positivos.

Segunda sesiónTarde del sábado 16 de enero de 2004

Problema 4.

Page 52: COMPENDIUM OMEFL - ToomatesXXXV Olimpiada Matemática Española Primera Fase Soluciones de la propuesta de problemas Problema 1 ¿Qué dígitos se han omitido en la siguiente multiplicación?

4/2/2021 XL Olimpiada Matemática Española

www.olimpiadamatematica.es/platea.pntic.mec.es/_csanchez/loc2004.html 3/4

Calculad todos los posibles valores de , donde es una función

que cumple:· para todo par de números naturales m, n.

· para todo número natural n.

·

Problema 5.

¿Existe algún triángulo tal que las medidas de sus lados son tres números

consecutivos y el ángulo mayor es el doble que el menor?. Si existe, determinad susmedidas.

Problema 6.

Hallad las cuatro últimas cifras de .

Soluciones en formato Microsoft Word 2000 comprimido .zip (180 Kb)

|Página principal| |Info Alumnos| |Info Profesores| |Otros sitios de interés| |Problemas|

Copyright © C. Sánchez-Rubio

Actualizado 21 Enero 2004

Page 53: COMPENDIUM OMEFL - ToomatesXXXV Olimpiada Matemática Española Primera Fase Soluciones de la propuesta de problemas Problema 1 ¿Qué dígitos se han omitido en la siguiente multiplicación?

4/2/2021 XL Olimpiada Matemática Española

www.olimpiadamatematica.es/platea.pntic.mec.es/_csanchez/loc2004.html 4/4

Page 54: COMPENDIUM OMEFL - ToomatesXXXV Olimpiada Matemática Española Primera Fase Soluciones de la propuesta de problemas Problema 1 ¿Qué dígitos se han omitido en la siguiente multiplicación?

Soluciones Prueba 2004

Problema 1

Consideremos los polinomios 3 2( )P x x ax bx c , 3 2( )Q x x Ax Bx C (x es la

variable, a, b, c, A, B, C son parámetros). Sabemos que las tres raíces de P son positivas y que las

raíces de Q son los números inversos de las raíces de P. Probad que · 9a A , · 9b B .

Solución:

Denotamos por 1 2 3, ,x x x las raíces de P. Las raíces de Q son 1 2 3

1 1 1, ,

x x x. Utilizamos las fórmulas de

Cardano para conseguir

1 2 3

1 2 3

1 1 1;a x x x A

x x x

;

1 2 1 3 2 3

1 2 1 3 2 3

1 1 1;b x x x x x x B

x x x x x x .

Ahora, ambas desigualdades son del mismo tipo. Basta probar que, si 1 2 3, ,y y y son números

positivos, se tiene que 1 2 3

1 2 3

1 1 19y y y

y y y

. Pero

3 31 2 1 21 2 3

1 2 3 2 1 3 1 3 2

1 1 13

y yy y y yy y y

y y y y y y y y y

.

Así pues, basta probar que si s y t son números positivos, 2s t

t s . Finalmente,

22 2 2

2 0.s ts t s t st

t s st st

Problema 2

En un tablero de damas (8 x 8), colocamos las 24 fichas del juego de modo que llenen las 3

filas de arriba. Podemos cambiar la posición de las fichas según el siguiente criterio: una ficha

puede saltar por encima de otra a un hueco libre, ya sea horizontal (a izquierda o derecha), vertical

(hacia arriba o hacia abajo) o diagonalmente. ¿Podemos lograr colocar todas las fichas en las 3 filas

de abajo?

Solución:

No podemos lograrlo:

Clasificamos (o coloreamos) las casillas del tablero en cuatro tipos, según la paridad de la fila y la

columna que ocupan. Cada ficha se mueve siempre por el mismo tipo de casilla. Pero el número de

casillas de cada tipo que están ocupadas en las posiciones inicial y final esdistinto:

Denotamos II, PI, IP, PP, los cuatro tipos de casillas, donde P indica paridad, I imparidad, la

primera entrada alude a la fila y la segunda a la columna. En la posición incial las ficha ocupan 8

casillas de tipo II, 8 de tipo PI, 4 de tipo IP y 4 de tipo PP.

En la pretendida posición final ocupan 4 casillas de tipo II, 4 de tipo PI, 8 de tipo IP y 8 de tipo PP.

Page 55: COMPENDIUM OMEFL - ToomatesXXXV Olimpiada Matemática Española Primera Fase Soluciones de la propuesta de problemas Problema 1 ¿Qué dígitos se han omitido en la siguiente multiplicación?

Problema 3.

¿Podemos trazar 2003 segmentos en el plano de forma que cada uno de ellos corte

exactamente a otros tres?.

Solución.

No es posible:

Llamamos N al número de cortes. Si sumamos, desde 1 hasta 2003, el número de segmentos que se

cortan con uno dado, cada corte locontamos dos veces. Por tanto ontenemos el número 2N.

Si la hipótesis del enunciado se cumple, se tiene 2003 · 3 = 2N. Pero 2003 ·3 es impar y 2N es par,

algo absurdo.

Problema 4 - 1

Encontrad todas las funciones :f tales que 2f f n n para todo número

natural n.

Solución:

Sea f una función que cumple las condiciones del enunciado.

Sea f(1) = a. Reiterando obtenemos 3, 3 2, , 1f a f a f n n a si n es impar.

Sea f(2) = b. De igual manera, 4, 2f b f n n b si n es par.

De hecho, las condiciones 3, 4, 1f a f b f n n a si n es impar, 2f n n b si n

es par, son necesarias y suficientes para que se cumpla la condición dada en el enunciado.

Para proseguir, debemos distinguir si a y b son pares o impares.

Si a es impar, 3 2 1.f a a Luego a = 2, hecho contradictorio.

Si b es par, 4 2 2f b b . Lugo b = 3, hecho contradictorio.

Así, a es par y b impar. Se tiene 3 2, 4 1f a a b f b a b . En ambos casos,

obtenemos a + b = 5.

Nuevamente las cinco condiciones 1f n n a si n es impar, 2f n n b si n es par, a es

par, b es impar, a + b = 5 son necesarias y suficientes para que f cumpla la condición dada.

Las únicas posibilidades son, o bien a = 2, b = 3; a = 4, b = 1. en un caso obtenemos

1f n n para todo n .

En el otro,

3 si es impar;

1 si es par.

n nf n

n n

Problema 5-2.

Un triángulo tiene sus vértices en cada uno de los ejes de un sistema de coordenadas

cartesianas en el espacio; ninguno está en el origen, ni dos de ellos coinciden el mismo eje,

Demostrad que el triángulo es acutángulo.

Solución.

Sean A, B, C los vértices del triángulo. Denotamos, respectivamente, x, y, z las distancias de los

vértices al origen de coordenadas; y, también respectivamente, a, b, c las longitudes de los lados

opuestos a los vértices.

Basta probar que uno de los ángulos es agudo. Probaremos que cos 0A

. Por el teorema del

coseno 2 2 2 ˆ2 cosa b c bc A . Luego debemos demostrar que 2 2 2a b c .

Page 56: COMPENDIUM OMEFL - ToomatesXXXV Olimpiada Matemática Española Primera Fase Soluciones de la propuesta de problemas Problema 1 ¿Qué dígitos se han omitido en la siguiente multiplicación?

Gracias al teorema de Pitágoras, se tiene que 2 2 2 2 2 2 2 2 2, ,a y z b x y c x y .

Por tanto 2 2 2 2 2 2 2 22b c x y z y z a .

Problema 6 – 3.

Hallad el número mínimo de apuestas de quiniela que debemos rellenar para asegurar que

obtenemos, al menos, 5 aciertos en una de ellas. (Una apuesta de quiniela consiste en un pronóstico

de resultado para 14 partidos, en cada partido hay 3 posibles resultados).

Solución.

Hay que rellenar 3 apuestas:

En 14 partidos, hay un resultado (1, X o 2) que se repite al menos 5 veces (en caso contario, el

número de partidos sería menor o igual qie 4·3 = 12, pero 14 > 12). Hacemos las tres apuestas que

siguen: todo 1, todo X, todo 2. En una de ellas tenemos al menos 5 aciertos.

Por otra parte, si hacemos 2 apuestas, es posible que no obtengamos ningún acierto. Para cada

partido hacemos uno o dos pronósticos distintos y puede suceder el tercero.

Problema 1 - 4

Demostrad que si 1 1x , 1 1y ,

1 1

x yx y

xy xy

Solución.

Si x, y tienen signos opuestos, se tiene , 1 1 1x y x y xy xy xy .

Así pues, la desigualdad es, realmente, una igualdad.

Si la desigualdad se cumple para un par de números (x, y), se cumple para el par opuesto (-x,-y). Por

tanto, sin pérdida de generalidad, podemos suponer que , 0x y .

Si la desigualdad se cumple para un par de números (x, y), se cumple para el par simétrico (y, x).

Por tanto, sin pérdida de generalidad, podemos suponer que 0 y x .

En este caso, 0,1 0, 0, 0, 0,x y xy x y xy y la desigualdad que debemos probar queda

1 1x y xy x y xy .

Si expandimos los términos, esta desigualdad es la misma que

2 2 2 2x y x y xy x y x y xy

Si simplificamos, obtenemos la desigualdad equivalente

22 2x y y

que, puesto que 2 1, 0x y , escierta.

Problema 2 - 5.

Consideramos los polinomios 3 2 2, 3 2P x x Ax Bx C Q x x Ax B (x es la

variable, A, B, C son parámetros). Supongamos que, si a, b, c son las tres raíces de P, las de Q son

,2 2

a b b c . Determinad todos los posibles polinomios P, Q.

Page 57: COMPENDIUM OMEFL - ToomatesXXXV Olimpiada Matemática Española Primera Fase Soluciones de la propuesta de problemas Problema 1 ¿Qué dígitos se han omitido en la siguiente multiplicación?

Solución.

Q es la derivada de P. Por tanto,

,P x x a x b x c Q x x a x b x a x c x b x c

Sea 2

a bd

, punto medio del segmento que une a con b. Se tiene que

2

a bd a d b

.

El valor de Q en d es

2

2 2 2 2 2

b a a b b a a b b aQ d d c d c

Sea 2

b ce

. Si cambiamos los papeles de a, b por los de b, c la igualdad anterior se transforma en

2

2

c bQ e

.

Por tanto d y e son raíces de Q si y sólo si

2 2

02 2

b a c b

: osea, si y sólo si a = b = c.

Ha de ser 3 2, 3P x x a Q x x a para un cierto parámetro a.

Problema 3 - 6

Hallad todas las posibles formas de escribir 2003 como suma de dos cuadrados de números

enteros positivos.

Solución.

No es posible escribir 2003 como suma de dos cuadrados de números enteros positivos:

Tenemos que, si a es entero, 20

mod 41

a

. Luego si a y b son enteros, 2 2

0

1 mod 4

4

a b

.

Pero 2003 3 (mod 4).

Problema 4.

Calculad todos los posibles valores de 2004f , donde :f es una función que

cumple:

o f nm f n f m para todo par de números naturales m, n.

o 2f n n para todo número natural n.

o 1002 1003969f

Solución.

Sea f una función que cumple las condiciones del enunciado.

Descomponemos 1002 y 2004 en factores primos. Se tiene que

1002 = 2 · 3 · 167; 2004 = 22 · 3 · 167

Si 22 2 1f , obtenemos que

Page 58: COMPENDIUM OMEFL - ToomatesXXXV Olimpiada Matemática Española Primera Fase Soluciones de la propuesta de problemas Problema 1 ¿Qué dígitos se han omitido en la siguiente multiplicación?

2 2 22 2 2 21002 2 3 167 2 1 ·3 ·167 2·3·167 3·167 1002 3·167f f f f

Si 23 3 1f , obtenemos que

2 2 22 2 2 21002 2 · 3 1 ·167 2·3·167 2·167 1002 2·167 .f

Si 2167 167 1f , obtenemos que

2 2 22 2 2 21002 2 ·3 167 1 2·3·167 2·3 1002 2·3 .f

En los tres casos anteriores, la menor cantidad que restamos a 10022 es (2 · 3)

2. Por tanto, en

cualquiera de los tres casos 22 21002 1002 2·3 1002 36 1003986f , que contradice las

condiciones de f.

Así pues, 2 2 22 2 , 3 3 , 167 167 .f f f

Por tanto 2 4 2 2 22004 2 3 167 2 ·3 ·167 2004f f f f

Por último, notamos que, al menos, hay una función que cumple las condiciones del enunciado: la

función :f definida por 2f n n lo hace.

Así pues, el único valor posible es 20042 = 4016016.

Problema 5.

¿Existe algún triángulo tal que las medidas de sus lados son tres números consecutivos y el

ángulo mayor es el doble que el menor?. Si existe, determinad sus medidas.

Solución.

Notamos que el triángulo debe ser acutángulo. Si a, b, c son sus lados, hay que probar, gracias al

teorema del coseno, que 2 2 2 2 2 2 2 2 2, ,a b c a c b b c a . Si tomamos a b c , basta probar 2 2 2a b c . Sea 1, , 1a x b x c x .Esta desigualdad equivale a 20 4 1.x x

Ahora, gracias al teorema del seno, el ángulo es más pequeño cuanto más pequeño sea el lado

opuesto. Así, el ángulo opuesto a a es , y el opuesto a c es 2.. El opuesto a b será 3 , y ha

de ser 3 2 ; o sea 5 4

.

Si plantamos el teorema del seno, obtenemos

21 2 32cos ; 4cos 1

1 1

x sen x sen

x sen x sen

Si eliminamos el ángulo, obtenemos 2

11

1 1

x x

x x

Esta expresión simplificada, da x = 5. También obtenemos que 3

cos4

.

O sea que, si el triángulo existe, sus dimensiones son 4, 5, 6. Para que exista, el ángulo debe

cumplir las condiciones cos cos cos4 5

. La primera desigualdad es

1 3,

42 que equivale,

elevando al cuadrado y quitando denominadores, a 16 < 18. La segunda es 3 1 5

,4 4

que

equivale, del mismo modo que antes a 4 < 5.

Así pues, el triángulo existe y sus lados miden 4, 5, 6.

Page 59: COMPENDIUM OMEFL - ToomatesXXXV Olimpiada Matemática Española Primera Fase Soluciones de la propuesta de problemas Problema 1 ¿Qué dígitos se han omitido en la siguiente multiplicación?

Problema 6.

Hallad las cuatro últimas cifras de 20043 .

Tenemos que 32 = 9 = 10 - 1. Gracias a ello, la fórmula del binomio de Newton nos permite

simplificar los cálculos:

20022004 3 2

3 2

4

1002 10023 10 1 10 10 1002·10 1

3 2

1002 1002·1001·1000·10 ·1001·10 1003·10 1

6 2

500 1 1000 1 ·100 1000 2 ·10 1 100 20 1 mod 10

Las últimas cuatro cifras son 0081.

Page 60: COMPENDIUM OMEFL - ToomatesXXXV Olimpiada Matemática Española Primera Fase Soluciones de la propuesta de problemas Problema 1 ¿Qué dígitos se han omitido en la siguiente multiplicación?

5/3/2020 Fase local 2005

www.olimpiadamatematica.es/platea.pntic.mec.es/_csanchez/loc2005.html 1/3

OLIMPIADA MATEMÁTICA ESPAÑOLA

Fase local 2005

Viernes 21 de enero de 2005 Sesión de Mañana

1. Sean a, b, c números reales no nulos y distintos. Probar que si las ecuaciones x2 + ax + bc = 0y x2 +bx+ ca = 0 tienen una raíz común, entonces las restantes raíces verifican la ecuación x2 + cx + ab = 0.

2. Sea M un punto interior del segmento AB. Se construyen cuadrados AMCD y BEHM en el mismo ladode AB. Si N es el segundo punto de intersección de las circunferencias circunscritas a dichos cuadrados,probar que:

1. los puntos B, N y C están alineados.2. el punto H es el ortocentro del triángulo ABC.

3. Sean x, y, z números reales positivos.

1. Si , ¿se verifica necesariamente que ?

2. Si , ¿se verifica necesariamente que ?

Viernes 21 de enero de 2005 Sesión de Tarde

4. Se considera un triángulo ABC con ÐBAC = 45º y ÐACB = 30º.Si M es el punto medio del lado BC, se pide demostrar que ÐAMB = 45º y que BC · AC = 2 · AM · AB.

5. Cuatro bolas negras y cinco bolas blancas se colocan, en orden arbitrario, alrededor de unacircunferencia.

Page 61: COMPENDIUM OMEFL - ToomatesXXXV Olimpiada Matemática Española Primera Fase Soluciones de la propuesta de problemas Problema 1 ¿Qué dígitos se han omitido en la siguiente multiplicación?

5/3/2020 Fase local 2005

www.olimpiadamatematica.es/platea.pntic.mec.es/_csanchez/loc2005.html 2/3

Si dos bolas consecutivas son del mismo color, se inserta una nueva bola negra entre ellas. En caso contrario,se inserta una nueva bola blanca.Se retiran las bolas negras y blancas previas a la inserción.Repitiendo el proceso, ¿es posible obtener nueve bolas blancas?

6. Se pide encontrar todos los números enteros positivos n tales que 3n + 5n es múltiplo de 3n-1 + 5n-1.

Sábado 22 de enero de 2005 Sesión de Mañana

1. Sean x1, x2 las raíces del polinomio P(x) = 3x2 + 3mx + m2 - 1, siendo m un número real. Probar queP(x1

3) = P(x23).

2. En el interior de un cuadrado ABCD se construye el triángulo equilátero ABE. Sea P el punto

intersección de las rectas AC y BE. Sea F el punto simétrico del P respecto de la recta DC. Se pide demostrarque:

a) el triángulo CEF es equilátero.b) el triángulo DEF es rectángulo e isósceles.c) el triángulo BDF es isósceles.d) el triángulo PDF es equilátero.

3. Encontrar todas las funciones tales que

x2·f(x) + f(1 - x) = 2x - x4.

Sábado 22 de enero de 2005 Sesión de tarde

4. Un grupo de chicos y chicas han comido en un restaurante en el que sólo se sirven pizzas cortadas en12 raciones. Cada chico comió 6 o 7 raciones y cada chica 2 o 3 raciones.Se sabe que 4 pizzas no fueron suficientes y que con 5 pizzas hubo de sobra.Calcular el número de chicos y de chicas del grupo.

5. Demostrar que la ecuación

x2 + y2 - z2 - x - 3y - z - 4 = 0

posee infinitas soluciones en números enteros.

6. En un tablero de ajedrez 10 ×10 se colocan 41 torres. Probar que se pueden elegir al menos 5 de ellasque no se coman entre sí.

Page 63: COMPENDIUM OMEFL - ToomatesXXXV Olimpiada Matemática Española Primera Fase Soluciones de la propuesta de problemas Problema 1 ¿Qué dígitos se han omitido en la siguiente multiplicación?

SOLUCIONES 2005

VIERNES MAÑANA – 1ª SESIÓN

Problema 1

Sean a , b , c números reales no nulos y ba . Probar que si las ecuaciones

02 bcaxx y 02 cabxx tienen una raíz común, entonces las restantes raíces

verifican la ecuación 02 abcxx .

Solución:

Sean 1x , 2x las raíces de la ecuación 02 bcaxx y 1x , 3x las de 02 cabxx .

La solución común 1x verifica la ecuación 022 cabxxbcaxx , de donde

resulta cbaxba 1 y cx 1 .

Se sigue que bx 2 y ax 3 , con lo que 2x y 3x son las raíces de la ecuación

02 abxbax . (1)

Pues cx 1 es raíz de 02 bcaxx , tenemos 02 bcacc , de donde 0 cba

y bac .

Sustituimos este valor de ba en (1) y hemos terminado.

Problema 2

Sea M un punto interior del segmento AB . Se construyen cuadrados AMCD y

BEHM en el mismo lado de AB . Si N es el segundo punto de intersección de las

circunferencias circunscritas a dichos cuadrados, probar que:

1. los puntos B , N y C están alineados.

2. el punto H es el ortocentro del triángulo ABC .

Solución:

1. Tenemos o45MNB .

figura 1figura 2

A BAB M

D

E

D

E

C

C

H

H

N

M

N

Page 64: COMPENDIUM OMEFL - ToomatesXXXV Olimpiada Matemática Española Primera Fase Soluciones de la propuesta de problemas Problema 1 ¿Qué dígitos se han omitido en la siguiente multiplicación?

Si es la figura 1, se verifica que ooo 18045135 MNBCNM ; si la figura 2,

o45 MNBMNC .

En ambas situaciones se concluye que B , N y C están alineados.

2. Por una parte, ABCH . Por otra, MEBH y ACME ; luego ACBH . Por

tanto, H es el ortocentro de ABC .

Problema 3

Sean x , y , z números reales positivos.

1. Si 3 zyx , ¿se verifica necesariamente que 3111

zyx?

2. Si 3 zyx , ¿se verifica necesariamente que 3111

zyx?

Solución:

1. La respuesta es no.

Contraejemplo con los números 1, 2 y 0.1

2. La respuesta es sí.

En efecto,

1 1 1 1 1 1

3 9x y zx y z x y z

de donde 3111

zyx.

Page 65: COMPENDIUM OMEFL - ToomatesXXXV Olimpiada Matemática Española Primera Fase Soluciones de la propuesta de problemas Problema 1 ¿Qué dígitos se han omitido en la siguiente multiplicación?

VIERNES TARDE - 2ª SESIÓN

VIERNES TARDE - 1ª SESIÓN

Problema 4

Se considera un triángulo ABC con o45BAC y o30ACB . Si M es el punto

medio del lado BC , se pide demostrar que o45AMB y que ABAMACBC 2 .

Solución:

Sea D el punto de AC tal que ACBD .

Puesto que oooo 45459090 BADDBA , el triángulo ADB es isósceles con

.DBAD

Pues CDB es rectángulo en D , MDCM y, por tanto, o30CDM . El teorema

del ángulo exterior aplicado en D al triángulo isósceles ADB da o15DAM .

El mismo teorema aplicado ahora al triángulo ACM en M da inmediatamente ooo 451530 AMB .

En consecuencia, los triángulos ABC y MBA son semejantes y, por tener la misma

altura, la razón de sus áreas es igual a la razón de sus bases:

2BM

BC

MBA

ABC

Por consiguiente, la razón de semejanza vale 2 . Tenemos, pues, que 2AM

AC y

2AB

BC.

La relación que se pide resulta al multiplicar miembro a miembro las dos igualdades

anteriores.

30

45

30

C B

A

M

D

Page 66: COMPENDIUM OMEFL - ToomatesXXXV Olimpiada Matemática Española Primera Fase Soluciones de la propuesta de problemas Problema 1 ¿Qué dígitos se han omitido en la siguiente multiplicación?

Problema 5

Cuatro bolas negras y cinco bolas blancas se colocan, en orden arbitrario, alrededor de

una circunferencia.

Si dos bolas consecutivas son del mismo color, se inserta una nueva bola negra entre

ellas. En caso contrario, se inserta una nueva bola blanca.

Se retiran las bolas negras y blancas previas a la inserción.

Repitiendo el proceso, ¿es posible obtener nueve bolas blancas?

Solución:

Si asignamos a cada bola negra el valor 1 y a cada bola blanca el valor –1, se observa

que dos bolas consecutivas se sustituyen por su producto.

Considerando el producto P de los nueve valores antes y después de cada operación,

vemos que el nuevo P es igual al cuadrado del anterior P . Así, siempre será 1P después

de cada operación.

Puesto que nueve bolas blancas darían 1P , no es posible obtener una tal

configuración.

Problema 6

Se pide encontrar todos los números enteros positivos n tales que nn 53 es múltiplo de 11 53 nn .

Solución:

Para un tal n , puesto que

1111 53553533 nnnnnn

se verifica

11 53453 nnnn

la cual se reduce a 11 35 nn

que implica 1n .

Pues 1n es solución (porque 8 es múltiplo de 2), se concluye que 1n es la única

solución.

Page 67: COMPENDIUM OMEFL - ToomatesXXXV Olimpiada Matemática Española Primera Fase Soluciones de la propuesta de problemas Problema 1 ¿Qué dígitos se han omitido en la siguiente multiplicación?

SÁBADO MAÑANA – 2ª SESIÓN

SÁBADO MAÑANA - 1ª SESIÓN

Problema 1

Sean 1x , 2x las raíces del polinomio 133 22 mmxxxP , siendo m un número

real. Probar que 3

2

3

1 xPxP .

Solución:

Tenemos mxx 21 , 3

12

21

mxx y

mxxxx

xxmxxxx

xxmxx

mmxxmmxxxPxP

3

2

3

1

3

2

3

1

3

2

3

1

3

2

3

1

3

2

3

1

3

2

3

1

6

2

6

1

23

2

6

2

23

1

6

1

3

2

3

1

3

33

33

133133

Pues mmm

mxxxxxxxx

3

133

23

2121

3

21

3

2

3

1 , resulta

03

2

3

1 mxx que implica 03

2

3

1 xPxP por lo visto anteriormente.

Problema 2

En el interior de un cuadrado ABCD se construye el triángulo equilátero ABE . Sea

P el punto intersección de las rectas AC y BE . Sea F el punto simétrico del P respecto de

la recta DC . Se pide demostrar que:

a) el triángulo CEF es equilátero.

b) el triángulo DEF es rectángulo e isósceles.

c) el triángulo BDF es isósceles.

d) el triángulo PDF es equilátero.

Solución:

75

30

75

4530

A B

D C

E

P

F

Page 68: COMPENDIUM OMEFL - ToomatesXXXV Olimpiada Matemática Española Primera Fase Soluciones de la propuesta de problemas Problema 1 ¿Qué dígitos se han omitido en la siguiente multiplicación?

a) Puesto que ooo 306090 EBC , los ángulos en la base del triángulo isósceles

BEC ( BCBE por construcción) son iguales a o75 . Resulta que ooo 304575 PCBECBECP , de donde se sigue que los triángulos

CEP y BEC son semejantes, y siendo, como se ha dicho, isósceles este último, lo

es también CEP con CPCE .

Por ser CD mediatriz del segmento PF , es CFCP .

Luego CFCE .

Ahora,

ooooo 6045759090 PCDBCEDCFECDECF .

Luego CEF es equilátero.

b) Tenemos EDCE porque AEDBED . Se sigue que los triángulos BCF y

BED son iguales porque tienen iguales dos lados y el ángulo comprendido

o135 . Por tanto, BDBF .

Luego BDF es isósceles.

c) Tenemos EFECDE y ooooo 27060756075 CEFBECAEBDEA .

Por tanto, EFDE y ooo 90270360 DEF .

Luego DEF es rectángulo e isósceles.

d) Tenemos, en fin, FDPF por ser iguales los triángulos rectángulos DEF y

PCF . Siendo, además,

oooo 60456045 PFCEFCDFEEFPDFEDFP

resulta que PDF es, en efecto, equilátero.

Problema 3

Encontrar todas las funciones :f tales que

42 21 xxxfxfx .

Solución:

Sustituimos x por x1 y obtenemos

4211211 xxxfxfx

Al eliminar xf 1 entre esta ecuación y la dada, resulta 21 xxf .

Se comprueba que 21 xxf satisface la ecuación propuesta.

La solución buscada es, por tanto, 21 xxf .

Page 69: COMPENDIUM OMEFL - ToomatesXXXV Olimpiada Matemática Española Primera Fase Soluciones de la propuesta de problemas Problema 1 ¿Qué dígitos se han omitido en la siguiente multiplicación?

SÁBADO TARDE – 2ª SESIÓN

Problema 4

Un grupo de chicos y chicas han comido en un restaurante en el que sólo se sirven

pizzas cortadas en 12 raciones. Cada chico comió 6 o 7 raciones y cada chica 2 o 3 raciones.

Se sabe que 4 pizzas no fueron suficientes y que con 5 pizzas hubo de sobra.

Calcular el número de chicos y de chicas del grupo.

Solución:

Sean x e y el número de chicos y de chicas, respectivamente.

Tenemos

5937 yx (1)

y

4926 yx (2)

Restando miembro a miembro obtenemos 10 yx

y por (2),

491026 xx

de donde 8x .

Pero (1) implica que 8x .

Luego 8x .

Sustituyendo este valor en las desigualdades anteriores, debe ser 1y .

Problema 5

Demostrar que la ecuación

043222 zyxzyx

posee infinitas soluciones en números enteros.

Solución:

yyzxzx

yzx

zyxzyxzyx

411

2

3

4

25

2

1

2

1

04

25

2

1

2

3

2

1043

222

222

222

La última ecuación se satisface si

11 yzx y yzx 4

esto es, si

3x , yz 1 .

Page 70: COMPENDIUM OMEFL - ToomatesXXXV Olimpiada Matemática Española Primera Fase Soluciones de la propuesta de problemas Problema 1 ¿Qué dígitos se han omitido en la siguiente multiplicación?

Luego las ternas aazyx 1,,3,, con Za son soluciones de la ecuación.

> Son también soluciones las ternas aazyx 2,,3,,

Problema 6

En un tablero de ajedrez 1010 se colocan 41 torres. Probar que se pueden elegir al

menos 5 de ellas que no se coman entre sí.

Solución:

Pues 110441 , existe una fila A que contiene al menos 5 torres.

Quedan 9 filas con, al menos, 31 torres. Una fila B , de entre estas 9, contiene, al

menos, 4 torres.

Quedan 8 filas con, al menos, 21 torres. Una fila C , de entre estas 8, contiene, al

menos, 3 torres.

Quedan 7 filas con, al menos, 11 torres. Una fila D , de entre estas 7, contiene, al

menos, 2 torres.

Quedan ahora 6 filas una de las cuales, sea E , contiene al menos 1 torre.

Elegimos una torre 1T de la fila E .

Elegimos una torre 2T de la fila D que no esté en la misma columna que 1T .

Elegimos una torre 3T de la fila C que no esté en la columna de 1T ni en la de 2T .

Elegimos una torre 4T de la fila B que no esté en la columna de 1T ni en la de 2T ni

en la de 3T .

Elegimos, en fin, una torre 5T de la fila A que no esté en la columna de 1T ni en la de

2T ni en la de 3T ni en la de 4T .

Las torres 1T , 2T , 3T , 4T y 5T no se comen entre sí.

Page 71: COMPENDIUM OMEFL - ToomatesXXXV Olimpiada Matemática Española Primera Fase Soluciones de la propuesta de problemas Problema 1 ¿Qué dígitos se han omitido en la siguiente multiplicación?

5/3/2020 Fase local 2005

www.olimpiadamatematica.es/platea.pntic.mec.es/_csanchez/loc2006.html 1/5

OLIMPIADA MATEMÁTICA ESPAÑOLA

Fase local 2006

Viernes 20 de enero de 2006 Primera Sesión (mañana)

1. Se da un triángulo rectángulo isósceles ABC, con el ángulo recto en C, y los catetos de longitud 2. Unarco de círculo l con centro A divide al triángulo en dos partes de la misma área, mientras que el arco decírculo m con centro en B es tangente al arco l en un punto de la hipotenusa AB.Hallar el área de la porción del triángulo no cubierta por los sectores circulares correspondientes a los dosarcos.

2. Se suponen conocidas las raíces reales de las n ecuaciones de segundo grado que se indican en elsiguiente cuadro:

Ecuación Raíces

Encontrar, razonadamente, las raíces de la ecuación

.

3. En el triángulo ABC se traza la bisectriz interior CD. Se sabe que el centro del círculo inscrito en eltriángulo BCD coincide con el centro del círculo circunscrito del triángulo ABC. Calcular los ángulos deltriángulo ABC.

Fase local 2006

Page 72: COMPENDIUM OMEFL - ToomatesXXXV Olimpiada Matemática Española Primera Fase Soluciones de la propuesta de problemas Problema 1 ¿Qué dígitos se han omitido en la siguiente multiplicación?

5/3/2020 Fase local 2005

www.olimpiadamatematica.es/platea.pntic.mec.es/_csanchez/loc2006.html 2/5

Viernes 20 de enero de 2006 Segunda Sesión (tarde)

4. Encontrar, razonadamente, dos números enteros positivos a y b, tales que

sea múltiplo de , sea múltiplo de , sea múltiplo de , sea múltiplo de ,

pero no sea múltiplo de .

5. Un número positivo x verifica la relación

.Demostrar que

es entero y calcular su valor.

6. Se considera la inecuación

,donde a es un parámetro real.a) Discutir la inecuación según los valores de a.b) Caracterizar los valores de a para los cuales la inecuación tiene exactamente DOS soluciones enteras.

Fase local 2006

Viernes 20 de enero de 2006 Primera Sesión (tarde)

1. En el sótano del castillo, 7 gnomos guardan su tesoro. El tesoro está detrás de 12 puertas, cada una deellas con 12 cerraduras. Todas las cerraduras son distintas. Cada gnomo tiene llaves para algunas de lascerraduras. Tres gnomos cualesquiera tienen conjuntamente llaves para todas las cerraduras. Probar que entretodos los gnomos tienen por lo menos 336 llaves.

2. Determinar todos los enteros n tales que

Page 73: COMPENDIUM OMEFL - ToomatesXXXV Olimpiada Matemática Española Primera Fase Soluciones de la propuesta de problemas Problema 1 ¿Qué dígitos se han omitido en la siguiente multiplicación?

5/3/2020 Fase local 2005

www.olimpiadamatematica.es/platea.pntic.mec.es/_csanchez/loc2006.html 3/5

es entero.

3. Dos esferas de radio r son tangentes exteriores. Tres esferas de radio R son tangentes exteriores entre

sí, cada una tangente a las otras dos. Cada una de estas esferas es, además, tangente exterior a las dosprimeras.Encontrar la relación existente entre R y r.

Fase local 2006

Sábado 20 de enero de 2006 Segunda Sesión (mañana)

4. Calcular los números p y q tales que las raíces de la ecuación

sean D y 1 – D, siendo D el discriminante de esa ecuación de segundo grado.

5. Los números naturales 22, 23, y 24 tienen la siguiente propiedad: los exponentes de los factores

primos de su descomposición son todos impares:

.¿Cuál es el mayor número de naturales consecutivos que pueden tener esa propiedad?. Razónese lacontestación.

6. Los vértices del cuadrilátero convexo ABCD están situados en una circunferencia. Sus diagonalesAC y BD se cortan en el punto E. Sea el centro del círculo inscrito en el triángulo ABC, y el centro delcírculo inscrito en el triángulo ABD. La recta corta a EB en M y a EA en N.Demostrar que el triángulo EMN es isósceles.

Fase local 2006

Sábado 20 de enero de 2006 Primera Sesión (mañana)

1. Los números reales no nulos a y b verifican la igualdad

.Encontrar, razonadamente, todos los valores tomados por la expresión

Page 74: COMPENDIUM OMEFL - ToomatesXXXV Olimpiada Matemática Española Primera Fase Soluciones de la propuesta de problemas Problema 1 ¿Qué dígitos se han omitido en la siguiente multiplicación?

5/3/2020 Fase local 2005

www.olimpiadamatematica.es/platea.pntic.mec.es/_csanchez/loc2006.html 4/5

.

2. ¿Existe un conjunto infinito de números naturales que NO se pueden representar en la forma

,siendo n natural y p primo? Razónese la contestación.

3. En el triángulo ABC, se trazan la bisectriz interior AL (L pertenece al lado BC), la altura BH (H

pertenece al lado AC) y la mediana CM (M pertenece al lado AB).Se sabe que los ángulos y son iguales.Determinar, razonadamente, las medidas de los ángulos del triángulo ABC.

Fase local 2006

Sábado 20 de enero de 2006 Segunda Sesión (tarde)

4. Determinar todas las ternas de números reales , con , tales que las parábolas

tienen el mismo vértice.

5. Encontrar todas la soluciones reales del sistema de ecuaciones

6. Decimos que tres números naturales distintos forman una terna aditiva si la suma de los dos primerosde ellos es igual al tercero. Hallar, razonadamente, el máximo número de ternas aditivas que puede haber enun conjunto dado de 20 números naturales.

Page 76: COMPENDIUM OMEFL - ToomatesXXXV Olimpiada Matemática Española Primera Fase Soluciones de la propuesta de problemas Problema 1 ¿Qué dígitos se han omitido en la siguiente multiplicación?

1

SOLUCIONES 2006

TANDA I (En sesiones de viernes, mañana y tarde)

Problema I - 1

Se da un triángulo rectángulo isósceles ABC, con el ángulo recto en C, y los catetos de longitud 2. Un

arco de círculo l con centro A divide al triángulo en dos partes de la misma área, mientras que el arco

de círculo m con centro en B es tangente al arco l en un punto de la hipotenusa AB.

Hallar el área de la porción del triángulo no cubierta por los sectores circulares correspondientes a los

dos arcos.

Solución.

Se r el radio del arco l. El área del sector determinado así en el triángulo es 1/8 del área del círculo. Por

lo tanto,

21 21 2

8r r

.

El radio del circulo m es

1

2 12 2 2 2 2 1r AB r

.

El área de la región buscada es entonces

2

1 11 ·8 1 2

8S

.

Problema I - 2

Se suponen conocidas las raíces reales de las n ecuaciones de segundo grado que se indican en el

siguiente cuadro:

Ecuación Raíces 2

1 1 0x a x b 0 1,x x

2

2 2 0x a x b 0 2,x x

2 0n nx a x b 0 , nx x

Encontrar, razonadamente, las raíces de la ecuación

2 1 2 1 2 0n na a a b b bx x

n n

.

Solución.

Por hipótesis, se tiene

2

0 1 0 1

2

0 0

0

0n n

x a x b

x a x b

Sumando estas igualdades, resulta

Page 77: COMPENDIUM OMEFL - ToomatesXXXV Olimpiada Matemática Española Primera Fase Soluciones de la propuesta de problemas Problema 1 ¿Qué dígitos se han omitido en la siguiente multiplicación?

2

2

0 1 0 1 0n nnx a a x b b

es decir, 0x es una raíz de la ecuación propuesta.

Por lo tanto, el discriminante de esta ecuación es no negativo y debe tener otra raíz a la que llamaremos

x . Por las fórmulas que relacionan los coeficientes y las raíces, se tiene

0 1 1

0

10

n n

n

x x a

x x a

a ax x

n

Multiplicando la última igualdad por n y restando de las anteriores, resulta

11 0 n

n

x xnx x x x

n

Problema I - 3

En el triángulo ABC se traza la bisectriz interior CD. Se sabe que el centro del círculo inscrito en el

triángulo BCD coincide con el centro del círculo circunscrito del triángulo ABC. Calcular los ángulos

del triángulo ABC.

Solución

Sea O el centro común de los círculos mencionados en el enunciado. Por la hipótesis, BO y CO son

bisectrices de los ángulos ABC y BCD (O es el centro del círculo inscrito en BCD).

Además AO = BO = CO (O es el centro del círculo circunscrito a ABC).

De aquí, si llamamos ABO , entonces tenemos

2,, BCDOBCOCBOAB (CD es bisectriz)

De aquí, 3 OCAOAC

Como la suma de los ángulos del triángulo ABC es 180º, resulta 18º , y 00 36,72 BCA .

Problema I - 4

Encontrar, razonadamente, dos números enteros positivos a y b, tales que

2b sea múltiplo de a , 3a sea múltiplo de 2b , 4b sea múltiplo de 3a , 5a sea múltiplo de 4b ,

pero 6b no sea múltiplo de 5a .

Solución

Escribamos

1 1

1 1,r r

r ra p p b p p

,

donde , 0,i i ip es primo para cada i, y i jp p si i j .

Las condiciones del problema son entonces equivalentes a

i) para cada i 2 3 4 5i i i i i y

Page 78: COMPENDIUM OMEFL - ToomatesXXXV Olimpiada Matemática Española Primera Fase Soluciones de la propuesta de problemas Problema 1 ¿Qué dígitos se han omitido en la siguiente multiplicación?

3

ii) existe i tal que 6

5i i .

Es claro entonces que basta considerar un solo primo, así que encontraremos 1 y 1 que satisfagan i)

y ii).

Esto puede hacerse fácilmente por tanteo, por ejemplo 1 4 y 1 3 sirven (también sirven 1 13

y 1 10 ).

Ahora tomamos para 1p cualquier primo, por ejemplo 2.

Una pareja que satisface las condiciones pedidas es 4 32 , 2a b .

Problema I - 5

Un número positivo x verifica la relación

2

2

17x

x .

Demostrar que

5

5

1x

x

es entero y calcular su valor.

Solución.

Se tiene

2

2

2

1 1 12 7 2 9 3x x x

x x x

.

Entonces

3

3 3 3

3 3 3

1 1 1 1 13·9 27 3 3·3 18x x x x x

x x x x x

.

Entonces

2 3 5 5 5

2 3 5 5 5

1 1 1 1 1 17·18 · 3 123x x x x x x

x x x x x x

.

Problema I - 6

Se considera la inecuación

1x ax ,

donde a es un parámetro real.

a) Discutir la inecuación según los valores de a.

b) Caracterizar los valores de a para los cuales la inecuación tiene exactamente DOS soluciones

enteras.

Solución.

a) En principio distinguiremos dos casos, según que 1x ó 1x .

Caso I: 1x . La desigualdad es equivalente a la siguiente:

1 1 1x ax a x .

Page 79: COMPENDIUM OMEFL - ToomatesXXXV Olimpiada Matemática Española Primera Fase Soluciones de la propuesta de problemas Problema 1 ¿Qué dígitos se han omitido en la siguiente multiplicación?

4

Subcaso I.1: Supongamos 1 0a , es decir, 1a . Entonces 1 1

, 1 0.1 1

x aa a

Por lo tanto,

11,1 .

1o a x

a

Subcaso I.2: 1 0 1a a y la desigualdad se escribe como 0·x < 1. Por lo tanto 1, 1.a x

Subcaso I.3: 1 0 1 1.a a x La desigualdad es equivalente a 1 1 1x ax a x .

Caso II: 1.x La desigualdad es equivalente a 1 1 1x ax a x .

Subcaso II.1: 1 1

1 0 1, 1, 1 0.1 1

a a x aa a

Por lo tanto, 1

0, 1,1

a xa

cuando 1 0a no hay solución.

Subcaso II.2: 1 0 1 1 0,a a no hay solución.

Subcaso II.3: 1

1 0 1 0.1

a a xa

Por lo tanto, 1

1, .1

a xa

Resumiendo:

Si 1,a entonces 1

.1

xa

Si 1 0,a no hay solución.

Si 0 1,a entonces 1 1

.1 1

xa a

Si 1 ,a entonces 1 .x

b) Por el análisis efectuado en a), la desigualdad puede tener dos soluciones enteras sólo si 0 1a .

Ya que en estos casos se tiene

1 10 1 ,

1 1a a

habrá dos soluciones enteras si y solamente si

12 3.

1 a

Por lo tanto la respuesta es

1 2.

2 3a

Page 80: COMPENDIUM OMEFL - ToomatesXXXV Olimpiada Matemática Española Primera Fase Soluciones de la propuesta de problemas Problema 1 ¿Qué dígitos se han omitido en la siguiente multiplicación?

5

TANDA II (En sesiones de tarde de viernes y mañana de sábado)

Problema II - 1

En el sótano del castillo, 7 gnomos guardan su tesoro. El tesoro está detrás de 12 puertas, cada una de

ellas con 12 cerraduras. Todas las cerraduras son distintas. Cada gnomo tiene llaves para algunas de

las cerraduras. Tres gnomos cualesquiera tienen conjuntamente llaves para todas las cerraduras. Probar

que entre todos los gnomos tienen por lo menos 336 llaves.

Solución.

Debe haber 4 gnomos de modo que cada uno de ellos tenga al menos 48 llaves (de lo contrario se

podrían elegir 3 de ellos de manera que tuvieran conjuntamente menos d 3·48 = 144 llaves y por lo

tanto no podrían abrir las cerraduras, contra lo supuesto). Los otro 3 gnomos tienen conjuntamente al

menos 144 llaves. Por lo tanto hay 4 gnomos con, al menos 48 llaves cada uno y otros 3 gnomos con al

menos 144 llaves, luego los 7 gnomos tienen al menos 4·48 +144 = 336 llaves.

Problema II - 2

Determinar todos los enteros n tales que

25 625 25 625

2 4 2 4n n

es entero.

Solución.

Llamemos

25 625 25 62525 2 .

2 4 2 4p n n n

(la última igualdad se comprueba elevando al cuadrado la expresión dada a la que se llama p).

Entonces

22 25

,2

pn

y p es un número impar mayor o igual que 5.

Si 9p , entonces 625

4n y la expresión de p no sería real.

Los posibles valores de p son 5 y 7, que dan respectivamente las soluciones

n = 0 y 144.

Problema II - 3

Dos esferas de radio r son tangentes exteriores. Tres esferas de radio R son tangentes exteriores entre

sí, cada una tangente a las otras dos. Cada una de estas esferas es, además, tangente exterior a las dos

primeras.

Encontrar la relación existente entre R y r.

Solución.

Los centros 1 2 3, ,O O O de las tres esferas de radio R son los vértices de un triángulo equilátero.

Page 81: COMPENDIUM OMEFL - ToomatesXXXV Olimpiada Matemática Española Primera Fase Soluciones de la propuesta de problemas Problema 1 ¿Qué dígitos se han omitido en la siguiente multiplicación?

6

Los centros de las dos esferas de radio r determinan una recta perpendicular al plano determinado por

los centros de las esferas de radio R; el punto de tangencia T de las esferas de radio r debe ser el centro

del triángulo equilátero anterior. Los cinco centros son los vértices de un doble tetraedro. Sea 1C uno

de los centros de las esferas de radio r y consideremos el tetraedro 1 2 3 1O O O C .

T es el pie de la altura desde 1C y se verifican las relaciones

1 2 1 1 12 , ,OO R OC r R C T r

El triángulo 1 1C OT es rectángulo en T, y se tiene

1

23.

3OT R

Una aplicación del teorema de Pitágoras en este último triangulo nos permite escribir

2

2 24

3

Rr R r ,

que se simplifica para dar

212 6 .

3Rr R R r

Problema II - 4

Calcular los números p y q tales que las raíces de la ecuación

2 0x px q

sean D y 1 – D, siendo D el discriminante de esa ecuación de segundo grado.

Solución

Por las fórmulas que relacionan las raíces y los coeficientes de la ecuación, se tiene

1 ,

· 1 .

D D p

D D q

De la primera se obtiene inmediatamente 1p ; de la segunda, 2q D D .

Pero 2 24 1 4 1 4 ,D p q q D D es decir

24 5 1 0,D D

deberá ser 1D ó 14

D .

Si 1D , entonces 21 1 0q .

Si 14

D , entonces

21 1 3

.4 4 16

q

Los valores obtenidos son

, 1,0

3, 1,

16

p q

p q

Page 82: COMPENDIUM OMEFL - ToomatesXXXV Olimpiada Matemática Española Primera Fase Soluciones de la propuesta de problemas Problema 1 ¿Qué dígitos se han omitido en la siguiente multiplicación?

7

Finalmente se comprueba por sustitución directa que en esos dos casos la ecuación tiene como raíces D

y –D.

Problema II - 5

Los números naturales 22, 23, y 24 tienen la siguiente propiedad: los exponentes de los factores primos

de su descomposición son todos impares:

1 1 1 3 122 2 ·11 ; 23 23 ; 24 2 ·3 .

¿Cuál es el mayor número de naturales consecutivos que pueden tener esa propiedad?. Razónese la

contestación.

Solución

Vamos a demostrar que es imposible encontrar 8 números consecutivos con esta propiedad.

Supongamos, para razonar por reducción al absurdo, que tale 8 números consecutivos existen. Uno de

ellos al que llamaremos n, es divisible por 8.

Entre los 8 números deberá estar, o bien n + 4, o bien n – 4. Tanto uno como otro son divisibles por 4,

pero no por 8, y esto es una contradicción porque el exponente de 2 en es número es necesariamente

PAR.

El ejemplo de los números

29, 30, 31, 32, 33, 34, 35

demuestra finalmente que el máximo buscado es 7, y que efectivamente se alcanza.

Problema II - 6

Los vértices del cuadrilátero convexo ABCD están situados en una circunferencia. Sus diagonales AC y

BD se cortan en el punto E. Sea 1O el centro del círculo inscrito en el triángulo ABC, y 2O el centro del

círculo inscrito en el triángulo ABD. La recta 1 2O O corta a EB en M y a EA en N.

Demostrar que el triángulo EMN es isósceles.

Solución

El centro del círculo inscrito es el punto de intersección de las bisectrices, así que

ABOABOBDOBAOACOABOBCOBAO 22221111 , , ,

Entonces se tiene

)( )( 21 ABOABO

de donde ABOABO 21 porque CADCBD 2222 (ángulos

inscritos que abarcan el arco CD).

Resulta entonces que los puntos 1 2, , ,B O O A están en una circunferencia. Pero entonces

ABOBOOBMO 2121

es decir,

211 BMOMBOAMN

Análogamente,

BAOANONAOEMN 211

de donde resulta EMN ENM porque 2 .

Por tanto, EMN es isósceles.

Page 83: COMPENDIUM OMEFL - ToomatesXXXV Olimpiada Matemática Española Primera Fase Soluciones de la propuesta de problemas Problema 1 ¿Qué dígitos se han omitido en la siguiente multiplicación?

8

TANDA III (Mañana y tarde del sábado)

Problema III - 1

Los números reales no nulos a y b verifican la igualdad

2 2

4 41

2

a b

a b

.

Encontrar, razonadamente, todos los valores tomados por la expresión

2 2

2 2

a b

a b

.

Solución

La primera igualdad del enunciado la escribimos como

2 2 4 4 4 2 2 4 2 2 2 22 2 0 2 0.a b a b a b a b a b a b

De ahí que, o bien 2 2a b (lo que es claramente imposible), o bien 2 22a b .

En este último caso, sustituyendo en la segunda expresión, obtenemos

2 2

2 2

2 1

2 3

b b

b b

.

Problema III - 2

¿Existe un conjunto infinito de números naturales que NO se pueden representar en la forma

2n p ,

siendo n natural y p primo? Razónese la contestación.

Solución

La respuesta es afirmativa. Vamos a demostrar que hay un número no acotado de cuadrados que no se

puede expresar de esa manera.

Sea 2 2m n p . Entonces 2 2 ,m n p m n m n p y al ser p primo, necesariamente

1, .m n m n p

De aquí que

1.

2

pm

Luego 2 2m n p es posible sólo para números de la forma 1

2

p , con p primo.

Pero existe un conjunto no acotado de números impares que no son primos, como por ejemplo 3r con

2r .

Problema III - 3

En el triángulo ABC, se trazan la bisectriz interior AL (L pertenece al lado BC), la altura BH (H

pertenece al lado AC) y la mediana CM (M pertenece al lado AB).

Se sabe que los ángulos ABHCAL , y BCM son iguales.

Page 84: COMPENDIUM OMEFL - ToomatesXXXV Olimpiada Matemática Española Primera Fase Soluciones de la propuesta de problemas Problema 1 ¿Qué dígitos se han omitido en la siguiente multiplicación?

9

Determinar, razonadamente, las medidas de los ángulos del triángulo ABC.

Solución

Llamemos BCMABHCAL Entonces 2BAC .

Si el ángulo BAC fuera obtuso, el punto H estaría en la prolongación del lado AC mas allá de A, y por

lo tanto 2 90º , de donde 90º . Esto es imposible, porque todos los ángulos mitad de un

triángulo son agudos (suman 90º entre los tres). Además 090BAC , pues 0 . Por lo tanto A es

agudo, y entonces, considerando el triángulo ABH se obtiene

2 90º 30º .

Es fácil ver que el recíproco también es cierto, es decir que si en un triángulo el ángulo A es de 60º,

entonces ABMCAL .

Por consiguiente, los triángulos buscados son aquellos que tienen el ángulo 060BAC , y cuya

mediana CM forma un ángulo de 30º con el lado BC.

El conjunto de los puntos desde los que el segmento BM se ve bajo ángulo de 30º, situados a un lado de

la recta AB, es un arco de circunferencia que puede cortar a AC a lo sumo en dos puntos.

Esta condición la verifican los puntos C con ABC equilátero, y H, con HM la paralela media de ABC

equilátero ( 030 HBCBHM ).

Por lo tanto las únicas soluciones del problema son

60ºA B C o bien 60º , 30º , 90º.A B C

Problema III - 4

Determinar todas las ternas de números reales , ,a b c , con , 0, 0a b a b , tales que las parábolas

2 2,y ax bx c y bx cx a

tienen el mismo vértice.

Solución.

Se observa en primer lugar que las parábolas pasan por el punto común 1,N a b c .

Primera solución

Sea 0 0,V x y el vértice de las dos parábolas. El cambio de variable

0 0,X x x Y y y

transforma V en el punto O’ que es el origen de coordenadas, es decir, las ecuaciones de las parábolas

en el nuevo sistema son

2 2, ,Y aX Y bX

que pasan por el punto común

0 0' 1 , .N x a b c y

Dado que a b , estas dos parábolas sólo tienen el único punto común O’, luego N’ = O’ y N es el

vértice de las dos parábolas.

Entonces

1 2 , 4 .2 2

b cb a c a

a b

Segunda solución.

Page 85: COMPENDIUM OMEFL - ToomatesXXXV Olimpiada Matemática Española Primera Fase Soluciones de la propuesta de problemas Problema 1 ¿Qué dígitos se han omitido en la siguiente multiplicación?

10

Anulando la primera derivada para hallar la abscisa del vértice e igualando, se obtiene directamente

;2 2

b c

a a

igualando ambos valores a 1, como abscisa del punto común, resulta lo anterior.

En conclusión, los valores buscados son

, , , 2 4a b c a a a con 0a .

Problema III - 5

Encontrar todas la soluciones ,x y reales del sistema de ecuaciones

2 2

2 2

7

2

x xy y

x y xy

Solución.

Como la segunda ecuación se puede escribir en la forma

2xy x y ,

vamos a escribir la primera de manera relativamente parecida:

2

3 7.x y xy

Haciendo el cambio de variables ,x y s xy p obtenemos el sistema equivalente

2 3 7

12

s p

sp

La segunda ecuación implica que 0s , y 2

ps

. Sustituyendo en la primera ecuación se obtiene la

ecuación cúbica 3 7 6 0,s s

que tiene las raíces enteras 1 2 31, 2, 3s s s .

A estos valores des les corresponden los valores de p

1 2 3

22, 1, ,

3p p p

respectivamente.

Los números x e y son las raíces de la ecuación cuadrática 2 0,t st p que en cada uno de los casos

anteriores da las tres ecuaciones de segundo grado

2 2 2 32 0, 2 1 0, 9 0.

5t t t t t t

Resolviendo obtenemos las soluciones del sistema dado:

9 57 9 57, 1,2 1 2,1 2 ,

6 6x y

Page 86: COMPENDIUM OMEFL - ToomatesXXXV Olimpiada Matemática Española Primera Fase Soluciones de la propuesta de problemas Problema 1 ¿Qué dígitos se han omitido en la siguiente multiplicación?

11

Problema III - 6

Decimos que tres números naturales distintos forman una terna aditiva si la suma de los dos primeros

de ellos es igual al tercero. Hallar, razonadamente, el máximo número de ternas aditivas que puede

haber en un conjunto dado de 20 números naturales.

Solución.

Sean veinte números naturales mutuamente distintos, arbitrariamente elegidos:

1 2 20x x x .

Vamos a obtener una cota superior para el número de ternas aditivas que se pueden encontrar entre

ellos, es decir, ternas , ,i j kx x x que verifiquen 1 20i j k , y i j kx x x . Primero haremos

estos para un valor fijo del índice 3,4, ,20k : sean estas ternas

1 1 2 2, , , , , , , , .

p pi j k i j k i j kx x x x x x x x x

Entonces los números

1 1 2 2, , , , , ,

p pi j i j i jx x x x x x

son mutuamente distintos y están todos en el conjunto

1 2 1, , , .kx x x

Por lo tanto, su número 2p debe estar sometido a la limitación

2 1p k ,

es decir p debe ser menor o igual que la parte entera de 1

12

k :

1

2

kp

.

En consecuencia, el número total de ternas aditivas no puede exceder de la suma

20

3

11 1 2 2 3 3 9 9 90

2k

k

.

El ejemplo del conjunto

1,2,3, ,20M

demuestra que la cota superior 90 puede ser efectivamente alcanzada, ya que para cualquier

3,4, ,20k ,

podemos elegir para i cualquier número del conjunto

11,2, , ;

2

k

el correspondiente j k i satisface, en efecto las desigualdades i j k , así que , ,i j k es una

terna contenida en M.

Page 87: COMPENDIUM OMEFL - ToomatesXXXV Olimpiada Matemática Española Primera Fase Soluciones de la propuesta de problemas Problema 1 ¿Qué dígitos se han omitido en la siguiente multiplicación?

5/3/2020 Fase local 2007

www.olimpiadamatematica.es/platea.pntic.mec.es/_csanchez/loc2007.html 1/3

OLIMPIADA MATEMÁTICA ESPAÑOLA

Fase local 2007

Viernes 19 de enero de 2007 Primera Sesión (mañana)

1. Un poliedro convexo tiene por caras 12 cuadrados, 8 hexágonos regulares y 6 octógonos regulares.En cada vértice del poliedro concurren exactamente un cuadrado, un hexágono y un octógono. ¿Cuantossegmentos que unen pares de vértices del poliedro son interiores al mismo, es decir, no son aristas ni estáncontenidos en una cara?

2. Se Resolver, en el conjunto de los números reales, el sistema de ecuaciones

3. Sea ABC un triangulo y D, E y F puntos situados en los segmentos AC, BA y CB respectivamente, deforma que los segmentos AF, BD, CE concurren en un punto P interior al triángulo. Sabemos que BP = 6,PD = 6, PC = 9, PE = 3 y AF = 20. Hallar el área del triángulo ABC.

Fase local 2007

Viernes 20 de enero de 2007 Segunda Sesión (tarde)

4. Demostrar que es imposible obtener un cubo yuxtaponiendo tetraedros regulares, todos del mismotamaño.

Page 88: COMPENDIUM OMEFL - ToomatesXXXV Olimpiada Matemática Española Primera Fase Soluciones de la propuesta de problemas Problema 1 ¿Qué dígitos se han omitido en la siguiente multiplicación?

5/3/2020 Fase local 2007

www.olimpiadamatematica.es/platea.pntic.mec.es/_csanchez/loc2007.html 2/3

5. Demostrar que, en un triángulo, la distancia de un vértice cualquiera al ortocentro es el doble de ladistancia del circuncentro al lado opuesto a ese vértice.

6. Hallar todas las soluciones reales de la ecuación

Fase local 2007

Sábado 20 de enero de 2007 Primera Sesión (mañana)

1. Para cuatro puntos no coplanarios, un plano ecualizador es un plano tal que las distancias respectivasde cada uno de los puntos a ese plano son todas iguales. Dado un conjunto de cuatro puntos no coplanarios,¿cuántos planos ecualizadores hay?

2. Encontrar todas las soluciones enteras posibles, x e y, de la ecuación:

siendo p un cierto número primo. .

3. Sea la sucesión {2, 9, 28, 65, . . .} y Hallar el máximo valor que puedetomar .

Fase local 2007

Page 89: COMPENDIUM OMEFL - ToomatesXXXV Olimpiada Matemática Española Primera Fase Soluciones de la propuesta de problemas Problema 1 ¿Qué dígitos se han omitido en la siguiente multiplicación?

5/3/2020 Fase local 2007

www.olimpiadamatematica.es/platea.pntic.mec.es/_csanchez/loc2007.html 3/3

Sábado 20 de enero de 2007 Segunda Sesión (tarde)

4. Sean a, b, c, d números enteros positivos que satisfacen ab = cd. Demostrar que a + b + c + d no es unnúmero primo.

5. Dado un entero , definimos como el número entero que en base diez se escribe

(es decir, un 1 repetido k veces). Demostrar que divide a si y sólo si k divide a l.

6. Sea P un punto interior a un triángulo ABC. Por P se trazan paralelas KP, MP y NP a los lados AB,AC y BC que dividen el triángulo inicial en tres triángulos y tres paralelogramos. Sean S1, S2, S3 las áreas delos nuevos triángulos y S el área del triángulo ABC. Probar que

Soluciones en formato Microsoft Word 2000 comprimido .zip (95 Kb)

|Página principal||InfoAlumnos||InfoProfesores||Otros sitios de interés||Problemas|

Copyright © C. Sánchez-Rubio

Actualizado 22 Enero 2007

Page 90: COMPENDIUM OMEFL - ToomatesXXXV Olimpiada Matemática Española Primera Fase Soluciones de la propuesta de problemas Problema 1 ¿Qué dígitos se han omitido en la siguiente multiplicación?

SOLUCIONES 2007

Primera sesión

Mañana del viernes 19 de enero de 2007

Problema 1. Un poliedro convexo tiene por caras 12 cuadrados, 8 hexágonos regulares y 6 octógonos regulares.

En cada vértice del poliedro concurren exactamente un cuadrado, un hexágono y un octógono.

¿Cuantos segmentos que unen pares de vértices del poliedro son interiores al mismo, es decir, no

son aristas ni están contenidos en una cara?

Solución: Sea V el número de vértices, A el número de aristas, D el número de diagonales sobre las

caras, e I el número de diagonales interiores.

Puesto que cada vértice del poliedro está exactamente en una cara cuadrada, debe haber

V = 4 · 12 = 48 vértices.

(Obtendríamos el mismo resultada contando con hexágonos o con octógonos).

Puesto que de cada vértice salen exactamente 3 aristas, hay A = 3V/2 = 72 aristas.

Como que cada cuadrado tiene 2 diagonales, cada hexágono tiene 9 y cada octógono tiene 20,

resulta D = 12 · 2 + 8 · 9 + 6 · 20 = 216 diagonales sobre las caras.

Finalmente, el número pedido I será igual al total de pares de segmentos que se pueden formar

uniendo pares de vértices de todas las formas posibles, y restando las aristas y las diagonales sobre

las caras.

48840.

2I A D

Problema 2. Resolver, en el conjunto de los números reales, el sistema de ecuaciones

3 2

3 2

3 2

6 12 8 0

6 12 8 0

6 12 8 0

y x x

z y y

x z z

Solución. Dado que 23 6 4 0t t para todo t R , entonces cualquier solución 0 0 0, ,x y z del

sistema verifica que 3 3 3

0 0 00, 0, 0x y z . Es decir, 0 0 0, ,x y z son números positivos.

Además, sumando las tres ecuaciones resulta

3 2 3 2 3 26 12 8 6 12 8 6 12 8 0x x x y y y z z z

o equivalentemente,

3 3 3

2 2 2 0 1x y z

Ahora distinguiremos dos casos: (a) 0 2x y (b) 00 2x .

(a) Si 0 2x , de la última ecuación obtenemos

3 3

0 0 0 0 06 12 8 8 6 2 0x z z z z

lo cual solo puede ocurrir si 0 0z (imposible) o 0 2z .

Análogamente, resulta que 0 2y y de (1) se deduce que 0 0 02, 2, 2x y z es la única solución

en este caso.

Page 91: COMPENDIUM OMEFL - ToomatesXXXV Olimpiada Matemática Española Primera Fase Soluciones de la propuesta de problemas Problema 1 ¿Qué dígitos se han omitido en la siguiente multiplicación?

(b) 00 2x , entonces tenemos

3 3

0 0 0 0 06 12 8 8 6 2 0x z z z z

lo cual solo es posible, al ser 0 0z , si 0 2z . Análogamente se obtiene que 0 2y lo que

contradice la igualdad (1). En consecuencia, la única solución real del sistema es (2, 2, 2) y hemos

terminado.

Problema 3. Sea ABC un triangulo y D, E y F puntos situados en los segmentos AC, BA y CB respectivamente,

de forma que los segmentos AF, BD, CE concurren en un punto P interior al triángulo. Sabemos que

BP = 6, PD = 6, PC = 9, PE = 3 y AF = 20. Hallar el área del triángulo ABC.

Solución.

Utilizamos varias veces que las áreas de dos triángulos de la misma base son proporcionales a las

alturas y que las de dos triángulos de la misma altura son proporcionales a las bases.

Las letras x, y, z, . . . de la figura de la izquierda indican las áreas de los triángulos pequeños donde

están situadas. Indicaremos por S el área total del triangulo ABC pedida. Tenemos

1 1, .

2 4

u v PD x y EP

S BD S EC

También se cumple

1 11 ,

20 2 4

S x y u vPF PF z t

AF S S

de donde PF = 5 y AP = 15.

Comparando las áreas de los triángulos ABD y BDC y de PDA y PDC resulta

14 1

14

SCD z t u u z t

AD x y v v x y S

, de donde CD = AD y u v ; por lo que 4S u .

Trazamos ahora una paralela DG a AF (figura de la derecha). Los triángulos CAP y CDG son

semejantes con razón 1/2. Por tanto 1 15

2 2DG AP y

9

2PG GC .

Puesto que 2 2 2PD PG DG resulta que el triangulo DPG es rectángulo con ángulo recto en P

indicado en la figura. Su área es u/2 = 27/2 de donde u = 27 y S = 4u = 108.

t

z

y

x

v

u

A

E

F

G

D

C

P

Page 92: COMPENDIUM OMEFL - ToomatesXXXV Olimpiada Matemática Española Primera Fase Soluciones de la propuesta de problemas Problema 1 ¿Qué dígitos se han omitido en la siguiente multiplicación?

Segunda sesión

Tarde del viernes 19 de enero de 2007

Problema 4. Demostrar que es imposible obtener un cubo yuxtaponiendo tetraedros regulares, todos del mismo

tamaño.

Solución.

Un tetraedro regular de lado c tiene volumen 32

12c y cada una de sus caras tiene área 23

4c .

Supongamos (sin perder generalidad) que un cubo de lado 1 se puede obtener uniendo N tetraedros

regulares de lado c. Entonces se satisface

3 21

12Nc

Por otro lado, cada cara del cubo estará formada por un número entero, digamos k, de caras de

tetraedros. Como que el área de una cara del cubo es 1, tenemos

2 31

4kc

Dividiendo la primera igualdad por la segunda podemos despejar c y en particular deducimos que 2c es un número racional. Pero esto es incompatible con la segunda de las ecuaciones.

Problema 5. Demostrar que, en un triángulo, la distancia de un vértice cualquiera al ortocentro es el doble de la

distancia del circuncentro al lado opuesto a ese vértice.

Solución 1.

Sean el triángulo ABC , su ortocentro H y su circuncentro O. Sean H’ y O’ sus simétricos respecto

del lado BC.

i) Puesto que los triángulos BPA y BCR son rectángulos y comparten el ángulo CBA , son

semejantes y, por lo tanto 'BAH HCP .

Pero por ser H’ simétrico de H,

'HCP PCH y ' 'BAH BCH

Cosa que prueba que H’ está sobre la circunferencia circunscrita al triángulo ABC .

O''

O'H'

H

R

P

O

A

B

C

Page 93: COMPENDIUM OMEFL - ToomatesXXXV Olimpiada Matemática Española Primera Fase Soluciones de la propuesta de problemas Problema 1 ¿Qué dígitos se han omitido en la siguiente multiplicación?

ii) Nuevamente por ser H’ simétrico de H, ' ' 'OH H H HO y, por ser HH’ y OO’ dos paralelas

cortadas por la secante HO’, ' ' 'H HO OO H , obteniendo

' 'OH H OO H

iii) Pero OA y OH’ sor radios de la circunferencia circunscrita y, en consecuencia, el triángulo

'H OH es isósceles, por lo que 'OH H HAO y, finalmente,

'OO H HAO

y el cuadrilátero AHO’O es un paralelogramo. Resulta:

' 2 ''AH OO OO .

Solución 2. (Aportada por Arnau Messegué Buisan, clasificado en la Fase Local de Cataluña).

Sean A’, B’, C’ los puntos medios de los lados BC, CA y AB respectivamente.

La circunferencia que pasa por A’, B’, C’ (circunferencia medial) es la imagen de la circunscrita a

A, B, C en la semejanza de centro el baricentro G y razón 12 .

Obviamente el circuncentro O de ABC es el ortocentro de A’B’C’ y se sigue el resultado al

corresponderse los segmentos AH y A’O en la semejanza anterior.

Problema 6. Hallar todas las soluciones reales de la ecuación

2 2 2

3 3 3 1x x y y y z z z x

Solución. Aplicando la desigualdad entre las medias aritmética y geométrica resulta

2 2 22 2 2 2 2 2

2 2 2

12 2 2 1

3 2 2 2 3

11 1 1

03

3 3 3 3 3 3

3 3 1

x y z x y zx x y y y z z x x x y z x y z

x y z

La igualdad se verifica cuando x = y = z = 1. Por tanto, la única solución es (x, y, z) = (1, 1, 1) y

hemos terminado.

GO

H

B'

A'

C'

A

B C

Page 94: COMPENDIUM OMEFL - ToomatesXXXV Olimpiada Matemática Española Primera Fase Soluciones de la propuesta de problemas Problema 1 ¿Qué dígitos se han omitido en la siguiente multiplicación?

Primera sesión

Mañana del sábado 20 de enero de 2007

Problema 1. Para cuatro puntos no coplanarios, un plano ecualizador es un plano tal que las distancias

respectivas de cada uno de los puntos a ese plano son todas iguales. Dado un conjunto de cuatro

puntos no coplanarios, ¿cuántos planos ecualizadores hay?

Solución:

Sean A, B, C y D cuatro puntos no coplanarios. Sea p un plano ecualizador de esos puntos y

examinemos las posibilidades:

i) Si A, B, C y D están en el mismo semiespacio en que p divide al espacio, está claro que uno de los

dos planos paralelos a p, a la misma distancia que están los cuatro puntos de p, contiene a todos

esos puntos, contra la hipótesis de no coplanariedad. Por lo tanto, los puntos no pueden estar todos

en el mismo lado del plano ecualizador.

ii) Sea A en un lado del plano y B, C y D en el otro. Es obvio que p es paralelo al plano q

determinado por los puntos B, C y D y corta al segmento que proyecta A sobre el plano q en su

punto medio. Por lo tanto, en esas condiciones, el plano ecualizador existe y es único. Ahora,

tomando cada vez uno cualquiera de los otros puntos como “punto aislado” en un lado del plano,

obtenemos otros tantos planos ecualizadores. En consecuencia, con un punto en un lado y los otros

tres en el otro, hay exactamente cuatro planos ecualizadores.

iii) Sean ahora A y B en un lado del plano p y C y D en el otro. Consideremos el plano r que

contiene a A y B y es paralelo al segmento CD y el plano s que contiene a C y D y es paralelo al

segmento AB.

Entonces, los planos r y s son paralelos y el plano ecualizador p es el plano equidistante de los

planos r y s, que también está determinado de forma única. Dado el punto A, la elección de su

“pareja” en uno de los lados de p determina otros tantos planos ecualizadores, tres en total.

Así, pues, para cuatro puntos no coplanarios dados, hay exactamente siete planos ecualizadores.

Problema 2. Encontrar todas las soluciones enteras posibles, x e y, de la ecuación:

p x y xy

siendo p un cierto número primo.

Solución:

De p x y xy y del hecho que p es un número primo se deduce que p divide a x o a y. Puesto

que, en el enunciado, los papeles de x e y son completamente simétricos, se puede, sin pérdida de

generalidad, suponer que p divide a x y que, en consecuencia, hay un número k tal que

x kp

Entonces, la ecuación propuesta queda

p kp y kpy

o sea,

kp y ky

Ahora, unas cuantas manipulaciones:

0 1kp y ky ky kp y k y p y p k y p y p k y p

Page 95: COMPENDIUM OMEFL - ToomatesXXXV Olimpiada Matemática Española Primera Fase Soluciones de la propuesta de problemas Problema 1 ¿Qué dígitos se han omitido en la siguiente multiplicación?

Ponen de manifiesto que k – 1 es un divisor de p y, dado que p es primo, hay cuatro posibilidades:

i) Que 1k p . Obtenemos que 1k p y, por lo tanto,

1 , 1x p p y p

ii) Que 1 1k . Resulta que 0k , o sea,

0, 0x y

iii) Que 1 1k . Entonces 2k y resulta:

2 , 2x p y p

iiii) Que 1k p . Entonces 1k p y resulta:

1 , 1x p p y p

que son todas las posibles soluciones de la ecuación propuesta.

Problema 3.

Sea 31na n la sucesión {2, 9, 28, 65, . . .} y 1mcd ,n n na a Hallar el máximo valor que

puede tomar n .

Solución: n divide a 1na y a na , y por tanto a su diferencia 2

1 3 3 1n n nb a a n n .

También divide a 23 3 3n n nc a nb n n y a la suma 4 2n n nd b c n . Pero entonces n

también divide a 2 3 2 6n n ne b nd n . Finalmente, divide a 3 14n nd e .

Pero 23 3 1 3 1 1nb n n n n es un número impar, luego n solamente puede ser 1 o 7. El

máximo es 7 ya que mcd (53 + 1, 6

3 + 1) = 7.

Page 96: COMPENDIUM OMEFL - ToomatesXXXV Olimpiada Matemática Española Primera Fase Soluciones de la propuesta de problemas Problema 1 ¿Qué dígitos se han omitido en la siguiente multiplicación?

Segunda sesión

Tarde del sábado 20 de enero de 2007

Problema 4. Sean a, b, c, d números enteros positivos que satisfacen ab = cd. Demostrar que a + b + c + d no es

un número primo.

Solución:

Usando la hipótesis ab = cd se escribe

a a b c d a c a d

de donde se obtiene que si a + b + c + d fuese primo debería dividir a a + c o a + d que son

menores que él.

Problema 5.

Dado un entero 1k , definimos ka como el número entero que en base diez se escribe

11 1

k

ka

(es decir, un 1 repetido k veces). Demostrar que ka divide a la si y sólo si k divide a l.

Solución:

Si l dk r donde r es más pequeño que k, entonces

1210 1 10 10 10d kr k k

l k ra a a

(Pondremos 0 0a .) Si k divide a l será r = 0 y es evidente que 0ra de forma que ka divide a la .

Recíprocamente, si ka divide a la entonces debe ser 0ra y por tanto r = 0.

Problema 6. Sea P un punto interior a un triángulo ABC. Por P se trazan paralelas KP, MP y NP a los lados AB,

AC y BC que dividen el triángulo inicial en tres triángulos y tres paralelogramos. Sean S1, S2, S3 las

áreas de los nuevos triángulos y S el área del triángulo ABC. Probar que

1 2 33S S S S

Solución. Sea L el punto de intersección de KP con el lado BC y sean , 1,2,3ih i las alturas de los

nuevos triángulos y h la altura de ABC . Dado que cada uno de los triángulos son semejantes con

ABC, se tiene

2

1 1

2

2

2 2

2

2

31

2

·

·

·

·

·

·

S KP h KP

S AB h AB

S MN h MN

S AB h AB

PL hS PL

S AB h AB

De donde resulta

31 2 1SS S KP MN PL

ABS S S

Page 97: COMPENDIUM OMEFL - ToomatesXXXV Olimpiada Matemática Española Primera Fase Soluciones de la propuesta de problemas Problema 1 ¿Qué dígitos se han omitido en la siguiente multiplicación?

y

2

1 2 3S S S S

Por otro lado, teniendo en cuenta la desigualdad entre las medias aritmética y cuadrática resulta

1 2 31 2 3 3

3

S S SS S S

con lo que

2

1 2 3 1 2 33S S S S S S S

Finalmente, obsérvese que la igualdad tiene lugar cuando P coincide con el baricentro G del

triángulo.

Page 98: COMPENDIUM OMEFL - ToomatesXXXV Olimpiada Matemática Española Primera Fase Soluciones de la propuesta de problemas Problema 1 ¿Qué dígitos se han omitido en la siguiente multiplicación?

FASE LOCAL DE LA XLIV OME

PRIMERA SESIÓN

Mañana del viernes 18 de Enero de 2008 1. Sea P una familia de puntos en el plano tales que por cada cuatro puntos de P pasa una circunferencia. ¿Se puede afirmar que necesariamente todos los puntos de P están en la misma circunferencia? Justifica la respuesta. 2. En un cuadrilátero convexo se trazan las perpendiculares desde cada vértice a la diagonal que no pasa por él. Demuestra que los cuatro puntos de intersección de cada perpendicular con su correspondiente diagonal forman un cuadrilátero semejante al dado.

3. Halla las soluciones reales de la ecuación: .81

61

6=⎟

⎠⎞

⎜⎝⎛ +

+−

⎟⎠⎞

⎜⎝⎛

+− x

xx

xxx

No está permitido el uso de calculadoras. Cada problema se califica sobre 7 puntos. El tiempo de cada sesión es de tres horas y media.

Page 99: COMPENDIUM OMEFL - ToomatesXXXV Olimpiada Matemática Española Primera Fase Soluciones de la propuesta de problemas Problema 1 ¿Qué dígitos se han omitido en la siguiente multiplicación?

FASE LOCAL DE LA XLIV OME

SEGUNDA SESIÓN

Tarde del viernes 18 de enero de 2008

4. Demuestra que 5555 22222222 5555+ es múltiplo de 7. 5. Dada una circunferencia y dos puntos P y Q en su interior, inscribir un triángulo rectángulo cuyos catetos pasen por P y Q. ¿Para qué posiciones de P y Q el problema no tiene solución? 6. Sean cba ,, tres números positivos de suma uno. Demuestra que

.31222 222

≥+++ bccabbcaa cba

No está permitido el uso de calculadoras. Cada problema se califica sobre 7 puntos. El tiempo de cada sesión es de tres horas y media.

Page 100: COMPENDIUM OMEFL - ToomatesXXXV Olimpiada Matemática Española Primera Fase Soluciones de la propuesta de problemas Problema 1 ¿Qué dígitos se han omitido en la siguiente multiplicación?

FASE LOCAL DE LA XLIV OME

PRIMERA SESIÓN

Tarde del viernes 18 de enero de 2008

1. Demuestra que no existen enteros a, b, c, d tales que el polinomio

dcxbxaxxP +++= 23)( ),0( ≠a cumpla que 1)4( =P y .2)7( =P . 2. En el triángulo ABC, el área S y el ángulo C son conocidos. Halla el valor de los lados a y b para que el lado c sea lo más corto posible. 3. Determina todas las ternas de números reales ),,,( cba que satisfacen el sistema de

ecuaciones siguiente: .454545

35

35

35

⎪⎩

⎪⎨

−=

−=

−=

bacacbcba

No está permitido el uso de calculadoras. Cada problema se califica sobre 7 puntos. El tiempo de cada sesión es de tres horas y media.

Page 101: COMPENDIUM OMEFL - ToomatesXXXV Olimpiada Matemática Española Primera Fase Soluciones de la propuesta de problemas Problema 1 ¿Qué dígitos se han omitido en la siguiente multiplicación?

FASE LOCAL DE LA XLIV OME

SEGUNDA SESIÓN

Mañana del sábado19 de enero de 2008

4. ¿Qué número es mayor: 999! ó 999500 ? Justifica la respuesta. 5. Sean FED ,, los puntos de tangencia del círculo inscrito al triángulo ABC con los lados ACBC, y AB respectivamente. Demuestra que

ABCDEF SS ≤4 donde XYZS denota el área del triángulo .XYZ 6. Las longitudes de los lados y de las diagonales de un cuadrilátero convexo plano ABCD son racionales. Si las diagonales AC y BD se cortan en el punto ,O demuestra que la longitud OA es también racional.

No está permitido el uso de calculadoras. Cada problema se califica sobre 7 puntos. El tiempo de cada sesión es de tres horas y media.

Page 102: COMPENDIUM OMEFL - ToomatesXXXV Olimpiada Matemática Española Primera Fase Soluciones de la propuesta de problemas Problema 1 ¿Qué dígitos se han omitido en la siguiente multiplicación?

FASE LOCAL DE LA XLIV OME

PRIMERA SESIÓN

Mañana del sábado 19 de enero de 2008

1. Sea m un entero positivo. Demuestra que no existen números primos de la forma

.1225 ++ mm 2. Un cuadrilátero convexo tiene la propiedad que cada una de sus dos diagonales biseca su área. Demuestra que este cuadrilátero es un paralelogramo. 3. Se consideran 17 enteros positivos tales que ninguno de ellos tiene un factor primo mayor que 7. Demuestra que hay al menos una pareja de estos números cuyo producto es un cuadrado perfecto. No está permitido el uso de calculadoras. Cada problema se califica sobre 7 puntos. El tiempo de cada sesión es de tres horas y media.

Page 103: COMPENDIUM OMEFL - ToomatesXXXV Olimpiada Matemática Española Primera Fase Soluciones de la propuesta de problemas Problema 1 ¿Qué dígitos se han omitido en la siguiente multiplicación?

FASE LOCAL DE LA XLIV OME

SEGUNDA SESIÓN

Tarde del sábado 19 de enero de 2008

4. Determina el triángulo de menor perímetro entre todos los que tienen la circunferencia inscrita con el mismo radio y el mismo valor de un ángulo. 5. Un club tiene 25 miembros. Cada comité está formado por 5 miembros. Dos comités cualesquiera tienen como mucho un miembro en común. Prueba que el número de comités no puede ser superior a .30 6. Halla todas las ternas ),,( zyx de números reales que son solución de la ecuación

( ).7532)53(7)37(5)75(3 zyxyxzxzyzyx ++=+++++

No está permitido el uso de calculadoras. Cada problema se califica sobre 7 puntos. El tiempo de cada sesión es de tres horas y media.

Page 104: COMPENDIUM OMEFL - ToomatesXXXV Olimpiada Matemática Española Primera Fase Soluciones de la propuesta de problemas Problema 1 ¿Qué dígitos se han omitido en la siguiente multiplicación?

1

FASE LOCAL DE LA XLIV OME SESIONES PRIMERA Y SEGUNDA

18 DE ENERO DE 2008 (MAÑANA Y TARDE)

1. Sea P una familia de puntos en el plano tales que por cada cuatro puntos de P pasa una circunferencia. ¿Se puede afirmar que necesariamente todos los puntos de P están en la misma circunferencia? Justifica la respuesta. SOLUCIÓN: Sea { }4321 ,,, xxxxT = un subconjunto de P con cuatro elementos. Por hipótesis existe una circunferencia α que pasa por estos cuatro puntos. Supongamos que exista un punto ,Px∈ tal que .α∉x Por la condición del enunciado existe una circunferencia β que pasa por los puntos 32 ,, xxx y .4x Entonces las circunferencias α y β tienen tres puntos comunes, lo que implica que deben coincidir. Por la condición del enunciado existe una circunferencia β que pasa por los puntos

32 ,, xxx y .4x Entonces las circunferencias α y β tienen tres puntos comunes, lo que implica que deben coincidir. 2. En un cuadrilátero convexo se trazan las perpendiculares desde cada vértice a la diagonal que no pasa por él. Demostrar que los cuatro puntos de intersección de cada perpendicular con su correspondiente diagonal forman un cuadrilátero semejante al dado. SOLUCIÓN:

Vamos a probar que ambos cuadriláteros tienen todos sus ángulos iguales. Por construcción ´´DADA es circunscriptible y de ahí que ´´´ ADAACD ∠=∠ por ser ambos suplementarios del mismo ángulo ´.´AAD∠ También el cuadrilátero ´´DDCC es circunscriptible y DCCCDC ´´´ ∠=∠ por estar inscritos en el mismo arco. Sumando resulta ´.´ACDADC ∠=∠ Análogamente se prueba para los restantes vértices.

B'

D'

A'

C'

P

AB

C

D

Page 105: COMPENDIUM OMEFL - ToomatesXXXV Olimpiada Matemática Española Primera Fase Soluciones de la propuesta de problemas Problema 1 ¿Qué dígitos se han omitido en la siguiente multiplicación?

2

3. Halla las soluciones reales de la ecuación: .81

61

6=⎟

⎠⎞

⎜⎝⎛ +

+−

⎟⎠⎞

⎜⎝⎛

+− x

xx

xxx

SOLUCIÓN: Sea .1+= xt Entonces sustituyendo en la ecuación dada, se tiene

,82717)1( =⎟⎠⎞

⎜⎝⎛ −+⎟⎠⎞

⎜⎝⎛ −− t

ttt que equivale a .827177 =⎟

⎠⎞

⎜⎝⎛ −+⎟⎠⎞

⎜⎝⎛ +−− t

ttt Haciendo

el cambio ,7 tt

u += se obtiene la ecuación 8)2)(8( =−− uu equivalente a

,024102 =+− uu cuyas soluciones son 4=u y .6=u

De ,47=+= t

tu se obtiene 0742 =+− tt que no tiene soluciones reales y si

,67=+= t

tu se llega a la ecuación ,0762 =+− tt cuyas dos soluciones son reales:

231 +=t y .232 −=t Como ,1−= tx la ecuación inicial tiene las soluciones reales 221 −=x y .222 +=x 4. Demuestra que 5555 22222222 5555+ es múltiplo de 7. SOLUCIÓN: Tenemos las siguientes congruencias módulo 7

0 1 2 3 4 5 62222 1, 2222 3, 2222 2, 2222 6, 2222 4, 2222 5, 2222 1≡ ≡ ≡ ≡ ≡ ≡ ≡ K 0 1 2 35555 1, 5555 4, 5555 2, 5555 1,≡ ≡ ≡ ≡ K

Los restos potenciales de 2222 forman un ciclo de longitud 6, los de 5555 otro ciclo de longitud 3; entonces

5555 55555 925 6 5 2222 2222 5= × + ⇒ ≡ ≡ 2222 22222 740 3 2 5555 5555 2= × + ⇒ ≡ ≡

La demostración concluye sumando las dos últimas congruencias. 5. Dada una circunferencia y dos puntos P y Q en su interior, inscribir un triángulo rectángulo cuyos catetos pasen por P y Q. ¿Para qué posiciones de P y Q el problema no tiene solución? SOLUCIÓN:

Page 106: COMPENDIUM OMEFL - ToomatesXXXV Olimpiada Matemática Española Primera Fase Soluciones de la propuesta de problemas Problema 1 ¿Qué dígitos se han omitido en la siguiente multiplicación?

3

Basta trazar la circunferencia de diámetro PQ, las eventuales intersecciones con la circunferencia inicial de centro O nos proporcionan dos puntos A y B que unidos con P y Q definen las soluciones. No existe solución cuando ambas circunferencias no se cortan es decir cuando

2PQOM r+ <

siendo r el radio de la circunferencia dada. 6. Sean cba ,, tres números positivos de suma uno. Demuestra que

.31222 222

≥+++ bccabbcaa cba

SOLUCIÓN: Dado que ,1=++ cba entonces .1)(2)( 2222 =+++++=++ bcacabcbacba

Además, .3ln121212111ln 222 =⎟⎠⎞

⎜⎝⎛ +++++

aca

cbc

bab

cc

bb

aa Aplicando la

desigualdad de Jensen a la función xxf ln)( = que es cóncava en su dominio ),0( >x

se tiene, ≥⎟⎠⎞

⎜⎝⎛ +++++=

aca

cbc

bab

cc

bb

aa 121212111ln3ln 222

+⎟⎠⎞

⎜⎝⎛

aa 1ln2 +⎟

⎠⎞

⎜⎝⎛b

b 1ln2 +⎟⎠⎞

⎜⎝⎛

cc 1ln2 +⎟

⎠⎞

⎜⎝⎛

aca 1ln2 +⎟

⎠⎞

⎜⎝⎛b

ab 1ln2 =⎟⎠⎞

⎜⎝⎛

cbc 1ln2

.111ln222 222

⎥⎥⎦

⎢⎢⎣

⎡⎟⎠⎞

⎜⎝⎛

⎟⎠⎞

⎜⎝⎛

⎟⎠⎞

⎜⎝⎛

+++ bccabbcaa

cba

Teniendo en cuenta que la función xxf ln)( = es inyectiva resulta que

caa

a

22

1 +

⎟⎠⎞

⎜⎝⎛

abb

b

22

1 +

⎟⎠⎞

⎜⎝⎛

bcc

c

22

1 +

⎟⎠⎞

⎜⎝⎛ ,3≤ o equivalentemente, .

31222 222

≥+++ bccabbcaa cba

La igualdad se alcanza cuando .31

=== cba

BA

M

O

P Q

Page 107: COMPENDIUM OMEFL - ToomatesXXXV Olimpiada Matemática Española Primera Fase Soluciones de la propuesta de problemas Problema 1 ¿Qué dígitos se han omitido en la siguiente multiplicación?

1

FASE LOCAL DE LA XLIV OME SESIONES PRIMERA Y SEGUNDA

18 y 19 DE ENERO DE 2008 (TARDE Y MAÑANA)

1. Demuestra que no existen enteros a, b, c, d tales que el polinomio )0()( 23 ≠+++= adcxbxaxxP cumpla que 1)4( =P y .2)7( =P

SOLUCIÓN: Supongamos que tal polinomio existe. Por el teorema del resto ,1)()4()( +−= xQxxP , siendo )(xQ un polinomio de grado dos con coeficientes enteros.

Entonces 1)7()47(2)7( +−== QP ⇒ 31)7( =Q que no es entero, en contra de la

hipótesis. 2. En el triángulo ABC, el área S y el ángulo C son conocidos. Hallar el valor de los lados a y b para que el lado c sea lo más corto posible. SOLUCIÓN: Por una parte

( ) ( )22 2 2 2 cos 2 1 cosc a b ab C a b ab C= + − = − + − y por otra

1 22

SS absenC absenC

= ⇒ = . Entonces,

( ) ( )22 4 1 cosS Cc a b

senC−

= − + será mínimo cuando 2Sa bsenC

= = .

3. Determina todas las ternas de números reales ),,,( cba que satisfacen el sistema de

ecuaciones siguiente: .454545

35

35

35

⎪⎩

⎪⎨

−=

−=

−=

bacacbcba

SOLUCIÓN: Sin pérdida de generalidad podemos suponer que =a máximo{ }.,, cba Primer caso: .bc ≥ Entonces bcca 44 55 +≥+ y .ab ≥ De este modo .cba == Segundo caso: .cb ≥ Entonces bcab 44 55 +≥+ y .ac ≥ Por tanto .cba == Así todo se reduce a resolver la ecuación ,045 35 =+− ttt donde .cbat === Claramente { }2,2,1,1,0 −−∈t y hay cinco posibles ternas que cumplen el sistema dado.

Page 108: COMPENDIUM OMEFL - ToomatesXXXV Olimpiada Matemática Española Primera Fase Soluciones de la propuesta de problemas Problema 1 ¿Qué dígitos se han omitido en la siguiente multiplicación?

2

4. ¿Qué número es mayor 999! ó 999500 ? Justifica la respuesta. SOLUCIÓN: Pongamos 999999!, 500A B= = , tenemos

=+

⋅++

⋅⋅−−

⋅−

=500

499500500

498500...500

1500500500

5001500...

500498500

500499500

BA

=⎟⎠⎞

⎜⎝⎛ +⎟⎠⎞

⎜⎝⎛ +⎟

⎠⎞

⎜⎝⎛ +⎟

⎠⎞

⎜⎝⎛ −⎟

⎠⎞

⎜⎝⎛ −⎟⎠⎞

⎜⎝⎛ −

5004991

5004981...

500111

50011...

5004981

5004991

.150011

50021...

5004981

5004991

2222

<⎥⎥⎦

⎢⎢⎣

⎡⎟⎠⎞

⎜⎝⎛−

⎥⎥⎦

⎢⎢⎣

⎡⎟⎠⎞

⎜⎝⎛−

⎥⎥⎦

⎢⎢⎣

⎡⎟⎠⎞

⎜⎝⎛−

⎥⎥⎦

⎢⎢⎣

⎡⎟⎠⎞

⎜⎝⎛−

Por tanto A < B. 5. Sean FED ,, los puntos de tangencia del círculo inscrito al triángulo ABC con los lados ACBC, y AB respectivamente. Demuestra que

ABCDEF SS ≤4 donde XYZS denota el área del triángulo .XYZ SOLUCIÓN: Sea I el incentro del triángulo .ABC Tenemos que ACIEBCID ⊥⊥ , e .ABIF ⊥ Por otro lado, utilizando las notaciones usuales,

AsenbcAsenACABS ABC 21

21

=⋅= y .21

21 2 EIFsenrEIFsenFIEISEFI =⋅=

Como los ángulos A y EIF son suplementarios, entonces EIFsenAsen = y

.2

bcr

SS

ABC

EIF = Análogamente abr

SS

ABC

EID2

= y .2

car

SS

ABC

FID =

Sumando estas tres fracciones resulta:

.)(2

abccbar

SSSS

SS

ABC

FIDEIDEIF

ABC

DEF ++=

++=

Como ⎟⎠⎞

⎜⎝⎛ ++

=2

cbarS ABC y ,4 abcSR ABC = se obtiene .2

)(2

Rr

abccbar

=++

Aplicando ahora la desigualdad de Euler ,2rR ≥ se obtiene 41

2≤=

Rr

SS

ABC

DEF y la

igualdad se cumple cuando el triángulo ABC es equilátero.

Page 109: COMPENDIUM OMEFL - ToomatesXXXV Olimpiada Matemática Española Primera Fase Soluciones de la propuesta de problemas Problema 1 ¿Qué dígitos se han omitido en la siguiente multiplicación?

3

6. Las longitudes de los lados y de las diagonales de un cuadrilátero convexo plano ABCD son racionales. Si las diagonales AC y BD se cortan en el punto ,O demuestra que la longitud OA es también racional. SOLUCIÓN: Sean ,α=∠ABD γ=∠CBD y .β=∠CBA

Por el teorema del coseno en el triángulo ,ABC∆BCAB

CABCAB⋅−+

=2

cos222

β es un

número racional. Análogamente αcos y γcos son números racionales. Por otra parte, .coscos)(coscos γαγαγαβ sensen−=+= Y así γα sensen es un

número racional. También es racional .cos1 22 γγ −=sen Por tanto γγα

γα

2sensensen

sensen

=

es racional. Aplicando el teorema de los senos a los triángulos OAB∆ y OCB∆ respectivamente se

tiene que αsen

AOBOAsen

AB=

∠ y .

γsenOC

BOCsenBC

=∠

Se deduce que

,rsensen

BCAB

OCOA

=⋅=γα es un número racional. Entonces .)1( OArOCOAAC +=+=

Por tanto r

ACOA+

=1

es racional.

Page 110: COMPENDIUM OMEFL - ToomatesXXXV Olimpiada Matemática Española Primera Fase Soluciones de la propuesta de problemas Problema 1 ¿Qué dígitos se han omitido en la siguiente multiplicación?

1

FASE LOCAL DE LA XLIV OME SESIONES PRIMERA Y SEGUNDA

19 DE ENERO DE 2008 (MAÑANA Y TARDE)

1. Sea m un entero positivo. Demuestra que no existen números primos de la forma

.1225 ++ mm SOLUCIÓN: Sumando y restando m22 resulta,

=++ 1225 mm +−=+−++ )12(222122 32225 mmmmmm .1222 ++ mm Teniendo en cuenta que ),122()12(12 23 ++−=− mmmm resulta que,

),122()122(122 2235 +++−=++ mmmmmm que es compuesto porque cada uno de los dos factores es un entero positivo mayor que 1 2. Un cuadrilátero convexo tiene la propiedad que cada una de sus dos diagonales biseca su área. Demuestra que este cuadrilátero es un paralelogramo. SOLUCIÓN:

Sea ABCD el cuadrilátero dado. Es sabido que al trazar paralelas a cada diagonal por los extremos de la otra se forma un paralelogramo (XYZT en la figura) de área doble de la del cuadrilátero de partida. Si AC biseca a ABCD también biseca a XYZT, pero siendo XYZT un paralelogramo y AC paralela a los lados XY y TZ ; P es medio de BD. De modo análogo se prueba que P es punto medio de AC y entonces los triángulos APD y BPC son iguales (dos lados iguales y el ángulo comprendido) y también APB y CPD. En consecuencia el cuadrilátero inicial tiene iguales los lados opuestos y es un paralelogramo. 3. Se consideran 17 enteros positivos tales que ninguno de ellos tiene un factor primo mayor que 7. Demuestra que, al menos, el producto de dos de estos números es un cuadrado perfecto.

Z

Y

T

X

P

A

B

CD

Page 111: COMPENDIUM OMEFL - ToomatesXXXV Olimpiada Matemática Española Primera Fase Soluciones de la propuesta de problemas Problema 1 ¿Qué dígitos se han omitido en la siguiente multiplicación?

2

SOLUCIÓN: Todos estos 17 números se pueden descomponer en factores primos de la forma

,7532 dcba donde sus exponentes dcba ,,, son enteros no negativos. Si dos números de este tipo, es decir dcba 7532 y '''' 7532 dcbá se multiplican, su producto es ''´' 7532 ddccbbaa ++++ y si los cuatro exponentes ',',' ccbbaa +++ y 'dd + son pares este producto es un cuadrado perfecto. Para que esto ocurra las dos cuaternas ),,,( dcba y )',',','( dcba deben tener igual paridad, es decir a y ,'a b y ,'b c y ,'c d y 'd deben tener respectivamente la misma paridad. Como cada uno de los cuatro números dcba ,,, puede ser par o impar hay un total de 16 cuaternas con distinta paridad entre sí. Al tener 17 números y, por tanto, 17 cuaternas, por el principio del palomar, dos deben tener igual paridad y entonces su producto es un cuadrado perfecto. 4. Determina el triángulo de menor perímetro entre todos los que tienen la circunferencia inscrita con el mismo radio y el mismo valor de un ángulo. SOLUCIÓN: Sean ABC todos los triángulos con esta propiedad; es decir que el ángulo común con el mismo valor es A y el radio inscrito también común es .r Entonces el perímetro

.2

cot2

cot2

cot22

cot2

cot2

cot2

cot2

cot2

cot

2

⎟⎠⎞

⎜⎝⎛ ++=⎟

⎠⎞

⎜⎝⎛ ++⎟

⎠⎞

⎜⎝⎛ ++⎟

⎠⎞

⎜⎝⎛ +

=++=

CBArBArACrCBr

cbap

Como A y r son fijos el perímetro será mínimo cuando sea mínima la expresión siguiente:

cot +2B cot

2C ,

2/2/2/cos

2/2/)2/)((

CsenBsenA

CsenBsenCBsen

=+

= pues º.180=++ CBA

Y entonces 2/2/ CsenBsen = ⎟⎠⎞

⎜⎝⎛ −

−=⎟

⎠⎞

⎜⎝⎛ +

−−

2cos

2cos

21

2cos

2cos

21 ACBCBCB

debe ser máximo, lo que conduce a que 2

cos CB − sea 1 y entonces .CB =

Así los triángulos de perímetro mínimo con el ángulo A fijo y el radio inscrito fijo son

los triángulos isósceles con .2º180 ACB −

==

5. Un club tiene 25 miembros. Cada comité está formado por 5 miembros. Dos comités cualesquiera tienen como mucho un miembro en común. Prueba que el número de comités no puede ser superior a .30 SOLUCIÓN: Supongamos que haya 31 comités. Entonces hay, al menos, 155531 =× asientos en estos 31 comités. Como sólo hay 25 miembros, al menos, uno de ellos tendrá que

Page 112: COMPENDIUM OMEFL - ToomatesXXXV Olimpiada Matemática Española Primera Fase Soluciones de la propuesta de problemas Problema 1 ¿Qué dígitos se han omitido en la siguiente multiplicación?

3

ocupar, al menos, 7 de los 155 asientos. Consideremos este miembro A y 7 de los comités donde se sienta. Hay, al menos, otros 28 asientos en estos 7 comités. Pero como sólo hay 24 miembros más, al menos, uno de ellos tendrá que ocupar, al menos, 2 de esos asientos. Sin embargo, este miembro y A deberán estar juntos en, al menos, 2 comités, lo que contradice las condiciones del enunciado. 6. Halla todas las ternas ),,( zyx de números reales que son solución de la ecuación

( ).7532)53(7)37(5)75(3 zyxyxzxzyzyx ++=+++++

SOLUCIÓN: Poniendo yx ba 5,3 == y zc 7= la ecuación anterior se convierte en:

).(2)()()( cbabacacbcba ++=+++++ Aplicando la desigualdad entre las medias aritmética y geométrica resulta:

⎟⎠⎞

⎜⎝⎛ +

+≤+42

2)( cbacba

⎟⎠⎞

⎜⎝⎛ +

+≤+42

2)( acbacb

⎟⎠⎞

⎜⎝⎛ +

+≤+42

2)( bacbac

Sumando las desigualdades anteriores se obtiene ).(2)()()( cbabacacbcba ++≤+++++

La igualdad tiene lugar cuando .cba == Por tanto, las soluciones buscadas son aquellas para las que ,753 zyx == lo que equivale a que .7ln5ln3ln zyx == Es decir las soluciones de la ecuación dada son:

.,7ln

1,5ln

1,3ln

1 Rttztytx ∈===

(Se observa que en general ,ln

ln1

ln1

eepp

pp ==⎟⎟⎠

⎞⎜⎜⎝

con 0>p y 1≠p y tecba === ).

Page 113: COMPENDIUM OMEFL - ToomatesXXXV Olimpiada Matemática Española Primera Fase Soluciones de la propuesta de problemas Problema 1 ¿Qué dígitos se han omitido en la siguiente multiplicación?

XLV Olimpiada Matemática Española

Primera Fase Primera sesión

Viernes mañana, 23 de enero de 2008

1. Calcular la suma ,20092008...

20092

200912 ⎥

⎤⎢⎣

⎡⎟⎠⎞⎜

⎝⎛++⎟

⎠⎞⎜

⎝⎛+⎟

⎠⎞⎜

⎝⎛ hhh siendo

h (t) = 55 + 25t

, t ∈R.

2. Si la sección producida por un plano al cortar un tetraedro es un rombo, probar que

necesariamente el rombo es un cuadrado.

3. Se consideran un cubo de cm1 de arista y dos vértices, A y B , diagonalmente

opuestos de una cara del cubo. Se denomina camino de longitud n a una sucesión de

1+n vértices de forma que dos consecutivos están a cm1 de distancia.

¿Cuál de los siguientes números es mayor: el número de caminos de longitud 1000 cm

que empiezan y acaban en A, o el número de caminos de longitud 1000 cm que

empiezan en A y acaban en B ? Justificar la respuesta.

No está permitido el uso de calculadoras. Cada problema se puntúa sobre 7 puntos.

El tiempo de cada sesión es de 3 horas y media.

Page 114: COMPENDIUM OMEFL - ToomatesXXXV Olimpiada Matemática Española Primera Fase Soluciones de la propuesta de problemas Problema 1 ¿Qué dígitos se han omitido en la siguiente multiplicación?

XLV Olimpiada Matemática Española

Primera Fase Segunda sesión

Viernes tarde, 23 de enero de 2008

4. Dado un triángulo acutángulo ABC, determinar para que puntos de su interior se verifican las siguientes desigualdades:

y

5. La igualdad 2008 = 1111 + 444 + 222 + 99 + 77 + 55 es un ejemplo de descomposición

del número 2008 como suma de números distintos de más de una cifra, cuya

representación (en el sistema decimal) utiliza un sólo dígito.

i) Encontrar una descomposición de este tipo para el número 2009.

ii) Determinar para el número 2009 todas las posibles descomposiciones de este tipo

que utilizan el menor número posible de sumandos (el orden de los sumandos no se

tiene en cuenta).

6. Se tienen en el plano puntos: n de color blanco, n de color azul y n de color

negro. Cada uno de los puntos está unido con puntos de color distinto al suyo mediante

1+n segmentos exactamente. Probar que hay, al menos, un triángulo formado por

vértices de distinto color.

No está permitido el uso de calculadoras. Cada problema se puntúa sobre 7 puntos.

El tiempo de cada sesión es de 3 horas y media.

Page 115: COMPENDIUM OMEFL - ToomatesXXXV Olimpiada Matemática Española Primera Fase Soluciones de la propuesta de problemas Problema 1 ¿Qué dígitos se han omitido en la siguiente multiplicación?

XLV Olimpiada Matemática Española

Primera Fase Primera sesión

Viernes tarde, 23 de enero de 2008

1. Dado un triángulo acutángulo ABC, determinar para que puntos de su interior se verifican las siguientes desigualdades:

y

2. La igualdad 2008 = 1111 + 444 + 222 + 99 + 77 + 55 es un ejemplo de descomposición

del número 2008 como suma de números distintos de más de una cifra, cuya

representación (en el sistema decimal) utiliza un sólo dígito.

i) Encontrar una descomposición de este tipo para el número 2009.

ii) Determinar para el número 2009 todas las posibles descomposiciones de este tipo

que utilizan el menor número posible de sumandos (el orden de los sumandos no se

tiene en cuenta).

3. Se tienen en el plano puntos: n de color blanco, n de color azul y n de color

negro. Cada uno de los puntos está unido con puntos de color distinto al suyo mediante

1+n segmentos exactamente. Probar que hay, al menos, un triángulo formado por

vértices de distinto color.

No está permitido el uso de calculadoras. Cada problema se puntúa sobre 7 puntos.

El tiempo de cada sesión es de 3 horas y media.

Page 116: COMPENDIUM OMEFL - ToomatesXXXV Olimpiada Matemática Española Primera Fase Soluciones de la propuesta de problemas Problema 1 ¿Qué dígitos se han omitido en la siguiente multiplicación?

XLV Olimpiada Matemática Española

Primera Fase Segunda sesión

Sábado mañana, 24 de enero de 2008

4. Probar que para todo entero positivo ,n n19– n7 es divisible por 30.

5. Determinar el mayor número de planos en el espacio tridimensional para los que

existen seis puntos con las siguientes condiciones:

i) Cada plano contiene al menos cuatro de los puntos.

ii) Cuatro puntos cualesquiera no pertenecen a una misma recta.

6. Los puntos de una retícula m × n pueden ser de color blanco o negro. Una retícula

se dice que está equilibrada si para cualquier punto P de ella, la fila y columna que

pasan por este punto P tienen el mismo número de puntos de igual color que P .

Determinar todos los pares de enteros positivos (m, n) para los que existe una retícula

equilibrada.

No está permitido el uso de calculadoras. Cada problema se puntúa sobre 7 puntos.

El tiempo de cada sesión es de 3 horas y media.

Page 117: COMPENDIUM OMEFL - ToomatesXXXV Olimpiada Matemática Española Primera Fase Soluciones de la propuesta de problemas Problema 1 ¿Qué dígitos se han omitido en la siguiente multiplicación?

XLV Olimpiada Matemática Española

Primera Fase Primera sesión

Sábado mañana, 24 de enero de 2008

1. Probar que para todo entero positivo n19– n7 es divisible por 30.

2. Determinar el mayor número de planos en el espacio tridimensional para los que

existen seis puntos con las siguientes condiciones:

i) Cada plano contiene al menos cuatro de los puntos.

ii) Cuatro puntos cualesquiera no pertenecen a una misma recta.

3. Los puntos de una retícula m × n pueden ser de color blanco o negro. Una retícula

se dice que está equilibrada si para cualquier punto P de ella, la fila y columna que

pasan por este punto P tienen ambas el mismo número de puntos de igual color que

P . Determinar todos los pares de enteros positivos (m, n) para los que existe una

retícula equilibrada.

No está permitido el uso de calculadoras. Cada problema se puntúa sobre 7 puntos.

El tiempo de cada sesión es de 3 horas y media.

Page 118: COMPENDIUM OMEFL - ToomatesXXXV Olimpiada Matemática Española Primera Fase Soluciones de la propuesta de problemas Problema 1 ¿Qué dígitos se han omitido en la siguiente multiplicación?

XLV Olimpiada Matemática Española

Primera Fase Segunda sesión

Sábado tarde, 24 de enero de 2008

4. En el interior de un paralelogramo ABCD se dibujan dos circunferencias. Una es

tangente a los lados AB y AD, y la otra es tangente a los lados CD y CB. Probar que

si estas circunferencias son tangentes entre sí, el punto de tangencia está en la

diagonal AC.

5. Dado un número natural n mayor que 1, hallar todos los pares de números enteros a y

,b tales que las dos ecuaciones 02008 =−+ axxn y 02009 =−+ bxxn tengan, al

menos, una raíz común real.

6. Sean 1C y 2C dos circunferencias exteriores tangentes en el punto .P Por un punto A

de 2C trazamos dos rectas tangentes a 1C en los puntos M y ´.M Sean N y ´N los

puntos respectivos de corte, distintos ambos de A, de estas rectas con .2C

Probar que .´´´ NMPNMNPN ⋅=⋅

No está permitido el uso de calculadoras. Cada problema se puntúa sobre 7 puntos.

El tiempo de cada sesión es de 3 horas y media.

Page 119: COMPENDIUM OMEFL - ToomatesXXXV Olimpiada Matemática Española Primera Fase Soluciones de la propuesta de problemas Problema 1 ¿Qué dígitos se han omitido en la siguiente multiplicación?

XLV Olimpiada Matemática Española Primera Fase

Primera sesión Viernes mañana, 23 de enero de 2008

SOLUCIONES

1. Calcular la suma ,20092008...

20092

200912 ⎥

⎤⎢⎣

⎡⎟⎠⎞⎜

⎝⎛++⎟

⎠⎞⎜

⎝⎛+⎟

⎠⎞⎜

⎝⎛ hhh siendo

Solución:

Se observa que la función es simétrica respecto al punto Por tanto,

de donde La suma

vale entonces

2. Si la sección producida por un plano al cortar un tetraedro es un rombo, probar que

necesariamente el rombo es un cuadrado.

Solución:

Los lados opuestos de un rombo son paralelos. Las caras del tetraedro que contienen a

dos de estos lados se cortan en una arista que, a su vez, será paralela al plano de corte y

paralela a estos dos lados del rombo. Sean A, B, C y D los vértices del tetraedro, y

supongamos que la arista paralela al plano de corte es la arista AB. De modo análogo, la

otra arista paralela también al plano de corte y a los otros dos lados del rombo será la

arista CD. Como AB y CD son perpendiculares, se tendrá que lados contiguos del

rombo son perpendiculares, por lo que en efecto, se trata de un cuadrado.

Page 120: COMPENDIUM OMEFL - ToomatesXXXV Olimpiada Matemática Española Primera Fase Soluciones de la propuesta de problemas Problema 1 ¿Qué dígitos se han omitido en la siguiente multiplicación?

3. Se consideran un cubo de cm1 de arista y dos vértices A y B diagonalmente

opuestos de una cara del cubo. Se denomina camino de longitud a una sucesión de

1+n vértices de forma que dos consecutivos están a cm1 de distancia. Entonces:¿Cuál

de los siguientes números es mayor: el número de caminos de longitud 1000 cm que

empiezan y acaban en A, o el número de caminos de longitud 1000 que empiezan

en A y acaban en ? Justifica la repuesta.

Solución:

Llamemos al número de caminos de longitud n que empiezan y acaban en A, y

llamemos al número de caminos de longitud n que empiezan en A y acaban en

B.

El ésimo vértice de un camino que termina en A sólo puede ser el propio A, o

cualquiera de los vértices diagonalmente opuestos en alguna de las tres caras que

concurren en A, uno de los cuales es el vértice B y los otros dos, C y D.

Ahora bien, hay tres caminos de longitud de A a A, correspondientes a las tres

aristas del cubo por A. Además, hay dos caminos de longitud desde B hasta A, uno

por cada uno de los otros dos vértices de la cara del cubo que los contiene, y

similarmente para C y D.

Por tanto, como hay tantos caminos de longitud n de A a B como de A a C o a D,

an = 3an-2 + 2bn-2 + 2bn-2 + 2bn-2 .

Un estudio análogo para el (n – 1)-ésimo vértice de un camino que termina en B nos

conduce a sólo puede ser A, B, C o D.

bn = 2an-2 + 3bn-2 + 2bn-2 + 2bn-2 .

Restando ambas igualdades, tenemos an – bn = an-2 – bn-2 = ⋅⋅⋅ = a0 – b0 = 1 – 0 = 1.

Luego hay exactamente 1 camino más de longitud de A a A que de A a B.

Page 121: COMPENDIUM OMEFL - ToomatesXXXV Olimpiada Matemática Española Primera Fase Soluciones de la propuesta de problemas Problema 1 ¿Qué dígitos se han omitido en la siguiente multiplicación?

XLV Olimpiada Matemática Española Primera Fase

Primera y segunda sesión Viernes tarde, 23 de enero de 2008

SOLUCIONES

1 y 4. Dado un triángulo acutángulo ABC, determinar para que puntos de su interior se verifican las siguientes desigualdades:

y

Solución: Sea O el cincuncentro del triángulo ABC. El valor del ángulo AC B, por estar inscrito en la circunferencia, es la mitad del ángulo AOB. De nuevo, para cualquier punto, P, sobre el arco AOB se tiene ∠APB = ∠AOB = 2∠ACB. Por tanto, este arco separa el interior del triángulo en dos partes: para los puntos Q situados a un lado del arco AOB, el valor del ángulo ∠AQB es mayor que ∠AOB y para los situados al otro lado del arco el ángulo ∠AQB es menor que ∠AOB. Así pues, los puntos del interior del triángulo que están sobre el arco AOB o fuera del segmento circular AOC son los que satisfacen la

primera de las condiciones .

El mismo razonamiento para las cuerdas BC y CA nos conducen a que el punto O es el

único que puede cumplir las tres condiciones.

Page 122: COMPENDIUM OMEFL - ToomatesXXXV Olimpiada Matemática Española Primera Fase Soluciones de la propuesta de problemas Problema 1 ¿Qué dígitos se han omitido en la siguiente multiplicación?

2 y 5. La igualdad 2008 = 1111 + 444 + 222 + 99 + 77 + 55 es un ejemplo de

descomposición del número 2008 como suma de números distintos de más de una cifra,

cuya representación (en el sistema decimal) utiliza un sólo dígito.

i) Encontrar una descomposición de este tipo para el número 2009.

ii) Determinar para el número 2009 todas las posibles descomposiciones de este tipo

que utilizan el menor número posible de sumandos (el orden de los sumandos no se

tiene en cuenta).

Solución:

Agrupando los números con igual cantidad de cifras tendremos la ecuación

2009 = 1111a + 111b + 11c

donde a, b y c son números enteros menores o iguales que 1 + 2 + 3 + · · · + 9 = 45,

puesto que los sumandos de la descomposición han de ser diferentes.

Se tiene entonces 2009 = 182 · 11 + 7 = 11 (101a + 10b + c ) + b. De donde 182 = 101a +

10b + c + (b – 7)/11. Como a, b y c son números enteros, se tiene que b ha de ser de la

forma c = 11k + 7, para algún valor natural de k comprendido entre 0 y 3 (recordar que c

≤ 45).

Substituyendo, obtenemos que 182 = 101a + 110k + 70 + c + k, de donde 112 = 101a +

111k + c. Las posibles soluciones son (a = 1, k = 0, c = 11) y (a = 0, k = 1, c = 1). En el

primer caso tenemos b = 7, y en el segundo b = 18, luego una descomposición puede ser

2009 = 1111 + 777 + 66 + 55, en la que c = 11 se ha descompuesto como 6 + 5.

Page 123: COMPENDIUM OMEFL - ToomatesXXXV Olimpiada Matemática Española Primera Fase Soluciones de la propuesta de problemas Problema 1 ¿Qué dígitos se han omitido en la siguiente multiplicación?

Analizando las soluciones vemos que no es posible obtener una descomposición para

2009 con menos de 4 sumandos, siendo las del primer tipo a = 1, b = 7, c = 9 + 2 = 8 +

3 = 7 + 4 = 6 + 5, y las del segundo tipo a = 1, b = 9 + 8 + 1 = 9 + 7 + 2 = 9 + 6 +3 = 9

+ 5 + 4 = 8 + 7 + 3 = 8 + 6 + 4 = 7 + 6 + 5, c = 1.

3 y 6. Se tienen en el plano puntos: de color blanco, de color azul y de color

negro. Cada uno de los puntos está unido con puntos de color distinto al suyo mediante

segmentos exactamente. Probar que hay, al menos, un triángulo formado por

vértices de distinto color.

Solución:

Consideramos el punto que está conectado con el número más alto de puntos de otro

color. Supongamos que este punto N es de color negro y que está conectado a k puntos de

color blanco. Como k ≤ n y N está conectado a n + 1 puntos, existirá un punto A de color

azul al que está conectado N. El número de puntos negros con los que está conectado A es

necesariamente menor o igual que k, por lo que A está conectado con por lo menos n + 1 –

k puntos blancos. Como sólo hay n puntos de color blanco y el número de los conectados

con N más los conectados con A suman por lo menos n + 1, necesariamente hay un punto

blanco conectado a ambos, con lo que ya tenemos el triángulo buscado.

Page 124: COMPENDIUM OMEFL - ToomatesXXXV Olimpiada Matemática Española Primera Fase Soluciones de la propuesta de problemas Problema 1 ¿Qué dígitos se han omitido en la siguiente multiplicación?

XLV Olimpiada Matemática Española Primera Fase

Primera y segunda sesión Viernes tarde, 23 de enero de 2008

SOLUCIONES

1 y 4. Dado un triángulo acutángulo ABC, determinar para que puntos de su interior se verifican las siguientes desigualdades:

y

Solución: Sea O el cincuncentro del triángulo ABC. El valor del ángulo AC B, por estar inscrito en la circunferencia, es la mitad del ángulo AOB. De nuevo, para cualquier punto, P, sobre el arco AOB se tiene ∠APB = ∠AOB = 2∠ACB. Por tanto, este arco separa el interior del triángulo en dos partes: para los puntos Q situados a un lado del arco AOB, el valor del ángulo ∠AQB es mayor que ∠AOB y para los situados al otro lado del arco el ángulo ∠AQB es menor que ∠AOB. Así pues, los puntos del interior del triángulo que están sobre el arco AOB o fuera del segmento circular AOC son los que satisfacen la

primera de las condiciones .

El mismo razonamiento para las cuerdas BC y CA nos conducen a que el punto O es el

único que puede cumplir las tres condiciones.

Page 125: COMPENDIUM OMEFL - ToomatesXXXV Olimpiada Matemática Española Primera Fase Soluciones de la propuesta de problemas Problema 1 ¿Qué dígitos se han omitido en la siguiente multiplicación?

2 y 5. La igualdad 2008 = 1111 + 444 + 222 + 99 + 77 + 55 es un ejemplo de

descomposición del número 2008 como suma de números distintos de más de una cifra,

cuya representación (en el sistema decimal) utiliza un sólo dígito.

i) Encontrar una descomposición de este tipo para el número 2009.

ii) Determinar para el número 2009 todas las posibles descomposiciones de este tipo

que utilizan el menor número posible de sumandos (el orden de los sumandos no se

tiene en cuenta).

Solución:

Agrupando los números con igual cantidad de cifras tendremos la ecuación

2009 = 1111a + 111b + 11c

donde a, b y c son números enteros menores o iguales que 1 + 2 + 3 + · · · + 9 = 45,

puesto que los sumandos de la descomposición han de ser diferentes.

Se tiene entonces 2009 = 182 · 11 + 7 = 11 (101a + 10b + c ) + b. De donde 182 = 101a +

10b + c + (b – 7)/11. Como a, b y c son números enteros, se tiene que b ha de ser de la

forma c = 11k + 7, para algún valor natural de k comprendido entre 0 y 3 (recordar que c

≤ 45).

Substituyendo, obtenemos que 182 = 101a + 110k + 70 + c + k, de donde 112 = 101a +

111k + c. Las posibles soluciones son (a = 1, k = 0, c = 11) y (a = 0, k = 1, c = 1). En el

primer caso tenemos b = 7, y en el segundo b = 18, luego una descomposición puede ser

2009 = 1111 + 777 + 66 + 55, en la que c = 11 se ha descompuesto como 6 + 5.

Page 126: COMPENDIUM OMEFL - ToomatesXXXV Olimpiada Matemática Española Primera Fase Soluciones de la propuesta de problemas Problema 1 ¿Qué dígitos se han omitido en la siguiente multiplicación?

Analizando las soluciones vemos que no es posible obtener una descomposición para

2009 con menos de 4 sumandos, siendo las del primer tipo a = 1, b = 7, c = 9 + 2 = 8 +

3 = 7 + 4 = 6 + 5, y las del segundo tipo a = 1, b = 9 + 8 + 1 = 9 + 7 + 2 = 9 + 6 +3 = 9

+ 5 + 4 = 8 + 7 + 3 = 8 + 6 + 4 = 7 + 6 + 5, c = 1.

3 y 6. Se tienen en el plano puntos: de color blanco, de color azul y de color

negro. Cada uno de los puntos está unido con puntos de color distinto al suyo mediante

segmentos exactamente. Probar que hay, al menos, un triángulo formado por

vértices de distinto color.

Solución:

Consideramos el punto que está conectado con el número más alto de puntos de otro

color. Supongamos que este punto N es de color negro y que está conectado a k puntos de

color blanco. Como k ≤ n y N está conectado a n + 1 puntos, existirá un punto A de color

azul al que está conectado N. El número de puntos negros con los que está conectado A es

necesariamente menor o igual que k, por lo que A está conectado con por lo menos n + 1 –

k puntos blancos. Como sólo hay n puntos de color blanco y el número de los conectados

con N más los conectados con A suman por lo menos n + 1, necesariamente hay un punto

blanco conectado a ambos, con lo que ya tenemos el triángulo buscado.

Page 127: COMPENDIUM OMEFL - ToomatesXXXV Olimpiada Matemática Española Primera Fase Soluciones de la propuesta de problemas Problema 1 ¿Qué dígitos se han omitido en la siguiente multiplicación?

XLV Olimpiada Matemática Española Primera Fase

Primera y segunda sesión Sábado mañana, 24 de enero de 2008

SOLUCIONES

1. y 4. Probar que para todo entero positivo n19– n7 es divisible por 30. Solución: n19

– n7 = n7(n12 – 1) = n7(n6 + 1)(n6 – 1) = n7(n6 + 1)(n3

+ 1)(n3 – 1), con lo que en la

descomposición de n19 – n7 aparecen tres números consecutivos, n – 1, n, n + 1, de los

cuales al menos uno es divisible por 2 y exactamente uno es divisible por 3.

Completaremos la descomposición para probar que aparece un factor divisible por 5, y

habremos terminado.

n19 – n7 = n7(n2 + 1)(n4

– n2 + 1)(n + 1)(n2 – n + 1)(n – 1)(n2

+ n + 1)

Si ninguno de los números n – 1, n, n + 1 es múltiplo de 5, entonces n = 5k ± 2, con lo

que (n2 + 1) = 25k2 ± 20k + 5 es múltiplo de 5, como queríamos.

2. y 5. Determinar el mayor número de planos en el espacio tridimensional para los

que existen seis puntos con las siguientes condiciones:

i) Cada plano contiene al menos cuatro de los puntos.

ii) Cuatro puntos cualesquiera no pertenecen a una misma recta.

Solución:

Sean r y s dos rectas que se cruzan en el espacio. Sean A, B y C tres puntos distintos de r

y sean P, Q y R tres puntos distintos en s. Cada uno de los puntos de r define con s un

plano, y análogamente cada punto de s con r. Estos 6 planos cumplen las condiciones del

problema, por lo que el número buscado es mayor o igual que 6.

Probaremos que no es posible satisfacer las condiciones con más de 6 planos.

Comenzamos por ver que no puede haber tres puntos en una misma recta. En efecto, si

suponemos que los puntos H, J, K están sobre una recta l, ningunos de los restantes

Page 128: COMPENDIUM OMEFL - ToomatesXXXV Olimpiada Matemática Española Primera Fase Soluciones de la propuesta de problemas Problema 1 ¿Qué dígitos se han omitido en la siguiente multiplicación?

puntos, L, M, N, puede estar en l, por la condición b. Estos tres puntos L, M y N,

pertenecen como mucho a tres de los planos, por lo que los demás planos contienen al

menos a 2 de los puntos de l, y por tanto a toda la recta. Es decir, al menos cuatro planos

contienen a l, lo que es imposible, porque al menos uno de ellos no podría contener a

ninguno de los puntos L, M o N, contrario a la condición a.

Veremos ahora que ningún plano puede contener a más de cuatro de los puntos.

Supongamos que uno de los planos contiene a cinco de los puntos y deja fuera al punto X.

Como acabamos de ver que no puede haber tres puntos alineados, un plano que contenga

a X contendría como mucho a dos de los otros puntos, contrario a la condición a.

Resumiendo, cada uno de los planos contiene exactamente a cuatro de los seis puntos y no

hay tres que estén en la misma recta.

Cada plano deja fuera un par de puntos y dos planos distintos dejan fuera a puntos

distintos, de lo contrario habría tres puntos en ambos planos, y deberían estar alineados.

Como seis puntos sólo se pueden agrupar en tres pares disjuntos de puntos, es imposible

que existan más de seis planos en las condiciones del problema.

3. y 6. Los puntos de una retícula m × n pueden ser de color blanco o negro. Una retícula

se dice que está equilibrada si para cualquier punto P de ella, la fila y columna que pasan

por este punto tienen ambas el mismo número de puntos de igual color que .

Determinar todos los pares de enteros positivos (m, n) para los que existe una retícula

equilibrada.

Solución:

Denotaremos por BF(i) el número de puntos de color blanco que hay en la fila i y con

BC(j) el número de puntos blancos en la columna j. Análogamente, NF(i) y NC(j)

denotarán el número de puntos negros en la fila i y en la columna j, respectivamente.

Siendo Pij el punto que se encuentra en la fila i y en la columna j, suponiendo que es de

color blanco, la condición de ser equilibrada se leerá BF(i) = BC(j).

Supongamos que el punto P11 de una retícula equilibrada de n filas y m columnas es de

color negro, y sea k el número de puntos negros de la primera fila. Intercambiando las

columnas, si fuere necesario, podemos suponer que estos puntos de color negro son los

Page 129: COMPENDIUM OMEFL - ToomatesXXXV Olimpiada Matemática Española Primera Fase Soluciones de la propuesta de problemas Problema 1 ¿Qué dígitos se han omitido en la siguiente multiplicación?

k primeros, P11, …,P1k. Por la condición de equilibrio para P11, la primera columna

también tendrá exactamente k puntos de color negro que, reordenando las filas, si fuere

necesario, supondremos que son los k primeros puntos, P11, …, Pk1.

Sea Pij , con 1 < i ≤ k y 1 < j ≤ k. Supongamos que Pij es de color blanco. Se tendrá

entonces que BF(i) = BC(j). Pero por ser negro el punto P1j, NC(j) = NF(1) = k, y por

ser negro el punto Pi1, NF(i) = NC(1) = k. De donde,

n = BF(i) + NF(i) = BF(i) + k = BC(j) + k = BC(j) + NC(j) = m.

Suponiendo que, por ejemplo, n > m, tendremos que todos los puntos negros de las filas

1 a k están en las primeras columnas, y análogamente todos los puntos negros de las

columnas 1 a k están en las primeras filas

Suponiendo que m – k > 0, todos los puntos Pij, con i > k y j > k, deben ser negros. En

otro caso tendríamos un rectángulo con tres vértices de color blanco y uno negro, de

donde se seguiría que n = m, como vimos al principio.

Por lo tanto, la condición para cualquiera de estos puntos nos dice que

n – k = NF(i) = NC(j) = m – k,

lo que contradice nuestra suposición de n > m. Por tanto, m – k = 0, lo que resulta en

que k = n – k, por la condición para Pmn, de donde n = 2m.

Luego los posibles pares de números serán (n, n), (n, 2n) y (2n, n), con n un entero

positivo. 

Page 130: COMPENDIUM OMEFL - ToomatesXXXV Olimpiada Matemática Española Primera Fase Soluciones de la propuesta de problemas Problema 1 ¿Qué dígitos se han omitido en la siguiente multiplicación?

XLV Olimpiada Matemática Española Primera Fase

Primera y segunda sesión Sábado mañana, 24 de enero de 2008

SOLUCIONES

1. y 4. Probar que para todo entero positivo n19– n7 es divisible por 30. Solución: n19

– n7 = n7(n12 – 1) = n7(n6 + 1)(n6 – 1) = n7(n6 + 1)(n3

+ 1)(n3 – 1), con lo que en la

descomposición de n19 – n7 aparecen tres números consecutivos, n – 1, n, n + 1, de los

cuales al menos uno es divisible por 2 y exactamente uno es divisible por 3.

Completaremos la descomposición para probar que aparece un factor divisible por 5, y

habremos terminado.

n19 – n7 = n7(n2 + 1)(n4

– n2 + 1)(n + 1)(n2 – n + 1)(n – 1)(n2

+ n + 1)

Si ninguno de los números n – 1, n, n + 1 es múltiplo de 5, entonces n = 5k ± 2, con lo

que (n2 + 1) = 25k2 ± 20k + 5 es múltiplo de 5, como queríamos.

2. y 5. Determinar el mayor número de planos en el espacio tridimensional para los

que existen seis puntos con las siguientes condiciones:

i) Cada plano contiene al menos cuatro de los puntos.

ii) Cuatro puntos cualesquiera no pertenecen a una misma recta.

Solución:

Sean r y s dos rectas que se cruzan en el espacio. Sean A, B y C tres puntos distintos de r

y sean P, Q y R tres puntos distintos en s. Cada uno de los puntos de r define con s un

plano, y análogamente cada punto de s con r. Estos 6 planos cumplen las condiciones del

problema, por lo que el número buscado es mayor o igual que 6.

Probaremos que no es posible satisfacer las condiciones con más de 6 planos.

Comenzamos por ver que no puede haber tres puntos en una misma recta. En efecto, si

suponemos que los puntos H, J, K están sobre una recta l, ningunos de los restantes

Page 131: COMPENDIUM OMEFL - ToomatesXXXV Olimpiada Matemática Española Primera Fase Soluciones de la propuesta de problemas Problema 1 ¿Qué dígitos se han omitido en la siguiente multiplicación?

puntos, L, M, N, puede estar en l, por la condición b. Estos tres puntos L, M y N,

pertenecen como mucho a tres de los planos, por lo que los demás planos contienen al

menos a 2 de los puntos de l, y por tanto a toda la recta. Es decir, al menos cuatro planos

contienen a l, lo que es imposible, porque al menos uno de ellos no podría contener a

ninguno de los puntos L, M o N, contrario a la condición a.

Veremos ahora que ningún plano puede contener a más de cuatro de los puntos.

Supongamos que uno de los planos contiene a cinco de los puntos y deja fuera al punto X.

Como acabamos de ver que no puede haber tres puntos alineados, un plano que contenga

a X contendría como mucho a dos de los otros puntos, contrario a la condición a.

Resumiendo, cada uno de los planos contiene exactamente a cuatro de los seis puntos y no

hay tres que estén en la misma recta.

Cada plano deja fuera un par de puntos y dos planos distintos dejan fuera a puntos

distintos, de lo contrario habría tres puntos en ambos planos, y deberían estar alineados.

Como seis puntos sólo se pueden agrupar en tres pares disjuntos de puntos, es imposible

que existan más de seis planos en las condiciones del problema.

3. y 6. Los puntos de una retícula m × n pueden ser de color blanco o negro. Una retícula

se dice que está equilibrada si para cualquier punto P de ella, la fila y columna que pasan

por este punto tienen ambas el mismo número de puntos de igual color que .

Determinar todos los pares de enteros positivos (m, n) para los que existe una retícula

equilibrada.

Solución:

Denotaremos por BF(i) el número de puntos de color blanco que hay en la fila i y con

BC(j) el número de puntos blancos en la columna j. Análogamente, NF(i) y NC(j)

denotarán el número de puntos negros en la fila i y en la columna j, respectivamente.

Siendo Pij el punto que se encuentra en la fila i y en la columna j, suponiendo que es de

color blanco, la condición de ser equilibrada se leerá BF(i) = BC(j).

Supongamos que el punto P11 de una retícula equilibrada de n filas y m columnas es de

color negro, y sea k el número de puntos negros de la primera fila. Intercambiando las

columnas, si fuere necesario, podemos suponer que estos puntos de color negro son los

Page 132: COMPENDIUM OMEFL - ToomatesXXXV Olimpiada Matemática Española Primera Fase Soluciones de la propuesta de problemas Problema 1 ¿Qué dígitos se han omitido en la siguiente multiplicación?

k primeros, P11, …,P1k. Por la condición de equilibrio para P11, la primera columna

también tendrá exactamente k puntos de color negro que, reordenando las filas, si fuere

necesario, supondremos que son los k primeros puntos, P11, …, Pk1.

Sea Pij , con 1 < i ≤ k y 1 < j ≤ k. Supongamos que Pij es de color blanco. Se tendrá

entonces que BF(i) = BC(j). Pero por ser negro el punto P1j, NC(j) = NF(1) = k, y por

ser negro el punto Pi1, NF(i) = NC(1) = k. De donde,

n = BF(i) + NF(i) = BF(i) + k = BC(j) + k = BC(j) + NC(j) = m.

Suponiendo que, por ejemplo, n > m, tendremos que todos los puntos negros de las filas

1 a k están en las primeras columnas, y análogamente todos los puntos negros de las

columnas 1 a k están en las primeras filas

Suponiendo que m – k > 0, todos los puntos Pij, con i > k y j > k, deben ser negros. En

otro caso tendríamos un rectángulo con tres vértices de color blanco y uno negro, de

donde se seguiría que n = m, como vimos al principio.

Por lo tanto, la condición para cualquiera de estos puntos nos dice que

n – k = NF(i) = NC(j) = m – k,

lo que contradice nuestra suposición de n > m. Por tanto, m – k = 0, lo que resulta en

que k = n – k, por la condición para Pmn, de donde n = 2m.

Luego los posibles pares de números serán (n, n), (n, 2n) y (2n, n), con n un entero

positivo. 

Page 133: COMPENDIUM OMEFL - ToomatesXXXV Olimpiada Matemática Española Primera Fase Soluciones de la propuesta de problemas Problema 1 ¿Qué dígitos se han omitido en la siguiente multiplicación?

XLV Olimpiada Matemática Española Primera Fase

Segunda sesión Sábado tarde, 24 de enero de 2008

SOLUCIONES

4. En el interior de un paralelogramo ABCD se dibujan dos circunferencias. Una es

tangente a los lados AB y AD, y la otra es tangente a los lados CD y CB. Probar que

si estas circunferencias son tangentes entre si, el punto de tangencia está en la

diagonal AC.

Solución:

Veremos que los puntos A, K y C están alineados.

Sean O1 y O2 los centros de la primera y segunda circunferencia, respectivamente. Notar

que AO1, biseca el ángulo DAB, y análogamente CO2 biseca el ángulo DCB. Como los

lados son paralelos dos a dos y los ángulos O1AK y CO2K son iguales, entonces AO1 es

paralelo a CO2, y, como O1K y O2K están alineados, los ángulos AO1K y KO2C son

iguales.

Como O1P ⊥ AB y O1Q ⊥ CD, los triángulos APO1 y CQO2 son semejantes, por lo que

, y como |O1P| = |O1K| y |O2Q| = |O2K|, los triángulos AO1K y KO2C son

semejantes, por lo que los puntos A, K y C están alineados, como se quería.

5. Dado un número natural mayor que , hallar todos los pares de números enteros y

tales que las dos ecuaciones y tengan, al

menos, una raíz común real.

Solución:

Page 134: COMPENDIUM OMEFL - ToomatesXXXV Olimpiada Matemática Española Primera Fase Soluciones de la propuesta de problemas Problema 1 ¿Qué dígitos se han omitido en la siguiente multiplicación?

Restando ambas ecuaciones tenemos que (b – a)x = 1. Luego si estas ecuaciones van a

tener una raíz común, tiene que ser x = 1/(b – a). Notar que a no puede ser igual a b.

Substituyendo en una de las ecuaciones, tendremos que

(b – a)n–1 (a – 2008(b – a)) = –1,

y que, por ser a y b enteros, estos dos factores serán uno igual a +1 y otro igual a –1.

Si (b – a) = 1, se tendrá a = –1 + 2008 = 2007, y por tanto b = 2008.

Si (b – a) = – 1, se tendrá a = (–1) n–1 – 2008, y por tanto b = (–1) n–1 – 2009.

Luego los únicos pares de números (a, b) son

(2007, 2008) y ((–1) n–1 – 2008, (–1) n–1 – 2009).

6. Sean y dos circunferencias exteriores tangentes en el punto Por un punto

de trazamos dos rectas tangentes a en los puntos y Sean y los

puntos respectivos de corte, distintos ambos de A, de estas rectas con

Probar que

Solución:

Probaremos que para cualquier punto N de C2 y M de C1 tal que MN es tangente a C1, se

tiene que el cociente es constante.Sea Q el punto de corte con C1 de la recta por N y

P. Los triángulos NMP y NQM son congruentes porque comparten el ángulo en N y

por ser inscrito y semi-inscrito con cuerda MP. Por lo tanto,

se tiene:

Page 135: COMPENDIUM OMEFL - ToomatesXXXV Olimpiada Matemática Española Primera Fase Soluciones de la propuesta de problemas Problema 1 ¿Qué dígitos se han omitido en la siguiente multiplicación?

MNQN

=PNMN

. (*)

Siendo O1 y O2 los centros de C1 y C2, respectivamente, los triángulos isósceles PO1Q y

PO2N son congruentes porque .

De aquí se sigue que , siendo r1 y r2 los respectivos radios de C1

y C2.

Como |QN| = |QP| + |PN| = |PN| (1 + λ), substituyendo en (*) tenemos que |MN|2 = |PN|2

(1 + λ), de donde , como queríamos.

Page 136: COMPENDIUM OMEFL - ToomatesXXXV Olimpiada Matemática Española Primera Fase Soluciones de la propuesta de problemas Problema 1 ¿Qué dígitos se han omitido en la siguiente multiplicación?

XLVI Olimpiada Matematica Espanola

Primera Fase

Primera sesion

Viernes manana, 15 de enero de 2010

OlimpiadaMatemáticaEspañola RSME

1. Sea In el conjunto de los n primeros numeros naturales impares. Por ejemplo:I3 = {1, 3, 5}, I6 = {1, 3, 5, 7, 9, 11}, etc.

¿Para que numeros n el conjunto In se puede descomponer en dos partes(disjuntas) de forma que coincidan las sumas de los numeros en cada una deellas?

2. Determina los lados del triangulo rectangulo del que se conocen el perımetro,

p = 96, y la altura sobre la hipotenusa, h =965

.

3. Halla todos los numeros naturales n que verifican la condicion:

[n

2

]+

[2n

3

]= n + 335

donde [x] es la parte entera de x. (Esto es, [1,32] = 1, [2] = 2,[12

]= 0,

[π] = 3, etc.)

No esta permitido el uso de calculadoras.Cada problema se puntua sobre 7 puntos.

El tiempo de cada sesion es de 3 horas y media.

Page 137: COMPENDIUM OMEFL - ToomatesXXXV Olimpiada Matemática Española Primera Fase Soluciones de la propuesta de problemas Problema 1 ¿Qué dígitos se han omitido en la siguiente multiplicación?

XLVI Olimpiada Matematica Espanola

Primera Fase

Segunda sesion

Viernes tarde, 15 de enero de 2010

OlimpiadaMatemáticaEspañola RSME

4. Se considera un triangulo equilatero de lado 1 y centro O, como el de lafigura.

A

B C

O

Un rayo parte de O y se refleja en los tres lados, AB, AC y BC, (en el ordendado), hasta alcanzar el vertice A.

Determina la longitud mınima del recorrido del rayo.

Nota: Cuando el rayo se refleja en un lado, los angulos de entrada (incidencia)y salida (reflexion) coinciden.

5. Calcula las soluciones reales de la ecuacion:

4√

97 − X + 4√

X = 5.

6. Dado el polinomio P (X) = X4 + X3 + X2 + X + , en el que cadacuadrado representa un hueco donde se colocara un coeficiente, se plantea elsiguiente juego entre dos jugadores: Alternativamente, el primer y el segundojugador eligen un hueco vacıo y colocan en el un entero no nulo hasta rellenartodos los cuatro huecos. Si el polinomio resultante tiene al menos dos raıcesenteras gana el segundo jugador, en otro caso el ganador es el primero.

Prueba que, eligiendo la estrategia adecuada, el primer jugador siemprepuede ganar.

No esta permitido el uso de calculadoras.Cada problema se puntua sobre 7 puntos.

El tiempo de cada sesion es de 3 horas y media.

Page 138: COMPENDIUM OMEFL - ToomatesXXXV Olimpiada Matemática Española Primera Fase Soluciones de la propuesta de problemas Problema 1 ¿Qué dígitos se han omitido en la siguiente multiplicación?

XLVI Olimpiada Matematica Espanola

Primera Fase

Primera sesion

Viernes tarde, 15 de enero de 2010

OlimpiadaMatemáticaEspañola RSME

1. Se considera un triangulo equilatero de lado 1 y centro O, como el de lafigura.

A

B C

O

Un rayo parte de O y se refleja en los tres lados, AB, AC y BC, (en el ordendado), hasta alcanzar el vertice A.

Determina la longitud mınima del recorrido del rayo.

Nota: Cuando el rayo se refleja en un lado, los angulos de entrada (incidencia)y salida (reflexion) coinciden.

2. Calcula las soluciones reales de la ecuacion:

4√

97 − X + 4√

X = 5.

3. Dado el polinomio P (X) = X4 + X3 + X2 + X + , en el que cadacuadrado representa un hueco donde se colocara un coeficiente, se plantea elsiguiente juego entre dos jugadores: Alternativamente, el primer y el segundojugador eligen un hueco vacıo y colocan en el un entero no nulo hasta rellenartodos los cuatro huecos. Si el polinomio resultante tiene al menos dos raıcesenteras gana el segundo jugador, en otro caso el ganador es el primero.

Prueba que, eligiendo la estrategia adecuada, el primer jugador siemprepuede ganar.

No esta permitido el uso de calculadoras.Cada problema se puntua sobre 7 puntos.

El tiempo de cada sesion es de 3 horas y media.

Page 139: COMPENDIUM OMEFL - ToomatesXXXV Olimpiada Matemática Española Primera Fase Soluciones de la propuesta de problemas Problema 1 ¿Qué dígitos se han omitido en la siguiente multiplicación?

XLVI Olimpiada Matematica Espanola

Primera Fase

Segunda sesion

Sabado manana, 16 de enero de 2010

OlimpiadaMatemáticaEspañola RSME

4. Supongamos que tenemos un tablero con dieciseis casillas dispuestas en cua-tro filas y cuatro columnas.

(a) Prueba que se pueden colocar siete fichas, nunca dos en la misma casilla,de forma que al eliminar dos filas y dos columnas cualesquiera, siempre quedealguna ficha sin eliminar.

(b) Prueba que si se colocan seis fichas, nunca dos en la misma casilla, siemprese puede eliminar dos filas y dos columnas de forma que todas las fichas seaneliminadas.

5. Se considera un tetraedro regular como el de la figura. Si el punto E recorrela arista AB. ¿Cuando el angulo CED es maximo?

A

B

C

D

E

6. Decimos que un conjunto E de numeros naturales es especial cuando al tomardos elementos cualesquiera distintos a, b ∈ E se tiene que (a − b)2 divide alproducto ab.

(a) Encuentra un conjunto especial formado por tres elementos.

(b) ¿Existe un conjunto especial formado por cuatro numeros naturales queestan en progresion aritmetica?

No esta permitido el uso de calculadoras.Cada problema se puntua sobre 7 puntos.

El tiempo de cada sesion es de 3 horas y media.

Page 140: COMPENDIUM OMEFL - ToomatesXXXV Olimpiada Matemática Española Primera Fase Soluciones de la propuesta de problemas Problema 1 ¿Qué dígitos se han omitido en la siguiente multiplicación?

XLVI Olimpiada Matematica Espanola

Primera Fase

Primera sesion

Sabado manana, 16 de enero de 2010

OlimpiadaMatemáticaEspañola RSME

1. Supongamos que tenemos un tablero con dieciseis casillas dispuestas en cua-tro filas y cuatro columnas.

(a) Prueba que se pueden colocar siete fichas, nunca dos en la misma casilla,de forma que al eliminar dos filas y dos columnas cualesquiera, siempre quedealguna ficha sin eliminar.

(b) Prueba que si se colocan seis fichas, nunca dos en la misma casilla, siemprese puede eliminar dos filas y dos columnas de forma que todas las fichas seaneliminadas.

2. Se considera un tetraedro regular como el de la figura. Si el punto E recorrela arista AB. ¿Cuando el angulo CED es maximo?

A

B

C

D

E

3. Decimos que un conjunto E de numeros naturales es especial cuando al tomardos elementos cualesquiera distintos a, b ∈ E se tiene que (a − b)2 divide alproducto ab.

(a) Encuentra un conjunto especial formado por tres elementos.

(b) ¿Existe un conjunto especial formado por cuatro numeros naturales queestan en progresion aritmetica?

No esta permitido el uso de calculadoras.Cada problema se puntua sobre 7 puntos.

El tiempo de cada sesion es de 3 horas y media.

Page 141: COMPENDIUM OMEFL - ToomatesXXXV Olimpiada Matemática Española Primera Fase Soluciones de la propuesta de problemas Problema 1 ¿Qué dígitos se han omitido en la siguiente multiplicación?

XLVI Olimpiada Matematica Espanola

Primera Fase

Segunda sesion

Sabado tarde, 16 de enero de 2010

OlimpiadaMatemáticaEspañola RSME

4. Un jardinero tiene que plantar en una fila a lo largo de un camino tresrobles, cuatro encinas y cinco hayas. Planta los arboles al azar; siendo laprobabilidad de plantar un arbol u otro la misma.Halla la probabilidad de que, una vez plantados todos los arboles, no hayados hayas consecutivas.

5. Calcula las soluciones reales de la ecuacion:

3√

1729 − X + 3√

X = 19.

6. Averigua que numeros de cuatro cifras significativas, abcd (con a �= 0), soniguales a ab

2+ cd

2 − cd.Nota: La notacion ab representa, en este problema, el numero que tiene adecenas y b unidades; en este caso se tiene que a, b ∈ {0, 1, 2, 3, 4, 5, 6, 7, 8, 9}.

No esta permitido el uso de calculadoras.Cada problema se puntua sobre 7 puntos.

El tiempo de cada sesion es de 3 horas y media.

Page 142: COMPENDIUM OMEFL - ToomatesXXXV Olimpiada Matemática Española Primera Fase Soluciones de la propuesta de problemas Problema 1 ¿Qué dígitos se han omitido en la siguiente multiplicación?

Problema 1. Sea In el conjunto de los n primeros numeros naturales impares. Por ejemplo:I3 = {1, 3, 5}, I6 = {1, 3, 5, 7, 9, 11}, etc.

¿Para que numeros n el conjunto In se puede descomponer en dos partes (disjuntas) de formaque coincidan las sumas de los numeros en cada una de ellas?

Solucion. Los primeros casos son:

I1 = {1} no descomponeI2 = {1, 3} no descomponeI3 = {1, 3, 5} no descomponeI4 = {1, 3, 5, 7} descompone {1, 7} y {3, 5}I5 = no descomponeI6 = {1, 3, 5, 7, 9, 11} descompone {1, 3, 5, 9} y {7, 11}

Observa que para que In se pueda descomponer en dos partes que tengan la misma suma, tieneque ser n par, ya que en ese caso la suma de todos los elementos de In es un numero par.

Observa que si Im descompone, entonces tambien lo hace Im+4, ya que se tiene

Im+4 = Im ∪ {2m + 1, 2m + 3, 2m + 5, 2m + 7}.Si Im = P1∪P2 es una descomposicion de I2m, entonces P1∪{2m+1, 2m+7}, P2∪{2m+3, 2m+5}es una descomposicion de I2m+4.

Como I4 e I6 descomponen, podemos concluir que In descompone para cada n, par, mayor oigual que 4.

Problema 2. Determina los lados del triangulo rectangulo del que se conocen el perımetro,

p = 96, y la altura sobre la hipotenusa, h =96

5.

Solucion. Consideramos el triangulo rectangulo de la figura.

h b

ad

c

Buscamos relaciones entre estos segmentos.

El area del triangulo es: bc2

= ah2

, de aquı se deduce que

bc = ah. (1)

Por ser p = a + b + c, se tiene b + c = p− a, luego (b + c)2 = (p− a)2, y de aquı, utilizando quea2 = b2 + c2, se tiene 2bc = p2−2pa. Ahora utilizamos la relacion (1) y se tiene: 2ah = p2−2pa.Finalmente, podemos calcular a como:

a =p2

2(h + p). (2)

1

Page 143: COMPENDIUM OMEFL - ToomatesXXXV Olimpiada Matemática Española Primera Fase Soluciones de la propuesta de problemas Problema 1 ¿Qué dígitos se han omitido en la siguiente multiplicación?

Como ya es conocido a, y teniendo en cuenta que se tienen las relaciones: b+c = p−a y bc = ah,podemos calcular b y c como las soluciones de la ecuacion de segundo grado z2−(b+c)z+bc = 0,esto es de la ecuacion z2 − (p − a)z + ah = 0.

En nuestro caso, con los valores dados, se tiene:

p = 96,

h = 96/5,

a =p2

2(h + p)=

962

2(96 + 96/5)= 40.

Falta calcular b y c, que son raıces de la ecuacion z2 − (p− a)z + ah = 0, esto es, de la ecuacionz2 − 56z + 768 = 0; y cuyas raıces son: 32 y 24.

Los lados del triangulo dado son: 40, 32 y 24.

Problema 3. Halla todos los numeros naturales n que verifican la condicion:

[n2

]+

[2n

3

]= n + 335

donde [x] es la parte entera de x. (Esto es, [1,32] = 1, [2] = 2,

[1

2

]= 0, [π] = 3, etc.)

Solucion. Distinguimos casos segun n sea de la forma 6k, 6k +1, 6k +2, 6k +3, 6k +4 o 6k +5(observemos que n es siempre de alguna de estas seis formas) y hacemos la siguiente tabla:

n [n2] [2n

3] [n

2] + [2n

3] n + 335

6k 3k 4k 7k 6k + 3356k + 1 3k 4k 7k 6k + 3366k + 2 3k + 1 4k + 1 7k + 2 6k + 3376k + 3 3k + 1 4k + 2 7k + 3 6k + 3386k + 4 3k + 2 4k + 2 7k + 4 6k + 3396k + 5 3k + 2 4k + 3 7k + 5 6k + 340

Igualando las ultimas dos columnas obtenemos que:

Si n = 6k entonces 7k = 6k + 334, de donde k = 335 y n = 6 · 334 = 2010.

Si n = 6k + 1 entonces 7k = 6k + 335, de donde k = 336 y n = 6 · 335 + 1 = 2017.

Si n = 6k + 2 entonces 7k + 2 = 6k + 336, de donde k = 335 y n = 6 · 334 + 2 = 2012.

Si n = 6k + 3 entonces 7k + 3 = 6k + 337, de donde k = 335 y n = 6 · 334 + 3 = 2013.

Si n = 6k + 4 entonces 7k + 4 = 6k + 338, de donde k = 335 y n = 6 · 334 + 4 = 2014.

Si n = 6k + 5 entonces 7k + 5 = 6k + 339, de donde k = 335 y n = 6 · 334 + 5 = 2015.

Por tanto, hay seis numeros que cumplen la ecuacion: 2010, 2012, 2013, 2014, 2015 y 2017.

2

Page 144: COMPENDIUM OMEFL - ToomatesXXXV Olimpiada Matemática Española Primera Fase Soluciones de la propuesta de problemas Problema 1 ¿Qué dígitos se han omitido en la siguiente multiplicación?

Problema 4. Se considera un triangulo equilatero de lado 1 y centro O, como el de la figura.

�B �C

A

�O

Un rayo parte de O y se refleja en los tres lados, AB, AC y BC, (en el orden dado), hastaalcanzar el vertice A.

Determina la longitud mınima del recorrido del rayo.

Nota: Cuando el rayo se refleja en un lado, los angulos de entrada (incidencia) y salida (re-flexion) coinciden.

Solucion. Como el rayo se refleja en los lados indicados, basta con desarrollar el camino recorridopor el rayo, para ello desdoblamos el triangulo segun la siguiente figura.

�B �C

A

�O

�A′ �B′

�C ′

�O′�

P

Esta figura nos indica que existe un unico camino para ir del punto O al punto A reflejandoseen los lados del triangulo en el orden indicado. Para calcular la distancia recorrida por el rayo,basta considerar el triangulo APO′; es un triangulo rectangulo del que tenemos que calcular lahipotenusa AO′. Sabemos que O′P es igual a 1

2. La distancia PA es 1 + 1 + 1

3= 7

3de la altura

h =√

32

del triangulo. En este caso tenemos:

(AO′)2 = (O′P )2 + (PA)2 =

(1

2

)2

+

(7

3

√3

2

)2

=1

4+

72 × 3

32 × 22=

1

4+

49

12=

13

3.

Por lo tanto la distancia AO′ es:√

133

=√

393

.

Problema 5. Calcula las soluciones reales de la ecuacion:

4√

97 − X +4√

X = 5.

3

Page 145: COMPENDIUM OMEFL - ToomatesXXXV Olimpiada Matemática Española Primera Fase Soluciones de la propuesta de problemas Problema 1 ¿Qué dígitos se han omitido en la siguiente multiplicación?

Solucion. Si llamamos a = 4√

97 − x y b = 4√

x, se verifica:

a + b = 5a4 + b4 = 97

}

Para resolver este sistema procedemos como sigue:

25 = (a + b)2 = a2 + 2ab + b2

y de aquı se tiene a2 + b2 = 25 − 2ab. Por otro lado

625 = (a + b)4 = a4 + b4 + 4ab(a2 + b2) + 6a2b2 = 97 + 100ab − 2a2b2,

y de aquı se tiene a2b2 − 50ab + 264 = 0. Entonces ab es una raız de z2 − 50z + 264 = 0; comolas raıces son 44 y 6, estudiamos cada uno de estos casos.

Si ab = 44, entonces a y b son las soluciones del sistemaa + b = 5ab = 44

}, luego de la ecuacion

z2 − 5z + 44 = 0; esta no tiene raıces reales. Si ab = 6, entonces a y b son raıces de la ecuacionz2 − 5z + 6 = 0, que tiene las raıces 3 y 2.

Si b = 3, entonces x = 81, y si x = 2, entonces x = 16. Estas son las unicas raıces reales de laecuacion dada.

Solucion.[Alternativa] Si llamamos a = 4√

97 − x y b = 4√

x, se verifica:

a + b = 5a4 + b4 = 97

}

Utilizamos una variable temporal t y escribimos a = 52

+ t, b = 52− t. entonces se verifica:

97 = a4 + b4 =

(5

2+ t

)4

+

(5

2− t

)4

= 2t4 + 12

(5

2

)2

t2 + 2

(5

2

)4

.

Simplificando resulta:16t4 + 600t2 − 151 = 0

El valor de t es ±12. Se tiene entonces:

b =5

2+

1

2= 3;

b =5

2− 1

2= 2.

Entonces al igualar b = 4√

x, resulta:

b = 3 ⇒ x = 81,b = 2 ⇒ x = 16,

que son las unicas dos soluciones reales de la ecuacion dada.

Problema 6. Dado el polinomio P (X) = X4 + �X3 + �X2 + �X + � , en el que cadacuadrado representa un hueco donde se colocara un coeficiente, se plantea el siguiente juegoentre dos jugadores: Alternativamente, el primer y el segundo jugador eligen un hueco vacıo ycolocan en el un entero no nulo hasta rellenar todos los cuatro huecos. Si el polinomio resultantetiene al menos dos raıces enteras gana el segundo jugador, en otro caso el ganador es el primero.

Prueba que, eligiendo la estrategia adecuada, el primer jugador siempre puede ganar.

4

Page 146: COMPENDIUM OMEFL - ToomatesXXXV Olimpiada Matemática Española Primera Fase Soluciones de la propuesta de problemas Problema 1 ¿Qué dígitos se han omitido en la siguiente multiplicación?

Solucion.

Nota: El enunciado presenta cierta ambiguedad: hay que distinguir si “dos raıces enteras”significa que estas son distintas, o por el contrario pueden ser raıces dobles.

El caso de dos raıces distintas es sencillo; basta con la primera jugada para el primer jugador.En el caso de raıces dobles tiene que hacer uso tambien de la segunda jugada.

Una posible estrategia es:

(i) El primer jugador coloca un -1 en el lugar a4, quedando el polinomio en la forma:

X4 + �X3 + �X2 + �X − 1

De esta forma el primer jugador fuerza a que las posibles raıces enteras del polinomio sean 1 y−1.(ii) El segundo jugador coloca ai �= 0 en uno de los huecos.(iii) El primer jugador coloca aj �= 0 en uno de los huecos.(iv) El segundo jugador coloca ak �= 0 en el unico hueco restante.

Observa que el polinomio tiene ahora la forma X4 + a1X3 + a2X

2 + a3X − 1 y que las posiblesraıces enteras (distintas) de este polinomio son 1 y -1, ya que el termino independiente es −1.

Cuando 1 es una raız se tiene 1 + a1 + a2 + a3 − 1 = 0, esto es, a1 + a2 + a3 = 0, y cuando −1es una raız se tiene 1 − a1 + a2 − a3 − 1 = 0, esto es −a1 + a2 − a3 = 0.

Sean cuales sean los valores de a1, a−2, a3 se tendra siempre 2a2 = 0, lo que implica que a2 = 0,y esto no esta permitido por las reglas del juego. Por lo tanto el caso de dos raıces distintasesta resuelto.

En el caso en el que las dos raıces sean iguales.

Si 1 es una raız doble, entonces se verifica a1 + a2 + a3 = 0 y 4 + 3a1 + 2a2 + a3 = 0, por lotanto se tiene:

a1 + a2 + a3 = 04 + 2a1 + 2a2 = 0

}siendo a3 = 4 + a1 y a2 = −4 − 2a1.

Si −1 es una raız doble, entonces se verifica −a1 + a2 − a3 = 0 y 4− 3a1 + 2a2 − a3 = 0, por lotanto se tiene:

−a1 + a2 − a3 = 04 − 2a1 + 2a2 = 0

}siendo a3 = −4 + a1 y a2 = −4 + 2a1.

Esta claro que si el segundo jugador hace ai = a2, entonces el primer jugador puede colocaraj = a1 de forma que −4−2a1 �= a2 y −4+2a1 �= a2, de esta forma no habra dos raıces enteras.

Por el contrario, si el primer jugador hace ai = a1, basta tomar a2 de forma que a2 �= −4 − 2a1

y a2 �= −4 + 2a1. El caso de ai = a3 se hace de la misma forma.

5

Page 147: COMPENDIUM OMEFL - ToomatesXXXV Olimpiada Matemática Española Primera Fase Soluciones de la propuesta de problemas Problema 1 ¿Qué dígitos se han omitido en la siguiente multiplicación?

Problema 1. Se considera un triangulo equilatero de lado 1 y centro O, como el de la figura.

�B �C

�A

�O

Un rayo parte de O y se refleja en los tres lados, AB, AC y BC, (en el orden dado), hastaalcanzar el vertice A.

Determina la longitud mınima del recorrido del rayo.

Nota: Cuando el rayo se refleja en un lado, los angulos de entrada (incidencia) y salida (re-flexion) coinciden.

Solucion. Como el rayo se refleja en los lados indicados, basta con desarrollar el camino recorridopor el rayo, para ello desdoblamos el triangulo segun la siguiente figura.

�B �C

�A

�O

�A′ �B′

�C ′

�O′�P

Esta figura nos indica que existe un unico camino para ir del punto O al punto A reflejandoseen los lados del triangulo en el orden indicado. Para calcular la distancia recorrida por el rayo,basta considerar el triangulo APO′; es un triangulo rectangulo del que tenemos que calcular lahipotenusa AO′. Sabemos que O′P es igual a 1

2. La distancia PA es 1 + 1 + 1

3= 7

3de la altura

h =√

32

del triangulo. En este caso tenemos:

(AO′)2 = (O′P )2 + (PA)2 =

(1

2

)2

+

(7

3

√3

2

)2

=1

4+

72 × 3

32 × 22=

1

4+

49

12=

13

3.

Por lo tanto la distancia AO′ es:√

133

=√

393

.

Problema 2. Calcula las soluciones reales de la ecuacion:

4√

97 − X +4√

X = 5.

1

Page 148: COMPENDIUM OMEFL - ToomatesXXXV Olimpiada Matemática Española Primera Fase Soluciones de la propuesta de problemas Problema 1 ¿Qué dígitos se han omitido en la siguiente multiplicación?

Solucion. Si llamamos a = 4√

97 − x y b = 4√

x, se verifica:

a + b = 5a4 + b4 = 97

}

Para resolver este sistema procedemos como sigue:

25 = (a + b)2 = a2 + 2ab + b2

y de aquı se tiene a2 + b2 = 25 − 2ab. Por otro lado

625 = (a + b)4 = a4 + b4 + 4ab(a2 + b2) + 6a2b2 = 97 + 100ab − 2a2b2,

y de aquı se tiene a2b2 − 50ab + 264 = 0. Entonces ab es una raız de z2 − 50z + 264 = 0; comolas raıces son 44 y 6, estudiamos cada uno de estos casos.

Si ab = 44, entonces a y b son las soluciones del sistemaa + b = 5ab = 44

}, luego de la ecuacion

z2 − 5z + 44 = 0; esta no tiene raıces reales. Si ab = 6, entonces a y b son raıces de la ecuacionz2 − 5z + 6 = 0, que tiene las raıces 3 y 2.

Si b = 3, entonces x = 81, y si x = 2, entonces x = 16. Estas son las unicas raıces reales de laecuacion dada.

Solucion.[Alternativa] Si llamamos a = 4√

97 − x y b = 4√

x, se verifica:

a + b = 5a4 + b4 = 97

}

Utilizamos una variable temporal t y escribimos a = 52

+ t, b = 52− t. entonces se verifica:

97 = a4 + b4 =

(5

2+ t

)4

+

(5

2− t

)4

= 2t4 + 12

(5

2

)2

t2 + 2

(5

2

)4

.

Simplificando resulta:16t4 + 600t2 − 151 = 0

El valor de t es ±12. Se tiene entonces:

b =5

2+

1

2= 3;

b =5

2− 1

2= 2.

Entonces al igualar b = 4√

x, resulta:

b = 3 ⇒ x = 81,b = 2 ⇒ x = 16,

que son las unicas dos soluciones reales de la ecuacion dada.

Problema 3. Dado el polinomio P (X) = X4 + �X3 + �X2 + �X + � , en el que cadacuadrado representa un hueco donde se colocara un coeficiente, se plantea el siguiente juegoentre dos jugadores: Alternativamente, el primer y el segundo jugador eligen un hueco vacıo ycolocan en el un entero no nulo hasta rellenar todos los cuatro huecos. Si el polinomio resultantetiene al menos dos raıces enteras gana el segundo jugador, en otro caso el ganador es el primero.

Prueba que, eligiendo la estrategia adecuada, el primer jugador siempre puede ganar.

2

Page 149: COMPENDIUM OMEFL - ToomatesXXXV Olimpiada Matemática Española Primera Fase Soluciones de la propuesta de problemas Problema 1 ¿Qué dígitos se han omitido en la siguiente multiplicación?

Solucion.

Nota: El enunciado presenta cierta ambiguedad: hay que distinguir si “dos raıces enteras”significa que estas son distintas, o por el contrario pueden ser raıces dobles.

El caso de dos raıces distintas es sencillo; basta con la primera jugada para el primer jugador.En el caso de raıces dobles tiene que hacer uso tambien de la segunda jugada.

Una posible estrategia es:

(i) El primer jugador coloca un -1 en el lugar a4, quedando el polinomio en la forma:

X4 + �X3 + �X2 + �X − 1

De esta forma el primer jugador fuerza a que las posibles raıces enteras del polinomio sean 1 y−1.(ii) El segundo jugador coloca ai �= 0 en uno de los huecos.(iii) El primer jugador coloca aj �= 0 en uno de los huecos.(iv) El segundo jugador coloca ak �= 0 en el unico hueco restante.

Observa que el polinomio tiene ahora la forma X4 + a1X3 + a2X

2 + a3X − 1 y que las posiblesraıces enteras (distintas) de este polinomio son 1 y -1, ya que el termino independiente es −1.

Cuando 1 es una raız se tiene 1 + a1 + a2 + a3 − 1 = 0, esto es, a1 + a2 + a3 = 0, y cuando −1es una raız se tiene 1 − a1 + a2 − a3 − 1 = 0, esto es −a1 + a2 − a3 = 0.

Sean cuales sean los valores de a1, a−2, a3 se tendra siempre 2a2 = 0, lo que implica que a2 = 0,y esto no esta permitido por las reglas del juego. Por lo tanto el caso de dos raıces distintasesta resuelto.

En el caso en el que las dos raıces sean iguales.

Si 1 es una raız doble, entonces se verifica a1 + a2 + a3 = 0 y 4 + 3a1 + 2a2 + a3 = 0, por lotanto se tiene:

a1 + a2 + a3 = 04 + 2a1 + 2a2 = 0

}siendo a3 = 4 + a1 y a2 = −4 − 2a1.

Si −1 es una raız doble, entonces se verifica −a1 + a2 − a3 = 0 y 4− 3a1 + 2a2 − a3 = 0, por lotanto se tiene:

−a1 + a2 − a3 = 04 − 2a1 + 2a2 = 0

}siendo a3 = −4 + a1 y a2 = −4 + 2a1.

Esta claro que si el segundo jugador hace ai = a2, entonces el primer jugador puede colocaraj = a1 de forma que −4−2a1 �= a2 y −4+2a1 �= a2, de esta forma no habra dos raıces enteras.

Por el contrario, si el primer jugador hace ai = a1, basta tomar a2 de forma que a2 �= −4 − 2a1

y a2 �= −4 + 2a1. El caso de ai = a3 se hace de la misma forma.

Problema 4. Supongamos que tenemos un tablero con dieciseis casillas dispuestas en cuatrofilas y cuatro columnas.

(a) Prueba que se pueden colocar siete fichas, nunca dos en la misma casilla, de forma que aleliminar dos filas y dos columnas cualesquiera, siempre quede alguna ficha sin eliminar.

(b) Prueba que si se colocan seis fichas, nunca dos en la misma casilla, siempre se puede eliminardos filas y dos columnas de forma que todas las fichas sean eliminadas.

3

Page 150: COMPENDIUM OMEFL - ToomatesXXXV Olimpiada Matemática Española Primera Fase Soluciones de la propuesta de problemas Problema 1 ¿Qué dígitos se han omitido en la siguiente multiplicación?

Solucion. (a). Una solucion es:

� �

� �

� �

(b). Si se tienen 6 fichas en el tablero, alguna columna tendra al menos dos fichas; eliminamosesa columna.

Quedan, como maximo 4 fichas, y exactamente tres columnas. Por el mismo procedimientopodemos ahora eliminar una columna de forma que nos queden. como maximo, dos fichas en eltablero.

El tercer y cuarto paso consiste en eliminar dos filas de forma que no queden fichas en el tablero.

Problema 5. Se considera un tetraedro regular como el de la figura. Si el punto E recorre la

arista AB. ¿Cuando el angulo CED es maximo?

A

B

C

D

E

Solucion. Supongamos que el tetraedro tiene arista de longitud 1, sea α el angulo CED y x lalongitud del segmento AE.

Si aplicamos el Teorema del coseno al triangulo AEC tenemos:

EC2 = x2 + 1 − 2x cos 60o = x2 + 1 − x.

Por simetrıa se tiene que la longitud EC es igual a la longitud ED. De nuevo por el Teoremadel coseno, ahora para el triangulo ECD, se tiene

1 = (x2 − x + 1) + (x2 − x + 1) − 2(√

x2 − x + 1)2 cos α

4

Page 151: COMPENDIUM OMEFL - ToomatesXXXV Olimpiada Matemática Española Primera Fase Soluciones de la propuesta de problemas Problema 1 ¿Qué dígitos se han omitido en la siguiente multiplicación?

1 = 2(x2 − x + 1)(1 − cos α).

Despejando se tiene:

cos α = 1 − 1

2(x2 − x + 1). (1)

Por ser la funcion cos decreciente en el primer cuadrante, par encontrar el valor maximo de αtenemos que buscar el valor de x ∈ [0, 1] que haga mınima la funcion dada en (1), o equivalen-temente que haga mınimo el denominador x2 − x + 1. Es evidente que este mınimo se alcanzapara x = 1

2.

La respuesta es: cuando el punto E es el punto medio del lado AB. Podemos calcular en estecaso el valor de α; se tiene: α = arccos 1

3.

Problema 6. Decimos que un conjunto E de numeros naturales es especial cuando al tomardos elementos cualesquiera distintos a, b ∈ E se tiene que (a − b)2 divide al producto ab.

(a) Encuentra un conjunto especial formado por tres elementos.

(b) ¿Existe un conjunto especial formado por cuatro numeros naturales que estan en progresionaritmetica?

Solucion. (a). Un conjunto especial de tres elementos es {2, 3, 4}.(b). Supongamos que {x, x + y, x + 2y, x + 3y} forman un conjunto especial.

Podemos suponer que x e y son primos relativos, pues si llamamos d = mcd{x, y} y d �= 1,tomando x′ = x/d e y′ = y/d tenemos un conjunto especial {x′, x′ + y′, x′ + 2y′, x′ + 3y′} con x′

e y′ primos relativos.

Sean pues x e y primos relativos. Por ser el conjunto especial y2 divide a x(x + y), y existe unentero k tal que x(x + y) = y2k, por lo tanto x2 = y2k − xy = y(yk − x), lo que implica quey = 1 al ser un divisor comun de x e y.

Ası pues el conjunto especial es de la forma {x, x + 1, x + 2, x + 3}. Al ser especial 4 es undivisor de x(x + 2) y de (x + 1)(x + 3), pero uno de ellos es un numero impar, lo que es unacontradiccion. Por lo tanto la suposicion de que existe un conjunto especial formado por cuatroterminos en progresion aritmetica es falsa.

5

Page 152: COMPENDIUM OMEFL - ToomatesXXXV Olimpiada Matemática Española Primera Fase Soluciones de la propuesta de problemas Problema 1 ¿Qué dígitos se han omitido en la siguiente multiplicación?

Problema 1. Supongamos que tenemos un tablero con dieciseis casillas dispuestas en cuatrofilas y cuatro columnas.

(a) Prueba que se pueden colocar siete fichas, nunca dos en la misma casilla, de forma que aleliminar dos filas y dos columnas cualesquiera, siempre quede alguna ficha sin eliminar.

(b) Prueba que si se colocan seis fichas, nunca dos en la misma casilla, siempre se puede eliminardos filas y dos columnas de forma que todas las fichas sean eliminadas.

Solucion. (a). Una solucion es:

� �

� �

� �

(b). Si se tienen 6 fichas en el tablero, alguna columna tendra al menos dos fichas; eliminamosesa columna.

Quedan, como maximo 4 fichas, y exactamente tres columnas. Por el mismo procedimientopodemos ahora eliminar una columna de forma que nos queden. como maximo, dos fichas en eltablero.

El tercer y cuarto paso consiste en eliminar dos filas de forma que no queden fichas en el tablero.

Problema 2. Se considera un tetraedro regular como el de la figura. Si el punto E recorre la

arista AB. ¿Cuando el angulo CED es maximo?

A

B

C

D

E

1

Page 153: COMPENDIUM OMEFL - ToomatesXXXV Olimpiada Matemática Española Primera Fase Soluciones de la propuesta de problemas Problema 1 ¿Qué dígitos se han omitido en la siguiente multiplicación?

Solucion. Supongamos que el tetraedro tiene arista de longitud 1, sea α el angulo CED y x lalongitud del segmento AE.

Si aplicamos el Teorema del coseno al triangulo AEC tenemos:

EC2 = x2 + 1 − 2x cos 60o = x2 + 1 − x.

Por simetrıa se tiene que la longitud EC es igual a la longitud ED. De nuevo por el Teoremadel coseno, ahora para el triangulo ECD, se tiene

1 = (x2 − x + 1) + (x2 − x + 1) − 2(√

x2 − x + 1)2 cos α

1 = 2(x2 − x + 1)(1 − cos α).

Despejando se tiene:

cos α = 1 − 1

2(x2 − x + 1). (1)

Por ser la funcion cos decreciente en el primer cuadrante, par encontrar el valor maximo de αtenemos que buscar el valor de x ∈ [0, 1] que haga mınima la funcion dada en (1), o equivalen-temente que haga mınimo el denominador x2 − x + 1. Es evidente que este mınimo se alcanzapara x = 1

2.

La respuesta es: cuando el punto E es el punto medio del lado AB. Podemos calcular en estecaso el valor de α; se tiene: α = arccos 1

3.

Problema 3. Decimos que un conjunto E de numeros naturales es especial cuando al tomardos elementos cualesquiera distintos a, b ∈ E se tiene que (a − b)2 divide al producto ab.

(a) Encuentra un conjunto especial formado por tres elementos.

(b) ¿Existe un conjunto especial formado por cuatro numeros naturales que estan en progresionaritmetica?

Solucion. (a). Un conjunto especial de tres elementos es {2, 3, 4}.(b). Supongamos que {x, x + y, x + 2y, x + 3y} forman un conjunto especial.

Podemos suponer que x e y son primos relativos, pues si llamamos d = mcd{x, y} y d �= 1,tomando x′ = x/d e y′ = y/d tenemos un conjunto especial {x′, x′ + y′, x′ + 2y′, x′ + 3y′} con x′

e y′ primos relativos.

Sean pues x e y primos relativos. Por ser el conjunto especial y2 divide a x(x + y), y existe unentero k tal que x(x + y) = y2k, por lo tanto x2 = y2k − xy = y(yk − x), lo que implica quey = 1 al ser un divisor comun de x e y.

Ası pues el conjunto especial es de la forma {x, x + 1, x + 2, x + 3}. Al ser especial 4 es undivisor de x(x + 2) y de (x + 1)(x + 3), pero uno de ellos es un numero impar, lo que es unacontradiccion. Por lo tanto la suposicion de que existe un conjunto especial formado por cuatroterminos en progresion aritmetica es falsa.

Problema 4. Un jardinero tiene que plantar en una fila a lo largo de un camino tres robles,cuatro encinas y cinco hayas. Planta los arboles al azar; siendo la probabilidad de plantar unarbol u otro la misma.

Halla la probabilidad de que, una vez plantados todos los arboles, no haya dos hayas consecutivas.

2

Page 154: COMPENDIUM OMEFL - ToomatesXXXV Olimpiada Matemática Española Primera Fase Soluciones de la propuesta de problemas Problema 1 ¿Qué dígitos se han omitido en la siguiente multiplicación?

Solucion. Una forma de hacer una disposicion en la que no haya dos hayas consecutivas puedeser imaginar plantados todos los robles y todas las encinas y colocar las cinco haya entre loshuecos y los extremos; tenemos pues ocho huecos para colocar las hayas.

El problema puede plantearse con dos supuestos diferentes, aunque el resultado final es elmismo.

Supuesto 1. No es posible distinguir los robles entre sı, las encinas entre sı y las hayas entre sı.El numero total de disposiciones es igual a las permutaciones con repeticion de 12 elementos

de los que 3, 4 y 5 son iguales entre si; esto es12!

3! 4! 5!. El numero de disposiciones favorables

es igual a las combinaciones de 8 elementos tomados de 5 en 5 multiplicado por el numero depermutaciones con repeticion de 7 elementos de los que 3 y 4 son iguales entre sı. La probabilidades:

p1 =8!

5! 3!7!

3! 4!12!

3! 4! 5!

=7

11 × 9=

7

99.

Supuesto 2. Es posibles distinguir entre los robles, encinas y hayas. En este caso el numero totalde disposiciones es 12!, y el de disposiciones favorables es el producto de 7!, la forma de colocarlos robles y las encinas, por 8 × 7 × 6 × 5 × 4, la forma de colocar las cinco hayas de fora queno haya dos hayas consecutivas. La probabilidad es:

p2 =(8 × 7 × 6 × 5 × 4) (7!)

12!=

7

11 × 9=

7

99.

Problema 5. Calcula las soluciones reales de la ecuacion:

3√

1729 − X +3√

X = 19.

Solucion. Si llamamos a = 3√

1729 − x y b = 3√

x, se tiene que a y b son raıces del sistemaa + b = 19a3 + b3 = 1729

}. Para resolver este sistema procedemos como sigue:

193 = (a + b)3 = a3 + b3 + 3ab(a + b) = 1729 + 3 × 19ab,

de aquı se tiene ab = 90. Luego a y b son las raıces del sistemaa + b = 19ab = 90

}, y por tanto de

la ecuacion z2 − 19z + 90 = 0. Como las raıces de esta son 10 y 9, resulta que x = 103 = 1000y x = 93 = 729 son las unicas raıces reales de la ecuacion dada.

Problema 6. Averigua que numeros de cuatro cifras significativas, abcd (con a �= 0), son

iguales a ab2+ cd

2 − cd.

Nota: La notacion ab representa, en este problema, el numero que tiene a decenas y b unidades;en este caso se tiene que a, b ∈ {0, 1, 2, 3, 4, 5, 6, 7, 8, 9}.

Solucion. Tenemos abcd = ab2+ cd

2 − cd, por lo tanto

ab00 = ab2+ cd

2 − 2cd = ab2+ (cd − 1)2 − 1,

3

Page 155: COMPENDIUM OMEFL - ToomatesXXXV Olimpiada Matemática Española Primera Fase Soluciones de la propuesta de problemas Problema 1 ¿Qué dígitos se han omitido en la siguiente multiplicación?

y de aquı se tiene:

ab00 + 1 = ab2+ (cd − 1)2. (2)

Vamos a reordenar los terminos de esta expresion para determinar los valores de a, b, c y d.Escribimos

1 = ab2 − 100ab + (cd − 1)2 = (ab − 50)2 + (cd − 1)2 − 502. (3)

y ordenando los terminos

2501 = 502 + 1 = ab2 − 100ab + (cd − 1)2 = (ab − 50)2 + (cd − 1)2. (4)

Tenemos que descomponer 2501 como suma de dos cuadrados; las unica descomposiciones son:

2501 = 502 + 1 = 492 + 102.

Analizar los diferentes casos para una descomposicion del tipo 2501 = x2 + y2,utilizando lascifras de las unidades; modulo 10 estas deben sumar 1. Se tienen las siguientes tablas:

Unidades de n: 0 1 2 3 4 5 6 7 8 9Unidades de n2: 0 1 4 9 6 5 6 9 4 1

Tenemos entonces las siguientes posibilidades par comenzar el analisis:

Unidades de x2 Unidades de y2 Unidades de x Unidades de y0 1 0 10 1 0 91 0 1 01 0 9 05 6 5 45 6 5 66 5 4 56 5 6 5

Estudiamos ahora los posibles valores para ab − 50, siendo (ab − 50)2 = 502, 12, 492 y 102.

solucion:

ab − 50 = ab 2501 − (ab − 50)2 cd − 1 cd abcd

50 100 NO−50 0 NO

1 51 2500 50 51 5151−1 49 2500 50 51 495149 99 100 10 11 9911

−49 1 NO10 60 2401 49 50 6050

−10 40 2401 49 50 4050

4

Page 156: COMPENDIUM OMEFL - ToomatesXXXV Olimpiada Matemática Española Primera Fase Soluciones de la propuesta de problemas Problema 1 ¿Qué dígitos se han omitido en la siguiente multiplicación?

XLVII Olimpiada Matematica Espanola

Primera Fase

Primera sesion

Viernes manana, 21 de enero de 2011

OlimpiadaMatemáticaEspañola RSME

1. Los anos recientes se han podido expresar como sumas, restas y multiplica-ciones de numeros con un mismo y unico dıgito; por ejemplo:

2009 = 7×7×7×7−7×7×7−7×7 , 2010 = 66×6×6−66×6+6×6−6

¿Se puede hacer lo mismo con el 2011, sin repetir jamas sumandos iguales?Por ejemplo, no es admisible 2011 = 1 + 1 + 1 + . . ..

2. Dos semirrectas tienen su comun origen en el punto O. Se considera unacircunferencia C1 tangente a ambas semirrectas, cuyo centro esta situadoa distancia d1 de O, y cuyo radio es r1. Se construyen sucesivamente lascircunferencias Cn, de modo que Cn es tangente a las semirrectas, tangenteexterior a Cn−1 y tal que la distancia de su centro a O, dn, es menor quedn−1, para n > 1. Halla la suma de las areas de los cırculos limitados porlas circunferencias Cn, para todo n, en funcion de r1 y d1.

3. Saber cual es la ultima cifra de 20092011 es muy facil, pero ¿cuantos cerospreceden a esa ultima cifra?

No esta permitido el uso de calculadoras.Cada problema se puntua sobre 7 puntos.

El tiempo de cada sesion es de 3 horas y media.

Page 157: COMPENDIUM OMEFL - ToomatesXXXV Olimpiada Matemática Española Primera Fase Soluciones de la propuesta de problemas Problema 1 ¿Qué dígitos se han omitido en la siguiente multiplicación?

XLVII Olimpiada Matematica Espanola

Primera Fase

Segunda sesion

Viernes tarde, 21 de enero de 2011

OlimpiadaMatemáticaEspañola RSME

4. Calcula todos los numeros enteros a, b y c tales que a2 = 2 b2 + 3 c2.

5. Dos esferas de radio r son tangentes exteriores. Otras tres esferas de radioR son tangentes exteriores entre sı, dos a dos. Cada una de estas tres esferases, ademas, tangente exterior a las dos primeras. Encuentra la relacion entreR y r.

6. Denotamos por N = {1, 2, 3, . . .} el conjunto de numeros naturales excluidoel cero y por N

∗ = {0, 1, 2, 3, . . .} el conjunto de numeros naturales incluidoel cero. Encuentra todas las funciones f : N → N

∗ que sean crecientes, esdecir f(n) ≥ f(m) si n > m, y tales que f(nm) = f(n) + f(m), para todon, m ∈ N.

No esta permitido el uso de calculadoras.Cada problema se puntua sobre 7 puntos.

El tiempo de cada sesion es de 3 horas y media.

Page 158: COMPENDIUM OMEFL - ToomatesXXXV Olimpiada Matemática Española Primera Fase Soluciones de la propuesta de problemas Problema 1 ¿Qué dígitos se han omitido en la siguiente multiplicación?

XLVII Olimpiada Matematica Espanola

Primera Fase

Primera sesion

Viernes tarde, 21 de enero de 2011

OlimpiadaMatemáticaEspañola RSME

1. Se considera el polinomio de segundo grado p(x) = ax2+bx+c, (a �= 0), cuyasraıces x1 y x2 se suponen distintas. Justifica que para que p

(x3

1

)= p

(x3

2

)es suficiente que a2 + 3ac − b2 = 0. ¿Es tambien necesaria esta condicion?

2. Denotemos N∗ = {0, 1, 2, 3, . . .}. Encuentra todas las funciones crecientes

f : N → N∗ con las siguientes propiedades:

i) f(2) = 2,ii) f(nm) = f(n) + f(m), para todo par n, m ∈ N.

3. Un cuadrado C se recubre completamente con un numero entero de cuadra-dos de lado unidad, sin solapamientos. Si uno coloca dentro de C y sinsolapamientos tantos cuadrados como sea posible de area 2, con los ladosparalelos a los lados de C, se pueden cubrir las ocho novenas partes del areadel cuadrado. Determina todas las posibles dimensiones de tales cuadrados.

No esta permitido el uso de calculadoras.Cada problema se puntua sobre 7 puntos.

El tiempo de cada sesion es de 3 horas y media.

Page 159: COMPENDIUM OMEFL - ToomatesXXXV Olimpiada Matemática Española Primera Fase Soluciones de la propuesta de problemas Problema 1 ¿Qué dígitos se han omitido en la siguiente multiplicación?

XLVII Olimpiada Matematica Espanola

Primera Fase

Segunda sesion

Sabado manana, 22 de enero de 2011

OlimpiadaMatemáticaEspañola RSME

4. Consideremos un alfabeto de n letras, con el que formaremos palabras. Dire-mos que una palabra contiene un palındromo si un trozo de esa palabra, demas de una letra, se lee igual al derecho que al reves. Por ejemplo, la palabraOLIMPIADAS contiene el palındromo ADA. Siendo k un entero mayor que2, determina cuantas palabras de longitud k se pueden formar, con nuestroalfabeto de n letras, que no contengan ningun palındromo de longitud impar.

5. Se ordenan los numeros naturales en forma de tabla triangular, es decir:

12 3 4

5 6 7 8 910 11 12 13 14 15 16

· · · · · · · · ·Diremos que la posicion de un numero N en la tabla viene dada por dos “co-ordenadas”: el primer numero de su fila y el primer numero de su columna.Por ejemplo, si N = 15, su posicion es (10, 9). Cuando un numero N , en laposicion (n, m), verifica que N = n+m diremos que N esta bien colocado enla tabla; ası 12 y 14 estan bien colocados en la tabla y 15 no lo esta. ¿Esta22011 bien colocado?

6. En un triangulo llamaremos O al circuncentro, I al incentro y r al radiode la circunferencia inscrita. Si la mediatriz del segmento OI corta a lacircunferencia circunscrita en L, y LI vuelve a cortarla en M , demuestraque IM = 2r.

No esta permitido el uso de calculadoras.Cada problema se puntua sobre 7 puntos.

El tiempo de cada sesion es de 3 horas y media.

Page 160: COMPENDIUM OMEFL - ToomatesXXXV Olimpiada Matemática Española Primera Fase Soluciones de la propuesta de problemas Problema 1 ¿Qué dígitos se han omitido en la siguiente multiplicación?

XLVII Olimpiada Matematica Espanola

Primera Fase

Primera sesion

Sabado manana, 22 de enero de 2011

OlimpiadaMatemáticaEspañola RSME

1. Sean n1 y n2 dos numeros naturales. Demuestra que la suma√

n1 + 3√

n2 esun numero entero o un numero irracional.

2. Demuestra que en un triangulo se verifica: si r es una recta que pasa por subaricentro y no pasa por ningun vertice, la suma de las distancias a dicharecta de los vertices que quedan en un mismo semiplano es igual a la distanciadel tercer vertice a dicha recta.

3. En un hexagono regular de lado unidad se situan 19 puntos. Demuestra quehay al menos un par de ellos separados por una distancia no mayor que

√3/3.

No esta permitido el uso de calculadoras.Cada problema se puntua sobre 7 puntos.

El tiempo de cada sesion es de 3 horas y media.

Page 161: COMPENDIUM OMEFL - ToomatesXXXV Olimpiada Matemática Española Primera Fase Soluciones de la propuesta de problemas Problema 1 ¿Qué dígitos se han omitido en la siguiente multiplicación?

XLVII Olimpiada Matematica Espanola

Primera Fase

Segunda sesion

Sabado tarde, 22 de enero de 2011

OlimpiadaMatemáticaEspañola RSME

4. Halla todas las ternas de numeros enteros positivos a ≤ b ≤ c primitivas (esdecir, que no tengan ningun factor primo comun) tales que cada uno de ellosdivide a la suma de los otros dos.

5. Halla todas las ternas (x, y, z) de numeros reales que son soluciones del sis-tema de ecuaciones

3 · 2y − 1 = 2x + 2−x ,

3 · 2z − 1 = 2y + 2−y ,

3 · 2x − 1 = 2z + 2−z .

⎫⎪⎬⎪⎭

6. En una reunion entre cuatro paıses de la ONU, digamos A, B, C y D, el paısA tiene el doble de representantes que el B, el triple que el C, y el cuadrupleque el D. Se pretende distribuir a los representantes en mesas con el mismonumero de personas en cada una. Solo hay una condicion: en cada mesa,cualquiera de los paises debe estar en inferioridad numerica respecto de losotros tres juntos. ¿Cuantos representantes debe haber en cada mesa, comomınimo?

No esta permitido el uso de calculadoras.Cada problema se puntua sobre 7 puntos.

El tiempo de cada sesion es de 3 horas y media.

Page 162: COMPENDIUM OMEFL - ToomatesXXXV Olimpiada Matemática Española Primera Fase Soluciones de la propuesta de problemas Problema 1 ¿Qué dígitos se han omitido en la siguiente multiplicación?

XLVII Olimpiada Matematica Espanola

Primera Fase

Soluciones a los problemas propuestos

OlimpiadaMatemáticaEspañola RSME

Problema 1.1. Los anos recientes se han podido expresar como sumas, restas y multiplicaciones de numeroscon un mismo y unico dıgito; por ejemplo:

2009 = 7 × 7 × 7 × 7 − 7 × 7 × 7 − 7 × 7, 2010 = 66 × 6 × 6 − 66 × 6 + 6 × 6 − 6

¿Se puede hacer lo mismo con el 2011, sin repetir jamas sumandos iguales? Por ejemplo, no es admisible2011 = 1 + 1 + 1 + ....

Solucion Problema 1.1 Si 2011 fuera expresable como sumas, restas y multiplicaciones de numeros con elmismo dıgito a, como cada uno de estos numeros es divisible por a, se tiene que a es divisor de 2011. Ahorabien, 2011 es un numero primo, por tanto a = 1.

Es sencillo observar que

1000 = 1111− 1112 = 111 − 11 × 11 + 11 + 1

Multiplicando estas dos igualdades se tiene:

2000 =1111× 111 − 1111 × 11 × 11 + 1111× 11 + 1111− 111 × 111++111× 11 × 11 − 111 × 11 − 111

Compruebese que todos los sumandos son distintos entre sı y distintos a 11. Por tanto, sumando 11 al numeroanterior se tiene una solucion.

Existen infinidad de maneras distintas:

2011 = 1111× 1111 − 111 × 11111− 111 × 111 + 111 × 11 × 11 − 111 + 11 + 1

o bien2011 = 1111× 1111 − 111 × 11111 + 1111− 111 + 11

Problema 1.2. Dos semirrectas tienen su comun origen en el punto O. Se considera una circunferencia C1

tangente a ambas semirrectas, cuyo centro esta situado a distancia d1 de O, y cuyo radio es r1. Se construyensucesivamente las circunferencias Cn, de modo que Cn es tangente a las semirrectas, tangente exterior a Cn−1

y tal que la distancia de su centro a O, dn, es menor que dn−1, para n > 1. Halla la suma de las areas de loscırculos limitados por las circunferencias Cn, para todo n, en funcion de r1 y d1.

Solucion Problema 1.2 Es claro de la figura que, por el Teorema de Thales,rn

dn=

r1

d1para todo n. Llamaremos

Page 163: COMPENDIUM OMEFL - ToomatesXXXV Olimpiada Matemática Española Primera Fase Soluciones de la propuesta de problemas Problema 1 ¿Qué dígitos se han omitido en la siguiente multiplicación?

a este valor α. Ademas, se tiene que:

rn

rn+1=

dn

dn+1=

dn+1 + rn+1 + rn

dn+1= 1 + α +

rn

dn+1=

1 + α +rn

rn+1

rn+1

dn+1= 1 + α +

rn

rn+1α.

Despejando se tienern

rn+1=

1 + α

1 − α, que es constante, luego los radios de las circunferencias forman una progre-

sion geometrica de razon

r =1 − α

1 + α=

1 − r1/d1

1 + r1/d1=

d1 − r1

d1 + r1.

La suma de areas buscada es

S = π

∞∑n=1

r2n = π

r21

1 − (d1−r1d1+r1

)2=

π

4r1 (d1 + r1)2

d1.

Problema 1.3. Saber cual es la ultima cifra de 20092011 es muy facil, pero ¿cuantos ceros preceden a esa ultimacifra?

Solucion Problema 1.3 Si n ≥ 1,

2009n = (2000 + 9)n = 9n + 2000k

Por tanto las 3 ultimas cifras de 2009n coinciden con las de 9n. Por el desarrollo del binomio de Newton:

92011 = (10 − 1)2011 = (−1)2011 +(

20111

)(−1)2010 · 10+

+(

20112

)(−1)2009 · 102 + K · 103 = −1 + 20110− 2011 · 1005 · 100 + K · 103

= −202085391 + K · 103 = 609 + K ′ · 103

Luego la respuesta es que 9 es la ultima cifra y le precede un unico cero.

Problema 1.4. Calcula todos los numeros enteros a, b y c tales que a2 = 2 b2 + 3 c2.

Solucion Problema 1.4 Sea (a, b, c) una solucion distinta de (0, 0, 0), con |a| + |b| + |c| mınimo. Tomando laigualdad modulo 3, tenemos a2 = 2b2 modulo 3. Como a2 y b2 solo pueden ser congruentes con 1 o 0, se deduce

Page 164: COMPENDIUM OMEFL - ToomatesXXXV Olimpiada Matemática Española Primera Fase Soluciones de la propuesta de problemas Problema 1 ¿Qué dígitos se han omitido en la siguiente multiplicación?

que a y b son multiplos de 3. Por tanto, 3c2 es multiplo de 9, ası que c tambien es multiplo de 3. Pero entonces,(a/3, b/3, c/3) serıa otra solucion con |a/3|+ |b/3|+ |c/3| < |a|+ |b|+ |c|, lo que contradice la hipotesis supuesta.

Problema 1.5. Dos esferas de radio r son tangentes exteriores. Otras tres esferas de radio R son tangentesexteriores entre sı, dos a dos. Cada una de estas tres esferas es, ademas, tangente exterior a las dos primeras.Encuentra la relacion entre R y r.

Solucion Problema 1.5 Los centros de las tres esferas de radio R, O1, O2 y O3, son los vertices de un triangulo

equilatero de lado 2R. El punto de tangencia, T , de las dos esferas de radio r es el centro de ese triangulo y, portanto, dista de los vertices dos tercios de la altura. La altura del triangulo es h = 2R

√3

2 = R√

3 y dos tercios deh es 2R√

3.

Si llamamos Q1, Q2 a los centros de las circunferencias de radio r, el triangulo O1TQ1 es rectangulo en T ysus lados son: 2R√

3, r y R + r. Aplicando el teorema de Pitagoras, se tiene:

(2R√

3

)2

+ r2 = (R + r)2

y simplificando resulta: R = 6r.

Problema 1.6. Denotamos por N = {1, 2, 3, . . .} el conjunto de numeros naturales excluido el cero y porN

∗ = {0, 1, 2, 3, . . .} el conjunto de numeros naturales incluido el cero. Encontrar todas las funciones f : N → N∗

que sean crecientes, es decir f(n) ≥ f(m) si n > m, y tales que f(nm) = f(n) + f(m), para todo n, m ∈ N.

Solucion Problema 1.6

La funcion nula: f(n) = 0, para todo n ∈ N verifica evidentemente lo anterior.

Sea f una funcion no nula verificando las condiciones del enunciado. Entonces

1. f no es constante, ni esta acotada. En efecto, si f(a) �= 0 entonces f(an) = nf(a) > f(a) para cadan.

2. f no es estrictamente creciente:

• Si f(2) = f(3) ya esta.• Si f(2) = a < b = f(3), entonces 2b �= 3a, pero f(2b) = ab = f(3a).

Page 165: COMPENDIUM OMEFL - ToomatesXXXV Olimpiada Matemática Española Primera Fase Soluciones de la propuesta de problemas Problema 1 ¿Qué dígitos se han omitido en la siguiente multiplicación?

De los dos puntos anteriores se deduce que es posible encontrar un numero natural m tal que k = f(m) =f(m + 1) < f(m + 2). Entonces

f [(m + 1)2] = 2k < f [m(m + 2)]

Sin embargo m(m + 2) < (m + 1)2, contradiciendo el caracter creciente de f .

En consecuencia la unica funcion que verifica las condiciones del enunciado es la funcion nula.

Page 166: COMPENDIUM OMEFL - ToomatesXXXV Olimpiada Matemática Española Primera Fase Soluciones de la propuesta de problemas Problema 1 ¿Qué dígitos se han omitido en la siguiente multiplicación?

XLVII Olimpiada Matematica Espanola

Primera Fase

Soluciones a los problemas propuestos

OlimpiadaMatemáticaEspañola RSME

Problema 2.1. Se considera el polinomio de segundo grado p(x) = ax2 + bx + c, (a �= 0), cuyas raıces x1 yx2 se suponen distintas. Justifica que para que p(x3

1) = p(x32) es suficiente que a2 + 3ac − b2 = 0. ¿Es tambien

necesaria esta condicion?

Solucion Problema 2.1

p(x31) − p(x3

2) = a(x61 − x6

2) + b(x31 − x3

2) = (x31 − x3

2)[a(x31 + x3

2) + b]

x31 + x3

2 = (x1 + x2)3 − 3x1x2(x1 + x2) =(− b

a

)3

− 3c

a

(− b

a

)= − b3

a3+

3bc

a2

Por tanto

p(x31) − p(x3

2) = (x31 − x3

2)[a

(− b3

a3+

3bc

a2

)+ b

]=

b

a2(x3

1 − x32)(−b2 + 3ac + a2)

Para que esta diferencia se anule, es suficiente que −b2 + 3ac + a2 = 0.La condicion no es necesaria. El producto se anula al anularse cualquier factor; el primer parentesis no se

anula, pero b puede anularse. De modo que otra condicion suficiente para que p(x31) = p(x3

2) es que b = 0.

Problema 2.2. Denotemos N∗ = {0, 1, 2, 3, ...}. Encuentra todas las funciones crecientes f : N → N∗ con lassiguientes propiedades:

i) f(2) = 2,

ii) f(nm) = f(n) + f(m) para todo par n, m ∈ N.

Solucion Problema 2.2 De las propiedades se deduce:

1. Haciendo m = 1, sigue de ii) f(1) = 0.

2. Por induccion finita sobre ii) sigue que f(nk) = kf(n), ∀n, k ∈ N.

Veamos si puede construirse una funcion creciente con estas propiedades. Ya que f(4) = f(22) = 2 f(2) = 4,resulta que los unicos posibles valores de f(3) si f es creciente, son f(3) = 2, f(3) = 3, f(3) = 4.

1. Si f(3) = 2, nos encontramos con que 23 < 32, pero f(23) = 6 > f(32) = 4.

2. Si f(3) = 3, nos encontramos con que 211 < 37, pero f(211) = 22 > f(37) = 21.

3. Si f(3) = 4, nos encontramos con que 33 < 25, pero f(33) = 12 > f(25) = 10.

Page 167: COMPENDIUM OMEFL - ToomatesXXXV Olimpiada Matemática Española Primera Fase Soluciones de la propuesta de problemas Problema 1 ¿Qué dígitos se han omitido en la siguiente multiplicación?

Ası pues, no hay ninguna funcion creciente con estas propiedades.

Problema 2.3. Un cuadrado C se recubre completamente con un numero entero de cuadrados de lado unidad,sin solapamientos. Si uno coloca dentro de C y sin solapamientos tantos cuadrados como sea posible de area2, con los lados paralelos a los lados de C, se puede cubrir las ocho novenas partes del area del cuadrado.Determina todas las posibles dimensiones de tales cuadrados.

Solucion Problema 2.3 Sea l el lado del cuadrado y n el numero maximo de cuadrados de area 2 que cabenen cada lado del cuadrado. l y n son enteros. Las condiciones del problema son que

{2n2 = 8

9 l2

l2 < 2(n + 1)2

Poniendo n = 2n′ y l = 3l′ (que tienen que ser enteros, por la primera ecuacion anterior), resulta sustituyendoque {

n′ = l′

n′2 − 8n′ − 2 < 0

de modo que 1 ≤ n′ ≤ 8. Las posibles soluciones son, pues,⎧⎪⎪⎪⎪⎪⎪⎪⎪⎪⎪⎨⎪⎪⎪⎪⎪⎪⎪⎪⎪⎪⎩

n′ = l′ = 1 n = 2 l = 3n′ = l′ = 2 n = 4 l = 6n′ = l′ = 3 n = 6 l = 9n′ = l′ = 4 n = 8 l = 12n′ = l′ = 5 n = 10 l = 15n′ = l′ = 6 n = 12 l = 18n′ = l′ = 7 n = 14 l = 21n′ = l′ = 8 n = 16 l = 24

Problema 2.4. Consideremos un alfabeto de n letras, con el que formaremos palabras. Diremos que una palabracontiene un palındromo si un trozo de esa palabra, de mas de una letra, se lee igual al derecho que al reves. Porejemplo, la palabra OLIMPIADAS contiene el palındromo ADA. Siendo k un entero mayor que 2, determinacuantas palabras de longitud k se pueden formar, con nuestro alfabeto de n letras, que no contengan ningunpalındromo de longitud impar.

Solucion Problema 2.4 Observemos que una palabra contiene un palındromo de longitud impar si y solosi contiene un palındromo de longitud 3. Por tanto, solo hay que contar las palabras que no contengan unpalındromo de longitud 3.

Podemos enumerar todas las palabras pedidas, de la siguiente manera: para la primera letra tenemos nposibilidades. Para la segunda letra tambien tenemos n posibilidades. La tercera letra puede ser cualquieramenos la letra que esta en la posicion 1. Por tanto, para la tercera letra tenemos n− 1 posibilidades. La cuartaletra puede ser cualquiera menos la que esta en la posicion 2, por lo que tamiben tenemos n − 1 posibilidades.Ası llegaremos hasta la k-esima letra, que puede ser cualquiera menos la que esta en la posicion k − 2, y portanto tambien hay n − 1 posibilidades. Por tanto, en total hay n2(n − 1)k−2 palabras que no contienen unpalındromo de longitud impar.

Problema 2.5. Se ordenan los numeros naturales en forma de tabla triangular, es decir:

12 3 4

5 6 7 8 910 11 12 13 14 15 16

Page 168: COMPENDIUM OMEFL - ToomatesXXXV Olimpiada Matemática Española Primera Fase Soluciones de la propuesta de problemas Problema 1 ¿Qué dígitos se han omitido en la siguiente multiplicación?

· · · · · · · · ·Diremos que la posicion de un numero N en la tabla viene dada por dos “coordenadas”: el primer numero

de su fila y el primer numero de su columna. Por ejemplo, si N = 15, su posicion es (10, 9). Cuando un numeroN , en la posicion (n, m), verifica que N = n+m diremos que N esta bien colocado en la tabla; ası 12 y 14 estanbien colocados en la tabla y 15 no lo esta. ¿Esta 22011 bien colocado?

Solucion Problema 2.5En cada fila hay dos numeros mas que en la anterior: en la primera fila hay 1 numero, en la segunda fila

hay 3, en la tercera fila hay 5,ası sucesivamente, en la n-esima fila hay 2n − 1 elementos. Por tanto, el ultimoelemento de la fila n-esima es: 1 + 3 + 5 + · + (2n − 1) = n2, por la suma de los elementos de una progresionaritmetica. El primer elemento de la fila (n + 1)-esima es n2 + 1 y el ultimo (n + 1)2.

Supongamos que N esta bien colocado.

Si N este en la mitad derecha de la tabla triangular, incluida la “altura”, su posicion es (m2 +1, n2), luegoN = n2 + m2 + 1. Tomando restos modulo 4, como n2 ≡ 0, 1 (mod 4), m2 ≡ 0, 1 (mod 4), se tiene queN ≡ 1, 2, 3 (mod 4). Como 22011 ≡ 0 (mod 4), se llega a una contradiccion.

Si N esta en la mitad izquierda de la tabla, excluida la “altura”, su posicion es (m2 + 1, n2 + 1), luegoN = n2 + m2 + 2. Razonando como antes, solo en los casos de m y n impares puede conseguirse quem2 ≡ 1, n2 ≡ 1 y ası N ≡ 0. En ese caso m = 2p + 1, n = 2q + 1, luego

N = (2p + 1)2 + (2q + 1)2 + 2 = 4(p2 + p + q2 + q + 1)

Si N = 22011, se tiene 22009 = p2 + p + q2 + q + 1. Como p2 + p y q2 + q son siempre numeros pares,encontramos una contradiccion.

Problema 2.6.En un triangulo llamaremos O al circuncentro, I al incentro y r al radio de la circunferencia inscrita. Si la

mediatriz del segmento OI corta a la circunferencia circunscrita en L, y LI vuelve a cortarla en M , demuestraque IM = 2r.

Solucion Problema 2.6

Por el Teorema de Euler, (OI)2 = R2 − 2rR. Sean T y Q los puntos de corte de la recta OI con lacircunferencia circunscrita. Entonces tenemos

IL · IM = IT · IQ.

Por simetrıa, IL = OL = R. Por otra parte, IT = OI + OT = OI + R, y tambien tenemos IQ = OQ − OI =R − OI. Por tanto, sustituyendo en la ecuacion anterior, se obtiene:

R · IM = (R + OI)(R − OI) = R2 − (OI)2 = 2rR,

Page 169: COMPENDIUM OMEFL - ToomatesXXXV Olimpiada Matemática Española Primera Fase Soluciones de la propuesta de problemas Problema 1 ¿Qué dígitos se han omitido en la siguiente multiplicación?

de donde IM = 2r.

Otra solucion del Problema 2.6. Llamemos α, β y γ los angulos del triangulo ABC.

Sea V el otro punto de corte de la recta AI con la circunferencia circunscrita. Considerando las cuerdas AVy LM , que se cortan en I, se tiene AI · IV = LI · IM . Como LI = LO = R por simetrıa, esto significa queAI · IV = IM · R. Por tanto, probar que IM = 2r es equivalente a probar que AI · IV = 2rR.

Tracemos el triangulo AIX , rectangulo en X , donde el angulo en A es α/2. Sea ahora W el punto diame-tralmente opuesto a V en la circunferencia circunscrita. Observemos que el triangulo WV C es rectangulo (alser WV un diametro de la circunferencia circunscrita, que contiene a C). Ademas al ser angulos inscritos quedeterminan la misma cuerda, se tiene ∠CWV = ∠CAV = α/2.

Por tanto, los triangulos rectangulos AIX y WV C son semejantes. Esto implica que WV/V C = AI/IX ,esto es 2R/V C = AI/r, de donde 2rR = AI · V C. Por tanto, debemos probar que AI · IV = AI ·V C. Es decir,debemos probar que IV = V C.

Para ello consideremos el triangulo CV I. Su angulo en C es igual a γ/2+∠V CB = γ/2+∠V AB = γ/2+α/2.Por otra parte, su angulo en I es igual a ∠CIV = 180◦−∠CIA = 180◦− (180◦−α/2− γ/2) = α/2+ γ/2. Estetriangulo es pues isosceles, con lo que se tiene V C = V I, como querıamos demostrar.

Page 170: COMPENDIUM OMEFL - ToomatesXXXV Olimpiada Matemática Española Primera Fase Soluciones de la propuesta de problemas Problema 1 ¿Qué dígitos se han omitido en la siguiente multiplicación?

XLVII Olimpiada Matematica Espanola

Primera Fase

Soluciones a los problemas propuestos

OlimpiadaMatemáticaEspañola RSME

Problema 3.1. Sean n1, n2 dos numeros naturales. Demuestra que la suma√

n1 + 3√

n2 es un numero enteroo un numero irracional.

Solucion Problema 3.1 Si se pone x =√

n1 + 3√

n2, se obtiene

x −√n1 = 3

√n2 =⇒ n2 = (x −√

n1)3 =⇒ (3x2 + n1)√

n1 = x3 + 3n1x − n2 =⇒

(3x2 + n1)2 n1 = (x3 + 3n1x − n2)2 =⇒ x6 − 3n1x4 − 2n2x

3 + 3n21x

2 − 6n1n2x + n22 − n3

1 = 0

Esta ecuacion tiene solo soluciones enteras o irracionales por ser 1 el coeficiente principal.

Problema 3.2. Demuestra que en un triangulo se verifica: si r es una recta que pasa por su baricentro y no pasapor ningun vertice, la suma de las distancias a dicha recta de los vertices que quedan en un mismo semiplanoes igual a la distancia del tercer vertice a dicha recta.

Solucion Problema 3.2

El triangulo GMM ′ es semejante a GAA′ con razon de semejanza 2 (pues AG = 2GM). Por tanto, AA′ =2MM ′.

Por otro lado, MM ′ es la paralela media del trapecio BB′C′C, de donde MM ′ = (BB′ + CC′)/2.En consecuencia: AA′ = 2MM ′ = BB′ + CC′.

Problema 3.3. En un hexagono regular de lado unidad se situan 19 puntos. Demuestra que hay al menos unpar de ellos separados por una distancia no mayor que

√3/3.

Page 171: COMPENDIUM OMEFL - ToomatesXXXV Olimpiada Matemática Española Primera Fase Soluciones de la propuesta de problemas Problema 1 ¿Qué dígitos se han omitido en la siguiente multiplicación?

Solucion Problema 3.3 Dividamos el hexagono regular en 6 triangulos equilateros iguales. Cada uno de ellos,si trazamos sus alturas, quedara dividido en 6 triangulos rectangulos. Uniendo estos triangulos rectangulos, dosa dos, por sus hipotenusas, habremos dividido el hexagono original en 18 regiones iguales.

Como tenemos 19 puntos, en alguna de estas regiones debe haber al menos 2 puntos. Para ver que estosdos puntos estan como mucho a distancia

√3/3, solo hay que probar que dicha region esta inscrita en una

circunferencia de diametro√

3/3.Pero la circunferencia circunscrita a un triangulo rectangulo es la que tiene como diametro la hipotenusa, por

tanto, nuestra region esta inscrita en una circunferencia de diametro d, donde d es la hipotenusa de cualquierade los dos triangulos rectangulos que la componen. Solo hay que demostrar que d ≤ √

3/3. Para ello bastaobservar que la altura del triangulo equilatero de lado 1 es, por el teorema de Pitagoras,

√1 − 1/4 =

√3/2, y

de aquı se tiene, por como divide el baricentro a una mediana,√

3/2 = d + d/2. De donde d =√

3/3.

Problema 3.4. Halla todas las ternas de numeros enteros positivos a ≤ b ≤ c primitivas (es decir, que notengan ningun factor primo comun) tales que cada uno de ellos divide a la suma de los otros dos.

Solucion Problema 3.4Supongamos que a = b. Como a y b no tienen factores en comun, debe ser a = b = 1. Como c divide a

a + b = 2, esto da lugar a las ternas (1, 1, 1) y (1, 1, 2).Supongamos ahora que a < b. Como c divide a a + b < c + c = 2c, debe ser a + b = c. Pero entonces, como

b divide a a + c = 2a + b, se sigue que b divide a 2a, y como b no tiene factores comunes con a, b divide a 2. bno puede ser 1 ya que es mayor que a, luego la unica terna posible en este caso es (1, 2, 3).

Problema 3.5. Halla todas las ternas (x, y, z) de numeros reales que son soluciones del sistema de ecuaciones

3 · 2y − 1 = 2x + 2−x,3 · 2z − 1 = 2y + 2−y,3 · 2x − 1 = 2z + 2−z.

⎫⎬⎭

Solucion Problema 3.5 Haciendo la sustitucion 2x = a, 2y = b, y 2z = c, se observa que a, b, c > 0 y se obtiene

b = 13 (a + 1 + 1

a ),c = 1

3 (b + 1 + 1b ),

a = 13 (c + 1 + 1

c ).

⎫⎬⎭

Aplicando la desigualdad entre las medias aritmetica y geometrica, resulta

b =13(a + 1 +

1a) ≥ 3

√a · 1 · 1

a= 1,

y por tanto b ≥ 1/b. Analogamente, a ≥ 1, a ≥ 1/a, y c ≥ 1, c ≥ 1/c. Teniendo en cuenta lo anterior, de laprimera ecuacion resulta que

b =13(a + 1 +

1a) ≤ 1

3(a + a + a) = a

Page 172: COMPENDIUM OMEFL - ToomatesXXXV Olimpiada Matemática Española Primera Fase Soluciones de la propuesta de problemas Problema 1 ¿Qué dígitos se han omitido en la siguiente multiplicación?

y de las otras dos que c ≤ b y a ≤ c. Combinando las desigualdades anteriores, se obtiene a ≤ c ≤ b ≤ a. Esdecir, a = b = c.

Ahora tenemos

a =13(a + 1 +

1a) ⇔ 2a2 − a − 1 = (a − 1)(2a + 1) = 0

que tiene por solucion a = 1 y a = −1/2. Como solo nos vale la solucion positiva, tenemos que a = b = c = 1 ypor tanto, (x, y, z) = (0, 0, 0) es la unica terna solucion del sistema.

Otra solucion Problema 3.5. Haciendo la sustitucion 2x = a, 2y = b, y 2z = c, se observa que a, b, c > 0, yse obtiene:

3b − 1 = a + 1a ,

3c − 1 = b + 1b ,

3a − 1 = c + 1c .

⎫⎬⎭ ⇔

a(3b − 1) = a2 + 1,b(3c − 1) = b2 + 1,c(3a − 1) = c2 + 1.

⎫⎬⎭ ⇔

3ab = a2 + a + 1,3bc = b2 + b + 1,3ca = c2 + c + 1.

⎫⎬⎭

Observemos que como a2 − 2a + 1 = (a − 1)2 ≥ 0, se tiene a2 + 1 ≥ 2a, luego la primera ecuacion implica3ab ≥ 3a, de donde b ≥ 1. Analogamente, de las otras dos ecuaciones se tiene c ≥ 1 y a ≥ 1.

Supongamos que a es el menor de los tres valores a, b, c (los otros casos son analogos). En particular,0 < a ≤ b. Entonces se tiene a2 + a + 1 = 3ab ≥ 3a2, luego 2a2 ≤ a + 1. La funcion 2x2 solo puede ser menor oigual que la funcion x + 1 si x ≤ 1. Por tanto a ≤ 1, y ası se tiene a = 1.

La primera ecuacion queda entonces 3b = 3, luego b = 1. Y de la segunda ecuacion se sigue que c = 1. Portanto, 2x = 2y = 2z = 1, y ası la unica terna posible es (x, y, z) = (0, 0, 0).

Problema 3.6. En una reunion entre cuatro paıses de la ONU, digamos A, B, C y D, el paıs A tiene el doblede representantes que el B, el triple que el C, y el cuadruple que el D. Se pretende distribuir a los representantesen mesas con el mismo numero de personas en cada una. Solo hay una condicion: en cada mesa, cualquiera delos paıses debe estar en inferioridad numerica respecto de los otros tres juntos. ¿Cuantos representantes debehaber en cada mesa, como mınimo?

Solucion Problema 3.6 La respuesta es 25. Veamos la demostracion. Sean a, b, c y d el numero de represen-tantes de cada paıs. Como a debe ser multiplo de 3 y de 4, tambien debe ser multiplo de 12. Por tanto, existeun numero k tal que a = 12k, luego b = 6k, c = 4k y d = 3k. El numero total de representantes es entonces25k.

Si llamamos M al numero de mesas, y P al numero de personas en cada mesa, tenemos MP = 25k.Sea ai el numero de representantes del paıs A en la mesa numero i. La condicion impuesta nos dice que

ai < P2 , o bien 2ai < P . Como ai es un numero entero, esto implica que 2ai ≤ P − 1. Sumando todos los ai, se

obtiene:24k = 2a = 2a1 + · · · + 2aM ≤ M(P − 1) = 25k − M.

De aquı se deduce M ≤ k. Por tanto, como MP = 25k, deducimos finalmente que P ≥ 25.Solo queda demostrar que, en efecto, se puede conseguir una configuracion en la que haya 25 personas en

cada mesa. Pero esto se consigue, por ejemplo, con una sola mesa en la que haya 12 representantes del paıs A,6 del B, 4 del C y 3 del D.

Page 173: COMPENDIUM OMEFL - ToomatesXXXV Olimpiada Matemática Española Primera Fase Soluciones de la propuesta de problemas Problema 1 ¿Qué dígitos se han omitido en la siguiente multiplicación?

XLVIII Olimpiada Matematica Espanola

Primera Fase

Primera sesion

Viernes manana, 16 de diciembre de 2011

OlimpiadaMatemáticaEspañola RSME

1. Dado un entero positivo n, hallar la suma de todos los enteros positivosinferiores a 10n que no son multiplos de 2 ni de 5.

2. Sea ABC un triangulo acutangulo con A = 45◦, y sea P el pie de la alturapor B. Trazamos la circunferencia de centro P que pasa por C y que vuelvea cortar a AC en el punto X y a la altura PB en el punto Y . Sean r y slas rectas perpendiculares a la recta AY por P y X , respectivamente, y L,K las intersecciones de r, s con AB. Demostrar que L es el punto medio deKB.

3. Los puntos A1, A2, . . . , A2n+1 son los vertices de un polıgono regular de2n + 1 lados. Hallar el numero de ternas Ai, Aj, Ak tales que el trianguloAiAjAk es obtusangulo.

No esta permitido el uso de calculadoras.Cada problema se puntua sobre 7 puntos.

El tiempo de cada sesion es de 3 horas y media.

Page 174: COMPENDIUM OMEFL - ToomatesXXXV Olimpiada Matemática Española Primera Fase Soluciones de la propuesta de problemas Problema 1 ¿Qué dígitos se han omitido en la siguiente multiplicación?

XLVIII Olimpiada Matematica Espanola

Primera Fase

Segunda sesion

Viernes tarde, 16 de diciembre de 2011

OlimpiadaMatemáticaEspañola RSME

4. Sean a, b y c tres numeros reales positivos cuyo producto es 1. Demostrarque si la suma de estos numeros es mayor que la suma de sus recıprocos,entonces exactamente uno de ellos es mayor que 1.

5. En un triangulo rectangulo de hipotenusa unidad y angulos respectivos de30◦, 60◦ y 90◦, se eligen 25 puntos cualesquiera. Demostrar que siemprehabra 9 entre ellos que podran cubrirse con un semicırculo de radio 3/10.

6. Sea P un punto interior a un triangulo ABC y sean HA, HB , HC los orto-centros de los triangulos PBC, PAC y PAB, respectivamente. Demostrarque los triangulos HAHBHC y ABC tiene igual area.

No esta permitido el uso de calculadoras.Cada problema se puntua sobre 7 puntos.

El tiempo de cada sesion es de 3 horas y media.

Page 175: COMPENDIUM OMEFL - ToomatesXXXV Olimpiada Matemática Española Primera Fase Soluciones de la propuesta de problemas Problema 1 ¿Qué dígitos se han omitido en la siguiente multiplicación?

XLVIII Olimpiada Matematica Espanola

Primera Fase

Primera sesion

Viernes tarde, 16 de diciembre de 2011

OlimpiadaMatemáticaEspañola RSME

1. Sean a, b y c tres numeros reales positivos cuyo producto es 1. Demostrarque si la suma de estos numeros es mayor que la suma de sus recıprocos,entonces exactamente uno de ellos es mayor que 1.

2. En un triangulo rectangulo de hipotenusa unidad y angulos respectivos de30◦, 60◦ y 90◦, se eligen 25 puntos cualesquiera. Demostrar que siemprehabra 9 entre ellos que podran cubrirse con un semicırculo de radio 3/10.

3. Sea P un punto interior a un triangulo ABC y sean HA, HB , HC los orto-centros de los triangulos PBC, PAC y PAB, respectivamente. Demostrarque los triangulos HAHBHC y ABC tiene igual area.

No esta permitido el uso de calculadoras.Cada problema se puntua sobre 7 puntos.

El tiempo de cada sesion es de 3 horas y media.

Page 176: COMPENDIUM OMEFL - ToomatesXXXV Olimpiada Matemática Española Primera Fase Soluciones de la propuesta de problemas Problema 1 ¿Qué dígitos se han omitido en la siguiente multiplicación?

XLVIII Olimpiada Matematica Espanola

Primera Fase

Segunda sesion

Sabado manana, 17 de diciembre de 2011

OlimpiadaMatemáticaEspañola RSME

4. Sea ABCD un cuadrilatero convexo y P un punto interior. Determinar quecondiciones deben cumplir el cuadrilatero y el punto P para que los cuatrotriangulos PAB, PBC, PCD y PDA tengan la misma area.

5. Sean a, b y c las longitudes de los lados de un triangulo ABC. Si

b(a + b)(b + c) = a3 + b(a2 + c2) + c3,

demostrar que la medida (en radianes) de los angulos A, B y C cumple larelacion

1√A +

√B

+1√

B +√

C=

2√A +

√C

.

6. Tenemos una coleccion de esferas iguales que apilamos formando un tetraedrocuyas aristas tienen todas n esferas. Calcular, en funcion de n, el numerototal de puntos de tangencia (contactos) entra las esferas del monton.

No esta permitido el uso de calculadoras.Cada problema se puntua sobre 7 puntos.

El tiempo de cada sesion es de 3 horas y media.

Page 177: COMPENDIUM OMEFL - ToomatesXXXV Olimpiada Matemática Española Primera Fase Soluciones de la propuesta de problemas Problema 1 ¿Qué dígitos se han omitido en la siguiente multiplicación?

XLVIII Olimpiada Matematica Espanola

Primera Fase

Primera sesion

Sabado manana, 17 de diciembre de 2011

OlimpiadaMatemáticaEspañola RSME

1. Sea ABCD un cuadrilatero convexo y P un punto interior. Determinar quecondiciones deben cumplir el cuadrilatero y el punto P para que los cuatrotriangulos PAB, PBC, PCD y PDA tengan la misma area.

2. Sean a, b y c las longitudes de los lados de un triangulo ABC. Si

b(a + b)(b + c) = a3 + b(a2 + c2) + c3,

demostrar que la medida (en radianes) de los angulos A, B y C cumple larelacion

1√A +

√B

+1√

B +√

C=

2√A +

√C

.

3. Tenemos una coleccion de esferas iguales que apilamos formando un tetraedrocuyas aristas tienen todas n esferas. Calcular, en funcion de n, el numerototal de puntos de tangencia (contactos) entra las esferas del monton.

No esta permitido el uso de calculadoras.Cada problema se puntua sobre 7 puntos.

El tiempo de cada sesion es de 3 horas y media.

Page 178: COMPENDIUM OMEFL - ToomatesXXXV Olimpiada Matemática Española Primera Fase Soluciones de la propuesta de problemas Problema 1 ¿Qué dígitos se han omitido en la siguiente multiplicación?

XLVIII Olimpiada Matematica Espanola

Primera Fase

Segunda sesion

Sabado tarde, 17 de diciembre de 2011

OlimpiadaMatemáticaEspañola RSME

4. Hallar todas las funciones reales continuas f : R+ → R

+ que cumplen, paratodo x real positivo, la condicion

x +1x

= f(x) +1

f(x)

5. Consideremos el numero entero positivo

n = 2r − 16s

donde r y s son tambien enteros positivos. Hallar las condiciones que debencumplir r y s para que el resto de la division de n por 7 sea 5. Hallar elmenor numero que cumple esta condicion.

6. Los puntos A1, A2, . . . , A2n son los vertices de un polıgono regular de 2nlados. Hallar el numero de ternas Ai, Aj, Ak tales que el triangulo AiAjAk

es rectangulo y el numero de ternas tales que el triangulo es acutangulo.

No esta permitido el uso de calculadoras.Cada problema se puntua sobre 7 puntos.

El tiempo de cada sesion es de 3 horas y media.

Page 179: COMPENDIUM OMEFL - ToomatesXXXV Olimpiada Matemática Española Primera Fase Soluciones de la propuesta de problemas Problema 1 ¿Qué dígitos se han omitido en la siguiente multiplicación?

A1.-Dado un entero positivo n, hallar la suma de todos los enteros positivos inferiores a 10nque no son multiplos de 2 ni de 5.

Solucion.Sean los conjuntos

A = {1, 2, . . . , 10n},B = {2, 4, . . . , 2(5n)},C = {5, 10, . . . , 5(2n)},

B ∩ C = {10, 20, . . . , 10n}.Nos piden la suma de los elementos de A que no son de B ni de C. Las sumas de loselementos de cada uno de los conjuntos es

ΣA =10n(10n + 1)

2, ΣB = 2

5n(5n + 1)2

, ΣC = 52n(2n + 1)

2, y

Σ(B ∩ C) =10n(10n + 1)

2.

La suma pedida esΣA − ΣB − ΣC + Σ(B ∩ C) = 20n2.

Page 180: COMPENDIUM OMEFL - ToomatesXXXV Olimpiada Matemática Española Primera Fase Soluciones de la propuesta de problemas Problema 1 ¿Qué dígitos se han omitido en la siguiente multiplicación?

A2.-Sea ABC un triangulo acutangulo con A = 45◦, y sea P el pie de la altura por B.Trazamos la circunferencia de centro P que pasa por C y que vuelve a cortar a AC enel punto X y a la altura PB en el punto Y . Sean r y s las rectas perpendiculares ala recta AY por P y X , respectivamente, y L, K las intersecciones de r, s con AB.Demostrar que L es el punto medio de KB.

Solucion.Por construccion es PX = PY = PC. Los triangulos PAY y PCB, rectangulos en P ,son iguales ya que AP = PB (el triangulo rectangulo APB es isosceles) y PY = PC.Por tanto los angulos α y β son iguales.El triangulo rectangulo PY Q es semejante a los anteriores, de manera que el anguloγ = LPB es igual a α. Resulta que los segmentos PL y CB son paralelos, y por elteorema de Thales queda KL=LB ya que PX=PC.

A K L B

Y

C

P

X

Q

αβ

γ

Page 181: COMPENDIUM OMEFL - ToomatesXXXV Olimpiada Matemática Española Primera Fase Soluciones de la propuesta de problemas Problema 1 ¿Qué dígitos se han omitido en la siguiente multiplicación?

A3.-Los puntos A1, A2, . . . , A2n+1 son los vertices de un polıgono regular de 2n + 1 lados.Hallar el numero de ternas Ai, Aj, Ak tales que el triangulo AiAjAk es obtusangulo.

Solucion.

Al ser 2n+1 impar, no es posibleconstruir triangulos rectangulos.Observemos que cualquier trian-gulo obtusangulo dejara el centroO (su circuncentro) fuera de el.Si lo giramos en sentido directoo inverso alrededor de O pode-mos conseguir que uno de susvertices agudos este en A1. Losotros dos estan, bien en el con-junto {A2, . . . , An+1}, bien en{An+2, . . . , A2n+1}. El numerobuscado sera 2

(n2

). Como esto

lo podemos hacer con cada unode los 2n + 1 vertices, quedaran2(2n + 1)

(n2

)triangulos. Pero

cada triangulo lo hemos contadodos veces, una para cada verticeagudo. Luego la solucion bus-cada es (2n + 1)

(n2

).

A1

A2

An+1An+2

A2n+1

O

Solucion alternativa.Fijemos el vertice obtuso en un vertice, por ejemplo, el A1. Los tres lados del trianguloabarcaran respectivamente x, y y z lados del polıgono de 2n+1 lados. Sera x+y +z =2n + 1. El lado opuesto al angulo obtuso, digamos z, debera cumplir z ≥ n + 1.Calculemos el numero de soluciones enteras positivas de la ecuacion x + y + z = 2n + 1con la condicion fijada para la z.Si z = n+1, queda x+y = n que tiene n−1 soluciones. Si z = n+2, queda x+y = n−1que tiene n − 2 soluciones. . . . Si z = 2n − 1, queda x + y = 1 que tiene 1 solucion.En total hay

(n − 1) + (n − 2) + · · ·+ 2 + 1 =n(n − 1)

2=

(n

2

)soluciones con el angulo obtuso en A1. Si consideramos las otras posibles posicionespara dicho angulo queda en total

(2n + 1)(

n

2

).

Page 182: COMPENDIUM OMEFL - ToomatesXXXV Olimpiada Matemática Española Primera Fase Soluciones de la propuesta de problemas Problema 1 ¿Qué dígitos se han omitido en la siguiente multiplicación?

B1.-

Sean a, b i c tres numeros reales positivos cuyo producto es 1. Demuestra que, si lasuma de estos numeros es mayor que la suma de sus inveros, entonces exactamente unode ellos es mayor que 1.

Solucion.

Puesto que abc = 1 y a + b + c >1a

+1b

+1c, tenemos que

(a − 1)(b − 1)(c − 1) = abc − ab − bc − ca + a + b + c − 1

= a + b + c −(

1a

+1b

+1c

)> 0.

La desigualdad anterior se cumple cuando uno de los factores del numero

(a − 1)(b − 1)(c − 1)

es positivo o los tres factores son positivos. Si fuesen positivos los tres, tendrıamosa > 1, b > 1 y c > 1, cosa que no es possible ya que abc = 1. Por tanto, solo uno deellos es positivo i esto acaba la demostracion.

Page 183: COMPENDIUM OMEFL - ToomatesXXXV Olimpiada Matemática Española Primera Fase Soluciones de la propuesta de problemas Problema 1 ¿Qué dígitos se han omitido en la siguiente multiplicación?

B2.

En un triangulo rectangulo de hipotenusa unidad y angulos de 30◦, 60◦ i 90◦, se eligen25 puntos cualesquiera. Demuestra que siempre habra 9 de ellos que podran cubrirsecon un semicırculo de radio 3

10.

Solucion. Este triangulo se puede descomponer en tres triangulos congruentes y seme-jantes al triangulo inicial.

Tenemos 3 triangulos y 25 puntos. En algun triangulo habra al menos 9 puntos. Lahipotenusa de cada uno de estos triangulos semejantes al inicial mide

√3

3 . Los triangulosson rectangulos y por lo tanto estan cubiertos per la mitad del cırculo circunscrito. Estoacaba el problema ya que el radio de este cırculo circunscrito, r, cumple

r =12

√3

3<

310

.

Page 184: COMPENDIUM OMEFL - ToomatesXXXV Olimpiada Matemática Española Primera Fase Soluciones de la propuesta de problemas Problema 1 ¿Qué dígitos se han omitido en la siguiente multiplicación?

B3.

Sea ABC un triangulo arbitrario, P un punto interior y HA, HB i HC , respectivament,los ortocentros de los triangulos PBC, PAC y PAB. Demuestra que los triangulosHAHBHC y ABC tienen la misma area.

Solucion.

Calculemos la distancia de un vertice A al ortocentro del triangulo ABC.En las figuras siguientes podemos observar que los triangulos BCC′ y AHC′ son seme-jantes. (Recordemos que los angulos de lados perpendiculares son iguales o suplemen-tarios). De esta semejanza resulta

AH

CB=

AC′

CC′ ⇔AH

a=

AC′

CC′

i, per tant

AH = aAC′

CC′ .

Si el triangulo es acutangulo (figura de la izquierda) tenemos que en el triangulo ACC′

es, obviamente, AC′ = b cosA y CC′ = b sinA.

C Ba

H

A

b c

A′

C′

C Ba

A

H

bc

A′

C′

Sustituyendo, quedaAH = a cotA.

Si el triangulo ABC es rectangulo en A, la formula es tambien valida, pero en este casoes A = H y AH = a cot 90◦ = 0. Si es obtusangulo (figura de la derecha), el punto Hes exterior al triangulo y queda AC′ = b cos(180◦ − A) y CC′ = b sin(180◦ − A), y portanto, AH = −a cotA. Pero en este caso cot A es negativa.Les distancies del ortocentro H a los vertices agudos de un triangulo rectangulo uobtusangulo salen de manera parecida.

Page 185: COMPENDIUM OMEFL - ToomatesXXXV Olimpiada Matemática Española Primera Fase Soluciones de la propuesta de problemas Problema 1 ¿Qué dígitos se han omitido en la siguiente multiplicación?

Cuando unimos el punto arbitrario P con los vertices A, B y C del triangulo obtenemoslos tres triangulos PAB, PBC y PCA.Sean α = ∠BPC, β = ∠APC, γ = ∠APB. Evidentemente, α + β + γ = 360◦. De estostres angulos, com mınimo dos son obtusos. El otro puede ser obtuso, recto o agudo.Estudiaremos los tres casos per separado.

1) Supongamos que los tres angulos son obtusos (Figura 1). Por lo que hemos dichoal principio tenemos PHA = −a cotα y PHC = −c cot γ. Fijemonos que el anguloy = ∠HAPHC = 180◦ − ∠APC = 180◦ − B ya que los lados HAP y HCP son,respectivamente, perpendiculares a los lados, BC y AB y un es obtuso y el otro esagudo.El area A(PHAHC) del triangulo PHAHC es, obviamente,

A(PHAHC) =PHA PHC sin y

2=

ac cotα cot γ sin B

2= A(ABC) cotα cot γ.

HB

C

A

P

HA

B

HCα

βγ

x

yz Figura 1

Sumando, pues, las areas de los tres triangulos PHAHB , PHBHC y PHCHA obtenemos

A(HAHBHC) = A(PHAHB) + A(PHBHC) + A(PHCHA) i

A(HAHBHC) = A(ABC)(

cotα cotβ + cotβ cot γ + cot γ cotα).

Como que α + β = 360 − γ, tenemos que cot(α + β) = − cot γ o, equivalentemente,

cot γ = − cot(α + β) =1 − cotα cotβ

cot α + cot β

o bien, cotα cotβ + cotβ cot γ + cot γ cotα = 1. (*)

De aquı resulta A(HAHBHC) = A(ABC).

2) Supongamos que uno de los angulos es recto, per ejemplo β = 90◦ (Figura 2).Entonces HB = P y

A(HAHBHC) = A(HAPHC) =PHA PHC sin y

2=

ac cotα cot γ sin B

2= A(ABC) cotα cot γ

Pero la misma identidad (*) nos dice que si cot β = 0 tiene que ser cot α cot γ = 1, y deaquı el resultado en este caso.

Page 186: COMPENDIUM OMEFL - ToomatesXXXV Olimpiada Matemática Española Primera Fase Soluciones de la propuesta de problemas Problema 1 ¿Qué dígitos se han omitido en la siguiente multiplicación?

3) Supongamos ahora que uno de los angulos α, β, γ es agudo, por ejemplo, APC =β < 90◦ (Figura 3). El punto P es exterior al triangulo HAHBHC y tenemos A(HAHBHC) =A(PHAHC) −A(PHAHB) −A(PHCHB).

A

B C

P = HB

HA

HC

Figura 2

A

B C

P

HA

HC

HB

Figura 3

Pero en este caso tenemos PHB = b cotβ, PHA = −a cot α i PHC = −c cot γ y, por lotanto,

A(HAHBHC) = A(PHAHC) −A(PHAHB) −A(PHCHB) ==

(cotα cot γ − (− cotα cot β) − (− cot γ cotβ)

)A(ABC) =

= (cotα cotβ + cotβ cot γ + cot γ cotα)A(ABC) = A(ABC).

Page 187: COMPENDIUM OMEFL - ToomatesXXXV Olimpiada Matemática Española Primera Fase Soluciones de la propuesta de problemas Problema 1 ¿Qué dígitos se han omitido en la siguiente multiplicación?

B1.-

Sean a, b i c tres numeros reales positivos cuyo producto es 1. Demuestra que, si lasuma de estos numeros es mayor que la suma de sus inveros, entonces exactamente unode ellos es mayor que 1.

Solucion.

Puesto que abc = 1 y a + b + c >1a

+1b

+1c, tenemos que

(a − 1)(b − 1)(c − 1) = abc − ab − bc − ca + a + b + c − 1

= a + b + c −(

1a

+1b

+1c

)> 0.

La desigualdad anterior se cumple cuando uno de los factores del numero

(a − 1)(b − 1)(c − 1)

es positivo o los tres factores son positivos. Si fuesen positivos los tres, tendrıamosa > 1, b > 1 y c > 1, cosa que no es possible ya que abc = 1. Por tanto, solo uno deellos es positivo i esto acaba la demostracion.

Page 188: COMPENDIUM OMEFL - ToomatesXXXV Olimpiada Matemática Española Primera Fase Soluciones de la propuesta de problemas Problema 1 ¿Qué dígitos se han omitido en la siguiente multiplicación?

B2.

En un triangulo rectangulo de hipotenusa unidad y angulos de 30◦, 60◦ i 90◦, se eligen25 puntos cualesquiera. Demuestra que siempre habra 9 de ellos que podran cubrirsecon un semicırculo de radio 3

10.

Solucion. Este triangulo se puede descomponer en tres triangulos congruentes y seme-jantes al triangulo inicial.

Tenemos 3 triangulos y 25 puntos. En algun triangulo habra al menos 9 puntos. Lahipotenusa de cada uno de estos triangulos semejantes al inicial mide

√3

3 . Los triangulosson rectangulos y por lo tanto estan cubiertos per la mitad del cırculo circunscrito. Estoacaba el problema ya que el radio de este cırculo circunscrito, r, cumple

r =12

√3

3<

310

.

Page 189: COMPENDIUM OMEFL - ToomatesXXXV Olimpiada Matemática Española Primera Fase Soluciones de la propuesta de problemas Problema 1 ¿Qué dígitos se han omitido en la siguiente multiplicación?

B3.

Sea ABC un triangulo arbitrario, P un punto interior y HA, HB i HC , respectivament,los ortocentros de los triangulos PBC, PAC y PAB. Demuestra que los triangulosHAHBHC y ABC tienen la misma area.

Solucion.

Calculemos la distancia de un vertice A al ortocentro del triangulo ABC.En las figuras siguientes podemos observar que los triangulos BCC′ y AHC′ son seme-jantes. (Recordemos que los angulos de lados perpendiculares son iguales o suplemen-tarios). De esta semejanza resulta

AH

CB=

AC′

CC′ ⇔AH

a=

AC′

CC′

i, per tant

AH = aAC′

CC′ .

Si el triangulo es acutangulo (figura de la izquierda) tenemos que en el triangulo ACC′

es, obviamente, AC′ = b cosA y CC′ = b sinA.

C Ba

H

A

b c

A′

C′

C Ba

A

H

bc

A′

C′

Sustituyendo, quedaAH = a cotA.

Si el triangulo ABC es rectangulo en A, la formula es tambien valida, pero en este casoes A = H y AH = a cot 90◦ = 0. Si es obtusangulo (figura de la derecha), el punto Hes exterior al triangulo y queda AC′ = b cos(180◦ − A) y CC′ = b sin(180◦ − A), y portanto, AH = −a cotA. Pero en este caso cot A es negativa.Les distancies del ortocentro H a los vertices agudos de un triangulo rectangulo uobtusangulo salen de manera parecida.

Page 190: COMPENDIUM OMEFL - ToomatesXXXV Olimpiada Matemática Española Primera Fase Soluciones de la propuesta de problemas Problema 1 ¿Qué dígitos se han omitido en la siguiente multiplicación?

Cuando unimos el punto arbitrario P con los vertices A, B y C del triangulo obtenemoslos tres triangulos PAB, PBC y PCA.Sean α = ∠BPC, β = ∠APC, γ = ∠APB. Evidentemente, α + β + γ = 360◦. De estostres angulos, com mınimo dos son obtusos. El otro puede ser obtuso, recto o agudo.Estudiaremos los tres casos per separado.

1) Supongamos que los tres angulos son obtusos (Figura 1). Por lo que hemos dichoal principio tenemos PHA = −a cotα y PHC = −c cot γ. Fijemonos que el anguloy = ∠HAPHC = 180◦ − ∠APC = 180◦ − B ya que los lados HAP y HCP son,respectivamente, perpendiculares a los lados, BC y AB y un es obtuso y el otro esagudo.El area A(PHAHC) del triangulo PHAHC es, obviamente,

A(PHAHC) =PHA PHC sin y

2=

ac cotα cot γ sin B

2= A(ABC) cotα cot γ.

HB

C

A

P

HA

B

HCα

βγ

x

yz Figura 1

Sumando, pues, las areas de los tres triangulos PHAHB , PHBHC y PHCHA obtenemos

A(HAHBHC) = A(PHAHB) + A(PHBHC) + A(PHCHA) i

A(HAHBHC) = A(ABC)(

cotα cotβ + cotβ cot γ + cot γ cotα).

Como que α + β = 360 − γ, tenemos que cot(α + β) = − cot γ o, equivalentemente,

cot γ = − cot(α + β) =1 − cotα cotβ

cot α + cot β

o bien, cotα cotβ + cotβ cot γ + cot γ cotα = 1. (*)

De aquı resulta A(HAHBHC) = A(ABC).

2) Supongamos que uno de los angulos es recto, per ejemplo β = 90◦ (Figura 2).Entonces HB = P y

A(HAHBHC) = A(HAPHC) =PHA PHC sin y

2=

ac cotα cot γ sin B

2= A(ABC) cotα cot γ

Pero la misma identidad (*) nos dice que si cot β = 0 tiene que ser cot α cot γ = 1, y deaquı el resultado en este caso.

Page 191: COMPENDIUM OMEFL - ToomatesXXXV Olimpiada Matemática Española Primera Fase Soluciones de la propuesta de problemas Problema 1 ¿Qué dígitos se han omitido en la siguiente multiplicación?

3) Supongamos ahora que uno de los angulos α, β, γ es agudo, por ejemplo, APC =β < 90◦ (Figura 3). El punto P es exterior al triangulo HAHBHC y tenemos A(HAHBHC) =A(PHAHC) −A(PHAHB) −A(PHCHB).

A

B C

P = HB

HA

HC

Figura 2

A

B C

P

HA

HC

HB

Figura 3

Pero en este caso tenemos PHB = b cotβ, PHA = −a cot α i PHC = −c cot γ y, por lotanto,

A(HAHBHC) = A(PHAHC) −A(PHAHB) −A(PHCHB) ==

(cotα cot γ − (− cotα cot β) − (− cot γ cotβ)

)A(ABC) =

= (cotα cotβ + cotβ cot γ + cot γ cotα)A(ABC) = A(ABC).

Page 192: COMPENDIUM OMEFL - ToomatesXXXV Olimpiada Matemática Española Primera Fase Soluciones de la propuesta de problemas Problema 1 ¿Qué dígitos se han omitido en la siguiente multiplicación?

C1.

Sea ABCD un cuadrilatero convexo y P un punto interior. Determina cuales sonlas condiciones que deben cumplir el cuadrilatero y el punto P para que los cuatrotriangulos PAB, PBC, PCD i PDA tengan la misma area.

Solucion.

Consideremos, primero, los triangulos PCD y PCB. Tienen la base comun PC yalturas correspondientes DX y BY . Si queremos que tengan la misma area, las alturasdeben ser iguales. Por lo tanto, el punto Q tiene que ser el punto medio de la diagonalBD. La recta CP debe pasar per Q. Analogamente, consideremos los triangulos ADPy PAB de base comun AP . Por el mismo argumento de antes, han de tener alturesiguales y AP tiene que pasar per Q. De ahı que AP y CP tienen dos puntos comunes:P y Q. Los segmentos AP y PC esta, pues, alineados. Es decir, son la diagonal AC.Es pues necesario que las dos diagonales se corten en el punt medio de una de ellas.Pero mirando los triangulos PDA y PDC, que tienen la misma area, resulta que Ptiene que ser el punto medio de AC.La condicion pedida es que las diagonales del cuadrilatero se corten en el punto mediode una de ellas y el punto P sea el punto medio de la otra.

A

B

C

D

O

P

XQ

Y

Page 193: COMPENDIUM OMEFL - ToomatesXXXV Olimpiada Matemática Española Primera Fase Soluciones de la propuesta de problemas Problema 1 ¿Qué dígitos se han omitido en la siguiente multiplicación?

C2.

Tenemos una coleccion de esferas iguales que apilaamos formando un tetraedro cuyasaristas tienen todas n esferas. Calcula, en funcion de n, el numero total de puntos detangencia (contactos) que hay entre las esferas del monton.

Solucion.

El problema en el plano.

Analicemos primero el problema en el cas plano. Sea An elnumero de contactos de n esferes colocadas en un trianguloplano con n esferas en cada uno de los lados (figura dela derecha). Fijemonos que el numero total de esferas es,evidentemente, Tn = n(n+1)

2.

Podemos proceder por induccion. Si hay n = 2 filas elnumero de contactos es 3; es decir, A2 = 3. Observemosque coincide con el numero de bolas del triangulo de dosfilas.En un triangulo de n − 1 filas hay An−1 contactos. Obviamente, en un triangulo den files habra los contactos que ya habıa en un triangulo de n − 1 filas , mas los queprovengan de anadir la ultima fila, tal como esta indicado en la figura anterior. Peroesta claro que, al anadir esta ultima fila se producen contactos de dos tipos:• Los que hay entre las bolas de la fila n-esima, que son n − 1.• Los que tienen las bolas de la fila n-esima con la anterior. Son 2(n − 1).Ası pues, An = An−1 + 3(n − 1), o bien, An − An−1 = 3(n − 1). Sumando queda

An = 3((n − 1) + (n − 2) + · · ·+ 2 + 1

)= 3

n(n − 1)2

= 3Tn−1.

El problema en el espacio tridimensional.

Ahora ya podemos analizar el caso en el espacio. Sea Cn elnumero de contactos de un monton tetraedrico de esferascon aristas de n esferes. En la figura de la derecha hemosrepresentado las esferas de la base en trazo continuo y lasdel piso inmediato superior en trazo discontinuo, a vistade pajaro. Cuando anadimos el piso n-esimo, anadimoscontactos de dos tipos:• Los propios del piso – un triangulo plano de n boles delado.• Los que provienen de contactos entre el piso n − 1 y elpiso n.

Page 194: COMPENDIUM OMEFL - ToomatesXXXV Olimpiada Matemática Española Primera Fase Soluciones de la propuesta de problemas Problema 1 ¿Qué dígitos se han omitido en la siguiente multiplicación?

Los contactos del primer tipo son, como hemos visto en el cas plano, An = 3Tn−1.El numero de contactos entre un piso y el anterior es 3Tn−1, ya que cada bola del pison − 1 toca exactamente tres boles del piso n. (Vease la figura.) En total, pues, elnumero de contactos es Cn − Cn−1 = An + 3Tn−1 = 3n(n − 1). Si sumamos queda

Cn − C2 = 3n(n − 1) + · · ·+ 3 · 3(3 − 1) = 3(n2 + · · ·+ 32) − 3(n + · · ·+ 3),

o bien

Cn = 3(n2+ · · ·+22 +12)−3(n+ · · ·+2+1) = 3n(n + 1)(2n + 1)

6−3

n(n + 1)2

= n3−n.

Otro camino. La recurrencia Cn = Cn−1 + 3n(n − 1) se puede resolver escribiendoCn como un polinomio cubico y calculando sus coeficientes a partir de la recurrencia yde la condicion inicial C1 = 0.Si ponemos Cn = an(n− 1)(n− 2) + bn(n− 1) + cn + d, la condicion de recurrencia daa = 1, b = 3, c = 0, y la condicion C1 = 0 da d = 0. En resumen

Cn = n(n − 1)(n − 2) + 3n(n − 1) = n3 − n.

Page 195: COMPENDIUM OMEFL - ToomatesXXXV Olimpiada Matemática Española Primera Fase Soluciones de la propuesta de problemas Problema 1 ¿Qué dígitos se han omitido en la siguiente multiplicación?

C3.

Sean a, b i c las longitudes de los lados de un triangulo ABC. Si

b(a + b)(b + c) = a3 + b(a2 + c2) + c3,

demuestra que les medidas de los angulos A, B, C cumplen la relacion

1√A +

√B

+1√

B +√

C=

2√C +

√A

.

.

Solucion. La condicion del enunciado se puede escribir en la forma

b3 + b2a + b2c + abc − a2b − bc2 − a3 − c3 = 0

o, equivalentemente,

b2(a + b + c) − (a3 + b3 + c3 − 3abc) + b3 − 2abc − a2b − b2c = 0.

Si sustituimos la identidad a3 + b3 + c3 − 3abc = (a + b + c)(a2 + b2 + c2 − ab− bc− ca),se obtiene

b2(a + b + c) − (a + b + c)(a2 + b2 + c2 − ab − bc − ca) + b(a + b + c)(b − a − c) = 0

o, lo que es lo mismo, (a+ b + c)(b2 − a2 − c2 − ac) = 0. Puesto que a + b + c �= 0, tieneque ser b2 − a2 − c2 − ac = 0, de donde, por el teorema del coseno, resulta que

b2 − a2 − c2

2ac=

12

= cos B

Por lo tanto, B = π/3. Sabemos que A+B+C = π i de esto resulta A+C = 2π/3 = 2B.Es decir, los angulos A, B, C estan en progresion aritmetica. Pero la igualdad que hayque demostrar equivale, precisamente, a que A, B, C esten en progresion aritmetica.Efectivamente, si suponemos que B = A + d y C = A + 2d con d ≥ 0, tenemos

1√A +

√B

+1√

B +√

C=

√B −

√A

B − A+

√C −

√B

C − B

=

√C −

√A

d=

C − A

d(√

C +√

A)=

2√C +

√A

Si fuese d = 0, el triangulo seria equilatero y el enunciado se cumpliria trivialmente.

Page 196: COMPENDIUM OMEFL - ToomatesXXXV Olimpiada Matemática Española Primera Fase Soluciones de la propuesta de problemas Problema 1 ¿Qué dígitos se han omitido en la siguiente multiplicación?

C1.

Sea ABCD un cuadrilatero convexo y P un punto interior. Determina cuales sonlas condiciones que deben cumplir el cuadrilatero y el punto P para que los cuatrotriangulos PAB, PBC, PCD i PDA tengan la misma area.

Solucion.

Consideremos, primero, los triangulos PCD y PCB. Tienen la base comun PC yalturas correspondientes DX y BY . Si queremos que tengan la misma area, las alturasdeben ser iguales. Por lo tanto, el punto Q tiene que ser el punto medio de la diagonalBD. La recta CP debe pasar per Q. Analogamente, consideremos los triangulos ADPy PAB de base comun AP . Por el mismo argumento de antes, han de tener alturesiguales y AP tiene que pasar per Q. De ahı que AP y CP tienen dos puntos comunes:P y Q. Los segmentos AP y PC esta, pues, alineados. Es decir, son la diagonal AC.Es pues necesario que las dos diagonales se corten en el punt medio de una de ellas.Pero mirando los triangulos PDA y PDC, que tienen la misma area, resulta que Ptiene que ser el punto medio de AC.La condicion pedida es que las diagonales del cuadrilatero se corten en el punto mediode una de ellas y el punto P sea el punto medio de la otra.

A

B

C

D

O

P

XQ

Y

Page 197: COMPENDIUM OMEFL - ToomatesXXXV Olimpiada Matemática Española Primera Fase Soluciones de la propuesta de problemas Problema 1 ¿Qué dígitos se han omitido en la siguiente multiplicación?

C2.

Tenemos una coleccion de esferas iguales que apilaamos formando un tetraedro cuyasaristas tienen todas n esferas. Calcula, en funcion de n, el numero total de puntos detangencia (contactos) que hay entre las esferas del monton.

Solucion.

El problema en el plano.

Analicemos primero el problema en el cas plano. Sea An elnumero de contactos de n esferes colocadas en un trianguloplano con n esferas en cada uno de los lados (figura dela derecha). Fijemonos que el numero total de esferas es,evidentemente, Tn = n(n+1)

2.

Podemos proceder por induccion. Si hay n = 2 filas elnumero de contactos es 3; es decir, A2 = 3. Observemosque coincide con el numero de bolas del triangulo de dosfilas.En un triangulo de n − 1 filas hay An−1 contactos. Obviamente, en un triangulo den files habra los contactos que ya habıa en un triangulo de n − 1 filas , mas los queprovengan de anadir la ultima fila, tal como esta indicado en la figura anterior. Peroesta claro que, al anadir esta ultima fila se producen contactos de dos tipos:• Los que hay entre las bolas de la fila n-esima, que son n − 1.• Los que tienen las bolas de la fila n-esima con la anterior. Son 2(n − 1).Ası pues, An = An−1 + 3(n − 1), o bien, An − An−1 = 3(n − 1). Sumando queda

An = 3((n − 1) + (n − 2) + · · ·+ 2 + 1

)= 3

n(n − 1)2

= 3Tn−1.

El problema en el espacio tridimensional.

Ahora ya podemos analizar el caso en el espacio. Sea Cn elnumero de contactos de un monton tetraedrico de esferascon aristas de n esferes. En la figura de la derecha hemosrepresentado las esferas de la base en trazo continuo y lasdel piso inmediato superior en trazo discontinuo, a vistade pajaro. Cuando anadimos el piso n-esimo, anadimoscontactos de dos tipos:• Los propios del piso – un triangulo plano de n boles delado.• Los que provienen de contactos entre el piso n − 1 y elpiso n.

Page 198: COMPENDIUM OMEFL - ToomatesXXXV Olimpiada Matemática Española Primera Fase Soluciones de la propuesta de problemas Problema 1 ¿Qué dígitos se han omitido en la siguiente multiplicación?

Los contactos del primer tipo son, como hemos visto en el cas plano, An = 3Tn−1.El numero de contactos entre un piso y el anterior es 3Tn−1, ya que cada bola del pison − 1 toca exactamente tres boles del piso n. (Vease la figura.) En total, pues, elnumero de contactos es Cn − Cn−1 = An + 3Tn−1 = 3n(n − 1). Si sumamos queda

Cn − C2 = 3n(n − 1) + · · ·+ 3 · 3(3 − 1) = 3(n2 + · · ·+ 32) − 3(n + · · ·+ 3),

o bien

Cn = 3(n2+ · · ·+22 +12)−3(n+ · · ·+2+1) = 3n(n + 1)(2n + 1)

6−3

n(n + 1)2

= n3−n.

Otro camino. La recurrencia Cn = Cn−1 + 3n(n − 1) se puede resolver escribiendoCn como un polinomio cubico y calculando sus coeficientes a partir de la recurrencia yde la condicion inicial C1 = 0.Si ponemos Cn = an(n− 1)(n− 2) + bn(n− 1) + cn + d, la condicion de recurrencia daa = 1, b = 3, c = 0, y la condicion C1 = 0 da d = 0. En resumen

Cn = n(n − 1)(n − 2) + 3n(n − 1) = n3 − n.

Page 199: COMPENDIUM OMEFL - ToomatesXXXV Olimpiada Matemática Española Primera Fase Soluciones de la propuesta de problemas Problema 1 ¿Qué dígitos se han omitido en la siguiente multiplicación?

C3.

Sean a, b i c las longitudes de los lados de un triangulo ABC. Si

b(a + b)(b + c) = a3 + b(a2 + c2) + c3,

demuestra que les medidas de los angulos A, B, C cumplen la relacion

1√A +

√B

+1√

B +√

C=

2√C +

√A

.

.

Solucion. La condicion del enunciado se puede escribir en la forma

b3 + b2a + b2c + abc − a2b − bc2 − a3 − c3 = 0

o, equivalentemente,

b2(a + b + c) − (a3 + b3 + c3 − 3abc) + b3 − 2abc − a2b − b2c = 0.

Si sustituimos la identidad a3 + b3 + c3 − 3abc = (a + b + c)(a2 + b2 + c2 − ab− bc− ca),se obtiene

b2(a + b + c) − (a + b + c)(a2 + b2 + c2 − ab − bc − ca) + b(a + b + c)(b − a − c) = 0

o, lo que es lo mismo, (a+ b + c)(b2 − a2 − c2 − ac) = 0. Puesto que a + b + c �= 0, tieneque ser b2 − a2 − c2 − ac = 0, de donde, por el teorema del coseno, resulta que

b2 − a2 − c2

2ac=

12

= cos B

Por lo tanto, B = π/3. Sabemos que A+B+C = π i de esto resulta A+C = 2π/3 = 2B.Es decir, los angulos A, B, C estan en progresion aritmetica. Pero la igualdad que hayque demostrar equivale, precisamente, a que A, B, C esten en progresion aritmetica.Efectivamente, si suponemos que B = A + d y C = A + 2d con d ≥ 0, tenemos

1√A +

√B

+1√

B +√

C=

√B −

√A

B − A+

√C −

√B

C − B

=

√C −

√A

d=

C − A

d(√

C +√

A)=

2√C +

√A

Si fuese d = 0, el triangulo seria equilatero y el enunciado se cumpliria trivialmente.

Page 200: COMPENDIUM OMEFL - ToomatesXXXV Olimpiada Matemática Española Primera Fase Soluciones de la propuesta de problemas Problema 1 ¿Qué dígitos se han omitido en la siguiente multiplicación?

D1.-Hallar todas las funciones reales continuas f : R+ → R+ que cumplen, para todo x realpositivo, la condicion

x +1x

= f(x) +1

f(x).

Solucion.Escribimos

x − f =1f− 1

x=

x − f

xf

de donde sale(x − f)

(1 − 1

xf

)= 0.

De aquı resulta que, para cada a > 0, sera, o bien f(a) = a, o bien f(a) =1a.

Las funciones de R+ en R+ definidas por f(x) = x y por f(x) = 1/x cumplen lacondicion, pero tambien la cumplen las funciones

max(x,1x

) y min(x,1x

).

ya que las curvas f(x) = x y f(x) = 1/x se cortan en (1, 1) y solamente en este punto.

f(x)= 1x

f(x)=x

f(x)=max(x, 1x )

f(x)=min(x, 1x )

Observacion. Un razonamiento mas fino para deducir que estas funciones son lasunicas continuas es el siguiente. Sea f una funcion que cumple las condiciones delenunciado y supongamos que en el intervalo (0, 1) tomase valores x y 1/x. Sea α elsupremo de los x < 1 tales que f(x) = x. Si α fuese estrictamente menor que 1, lafuncion tendrıa una discontinuidad en el. Luego α = 1 y la funcion no puede saltar dex a 1/x en el intervalo (0, 1). Analogamente se hace a la derecha del 1.

Page 201: COMPENDIUM OMEFL - ToomatesXXXV Olimpiada Matemática Española Primera Fase Soluciones de la propuesta de problemas Problema 1 ¿Qué dígitos se han omitido en la siguiente multiplicación?

D2.-Consideremos el numero entero positivo

n = 2r − 16s

donde r y s son tambien enteros positivos. Hallar las condiciones que deben cumplirr y s para que el resto de la division de n por 7 sea 5. Hallar el menor numero quecumple esta condicion.

Solucion.Los restos obtenidos al dividir las potencias de 2 por 7 son {1, 2, 4, 1, 2, 4, . . .}, repitien-dose con periodo 3. Estos mismos restos se obtienen al dividir 16 por 7. Luego la unicaposibilidad para obtener resto 5 al restar es que

r = 3 + 1 y s = 3 + 2.

Estas son las condiciones pedidas.Para hallar el mınimo positivo n escribimos

n = 23k+1 − 163h+2 = 23k+1 − 212h+8.

Debera ser 2k +1 > 12h+8 (la funcion 2x es creciente) equivalente a 2k− 12h− 7 > 0.El mınimo se obtendra cuando 2k − 12h − 7 sea mınimo (la funcion 2x es convexa), yse ve facilmente que este mınimo se obtiene para k = 3 y h = 0 y resulta

n = 210 − 28 = 768.

Page 202: COMPENDIUM OMEFL - ToomatesXXXV Olimpiada Matemática Española Primera Fase Soluciones de la propuesta de problemas Problema 1 ¿Qué dígitos se han omitido en la siguiente multiplicación?

D3.-Los puntos A1, A2, . . . , A2n son los vertices de un polıgono regular de 2n lados. Hallarel numero de ternas Ai, Aj, Ak tales que el triangulo AiAjAk es rectangulo y el numerode ternas tales que el triangulo es acutangulo.

Solucion.Al ser 2n par, podremos formar triangulos rectangulos que tendran la hipotenusa sobrelos n diametros del polıgono. Para cada diametro fijado, el angulo recto del triangulorectangulo puede ser cualquiera de los 2n − 2 vertices sobrantes. En total habra Rn =n(2n − 2) = 2n(n − 1) triangulos rectangulos.

Para calcular los acutangulos podemos cal-cular ahora los obtusangulos y restar del to-tal. Observemos que cualquier triangulo ob-tusangulo dejara el centro O (su circuncen-tro) fuera de el. Si lo giramos en sentido di-recto o inverso alrededor de O podemos con-seguir que uno de sus vertices agudos esteen A1. Los otros dos estan, bien en el con-junto {A2, . . . , An}, bien en {An+2, . . . , A2n}.El numero buscado sera 2

(n−1

2

). Como

esto lo podemos hacer con cada uno de los2n vertices, quedaran 2(2n)

(n−1

2

)triangulos.

Pero cada triangulo lo hemos contado dos ve-ces, una para cada vertice agudo.

A1

A2

AnAn+1

An+2

A2n

O

Luego el numero de triangulos obtusangulos sera On = 2n(n−1

2

).

El numero de los acutangulos sera el numero total(2n3

)menos los rectangulo y ob-

tusangulos.

An =(

2n

3

)− Rn − On = · · · calculando · · · =

n(n − 1)(n − 2)3

= 2(

n

3

).

Solucion alternativa. Fijemos un vertice agudo en un vertice del polıgono, porejemplo, el A1. Los tres lados del triangulo abarcaran respectivamente x, y y z ladosdel polıgono de 2n lados. Sera x + y + z = 2n. Al ser los tres angulos agudos deberaser 0 < x, y, z < n. Calculemos el numero de soluciones enteras positivas de la ecuacionx + y + z = 2n con la condicion fijada para la z.Si z = 1, queda x + y = 2n− 1 que no tiene soluciones. Si z = 2, queda x + y = 2n− 2que tiene 1 solucion. Si z = 3, queda x + y = 2n − 3 que tiene 2 soluciones. Etc. Siz = n − 1, queda x + y = n + 1 que tiene n − 2 soluciones.En total hay (n− 2) + · · ·+2 +1 = (n−2)(n−1

2 =(n−1

2

)soluciones con un angulo en A1.

Si consideramos las otras posibles posiciones para dicho angulo queda en total 2n(n−1

2

).

Pero hemos contado cada triangulo tres veces, luego queda

An =n(n − 1)(n − 2)

3= 2

(n

3

).

Page 203: COMPENDIUM OMEFL - ToomatesXXXV Olimpiada Matemática Española Primera Fase Soluciones de la propuesta de problemas Problema 1 ¿Qué dígitos se han omitido en la siguiente multiplicación?

OLIMPIADA MATEMATICA ESPANOLA

Fase LocalManana del Viernes

 

1 Hallar todas las soluciones enteras (x, y) de la ecuacion

yk = x2 + x

donde k es un numero entero dado mayor que 1.

2 Busca un polinomio de grado tres cuyas raıces sean, precisamente,el cuadrado de las raıces del polinomio p(x) = x3 + 2x2 + 3x + 4.

3 Deslizamos un cuadrado de 10 cm de lado por el plano OXY deforma que los vertices de uno de sus lados esten siempre en contactocon los ejes de coordenadas, uno con el eje OX y otro con el eje OY .Determina el lugar geometrico que en ese movimiento describen:

1. El punto medio del lado de contacto con los ejes.

2. El centro del cuadrado.

3. Los vertices del lado de contacto y del opuesto en el primer cua-drante.

No esta permitido el uso de calculadoras.Cada problema se puntua sobre 7 puntos.

El tiempo de cada sesion es de 3 horas y media.

Page 204: COMPENDIUM OMEFL - ToomatesXXXV Olimpiada Matemática Española Primera Fase Soluciones de la propuesta de problemas Problema 1 ¿Qué dígitos se han omitido en la siguiente multiplicación?

OLIMPIADA MATEMATICA ESPANOLA

Fase LocalTarde del Viernes

 

4 Calcula la suma de los inversos de los dos mil trece primeros terminosde la sucesion de termino general

an = 1−1

4n2

5 Obten los dos valores enteros de x mas proximos a 2013◦, tanto pordefecto como por exceso, que cumplen esta ecuacion trigonometrica:

2sin2 x + 2cos2 x = 2√2

6 Por los puntos medios de dos lados de un triangulo ABC trazamoslas medianas y unimos los puntos que trisecan el tercer lado conel vertice opuesto. Ası, en el interior, se obtiene una pajarita (dostriangulos unidos por un vertice). Se pide calcular la fraccion de su-perficie total del triangulo que representa la pajarita.

No esta permitido el uso de calculadoras.Cada problema se puntua sobre 7 puntos.

El tiempo de cada sesion es de 3 horas y media.

Page 205: COMPENDIUM OMEFL - ToomatesXXXV Olimpiada Matemática Española Primera Fase Soluciones de la propuesta de problemas Problema 1 ¿Qué dígitos se han omitido en la siguiente multiplicación?

OLIMPIADA MATEMATICA ESPANOLA

Fase LocalTarde del Viernes

 

4 Calcula la suma de los inversos de los dos mil trece primeros terminosde la sucesion de termino general

an = 1−1

4n2

5 Obten los dos valores enteros de x mas proximos a 2013◦, tanto pordefecto como por exceso, que cumplen esta ecuacion trigonometrica:

2sin2 x + 2cos2 x = 2√2

6 Por los puntos medios de dos lados de un triangulo ABC trazamoslas medianas y unimos los puntos que trisecan el tercer lado conel vertice opuesto. Ası, en el interior, se obtiene una pajarita (dostriangulos unidos por un vertice). Se pide calcular la fraccion de su-perficie total del triangulo que representa la pajarita.

No esta permitido el uso de calculadoras.Cada problema se puntua sobre 7 puntos.

El tiempo de cada sesion es de 3 horas y media.

Page 206: COMPENDIUM OMEFL - ToomatesXXXV Olimpiada Matemática Española Primera Fase Soluciones de la propuesta de problemas Problema 1 ¿Qué dígitos se han omitido en la siguiente multiplicación?

OLIMPIADA MATEMATICA ESPANOLA

Fase LocalManana del Sabado

 

1 Dado un numero entero n escrito en el sistema de numeracion de-cimal, formamos el numero entero k restando del numero formado porlas tres ultimas cifras de n el numero formado por las cifras anterioresrestantes. Demostrar que n es divisible por 7, 11 o 13 si y solo si ktambien lo es.

2 Prueba que las sumas de las primeras, segundas y terceras poten-cias de las raıces del polinomio p(x) = x3 + 2x2 + 3x + 4 valen lomismo.

3 En una sala de baile hay 15 chicos y 15 chicas dispuestos en dosfilas paralelas de manera que se formaran 15 parejas de baile. Sucedeque la diferencia de altura entre el chico y la chica de cada pareja nosupera los 10 cm. Demostrar que si colocamos los mismos chicos ychicas en dos filas paralelas en orden creciente de alturas, tambiensucedera que la diferencia de alturas entre los miembros de las nuevasparejas ası formadas no superaran los 10 cm.

No esta permitido el uso de calculadoras.Cada problema se puntua sobre 7 puntos.

El tiempo de cada sesion es de 3 horas y media.

Page 207: COMPENDIUM OMEFL - ToomatesXXXV Olimpiada Matemática Española Primera Fase Soluciones de la propuesta de problemas Problema 1 ¿Qué dígitos se han omitido en la siguiente multiplicación?

OLIMPIADA MATEMATICA ESPANOLA

Fase LocalManana del Sabado

 

1 Dado un numero entero n escrito en el sistema de numeracion de-cimal, formamos el numero entero k restando del numero formado porlas tres ultimas cifras de n el numero formado por las cifras anterioresrestantes. Demostrar que n es divisible por 7, 11 o 13 si y solo si ktambien lo es.

2 Prueba que las sumas de las primeras, segundas y terceras poten-cias de las raıces del polinomio p(x) = x3 + 2x2 + 3x + 4 valen lomismo.

3 En una sala de baile hay 15 chicos y 15 chicas dispuestos en dosfilas paralelas de manera que se formaran 15 parejas de baile. Sucedeque la diferencia de altura entre el chico y la chica de cada pareja nosupera los 10 cm. Demostrar que si colocamos los mismos chicos ychicas en dos filas paralelas en orden creciente de alturas, tambiensucedera que la diferencia de alturas entre los miembros de las nuevasparejas ası formadas no superaran los 10 cm.

No esta permitido el uso de calculadoras.Cada problema se puntua sobre 7 puntos.

El tiempo de cada sesion es de 3 horas y media.

Page 208: COMPENDIUM OMEFL - ToomatesXXXV Olimpiada Matemática Española Primera Fase Soluciones de la propuesta de problemas Problema 1 ¿Qué dígitos se han omitido en la siguiente multiplicación?

OLIMPIADA MATEMATICA ESPANOLA

Fase LocalTarde del Sabado

 

4 Demuestra que el producto de los dos mil trece primeros terminosde la sucesion

an = 1 +1

n3

no llega a valer 3.

5 Resuelve esta ecuacion exponencial

2x · 35−x

+35x

2x= 6

6 Sean A,B y C los vertices de un triangulo y P,Q y R los respectivospies de las bisectrices trazadas desde esos mismos vertices. Sabiendoque PQR es un triangulo rectangulo en P , se te pide probar dos cosas:

a) Que ABC ha de ser obtusangulo.

b) Que en el cuadrilatero ARPQ, pese a no ser cıclico, la suma de susangulos opuestos es constante.

No esta permitido el uso de calculadoras.Cada problema se puntua sobre 7 puntos.

El tiempo de cada sesion es de 3 horas y media.

Page 209: COMPENDIUM OMEFL - ToomatesXXXV Olimpiada Matemática Española Primera Fase Soluciones de la propuesta de problemas Problema 1 ¿Qué dígitos se han omitido en la siguiente multiplicación?

OLIMPIADA MATEMATICA ESPANOLA

Fase LocalManana del Viernes

 

1 Hallar todas las soluciones enteras (x, y) de la ecuacion

yk = x2 + x

donde k es un numero entero dado mayor que 1.

Solucion. Puesto que yk = x2 + x = x(x + 1) y mcd(x, x + 1) = 1resulta que tanto x + 1 como x deben ser potencias k-esimas de unentero. Pero los dos unicos numeros enteros consecutivos que sonpotencias k-esimas, con k > 1 son 0 y 1 o bien -1 y 0 . Las dos unicassoluciones son, pues, x = 0, y = 0 y x = −1, y = 0.

2 Busca un polinomio de grado tres cuyas raıces sean, precisamente, elcuadrado de las raıces del polinomio p(x) = x3 + 2x2 + 3x+ 4.

Solucion. Sean r, s y t las raıces, reales o complejas, del polinomiop(x). Por tanto, p(x) = (x − r)(x − s)(x − t). El polinomio que bus-camos, salvo que multipliquemos por una constante, sera de la formaq(x) = (x− r2)(x− s2)(x− t2). De aquı resulta

q(x2) = (x2−r2)(x2−s2)(x2−t2) = (x−r)(x+r)(x−s)(x+s)(x−t)(x+t)

Y notando que

p(−x) = (−x− r)(−x− s)(−x− t) = −(x+ r)(x+ s)(x+ t),

entonces

q(x2) = (x− r)(x+ r)(x− s)(x+ s)(x− t)(x+ t) = p(x)[−p(−x)]= (x3 + 2x2 + 3x+ 4)(x3 − 2x2 + 3x− 4) = x6 + 2x4 − 7x2 − 16

Luego, q(x) = x3 + 2x2 − 7x− 16 es solucion (y, tambien, este mismopolinomio multiplicado por una constante cualquiera).

3 Deslizamos un cuadrado de 10 cm de lado por el planoOXY de formaque los vertices de uno de sus lados esten siempre en contacto con losejes de coordenadas, uno con el eje OX y otro con el eje OY . Determinael lugar geometrico que en ese movimiento describen:

1

Page 210: COMPENDIUM OMEFL - ToomatesXXXV Olimpiada Matemática Española Primera Fase Soluciones de la propuesta de problemas Problema 1 ¿Qué dígitos se han omitido en la siguiente multiplicación?

1. El punto medio del lado de contacto con los ejes.

2. El centro del cuadrado.

3. Los vertices del lado de contacto y del opuesto en el primer cua-drante.

Solucion. Sean PQRS el cuadrado de lado 10 cm, PQ el lado deapoyo, M(m1,m2) el punto medio de dicho lado y C(c1, c2) el centrodel cuadrado tal y como muestra la figura donde, ademas, senalamoslos puntos A, B, D y E.

C

A

O

Q

B

M

R

D

PE

S

 

a) Caso del punto medio M .

OM = PM =1

2PQ = 5,

luego m21 +m2

2 = 25.

b) Caso del centro del cuadrado C.

Los triangulosAQM , AOM , BMO yDMC son claramente congruen-tes

AM = OB = DC,AQ = OA = MD = BM, y OM = MQ = MC = 5.

Ası, resulta que las coordenadas del centro del cuadrado, en su desliza-miento, son iguales

c1 = OE = OB +BE = m1 +MD = m1 +m2,

c2 = EC = ED +DC = OA+AM = m2 +m1

Luego, el centro del cuadrado se mueve, en este primer cuadrante, so-bre un segmento de la lınea. Las posiciones extremas se dan cuandoel lado PQ se apoya sobre alguno de los ejes, C(5, 5), y cuando formauna escuadra, esto es, un triangulo rectangulo isosceles, con ellos,

2

Page 211: COMPENDIUM OMEFL - ToomatesXXXV Olimpiada Matemática Española Primera Fase Soluciones de la propuesta de problemas Problema 1 ¿Qué dígitos se han omitido en la siguiente multiplicación?

C(5√2, 5√2). Trabajando analogamente en los demas cuadrantes pode-

mos afirmar que el centro del cuadrado recorre el segmento de susbisectrices que viene dado por la expresion

C(c1, c2) = (±5λ,±5λ) con λ ∈ [1,√2].

c) Caso de los vertices del cuadrado en el lado de contacto: P y Q.

Los vertices P yQ se mueven sobre segmentos de los ejes coordenados,esto es, de las lıneas x = 0 y y = 0.

Soluciones a los Problemas Propuestos para la 49ª O. M. E. 5

Trabajando análogamente en los demás cuadrantes podemos afirmar que el centro del cuadrado recorre el segmento de sus bisectrices que viene dado por la expresión:

))))5555,,,,5555(((())))cccc,,,,cccc((((CCCC 22221111 λλλλ±±±±λλλλ±±±±==== con ]]]]2222,,,,1111[[[[∈∈∈∈λλλλ

III) III) III) III) Caso de los vértices del cuadrado:Caso de los vértices del cuadrado:Caso de los vértices del cuadrado:Caso de los vértices del cuadrado:

- Vértices del lado de contVértices del lado de contVértices del lado de contVértices del lado de contacto: P y Q.acto: P y Q.acto: P y Q.acto: P y Q.

Los vértices PPPP y QQQQ se mueven sobre segmentos de los ejes coordenados, esto es, de las líneas 0000yyyy ==== y 0000xxxx ==== .

Los casos extremos se dan cuando el lado de contacto descansa sobre los ejes.

Así: si las coordenadas de uno son )))),,,,0000(((( λλλλ , las del otro ))))0000,,,,100100100100(((( 2222λλλλ−−−−±±±± y

si las coordenadas de uno son ))))0000,,,,((((λλλλ , las del otro ))))100100100100,,,,0000(((( 2222λλλλ−−−−±±±±

con ]]]]10101010,,,,10101010[[[[−−−−∈∈∈∈λλλλ

- Vértices del lado opuesto al de contacto: R y S.Vértices del lado opuesto al de contacto: R y S.Vértices del lado opuesto al de contacto: R y S.Vértices del lado opuesto al de contacto: R y S.

De nuevo, apoyándonos en la figura, por ser congruentes los triángulos OQPOQPOQPOQP, QHRQHRQHRQHR y PFS PFS PFS PFS y, a la vez, semejantes a AQM AQM AQM AQM:

))))rrrr,,,,rrrr((((RRRR 22221111

22221111 mmmm2222rrrr ====

222211112222 mmmm2222mmmm2222rrrr ++++====

Total:: 2222

rrrrrrrrmmmm 11112222

1111−−−−

==== y2222rrrr

mmmm 11112222 ====

))))ssss,,,,ssss((((SSSS 22221111

222211111111 mmmm2222mmmm2222ssss ++++====

11112222 mmmm2222ssss ====

Total: 2222ssss

mmmm 22221111==== y

2222ssssssss

mmmm 222211112222

−−−−====

��

��

Como sabemos que 25252525mmmmmmmm 22222222

22221111 ====++++

En RRRR: 252525252222rrrr

2222rrrrrrrr

22221111

222211112222 ====⎟⎟⎟⎟

⎠⎠⎠⎠⎞⎞⎞⎞

⎜⎜⎜⎜⎝⎝⎝⎝⎛⎛⎛⎛++++⎟⎟⎟⎟

⎠⎠⎠⎠⎞⎞⎞⎞

⎜⎜⎜⎜⎝⎝⎝⎝⎛⎛⎛⎛ −−−−

→ 100100100100rrrr))))rrrrrrrr(((( 22221111

222211112222 ====++++−−−−

El lugar geométrico descrito tendría por ecuación: 100100100100xxxx))))xxxxyyyy(((( 22222222 ====++++−−−− ,un sector de elipse que también se puede expresar así:

2222xxxx100100100100xxxxyyyy −−−−++++==== con ]]]]10101010,,,,0000[[[[xxxx ∈∈∈∈ e ]]]]222210101010,,,,10101010[[[[yyyy ∈∈∈∈

Los casos extremos se dan cuando el lado de contacto descansa so-bre los ejes. Ası: si las coordenadas de uno son (0, λ) , las del otro(±√100− λ2, 0) y si las coordenadas de uno son (λ, 0) , las del otro

son (0,±√100− λ2), con λ ∈ [−10, 10].

d) Caso de los vertices del cuadrado en el lado opuesto al de contacto:R y S.

De nuevo, apoyandonos en la figura, por ser congruentes los triangulosOQP , QHR y PFS y, a la vez, semejantes a AQM :

R(r1, r2)r1 = 2m2r2 = 2m1 +m2

de donde m1 = r2−r12

, m2 = r1/2. Como sabemos que m21 +m2

2 = 25,tenemos para R

(r2 − r1

2)2 + (

r1

2)2 = 25

o bien (r2−r1)2+r21 = 100. El lugar geometrico esta, pues, en la elipsede ecuacion (y − x)2 + x2 = 100 y es un arco de elipse que se puedeparametrizar como

y = x+√100− x2

con x ∈ [0, 10] e y ∈ [10, 10√2]. Analogamente, para S sale el arco de

elipse y2 + (x− y)2 = 100 con

x = y +√100− y2

3

Page 212: COMPENDIUM OMEFL - ToomatesXXXV Olimpiada Matemática Española Primera Fase Soluciones de la propuesta de problemas Problema 1 ¿Qué dígitos se han omitido en la siguiente multiplicación?

con y ∈ [0, 10] y x ∈ [10, 10√2].

En los demas cuadrantes sale de forma parecida:

Segundo cuadrante:y = −x+

√100− x2

con x ∈ [−10, 0] e y ∈ [10, 10√2].

x = −y −√

100− y2

con y ∈ [0, 10] y x ∈ [−10√2,−10].

Tercer cuadrante:y = x−

√100− x2

con x ∈ [−10, 0] e y ∈ [−10,−10√2].

x = y −√

100− y2

con x ∈ [−10, 0] e y ∈ [−10√2,−10].

Cuarto cuadrante:y = −x−

√100− x2

con x ∈ [0, 10] e y ∈ [−10,−10√2].

x = −y +√

100− y2

con y ∈ [−10, 0] y x ∈ [10, 10√2].

4

Page 213: COMPENDIUM OMEFL - ToomatesXXXV Olimpiada Matemática Española Primera Fase Soluciones de la propuesta de problemas Problema 1 ¿Qué dígitos se han omitido en la siguiente multiplicación?

OLIMPIADA MATEMATICA ESPANOLA

Fase LocalTarde del Viernes

 

4 Calcula la suma de los inversos de los dos mil trece primeros terminosde la sucesion de termino general

an = 1−1

4n2

Solucion. El termino general se puede escribir como

an =4n2 − 1

4n2=

(2n− 1)(2n+ 1)

4n2

y su inverso es

1

an

=4n2

(2n− 1)(2n+ 1)=

n

2n− 1+

n

2n+ 1

Hemos de calcular

S =1

a1

+1

a2

+1

a3

+ . . .+1

a2012

+1

a2013

=

(1

1+

1

3

)+

(2

3+

2

5

)+ . . .+

(2012

4023+

2012

4025

)+

(2013

4025+

2013

4027

)= 1 +

(1

3+

3

3

)+

(2

5+

3

5

)+ . . .+

(2012

4025+

2013

4025

)+

2013

4027

= 2013 +2013

4027= 2013

(1 +

1

4027

)=

8108364

4027' 2013.5

5 Obten los dos valores enteros de x mas proximos a 2013◦, tanto pordefecto como por exceso, que cumplen esta ecuacion trigonometrica

2sin2 x + 2cos2 x = 2√2

5

Page 214: COMPENDIUM OMEFL - ToomatesXXXV Olimpiada Matemática Española Primera Fase Soluciones de la propuesta de problemas Problema 1 ¿Qué dígitos se han omitido en la siguiente multiplicación?

Solucion. Aplicando la desigualdad entre las medias aritmetica ygeometrica resulta

2sin2 x + 2cos2 x ≥ 2√2sin2 x · 2cos2 x = 2

√2sin2 x+cos2 x = 2

√2

La igualdad se alcanza cuando 2sin2 x = 2cos2 x. Es decir, cuando sin2 x =cos2 x o sinx = ± cosx. Los valores de x que satisfacen la igualdad an-terior son x = 45◦ + 90◦k con k ∈ Z. Los valores pedidos se obtienenpara

k1 =

[2013◦ − 45◦

90

]= 21 y k1 =

[2013◦ + 45◦

90

]= 22

y son x1 = 1935◦ y x2 = 2025◦.

6 Por los puntos medios de dos lados de un triangulo ABC trazamoslas medianas y unimos los puntos que trisecan el tercer lado con elvertice opuesto. Ası, en el interior, se obtiene una pajarita (dos triangulosunidos por un vertice). Se pide calcular la fraccion de superficie total deltriangulo que representa la pajarita.

Solucion. Supongamos, sin perdida de generalidad, que el area deltriangulo ABC es uno. Es decir, [ABC] = 1. Tenemos

GM

A P

D

EF

B

N

Q

H

MY

A

h

P

2h/3

E

Z

X

h

B

V

h/3

Q

H

Las medianas dividen al triangulo en seis partes de igual area, luego:

[AMF ] = [FNC] =1

6y [AFC] = 2 ·

1

6=

1

3. Trazamos, desde B,A, P

y Q, perpendiculares sobre la mediana CM y sean X,Y, Z y V susrespectivos pies tal y como muestra la figura de la derecha. En estafigura se tiene trabajando con la mediana CM :

• AMY ≡ BMX. Como AM = MB entonces AY = XB = h.

• AY C ∼= PZC ∼= QV C. Como AP = PQ = QC, entonces PZ =2

3h y QV =

1

3h.

6

Page 215: COMPENDIUM OMEFL - ToomatesXXXV Olimpiada Matemática Española Primera Fase Soluciones de la propuesta de problemas Problema 1 ¿Qué dígitos se han omitido en la siguiente multiplicación?

• PEZ ∼= XBE. Como PZ =2

3XB → PE =

2

3EB → PE =

2

5PB y EB =

3

5PB.

• QHV ∼= XBH. Como QV =1

3QB → QH =

1

3HB → QH =

1

4QB

Y trabajando, analogamente1, sobre la otra mediana AN, se obtiene

QG =2

5QB GB =

3

5QB y PD =

1

4PB

Nos fijamos, por ejemplo, en la ceviana PB y ya podemos conocer laproporcion en que los puntos D y E la dividen. Falta

DE = PE − PD =

(2

5−

1

4

)PB =

3

20PB

Por tanto,

PD =5

20PB DE =

3

20PB y EB =

12

20PB

Ahora dibujamos la lınea auxiliar AE. Sobre la ceviana PB,ABP dearea conocida queda dividido en tres triangulos, ABE,AED y ADP ,de identica altura. Por tanto,

[ADP ]

[ABP ]=PD

PB=

5

20,

[AED]

[ABP ]=DE

PB=

3

20,

[ABE]

[ABP ]=EB

PB=

12

20

De

[ABP ] =1

3→ [ADP ] =

5

60, [AED] =

3

60y [ABE] =

12

60

Por otro lado, como EM es mediana del triangulo ABE, se tiene

[AME] =1

2[ABE] =

6

60. Finalmente,

1

6= [AMF ] = [AME] + [AED] + [DEF ] =

6

60+

3

60+ [DEF ]

de donde resulta el area de media pajarita. Es decir, [DEF ] =1

60.

Analogamente, [FGH] =1

60, y el area pedida es

[DEF ] + [FGH] =1

30=

1

30[ABC]

1Todo esto es equivalente a aplicar el Teorema de Menelao sobre las transversalesde un triangulo

7

Page 216: COMPENDIUM OMEFL - ToomatesXXXV Olimpiada Matemática Española Primera Fase Soluciones de la propuesta de problemas Problema 1 ¿Qué dígitos se han omitido en la siguiente multiplicación?

OLIMPIADA MATEMATICA ESPANOLA

Fase LocalTarde del Viernes

 

4 Calcula la suma de los inversos de los dos mil trece primeros terminosde la sucesion de termino general

an = 1−1

4n2

Solucion. El termino general se puede escribir como

an =4n2 − 1

4n2=

(2n− 1)(2n+ 1)

4n2

y su inverso es

1

an

=4n2

(2n− 1)(2n+ 1)=

n

2n− 1+

n

2n+ 1

Hemos de calcular

S =1

a1

+1

a2

+1

a3

+ . . .+1

a2012

+1

a2013

=

(1

1+

1

3

)+

(2

3+

2

5

)+ . . .+

(2012

4023+

2012

4025

)+

(2013

4025+

2013

4027

)= 1 +

(1

3+

3

3

)+

(2

5+

3

5

)+ . . .+

(2012

4025+

2013

4025

)+

2013

4027

= 2013 +2013

4027= 2013

(1 +

1

4027

)=

8108364

4027' 2013.5

5 Obten los dos valores enteros de x mas proximos a 2013◦, tanto pordefecto como por exceso, que cumplen esta ecuacion trigonometrica

2sin2 x + 2cos2 x = 2√2

5

Page 217: COMPENDIUM OMEFL - ToomatesXXXV Olimpiada Matemática Española Primera Fase Soluciones de la propuesta de problemas Problema 1 ¿Qué dígitos se han omitido en la siguiente multiplicación?

Solucion. Aplicando la desigualdad entre las medias aritmetica ygeometrica resulta

2sin2 x + 2cos2 x ≥ 2√2sin2 x · 2cos2 x = 2

√2sin2 x+cos2 x = 2

√2

La igualdad se alcanza cuando 2sin2 x = 2cos2 x. Es decir, cuando sin2 x =cos2 x o sinx = ± cosx. Los valores de x que satisfacen la igualdad an-terior son x = 45◦ + 90◦k con k ∈ Z. Los valores pedidos se obtienenpara

k1 =

[2013◦ − 45◦

90

]= 21 y k1 =

[2013◦ + 45◦

90

]= 22

y son x1 = 1935◦ y x2 = 2025◦.

6 Por los puntos medios de dos lados de un triangulo ABC trazamoslas medianas y unimos los puntos que trisecan el tercer lado con elvertice opuesto. Ası, en el interior, se obtiene una pajarita (dos triangulosunidos por un vertice). Se pide calcular la fraccion de superficie total deltriangulo que representa la pajarita.

Solucion. Supongamos, sin perdida de generalidad, que el area deltriangulo ABC es uno. Es decir, [ABC] = 1. Tenemos

GM

A P

D

EF

B

N

Q

H

MY

A

h

P

2h/3

E

Z

X

h

B

V

h/3

Q

H

Las medianas dividen al triangulo en seis partes de igual area, luego:

[AMF ] = [FNC] =1

6y [AFC] = 2 ·

1

6=

1

3. Trazamos, desde B,A, P

y Q, perpendiculares sobre la mediana CM y sean X,Y, Z y V susrespectivos pies tal y como muestra la figura de la derecha. En estafigura se tiene trabajando con la mediana CM :

• AMY ≡ BMX. Como AM = MB entonces AY = XB = h.

• AY C ∼= PZC ∼= QV C. Como AP = PQ = QC, entonces PZ =2

3h y QV =

1

3h.

6

Page 218: COMPENDIUM OMEFL - ToomatesXXXV Olimpiada Matemática Española Primera Fase Soluciones de la propuesta de problemas Problema 1 ¿Qué dígitos se han omitido en la siguiente multiplicación?

• PEZ ∼= XBE. Como PZ =2

3XB → PE =

2

3EB → PE =

2

5PB y EB =

3

5PB.

• QHV ∼= XBH. Como QV =1

3QB → QH =

1

3HB → QH =

1

4QB

Y trabajando, analogamente1, sobre la otra mediana AN, se obtiene

QG =2

5QB GB =

3

5QB y PD =

1

4PB

Nos fijamos, por ejemplo, en la ceviana PB y ya podemos conocer laproporcion en que los puntos D y E la dividen. Falta

DE = PE − PD =

(2

5−

1

4

)PB =

3

20PB

Por tanto,

PD =5

20PB DE =

3

20PB y EB =

12

20PB

Ahora dibujamos la lınea auxiliar AE. Sobre la ceviana PB,ABP dearea conocida queda dividido en tres triangulos, ABE,AED y ADP ,de identica altura. Por tanto,

[ADP ]

[ABP ]=PD

PB=

5

20,

[AED]

[ABP ]=DE

PB=

3

20,

[ABE]

[ABP ]=EB

PB=

12

20

De

[ABP ] =1

3→ [ADP ] =

5

60, [AED] =

3

60y [ABE] =

12

60

Por otro lado, como EM es mediana del triangulo ABE, se tiene

[AME] =1

2[ABE] =

6

60. Finalmente,

1

6= [AMF ] = [AME] + [AED] + [DEF ] =

6

60+

3

60+ [DEF ]

de donde resulta el area de media pajarita. Es decir, [DEF ] =1

60.

Analogamente, [FGH] =1

60, y el area pedida es

[DEF ] + [FGH] =1

30=

1

30[ABC]

1Todo esto es equivalente a aplicar el Teorema de Menelao sobre las transversalesde un triangulo

7

Page 219: COMPENDIUM OMEFL - ToomatesXXXV Olimpiada Matemática Española Primera Fase Soluciones de la propuesta de problemas Problema 1 ¿Qué dígitos se han omitido en la siguiente multiplicación?

OLIMPIADA MATEMATICA ESPANOLA

Fase LocalManana del Sabado

 

1 Dado un numero entero n escrito en el sistema de numeracion de-cimal, formamos el numero entero k restando del numero formado porlas tres ultimas cifras de n el numero formado por las cifras anterioresrestantes. Demostrar que n es divisible por 7, 11 o 13 si y solo si ktambien lo es.

Solucion. SeaA el numero formado por las tres ultimas cifras de n yBel numero formado por las cifras anteriores. Entonces n = 1000B +Ay k = A − B. Tenemos n − k = 1001B = 7 · 11 · 13B y n y k soncongruentes modulo 7, 11 y 13.

2 Prueba que las sumas de las primeras, segundas y terceras potenciasde las raıces del polinomio p(x) = x3 + 2x2 + 3x+ 4 valen lo mismo.

Solucion. Sean r, s y t las raıces, reales o complejas, del polinomiop(x) y sea Sn la suma de sus n−esimas potencias, esto es, Sn = rn +sn+ tn. Por un lado, teniendo en cuenta las formulas de Cardano-Vietaresulta que S1 = r + s + t = −2, rs + st + tr = 3 y rst = −4 lo quenos permite calcular S2. En efecto,

S2 = r2 + s2 + t2 = (r + s+ t)2 − 2(rs+ st+ tr) = −2

Y por otro lado:

p(r) = r3 + 2r2 + 3r + 4 = 0 → r3 = −2r2 − 3r − 4p(s) = s3 + 2s2 + 3s+ 4 = 0 → s3 = −2s2 − 3s− 4p(t) = t3 + 2t2 + 3t+ 4 = 0 → t3 = −2t2 − 3t− 4

Sumando las expresiones anteriores, resulta

S3 = r3 + s3 + t3 = −2S2 − 3S1 − 12 = −2

quedando probado que las sumas de las tres primeras potencias de lasraıces del polinomio p(x) valen lo mismo: s1 = S2 = S3 = −2.

3 En una sala de baile hay 15 chicos y 15 chicas dispuestos en dos filasparalelas de manera que se formaran 15 parejas de baile. Sucede que

8

Page 220: COMPENDIUM OMEFL - ToomatesXXXV Olimpiada Matemática Española Primera Fase Soluciones de la propuesta de problemas Problema 1 ¿Qué dígitos se han omitido en la siguiente multiplicación?

la diferencia de altura entre el chico y la chica de cada pareja no superalos 10 cm. Demostrar que si colocamos los mismos chicos y chicas endos filas paralelas en orden creciente de alturas, tambien sucedera quela diferencia de alturas entre los miembros de las nuevas parejas asıformadas no superaran los 10 cm.

Solucion. Sean P1, P2,. . .P15 las quince parejas iniciales. Ordenemosahora los chicos por alturas a1 ≤ a2 ≤ · · · ≤ a15 y tambien las chicasb1 ≤ b2 ≤ · · · ≤ b15. Supongamos que una de las parejas tuviese unadiferencia de alturas superior a 10 cm, digamos ak − bk > 10. En-tonces las parejas formadas por las chicas de alturas b1, . . . , bk y loschicos de alturas ak, . . . , a15 tambien cumpliran ai − bj > 10. Colo-quemos ahora cada una de las 16 personas mencionadas, de alturasb1, . . . , bk, ak, ak+1, . . . , a15, en las parejas (cajas) Ps iniciales, segun ellugar que ocupaban. Por el principio de las casillas (palomar), dos per-sonas compartiran la misma caja. Por lo tanto en las parejas inicialeshabıa una cuya diferencia de alturas era mayor que 10 cm, contra losupuesto.

9

Page 221: COMPENDIUM OMEFL - ToomatesXXXV Olimpiada Matemática Española Primera Fase Soluciones de la propuesta de problemas Problema 1 ¿Qué dígitos se han omitido en la siguiente multiplicación?

OLIMPIADA MATEMATICA ESPANOLA

Fase LocalManana del Sabado

 

1 Dado un numero entero n escrito en el sistema de numeracion de-cimal, formamos el numero entero k restando del numero formado porlas tres ultimas cifras de n el numero formado por las cifras anterioresrestantes. Demostrar que n es divisible por 7, 11 o 13 si y solo si ktambien lo es.

Solucion. SeaA el numero formado por las tres ultimas cifras de n yBel numero formado por las cifras anteriores. Entonces n = 1000B +Ay k = A − B. Tenemos n − k = 1001B = 7 · 11 · 13B y n y k soncongruentes modulo 7, 11 y 13.

2 Prueba que las sumas de las primeras, segundas y terceras potenciasde las raıces del polinomio p(x) = x3 + 2x2 + 3x+ 4 valen lo mismo.

Solucion. Sean r, s y t las raıces, reales o complejas, del polinomiop(x) y sea Sn la suma de sus n−esimas potencias, esto es, Sn = rn +sn+ tn. Por un lado, teniendo en cuenta las formulas de Cardano-Vietaresulta que S1 = r + s + t = −2, rs + st + tr = 3 y rst = −4 lo quenos permite calcular S2. En efecto,

S2 = r2 + s2 + t2 = (r + s+ t)2 − 2(rs+ st+ tr) = −2

Y por otro lado:

p(r) = r3 + 2r2 + 3r + 4 = 0 → r3 = −2r2 − 3r − 4p(s) = s3 + 2s2 + 3s+ 4 = 0 → s3 = −2s2 − 3s− 4p(t) = t3 + 2t2 + 3t+ 4 = 0 → t3 = −2t2 − 3t− 4

Sumando las expresiones anteriores, resulta

S3 = r3 + s3 + t3 = −2S2 − 3S1 − 12 = −2

quedando probado que las sumas de las tres primeras potencias de lasraıces del polinomio p(x) valen lo mismo: s1 = S2 = S3 = −2.

3 En una sala de baile hay 15 chicos y 15 chicas dispuestos en dos filasparalelas de manera que se formaran 15 parejas de baile. Sucede que

8

Page 222: COMPENDIUM OMEFL - ToomatesXXXV Olimpiada Matemática Española Primera Fase Soluciones de la propuesta de problemas Problema 1 ¿Qué dígitos se han omitido en la siguiente multiplicación?

la diferencia de altura entre el chico y la chica de cada pareja no superalos 10 cm. Demostrar que si colocamos los mismos chicos y chicas endos filas paralelas en orden creciente de alturas, tambien sucedera quela diferencia de alturas entre los miembros de las nuevas parejas asıformadas no superaran los 10 cm.

Solucion. Sean P1, P2,. . .P15 las quince parejas iniciales. Ordenemosahora los chicos por alturas a1 ≤ a2 ≤ · · · ≤ a15 y tambien las chicasb1 ≤ b2 ≤ · · · ≤ b15. Supongamos que una de las parejas tuviese unadiferencia de alturas superior a 10 cm, digamos ak − bk > 10. En-tonces las parejas formadas por las chicas de alturas b1, . . . , bk y loschicos de alturas ak, . . . , a15 tambien cumpliran ai − bj > 10. Colo-quemos ahora cada una de las 16 personas mencionadas, de alturasb1, . . . , bk, ak, ak+1, . . . , a15, en las parejas (cajas) Ps iniciales, segun ellugar que ocupaban. Por el principio de las casillas (palomar), dos per-sonas compartiran la misma caja. Por lo tanto en las parejas inicialeshabıa una cuya diferencia de alturas era mayor que 10 cm, contra losupuesto.

9

Page 223: COMPENDIUM OMEFL - ToomatesXXXV Olimpiada Matemática Española Primera Fase Soluciones de la propuesta de problemas Problema 1 ¿Qué dígitos se han omitido en la siguiente multiplicación?

OLIMPIADA MATEMATICA ESPANOLA

Fase LocalTarde del Sabado

 

4 Demuestra que el producto de los dos mil trece primeros terminos dela sucesion

an = 1 +1

n3

no llega a valer 3.

Solucion. Veamos por induccion que pn = a1 · a2 · a3 . . . an ≤ 3 −1

ny, ası, quedara probado para el caso particular n = 2013 que se pide

en el enunciado. Para n = 1 es p1 = a1 = 1 +1

13= 2 ≤ 3 −

1

1.

Supongamos que es cierto para n = k, pk = a1 · a2 · a3 . . . ak ≤ 3−1

k.

Hemos de probar que se cumple para n = k + 1. Es decir, hemos de

ver que pk+1 = a1 · a2 · a3 . . . ak · ak+1 ≤ 3−1

k + 1. En efecto,

pk+1 = a1 · a2 · a3 . . . ak · ak+1 = pk · ak+1 ≤(3−

1

k

)(3−

1

k

)

= 3−1

k+

3

(k + 1)3−

1

k(k + 1)3

Ahora falta ver que

3−1

k+

3

(k + 1)3−

1

k(k + 1)3≤ 3−

1

k + 1

lo caul es equivalente a probar que

3

(k + 1)3−

1

k(k + 1)3≤

1

−1

k + 1⇔ k2 − k + 2 =

(k −

1

2

)2

+3

4≥ 0

5 Resuelve la ecuacion exponencial

2x · 35−x

+35x

2x= 6

10

Page 224: COMPENDIUM OMEFL - ToomatesXXXV Olimpiada Matemática Española Primera Fase Soluciones de la propuesta de problemas Problema 1 ¿Qué dígitos se han omitido en la siguiente multiplicación?

Solucion. Aplicando la desigualdad de las medias aritmetica y geometricay, despues, una de de sus mas conocidas consecuencias ( la suma deun numero real positivo y su inverso es siempre mayor o igual que 2,y la igualdad solo se da para el numero 1) tenemos,

6 = 2x35−x

+ 2−x35x ≥ 2√2x35−x2−x35x = 6.

Y la igualdad se dara cuando los numeros mediados sean iguales.

2x35−x

= 2−x35x ⇔ 22x = 35x−5−x ⇔ 5x = 5−x

esto es, cuando x = 0 que sera, pues, la unica solucion de la ecuacion.

6 Sean A,B y C los vertices de un triangulo y P,Q y R los respectivospies de las bisectrices trazadas desde esos mismos vertices. Sabiendoque PQR es un triangulo rectangulo en P , se te pide probar dos cosas:

a) Que ABC ha de ser obtusangulo.

b) Que en el cuadrilatero ARPQ, pese a no ser cıclico, la suma de susangulos opuestos es constante.

Solucion. Para resolver el problema utilizaremos dos herramientasfundamentales en los problemas geometricos relativos a triangulos:el teorema de la bisectriz (La bisectriz de un angulo de un triangulocorta al lado opuesto en dos segmentos de longitudes proporcionalesa los otros dos lados del triangulo) y el teorema de Stewart (Si en untriangulo ABC consideramos un punto P en el lado AB , entonces secumple que AP 2 a = PB b2 + PC c2 − PB PC a).Sea ahora el punto P pie de la bisectriz de nuestro problema. Por elteorema de la bisectriz tenemos

PB =ac

b+ cy PC =

ab

b+ c.

Ahora, aplicando el teorema de Stewart queda

AP 2 =bc

(b+ c)2(b2 + 2bc+ c2 − a2)

y, substituyendo a2 por su expresion obtenida del teorema del coseno

AP =2bc

(b+ c)2cos

A

2.

Calculemos ahora los lados del triangulo PQR.

11

Page 225: COMPENDIUM OMEFL - ToomatesXXXV Olimpiada Matemática Española Primera Fase Soluciones de la propuesta de problemas Problema 1 ¿Qué dígitos se han omitido en la siguiente multiplicación?

Soluciones a los Problemas Propuestos para la 49ª O. M. E. 23

Solución. Solución. Solución. Solución.

Al hacernos un primer croquis de la situación intuimos que es el ángulo AAAA el que ha de ser obtuso:

C P

Q

A

B

R

Calculemos la longitud de los lados del triángulo PQRPQRPQRPQR:

- Aplicando el teorema del coseno en APQAPQAPQAPQ, haciendo uso de los resultados de los teoremas de la bisectriz y de Stewart anteriores, y operando con sumo cuidado:

[[[[ ]]]]

2222

22222222

2222

222222222222

2222

22222222

2222

222222222222

222222222222

2222

22222222

2222

222222222222

222222222222

))))ccccaaaa((((

ccccbbbb

))))ccccaaaa(((())))ccccbbbb((((

))))2222////AAAA((((coscoscoscos))))bbbbaaaa((((ccccbbbb4444

))))ccccaaaa((((

ccccbbbbccccbbbb(((())))ccccaaaa((((

))))ccccaaaa(((())))ccccbbbb((((

))))2222////AAAA((((coscoscoscosccccbbbb4444

))))ccccaaaa)()()()(ccccbbbb(((())))2222////AAAA((((coscoscoscosccccbbbb4444

))))ccccaaaa((((

ccccbbbb

))))ccccbbbb((((

))))2222////AAAA((((coscoscoscosccccbbbb4444

))))2222////AAAAcos(cos(cos(cos(QAQAQAQAAPAPAPAP2222QAQAQAQAAPAPAPAPPQPQPQPQ

++++++++

++++++++⋅⋅⋅⋅−−−−⋅⋅⋅⋅====

++++++++++++−−−−++++⋅⋅⋅⋅

++++++++⋅⋅⋅⋅⋅⋅⋅⋅

====

====++++++++

⋅⋅⋅⋅⋅⋅⋅⋅−−−−

++++++++

++++⋅⋅⋅⋅⋅⋅⋅⋅

====

====⋅⋅⋅⋅⋅⋅⋅⋅⋅⋅⋅⋅−−−−++++====

- Análogamente, aplicando el teorema del coseno en APRAPRAPRAPR.

2222

22222222

2222

222222222222

2222

22222222

2222

222222222222

222222222222

2222

22222222

2222

222222222222

222222222222

))))bbbbaaaa((((

ccccbbbb

))))bbbbaaaa(((())))ccccbbbb((((

))))2222////AAAA((((coscoscoscos))))ccccaaaa((((ccccbbbb4444

))))bbbbaaaa((((

ccccbbbb)])])])]ccccbbbb(((())))bbbbaaaa[([([([(

))))bbbbaaaa(((())))ccccbbbb((((

))))2222////AAAA((((coscoscoscosccccbbbb4444

))))bbbbaaaa)()()()(ccccbbbb(((())))2222////AAAA((((coscoscoscosccccbbbb4444

))))bbbbaaaa((((

ccccbbbb

))))ccccbbbb((((

))))2222////AAAA((((coscoscoscosccccbbbb4444

))))2222////AAAAcos(cos(cos(cos(RARARARAAPAPAPAP2222RARARARAAPAPAPAPPRPRPRPR

++++++++

++++++++⋅⋅⋅⋅−−−−⋅⋅⋅⋅====

====++++

++++++++−−−−++++⋅⋅⋅⋅++++++++

⋅⋅⋅⋅⋅⋅⋅⋅====

====++++++++

⋅⋅⋅⋅⋅⋅⋅⋅−−−−

++++++++

++++⋅⋅⋅⋅⋅⋅⋅⋅

====

====⋅⋅⋅⋅⋅⋅⋅⋅⋅⋅⋅⋅−−−−++++====

- Y, lo mismo, aplicando el teorema del coseno en AQRAQRAQRAQR.

))))ccccaaaa)()()()(bbbbaaaa((((AAAAcoscoscoscosccccbbbb2222

))))bbbbaaaa((((

ccccbbbb

))))ccccaaaa((((

ccccbbbb

AAAAcoscoscoscosbbbbaaaa

bcbcbcbcccccaaaa

bcbcbcbc2222

))))bbbbaaaa((((

ccccbbbb

))))ccccaaaa((((

ccccbbbb

AAAAcoscoscoscosRARARARAQAQAQAQA2222RARARARAQAQAQAQAQRQRQRQR

22222222

2222

22222222

2222

22222222

2222

22222222

2222

22222222

222222222222

++++++++⋅⋅⋅⋅⋅⋅⋅⋅−−−−

++++++++

++++====

====⋅⋅⋅⋅++++

⋅⋅⋅⋅++++

⋅⋅⋅⋅−−−−++++

++++++++

====

====⋅⋅⋅⋅⋅⋅⋅⋅⋅⋅⋅⋅−−−−++++====

Por el teorema del coseno y los teoremas de la bisectriz y de Stewartobtendremos

PR2 = AP 2+QA2−2AP RA cosA =4b2c2(a− b) cos2A/2

(b+ c)2(a+ b)+

b2c2

(a+ b)2,

QR2 = QA2+RA2−2QARA cosA =4b2c2(a− b) cos2A/2

(b+ c)2(a+ b)+

b2c2

(a+ b)2,

PR2 = AP 2+QA2−2AP RA cosA =b2c2

(a+ c)2+

b2c2

(a+ b)2−

2b2c2 cosA

(a+ b)(a+ c).

Como que el triangulo PQR es rectangulo PQ2 + PR2 = QR2, substi-tuyendo y simplificando, tenemos

2a2 − b2 − c2 + (2a2 + 2bc) cosA = 0

de donde sale cosA = −1/2, A = 120◦ y de aquı R+Q = 150◦.

12

Page 226: COMPENDIUM OMEFL - ToomatesXXXV Olimpiada Matemática Española Primera Fase Soluciones de la propuesta de problemas Problema 1 ¿Qué dígitos se han omitido en la siguiente multiplicación?

L Olimpiada Matematica Espanola

Primera Fase

Primera sesion

Viernes manana, 17 de enero de 2014

OlimpiadaMatemáticaEspañola RSME

1. Tenemos 50 fichas numeradas del 1 al 50, y hay que colorearlas de rojo oazul. Sabemos que la ficha 5 es de color azul. Para la coloracion del resto defichas se siguen las siguientes reglas:

a) Si la ficha con el numero x y la ficha con el numero y son de distinto color,entonces la ficha con el numero |x− y| se pinta de color rojo.

b) Si la ficha con el numero x y la ficha con el numero y son de distinto color yx · y es un numero entre 1 y 50 (incluyendo ambos), entonces la ficha con elnumero x · y se pinta de color azul.

Determinar cuantas coloraciones distintas se pueden realizar en el conjuntode fichas.

2. Determinar cuantas soluciones reales tiene la ecuacion

√2− x2 =

3√

3− x3

3. Sea ΔABC un triangulo y D, E y F tres puntos cualesquiera sobre los ladosAB, BC y CA respectivamente. Llamemos P al punto medio de AE, Q alpunto medio de BF y R al punto medio de CD. Probar que el area deltriangulo ΔPQR es la cuarta parte del area del triangulo ΔDEF .

No esta permitido el uso de calculadoras.Cada problema se puntua sobre 7 puntos.

El tiempo de cada sesion es de 3 horas y media.

Page 227: COMPENDIUM OMEFL - ToomatesXXXV Olimpiada Matemática Española Primera Fase Soluciones de la propuesta de problemas Problema 1 ¿Qué dígitos se han omitido en la siguiente multiplicación?

L Olimpiada Matematica Espanola

Primera Fase

Segunda sesion

Viernes tarde, 17 de enero de 2014

OlimpiadaMatemáticaEspañola RSME

4. Se considera un polıgono regular de 90 vertices, numerados del 1 al 90 demanera aleatoria. Probar que siempre podemos encontrar dos vertices con-secutivos cuyo producto es mayor o igual que 2014.

5. Hallar las soluciones enteras de la ecuacion

x4 + y4 = 3x3y

6. Probar que20142013 − 10132013 − 10012013

es multiplo de20143 − 10133 − 10013

No esta permitido el uso de calculadoras.Cada problema se puntua sobre 7 puntos.

El tiempo de cada sesion es de 3 horas y media.

Page 228: COMPENDIUM OMEFL - ToomatesXXXV Olimpiada Matemática Española Primera Fase Soluciones de la propuesta de problemas Problema 1 ¿Qué dígitos se han omitido en la siguiente multiplicación?

L Olimpiada Matematica Espanola

Primera Fase

Primera sesion

Viernes tarde, 17 de enero de 2014

OlimpiadaMatemáticaEspañola RSME

1. Se considera un polıgono regular de 90 vertices, numerados del 1 al 90 demanera aleatoria. Probar que siempre podemos encontrar dos vertices con-secutivos cuyo producto es mayor o igual que 2014.

2. Hallar las soluciones enteras de la ecuacion

x4 + y4 = 3x3y

3. Probar que20142013 − 10132013 − 10012013

es multiplo de20143 − 10133 − 10013

No esta permitido el uso de calculadoras.Cada problema se puntua sobre 7 puntos.

El tiempo de cada sesion es de 3 horas y media.

Page 229: COMPENDIUM OMEFL - ToomatesXXXV Olimpiada Matemática Española Primera Fase Soluciones de la propuesta de problemas Problema 1 ¿Qué dígitos se han omitido en la siguiente multiplicación?

L Olimpiada Matematica Espanola

Primera Fase

Segunda sesion

Sabado manana, 18 de enero de 2014

OlimpiadaMatemáticaEspañola RSME

4. Sean a, b numeros positivos. Probar que

a+ b ≥√ab+

√a2 + b2

2

5. Encontrar las tres ultimas cifras del numero 72014

6. De un prisma recto de base cuadrada, con lado de longitud L1, y alturaH, ex-traemos un tronco de piramide, no necesariamente recto, de bases cuadradas,con lados de longitud L1 (para la inferior) y L2 (para la superior), y alturaH. Las dos piezas obtenidas aparecen en la imagen siguiente:

L1L1

L2

Si el volumen del tronco de piramide es 2/3 del total del volumen del prisma,¿cual es el valor de L1/L2?

No esta permitido el uso de calculadoras.Cada problema se puntua sobre 7 puntos.

El tiempo de cada sesion es de 3 horas y media.

Page 230: COMPENDIUM OMEFL - ToomatesXXXV Olimpiada Matemática Española Primera Fase Soluciones de la propuesta de problemas Problema 1 ¿Qué dígitos se han omitido en la siguiente multiplicación?

L Olimpiada Matematica Espanola

Primera Fase

Primera sesion

Sabado manana, 18 de enero de 2014

OlimpiadaMatemáticaEspañola RSME

1. Sean a, b numeros positivos. Probar que

a+ b ≥√ab+

√a2 + b2

2

2. Encontrar las tres ultimas cifras del numero 72014

3. De un prisma recto de base cuadrada, con lado de longitud L1, y alturaH, ex-traemos un tronco de piramide, no necesariamente recto, de bases cuadradas,con lados de longitud L1 (para la inferior) y L2 (para la superior), y alturaH. Las dos piezas obtenidas aparecen en la imagen siguiente:

L1L1

L2

Si el volumen del tronco de piramide es 2/3 del total del volumen del prisma,¿cual es el valor de L1/L2?

No esta permitido el uso de calculadoras.Cada problema se puntua sobre 7 puntos.

El tiempo de cada sesion es de 3 horas y media.

Page 231: COMPENDIUM OMEFL - ToomatesXXXV Olimpiada Matemática Española Primera Fase Soluciones de la propuesta de problemas Problema 1 ¿Qué dígitos se han omitido en la siguiente multiplicación?

L Olimpiada Matematica Espanola

Primera Fase

Segunda sesion

Sabado tarde, 18 de enero de 2014

OlimpiadaMatemáticaEspañola RSME

4. Hallar para que valores del numero real a todas las raıces del polinomio, enla variable x,

x3 − 2x2 − 25x+ a

son numeros enteros.

5. Sean x e y numeros reales entre 0 y 1. Probar que

x3 + xy2 + 2 xy ≤ 2 x2y + x2 + x+ y

6. Consideramos un numero primo p. Debemos disenar un torneo de p-parchıssujeto a las siguientes reglas:

• En el torneo participan p2 jugadores.

• En cada partida juegan p jugadores.

• El torneo se divide en rondas. Las rondas se dividen en partidas. Cadajugador juega una, o ninguna, partida en cada ronda.

• Al final del torneo cada jugador se ha enfrentado exactamente una vez concada uno de los otros jugadores.

Determinar si es posible disenar un torneo ası. En caso afirmativo, obtenerel mınimo numero de rondas que puede tener el torneo.

No esta permitido el uso de calculadoras.Cada problema se puntua sobre 7 puntos.

El tiempo de cada sesion es de 3 horas y media.

Page 232: COMPENDIUM OMEFL - ToomatesXXXV Olimpiada Matemática Española Primera Fase Soluciones de la propuesta de problemas Problema 1 ¿Qué dígitos se han omitido en la siguiente multiplicación?

Sesion 1

1. Tenemos 50 fichas numeradas del 1 al 50, y hay que colorearlas de rojoo azul. Sabemos que la ficha 5 es de color azul. Para la coloracion delresto de fichas se siguen las siguientes reglas:

a) Si la ficha con el numero x y la ficha con el numero y son dedistinto color, entonces la ficha con el numero |x− y| se pinta decolor rojo.

b) Si la ficha con el numero x y la ficha con el numero y son dedistinto color y x·y es un numero entre 1 y 50 (incluyendo ambos),entonces la ficha con el numero x · y se pinta de color azul.

Determinar cuantas coloraciones distintas se pueden realizar en el con-junto de fichas.

Solucion. Observemos que dos numeros que se diferencian en 5 tienenel mismo color. En efecto, si fueran de distinto color, su diferenciadeberıa ser de color rojo, por la regla a). Pero su diferencia es 5, quees de color azul. Por tanto basta con saber el color de los 4 primerosnumeros. Aquı, distinguimos dos casos: 1) que la ficha 1 sea de colorazul y 2) que la ficha 1 sea de color rojo.

Caso 1) Si 1 es de color azul, el resto de fichas debera ser de color azul,por la regla b). Esto es ası porque si la ficha k 6= 1 fuera roja, entoncespor b), k = k · 1 tendrıa que ser azul, lo que contradice que k sea roja.

Caso 2) Si 1 es roja, por la regla a) 4 = 5− 1 es roja. Para determinarel color de 2 y 3, supongamos que 3 es azul. Por ser 2 = 3 − 1 y ser 3y 1 de diferente color, entonces 2 es roja. Ahora bien 3 = 5− 2 y 5 esazul y 2 roja, por lo tanto 3 es roja. Esto no puede ser, por lo tanto 3no puede ser azul y es roja, por lo que 2 tambien es roja. Ası pues, 1,2, 3 y 4 son rojas, lo mismo que el resto de fichas que no son multiplode 5.

Por tanto, solo hay dos coloraciones posibles, o todas las fichas de colorazul o todas rojas, excepto los multiplos de 5 que son azules.

2. Determinar cuantas soluciones reales tiene la ecuacion

√2− x2 =

3√

3− x3.

1

Page 233: COMPENDIUM OMEFL - ToomatesXXXV Olimpiada Matemática Española Primera Fase Soluciones de la propuesta de problemas Problema 1 ¿Qué dígitos se han omitido en la siguiente multiplicación?

Solucion. Para que existan soluciones reales tiene que ser x ∈ [−√

2,√

2].Ahora bien, si x ∈ [−

√2, 0] se tiene que

2− x2 ≤ 2, 3− x3 ≥ 3,

pero√

2 < 3√

3, por lo que no hay soluciones cuando x ∈ [−√

2, 0]. Porotra parte, cuando x ∈ (0,

√2] podemos ver que

3√

3− x3 > 3

√2√

2− x3.

Puesto que la ecuacion

√2− x2 =

3

√2√

2− x3

tiene como unicas soluciones x = 0 y x =√

2 y, para x = 1, resulta3√

2√

2− 1 >√

2− 1 = 1, es evidente que

3√

3− x3 > 3

√2√

2− x3 ≥√

2− x2,

por lo que tampoco pueden existir soluciones cuando x ∈ (0,√

2].

3. Sea ∆ABC un triangulo y D, E y F tres puntos cualesquiera sobre loslados AB, BC y CA respectivamente. Llamemos P al punto medio deAE, Q al punto medio de BF y R al punto medio de CD. Probar queel area del triangulo ∆PQR es la cuarta parte del area del triangulo∆DEF .

Solucion. Hagamos primero un dibujo donde queden reflejados loselementos que intervienen en el problema.Solución:

a) Observemos que, Como Q, P y R son los puntos medios de las correspondientes Cevianas AE, BF y CD , estos puntos se encuentran en los lados del triángulo que determinan los pies de las medianas de cada lado.

b) Ya sabemos que los triángulos {ABC} y {Ma Mb Mc} son semejantes con razón de semejanza ଵଶ

2

Page 234: COMPENDIUM OMEFL - ToomatesXXXV Olimpiada Matemática Española Primera Fase Soluciones de la propuesta de problemas Problema 1 ¿Qué dígitos se han omitido en la siguiente multiplicación?

Observemos que, como P , Q y R son los puntos medios de las corres-pondientes Cevianas AE, BF y CD, estos puntos se encuentran en loslados del triangulo que determinan los pies de las medianas de cadalado, como se ve en la figura siguiente.

Solución:

a) Observemos que, Como Q, P y R son los puntos medios de las correspondientes Cevianas AE, BF y CD , estos puntos se encuentran en los lados del triángulo que determinan los pies de las medianas de cada lado.

b) Ya sabemos que los triángulos {ABC} y {Ma Mb Mc} son semejantes con razón de semejanza ଵଶ

Los triangulos ∆ABC y ∆MaMbMc son semejantes con razon de se-mejanza 1/2, por lo que se tiene

McMa

AC=McQ

AF=QMa

FC=

1

2,

MaMb

BA=MaR

BD=RMb

DA=

1

2,

MbMc

CB=MbP

CE=PMc

EB=

1

2.

Ademas, los angulos en A, B y C son iguales, respectivamente, a losangulos en Ma, Mb y Mc.

Sea u =AD

AB, v =

BE

BCy w =

CF

CA, por lo que

DB

AB= 1− u, EC

BC= 1− v, EC

BC= 1− w.

Aplicando la semejanza de los triangulos ∆ABC y ∆MaMbMc, resulta

u =RMb

MaMb

, v =PMc

MbMc

, w =QMa

McMa

,

3

Page 235: COMPENDIUM OMEFL - ToomatesXXXV Olimpiada Matemática Española Primera Fase Soluciones de la propuesta de problemas Problema 1 ¿Qué dígitos se han omitido en la siguiente multiplicación?

MaR

MaMb

= 1− u, MbP

MbMc

= 1− v, McQ

McMa

= 1− w.

Con esto, calculemos ahora el area de los triangulos complementariosdel triangulo ∆DEF .

c) En consecuencia:

d) Y además los ángulos en A , en B y en C son iguales, respectivamente a los ángulos en Ma , en Mb y en Mc

e) Si llamamos 𝑢 =   , 𝑣 =   , 𝑤 =   , tendremos:   = 1 − 𝑢 ,   = 1 − 𝑣 ,   = 1 − 𝑤

Y, obviamente:

1 − 𝑢 =   , 1 − 𝑣 =   , 1-𝑤 =   , = 𝑢 , = 𝑣 , = 𝑤

f) Observemos ahora el área de los triángulos complementarios del triángulo DEF:

Si [ABC] denota el area de un triangulo ∆ABC, tenemos que

[AFD] = FAAD senα = (1− w)uCAAB senα = (1− w)u[ABC],

[BED] = DBBE sen β = (1− u)v AB BC sen β = (1− u)v[ABC],

[CFE] = EC CF sen γ = (1− v)wBC CA sen γ = (1− v)w[ABC].

Haciendo lo mismo con los triangulos complementarios del triangulo∆PQR, con respecto al triangulo ∆MaMbMc se tieneg) Y hagamos lo mismo con los triángulos complementarios del triángulo PQR con respecto al

triángulo MaMbMc:

h) Basta entonces darnos cuenta que:

i) Concluimos entonces de b) , f) g) y h) lo que queríamos demostrar.

Variante un pelín más sencilla:

Sea un triángulo ABC. Sea los puntos D, E y F tres puntos cualesquiera de los lados AB, BC y CA respectivamente. Y sean D’ , E’ y F’ los simétricos de D, E y F respecto al punto medio del lado al que pertenecen. Probar que el área del triángulo DEF es igual que el área del triángulo D’E’F’.

4

Page 236: COMPENDIUM OMEFL - ToomatesXXXV Olimpiada Matemática Española Primera Fase Soluciones de la propuesta de problemas Problema 1 ¿Qué dígitos se han omitido en la siguiente multiplicación?

[MaRQ] = QMaMaR senα = (1−u)wMaMbMaMc senα = (1−u)w[MaMbMc],

[MbPR] = RMbMbP sen β = (1−v)uMaMbMbMc sen β = (1−v)u[MaMbMc],

[McQP ] = PMcMcQ sen γ = (1−w)vMbMcMaMc sen γ = (1−w)v[MaMbMc].

Teniendo en cuenta que [ABC] = 4[MaMbMc] y que

(1− u)w + (1− v)u+ (1− w)v = (1− w)u+ (1− u)v + (1− v)w,

se sigue el resultado.

5

Page 237: COMPENDIUM OMEFL - ToomatesXXXV Olimpiada Matemática Española Primera Fase Soluciones de la propuesta de problemas Problema 1 ¿Qué dígitos se han omitido en la siguiente multiplicación?

Sesion 2

1. Se considera un polıgono regular de 90 vertices, numerados del 1 al90 de manera aleatoria. Probar que siempre podemos encontrar dosvertices consecutivos cuyo producto es mayor o igual que 2014.

Solucion. Consideremos el primer par de numeros consecutivos cuyoproducto es mayor o igual que 2014, que son el 45 y el 46. Por lo tanto,para que no se cumpliera el enunciado, los numeros que deben ir aizquierda y derecha de los vertices numerados del 46 al 90 tendrıan queser menores o iguales que 44. Sin embrago, entre los vertices numeradosdel 46 al 90 hay, al menos, 45 vertices.

2. Hallar las soluciones enteras de la ecuacion

x4 + y4 = 3x3y.

Solucion 1. Supongamos, en primer lugar, que x = 0. En este casose tiene y4 = 0, por lo que y tambien tiene que ser 0. Ası pues, unasolucion es x = y = 0.

Si x 6= 0 dividimos toda la ecuacion por x4, quedando

1 +(yx

)4= 3

y

x.

Sea t = y/x, entonces, las soluciones enteras de la ecuacion x4 + y4 =3x3y dan lugar a soluciones racionales de la ecuacion t4−3t+1 = 0. Sinembargo, esta ecuacion no tiene soluciones racionales pues, de tenerlas,el denominador de la fraccion deberıa ser un divisor del coeficiente det4 y el denominador un divisor del termino independiente. Es decir,las posibles soluciones racionales solo pueden ser 1 o −1, pero ningunade ellas verifica t4 − 3t + 1 = 0. Por tanto, como no hay solucionesracionales, no hay soluciones enteras de x4 + y4 = 3x3y con x 6= 0.

Solucion 2. En primer lugar observemos que si (x, y) es una solucion,tambien lo es (−x,−y) y que, por ser x4 + y4 ≥ 0, entonces o bienx, y ≥ 0, o bien x, y ≤ 0. Debido a esto, buscaremos soluciones talesque x, y ≥ 0.

Puesto que el segundo miembro de la ecuacion es un multiplo de 3,tambien ha de serlo el primero. Ahora bien, toda potencia cuarta escongruente con 0 o con 1 modulo 3, siendo congruente con 0 solo cuando

6

Page 238: COMPENDIUM OMEFL - ToomatesXXXV Olimpiada Matemática Española Primera Fase Soluciones de la propuesta de problemas Problema 1 ¿Qué dígitos se han omitido en la siguiente multiplicación?

se trata de un multiplo de 3. Por tanto, tanto x como y deben sermultiplos de 3. Es decir, existen k1 ≥ 0 y k2 ≥ 0 enteros tales que

x = 3k1, y = 3k2.

Sustituyendo en la ecuacion y simplificando resulta

k41 + k42 = 3k31k2.

Es decir k1, k2 es una solucion tal que 0 ≤ k1 < x y 0 ≤ k2 < y. Siaplicaramos el mismo razonamiento a k1, k2 obtendrıamos una nuevasolucion k3, k4 tal que 0 ≤ k3 < k1, 0 ≤ k4 < k2. Es decir podemosobtener una sucesion infinita de soluciones, decreciente, a partir de unadada. Pero esto no es posible ya que la solucion de partida es finita.Por tanto debe ser k1 = k2 = 0, o lo que es lo mismo x = y = 0, que esla unica solucion entera de la ecuacion.

3. Probar que20142013 − 10132013 − 10012013

es multiplo de20143 − 10133 − 10013.

Solucion. Se tiene que 2014 = 1013 + 1001. Sea a = 2014 y b = 1013,entonces deberemos probar que

a2013 − b2013 − (a− b)2013

es multiplo dea3 − b3 − (a− b)3 = 3ab(a− b).

Ahora bien, por el binomio de Newton, se obtiene

a2013 − b2013 − (a− b)2013 =2012∑n=1

(2013

n

)(−1)na2013−n bn,

y agrupando por parejas simetricas resulta

a2013 − b2013 − (a− b)2013 =1006∑n=1

(2013

n

)(−1)n(a2013−n bn − an b2013−n)

=1006∑n=1

(2013

n

)(−1)nanbn(a2013−2n − b2013−2n).

7

Page 239: COMPENDIUM OMEFL - ToomatesXXXV Olimpiada Matemática Española Primera Fase Soluciones de la propuesta de problemas Problema 1 ¿Qué dígitos se han omitido en la siguiente multiplicación?

Teniendo en cuenta que

ak − bk = (a− b)(ak−1 + ak−2b+ · · ·+ abk−2 + bk−1 = (a− b)pk(a, b),

podemos escribir

a2013−b2013−(a−b)2013 = a b (a−b)1006∑n=1

(2013

n

)(−1)nan−1bn−1pn(a, b).

Finalmente, observemos que 20142013 − 10132013 − 10012013 es multiplode 3, pero no a = 2014 ni b = 1013 ni a − b = 1001, de donde seconcluye que

a2013 − b2013 − (a− b)2013 es multiplo de 3ab(a− b).

8

Page 240: COMPENDIUM OMEFL - ToomatesXXXV Olimpiada Matemática Española Primera Fase Soluciones de la propuesta de problemas Problema 1 ¿Qué dígitos se han omitido en la siguiente multiplicación?

Sesion 2

1. Se considera un polıgono regular de 90 vertices, numerados del 1 al90 de manera aleatoria. Probar que siempre podemos encontrar dosvertices consecutivos cuyo producto es mayor o igual que 2014.

Solucion. Consideremos el primer par de numeros consecutivos cuyoproducto es mayor o igual que 2014, que son el 45 y el 46. Por lo tanto,para que no se cumpliera el enunciado, los numeros que deben ir aizquierda y derecha de los vertices numerados del 46 al 90 tendrıan queser menores o iguales que 44. Sin embrago, entre los vertices numeradosdel 46 al 90 hay, al menos, 45 vertices.

2. Hallar las soluciones enteras de la ecuacion

x4 + y4 = 3x3y.

Solucion 1. Supongamos, en primer lugar, que x = 0. En este casose tiene y4 = 0, por lo que y tambien tiene que ser 0. Ası pues, unasolucion es x = y = 0.

Si x 6= 0 dividimos toda la ecuacion por x4, quedando

1 +(yx

)4= 3

y

x.

Sea t = y/x, entonces, las soluciones enteras de la ecuacion x4 + y4 =3x3y dan lugar a soluciones racionales de la ecuacion t4−3t+1 = 0. Sinembargo, esta ecuacion no tiene soluciones racionales pues, de tenerlas,el denominador de la fraccion deberıa ser un divisor del coeficiente det4 y el denominador un divisor del termino independiente. Es decir,las posibles soluciones racionales solo pueden ser 1 o −1, pero ningunade ellas verifica t4 − 3t + 1 = 0. Por tanto, como no hay solucionesracionales, no hay soluciones enteras de x4 + y4 = 3x3y con x 6= 0.

Solucion 2. En primer lugar observemos que si (x, y) es una solucion,tambien lo es (−x,−y) y que, por ser x4 + y4 ≥ 0, entonces o bienx, y ≥ 0, o bien x, y ≤ 0. Debido a esto, buscaremos soluciones talesque x, y ≥ 0.

Puesto que el segundo miembro de la ecuacion es un multiplo de 3,tambien ha de serlo el primero. Ahora bien, toda potencia cuarta escongruente con 0 o con 1 modulo 3, siendo congruente con 0 solo cuando

6

Page 241: COMPENDIUM OMEFL - ToomatesXXXV Olimpiada Matemática Española Primera Fase Soluciones de la propuesta de problemas Problema 1 ¿Qué dígitos se han omitido en la siguiente multiplicación?

se trata de un multiplo de 3. Por tanto, tanto x como y deben sermultiplos de 3. Es decir, existen k1 ≥ 0 y k2 ≥ 0 enteros tales que

x = 3k1, y = 3k2.

Sustituyendo en la ecuacion y simplificando resulta

k41 + k42 = 3k31k2.

Es decir k1, k2 es una solucion tal que 0 ≤ k1 < x y 0 ≤ k2 < y. Siaplicaramos el mismo razonamiento a k1, k2 obtendrıamos una nuevasolucion k3, k4 tal que 0 ≤ k3 < k1, 0 ≤ k4 < k2. Es decir podemosobtener una sucesion infinita de soluciones, decreciente, a partir de unadada. Pero esto no es posible ya que la solucion de partida es finita.Por tanto debe ser k1 = k2 = 0, o lo que es lo mismo x = y = 0, que esla unica solucion entera de la ecuacion.

3. Probar que20142013 − 10132013 − 10012013

es multiplo de20143 − 10133 − 10013.

Solucion. Se tiene que 2014 = 1013 + 1001. Sea a = 2014 y b = 1013,entonces deberemos probar que

a2013 − b2013 − (a− b)2013

es multiplo dea3 − b3 − (a− b)3 = 3ab(a− b).

Ahora bien, por el binomio de Newton, se obtiene

a2013 − b2013 − (a− b)2013 =2012∑n=1

(2013

n

)(−1)na2013−n bn,

y agrupando por parejas simetricas resulta

a2013 − b2013 − (a− b)2013 =1006∑n=1

(2013

n

)(−1)n(a2013−n bn − an b2013−n)

=1006∑n=1

(2013

n

)(−1)nanbn(a2013−2n − b2013−2n).

7

Page 242: COMPENDIUM OMEFL - ToomatesXXXV Olimpiada Matemática Española Primera Fase Soluciones de la propuesta de problemas Problema 1 ¿Qué dígitos se han omitido en la siguiente multiplicación?

Teniendo en cuenta que

ak − bk = (a− b)(ak−1 + ak−2b+ · · ·+ abk−2 + bk−1 = (a− b)pk(a, b),

podemos escribir

a2013−b2013−(a−b)2013 = a b (a−b)1006∑n=1

(2013

n

)(−1)nan−1bn−1pn(a, b).

Finalmente, observemos que 20142013 − 10132013 − 10012013 es multiplode 3, pero no a = 2014 ni b = 1013 ni a − b = 1001, de donde seconcluye que

a2013 − b2013 − (a− b)2013 es multiplo de 3ab(a− b).

8

Page 243: COMPENDIUM OMEFL - ToomatesXXXV Olimpiada Matemática Española Primera Fase Soluciones de la propuesta de problemas Problema 1 ¿Qué dígitos se han omitido en la siguiente multiplicación?

Sesion 3

1. Sean a, b numeros positivos. Probar que

a+ b ≥√ab+

√a2 + b2

2.

Solucion 1. La desigualdad equivale a

√ab+

√a2+b2

2

2≤ a+ b

2.

Si aplicamos la desigualdad entre las medias aritmetica y geometricaal miembro de la izquierda obtenemos

√ab+

√a2+b2

2

2≤

√√√√ab+a2 + b2

22

=

√2ab+ a2 + b2

2 · 2=

√(a+ b)2

22=a+ b

2.

Solucion 2. Con el cambio de variable a = s2, b = t2 (0 ≤ s, t)obtenemos la desigualdad equivalente

st+

√s4 + t4

2≤ s2 + t2.

Aislando la raız cuadrada y elevando al cuadrado, tambien es equiva-lente

s4 + t4

2≤ s4 + t4 + s2t2 + 2s2t2 − 2s3t− 2t3s.

Multiplicando por 2 e igualando a 0 el miembro de la izquierda, esequivalente probar que

0 ≤ s4 + t4 + 6s2t2 − 4s3t− 4t3s.

Finalmente, gracias al binomio de Newton,

s4 + t4 + 6s2t2 − 4s3t− 4t3s = (t− s)4 ≥ 0.

Solucion 3. Denotamos

A =a+ b

2(media aritmetica de los numeros a y b.)

9

Page 244: COMPENDIUM OMEFL - ToomatesXXXV Olimpiada Matemática Española Primera Fase Soluciones de la propuesta de problemas Problema 1 ¿Qué dígitos se han omitido en la siguiente multiplicación?

G =√ab (media geometrica de los numeros a y b.)

Q =

√a2 + b2

2(media cuadratica de los numeros a y b.)

Con esta notacion debemos probar que

G+Q ≤ 2A

o, equivalentemente,Q− A ≤ A−G,

que expresamos, multiplicando por el conjugado en cada uno de losterminos, como

Q2 − A2

Q+ A≤ A2 −G2

A+G.

Puesto que Q ≥ G, se tiene que Q + A ≥ A + G > 0. Como Q ≥ A,tenemos Q2 − A2 ≥ 0. Puesto que A ≥ G, tenemos que A2 − G2 ≥ 0.Ası pues, basta probar que

Q2 − A2 ≤ A2 −G2,

que equivale aQ2 +G2 ≤ 2A2.

Es decir, basta probar que

a2 + b2

2+ ab ≤ (a+ b)2

2.

Simplificando observamos que esta ultima expresion es, en realidad,una igualdad.

Solucion 4. La desigualdad equivale a

1

2

√ab+

1

2

√a2 + b2

2≤ a+ b

2.

Puesto que la funcion f(x) =√x es concava, gracias a la desigualdad

de Jensen,

1

2

√ab+

1

2

√a2 + b2

2≤√

1

2ab+

1

2

a2 + b2

2=a+ b

2.

10

Page 245: COMPENDIUM OMEFL - ToomatesXXXV Olimpiada Matemática Española Primera Fase Soluciones de la propuesta de problemas Problema 1 ¿Qué dígitos se han omitido en la siguiente multiplicación?

2. Encontrar las tres ultimas cifras de 72014.

Solucion 1. Usaremos el teorema de Euler-Fermat: si mcd a,m = 1,entonces

aϕ(m) ≡ 1 (mod m).

En nuestro caso, queremos calcular 72014 (mod 1000). Por ser 1000 =2353, se tiene que ϕ(1000) = 22(2− 1) · 52(5− 1) = 400. Entonces,

72014 = 75·400+14 = (7400)5 · 714 ≡ 15 · 714 ≡ 849 (mod 1000).

En consecuencia, las tres ultimas cifras de 72014 son 849.

Solucion 2. Observemos que las tres ultimas cifras de 74 son 401. Portanto, multiplicando sucesivamente por 74, resulta que

Potencia ultimas cifras

74 40178 801712 201716 601720 001724 401

Puesto que 72014 = 72 · 72012 = 72 · 7503·4, resulta que las tres ultimascifras de 72012 son 201, ya que 503 deja resto 3 al dividirlo por 5 (5 esel periodo de la tabla y 3 corresponde a la tercera entrada de la tabla).Ası pues, las tres ultimas cifras de 72014 son las tres ultimas cifras delproducto 49 · 201, es decir 849.

3. De un prisma recto de base cuadrada, con lado de longitud L1, y alturaH, extraemos un tronco de piramide, no necesariamente recto, de basescuadradas, con lados de longitud L1 (para la inferior) y L2 (para lasuperior), y altura H. Las dos piezas obtenidas aparecen en la imagensiguiente.

L1

L2

L1

11

Page 246: COMPENDIUM OMEFL - ToomatesXXXV Olimpiada Matemática Española Primera Fase Soluciones de la propuesta de problemas Problema 1 ¿Qué dígitos se han omitido en la siguiente multiplicación?

Si el volumen del tronco de piramide es 2/3 del total del volumen delprisma, ¿cual es el valor de L1/L2?

Solucion. Si prolongamos una altura h el tronco de piramide hastaobtener una piramide completa de altura H + h tendra una seccioncomo la que se muestra en la figura siguiente.

L1

L2

H

h

Un argumento de semejanza de triangulos permite comprobar que

h+H

L1

=h

L2

y, por tanto,

h =HL2

L1 − L2

.

12

Page 247: COMPENDIUM OMEFL - ToomatesXXXV Olimpiada Matemática Española Primera Fase Soluciones de la propuesta de problemas Problema 1 ¿Qué dígitos se han omitido en la siguiente multiplicación?

Ademas, podemos observar que

Volumen del tronco de piramide = 13(L2

1(H + h)− L22h)

=1

3

(HL3

1

L1 − L2

− HL31

L1 − L2

)=H

3

L31 − L3

2

L1 − L2

=H

3(L2

1 + L1L2 + L22).

Ası, teniendo en cuenta que

Volumen del tronco de piramide =2

3Volumen del prisma,

tendremos la ecuacion

H

3(L2

1 + L1L2 + L22) =

2

3HL2

1,

que se transforma en (L1

L2

)2

− L1

L2

− 1 = 0,

cuya unica solucion positiva esL1

L2

=1 +√

5

2. Es decir, los lados deben

estar en relacion aurea.

13

Page 248: COMPENDIUM OMEFL - ToomatesXXXV Olimpiada Matemática Española Primera Fase Soluciones de la propuesta de problemas Problema 1 ¿Qué dígitos se han omitido en la siguiente multiplicación?

Sesion 3

1. Sean a, b numeros positivos. Probar que

a+ b ≥√ab+

√a2 + b2

2.

Solucion 1. La desigualdad equivale a

√ab+

√a2+b2

2

2≤ a+ b

2.

Si aplicamos la desigualdad entre las medias aritmetica y geometricaal miembro de la izquierda obtenemos

√ab+

√a2+b2

2

2≤

√√√√ab+a2 + b2

22

=

√2ab+ a2 + b2

2 · 2=

√(a+ b)2

22=a+ b

2.

Solucion 2. Con el cambio de variable a = s2, b = t2 (0 ≤ s, t)obtenemos la desigualdad equivalente

st+

√s4 + t4

2≤ s2 + t2.

Aislando la raız cuadrada y elevando al cuadrado, tambien es equiva-lente

s4 + t4

2≤ s4 + t4 + s2t2 + 2s2t2 − 2s3t− 2t3s.

Multiplicando por 2 e igualando a 0 el miembro de la izquierda, esequivalente probar que

0 ≤ s4 + t4 + 6s2t2 − 4s3t− 4t3s.

Finalmente, gracias al binomio de Newton,

s4 + t4 + 6s2t2 − 4s3t− 4t3s = (t− s)4 ≥ 0.

Solucion 3. Denotamos

A =a+ b

2(media aritmetica de los numeros a y b.)

9

Page 249: COMPENDIUM OMEFL - ToomatesXXXV Olimpiada Matemática Española Primera Fase Soluciones de la propuesta de problemas Problema 1 ¿Qué dígitos se han omitido en la siguiente multiplicación?

G =√ab (media geometrica de los numeros a y b.)

Q =

√a2 + b2

2(media cuadratica de los numeros a y b.)

Con esta notacion debemos probar que

G+Q ≤ 2A

o, equivalentemente,Q− A ≤ A−G,

que expresamos, multiplicando por el conjugado en cada uno de losterminos, como

Q2 − A2

Q+ A≤ A2 −G2

A+G.

Puesto que Q ≥ G, se tiene que Q + A ≥ A + G > 0. Como Q ≥ A,tenemos Q2 − A2 ≥ 0. Puesto que A ≥ G, tenemos que A2 − G2 ≥ 0.Ası pues, basta probar que

Q2 − A2 ≤ A2 −G2,

que equivale aQ2 +G2 ≤ 2A2.

Es decir, basta probar que

a2 + b2

2+ ab ≤ (a+ b)2

2.

Simplificando observamos que esta ultima expresion es, en realidad,una igualdad.

Solucion 4. La desigualdad equivale a

1

2

√ab+

1

2

√a2 + b2

2≤ a+ b

2.

Puesto que la funcion f(x) =√x es concava, gracias a la desigualdad

de Jensen,

1

2

√ab+

1

2

√a2 + b2

2≤√

1

2ab+

1

2

a2 + b2

2=a+ b

2.

10

Page 250: COMPENDIUM OMEFL - ToomatesXXXV Olimpiada Matemática Española Primera Fase Soluciones de la propuesta de problemas Problema 1 ¿Qué dígitos se han omitido en la siguiente multiplicación?

2. Encontrar las tres ultimas cifras de 72014.

Solucion 1. Usaremos el teorema de Euler-Fermat: si mcd a,m = 1,entonces

aϕ(m) ≡ 1 (mod m).

En nuestro caso, queremos calcular 72014 (mod 1000). Por ser 1000 =2353, se tiene que ϕ(1000) = 22(2− 1) · 52(5− 1) = 400. Entonces,

72014 = 75·400+14 = (7400)5 · 714 ≡ 15 · 714 ≡ 849 (mod 1000).

En consecuencia, las tres ultimas cifras de 72014 son 849.

Solucion 2. Observemos que las tres ultimas cifras de 74 son 401. Portanto, multiplicando sucesivamente por 74, resulta que

Potencia ultimas cifras

74 40178 801712 201716 601720 001724 401

Puesto que 72014 = 72 · 72012 = 72 · 7503·4, resulta que las tres ultimascifras de 72012 son 201, ya que 503 deja resto 3 al dividirlo por 5 (5 esel periodo de la tabla y 3 corresponde a la tercera entrada de la tabla).Ası pues, las tres ultimas cifras de 72014 son las tres ultimas cifras delproducto 49 · 201, es decir 849.

3. De un prisma recto de base cuadrada, con lado de longitud L1, y alturaH, extraemos un tronco de piramide, no necesariamente recto, de basescuadradas, con lados de longitud L1 (para la inferior) y L2 (para lasuperior), y altura H. Las dos piezas obtenidas aparecen en la imagensiguiente.

L1

L2

L1

11

Page 251: COMPENDIUM OMEFL - ToomatesXXXV Olimpiada Matemática Española Primera Fase Soluciones de la propuesta de problemas Problema 1 ¿Qué dígitos se han omitido en la siguiente multiplicación?

Si el volumen del tronco de piramide es 2/3 del total del volumen delprisma, ¿cual es el valor de L1/L2?

Solucion. Si prolongamos una altura h el tronco de piramide hastaobtener una piramide completa de altura H + h tendra una seccioncomo la que se muestra en la figura siguiente.

L1

L2

H

h

Un argumento de semejanza de triangulos permite comprobar que

h+H

L1

=h

L2

y, por tanto,

h =HL2

L1 − L2

.

12

Page 252: COMPENDIUM OMEFL - ToomatesXXXV Olimpiada Matemática Española Primera Fase Soluciones de la propuesta de problemas Problema 1 ¿Qué dígitos se han omitido en la siguiente multiplicación?

Ademas, podemos observar que

Volumen del tronco de piramide = 13(L2

1(H + h)− L22h)

=1

3

(HL3

1

L1 − L2

− HL31

L1 − L2

)=H

3

L31 − L3

2

L1 − L2

=H

3(L2

1 + L1L2 + L22).

Ası, teniendo en cuenta que

Volumen del tronco de piramide =2

3Volumen del prisma,

tendremos la ecuacion

H

3(L2

1 + L1L2 + L22) =

2

3HL2

1,

que se transforma en (L1

L2

)2

− L1

L2

− 1 = 0,

cuya unica solucion positiva esL1

L2

=1 +√

5

2. Es decir, los lados deben

estar en relacion aurea.

13

Page 253: COMPENDIUM OMEFL - ToomatesXXXV Olimpiada Matemática Española Primera Fase Soluciones de la propuesta de problemas Problema 1 ¿Qué dígitos se han omitido en la siguiente multiplicación?

Sesion 4

1. Hallar para que valores del numero real a todas las raıces del polinomio,en la variable x,

x3 − 2x2 − 25x+ a

son numeros enteros.

Solucion 1. Sean α, β y γ las raıces del polinomio. Aplicando lasformulas de Cardano-Vieta se tiene

α + β + γ = 2, αβ + αγ + βγ = −25.

Ahora bien

α2 + β2 + γ2 = (α + β + γ)2 − 2(αβ + αγ + βγ) = 54.

Como α, β y γ son enteros, buscamos soluciones enteras de la parejade ecuaciones

α + β + γ = 2, α2 + β2 + γ2 = 54.

De la segunda vemos que las unicas soluciones posibles son

(±1,±2,±7), (±2,±5,±5), (±3,±3,±6),

y teniendo en cuenta la primera ecuacion, la unica solucion posible es(2, 5,−5) y entonces a = 50.

Solucion 2. Notemos que el polinomio que nos dan es uno que seobtiene desplazando verticalmente a unidades el polinomio

P (x) ≡ x3 − 2x2 − 25x = x(x2 − 2x− 25),

que tiene raıces en x = 1 ±√

26 y x = 0. Si a > 0 habra dos raıcespositivas mayores que 0 y menores que 1 +

√26. Al ser enteras, solo

podran ser x1 = 1, x2 = 2, x3 = 3, x4 = 4, x5 = 5 o x6 = 6 y ademas,se tendra que verificar P (xj) = P (xk), para j 6= k y 1 ≤ j, k ≤ 6.

Teniendo en cuenta que P (2) = P (5) = −50, resulta que el polinomio

x3 − 2x2 − 25x+ 50

tiene sus tres raıces enteras, dos de ellas son las mencionadas 2 y 5 yla tercera −5.

En el caso en que a < 0, no existen soluciones, ya que en este casohabrıa dos raıces negativas, que podrıan ser −1, −2 −3 o −4, pero

P (−1) 6= P (−2) 6= P (−3) 6= P (−4).

14

Page 254: COMPENDIUM OMEFL - ToomatesXXXV Olimpiada Matemática Española Primera Fase Soluciones de la propuesta de problemas Problema 1 ¿Qué dígitos se han omitido en la siguiente multiplicación?

2. Sean x e y numeros reales entre 0 y 1. Probar que

x3 + xy2 + 2xy ≤ 2x2y + x2 + x+ y.

Solucion. La desigualdad equivale a

P = 2x2y + x2 + x+ y − x3 − xy2 − 2xy ≥ 0.

Escribimos P como un polinomio en la variable x,

P = −x3 + (2y + 1)x2 + (1− 2y − y2)x+ y.

Dividimos este polinomio entre x−1, mediante el algoritmo de Ruffini.

−1 2y + 1 1− 2y − y2 y1 −1 2y 1− y2−1 2y 1− y2 1 + y − y2.

Es decir,

P = (−x2 + 2xy + 1− y2)(x− 1) + 1 + y − y2.

Tambien

P = (−x2 + 2xy − y2)(x− 1) + x− 1 + 1 + y − y2

= (x2 − 2xy + y2)(1− x) + x+ y − y2

= (x− y)2(1− x) + x+ y(1− y).

Puesto que las cinco cantidades (x − y)2, 1 − x, x, y, 1 − y son nonegativas, P ≥ 0.

3. Consideramos un numero primo p. Debemos disenar un torneo de p-parchıs sujeto a las siguientes reglas.

En el torneo participan p2 jugadores.

En cada partida juegan p jugadores.

El torneo se divide en rondas. Las rondas se dividen en partidas.Cada jugador juega una, o ninguna, partida en cada ronda.

Al final del torneo cada jugador se ha enfrentado exactamente unavez con cada uno de los otros jugadores.

15

Page 255: COMPENDIUM OMEFL - ToomatesXXXV Olimpiada Matemática Española Primera Fase Soluciones de la propuesta de problemas Problema 1 ¿Qué dígitos se han omitido en la siguiente multiplicación?

Determinar si es posible disenar un torneo ası. En caso afirmativo,obtened el mınimo numero de rondas que puede tener el torneo.

Solucion. El numero de partidas que disputa cada jugador es

numero de jugadores a los que se enfrenta

numero de jugadores a los que se enfrenta en cada partida=p2 − 1

p− 1.

O sea, cada jugador juega p+1 partidas. Por tanto el numero de rondases, al menos, p+ 1, y es exactamente p+ 1 cuando todos los jugadoresjuegan en todas las rondas. En ese caso, en cada ronda se disputanp2

p= p partidas.

Vamos a probar que es posible organizar un torneo con p + 1 rondas.Representamos a cada jugador como un numero de dos cifras escritoen base p. Es decir, escribimos cada jugador de la forma

CiCd,

donde Ci, la cifra de la izquierda, y Cd, la cifra de la derecha, tomanvalores enteros entre 0 y p− 1.

Realizamos la siguiente planificacion.

Ronda 0. Agrupamos los jugadores en los que Ci coincide.

Ronda 1. Agrupamos los jugadores en los que Ci + Cd tiene elmismo resto al dividir por p.

. . . . . .

Ronda k. Agrupamos los jugadores en los que Ci + k · Cd tiene elmismo resto al dividir por p.

. . . .

Ronda p− 1. Agrupamos los jugadores en los que Ci + (p− 1) ·Cd

tiene el mismo resto al dividir por p.

Ronda p. Agrupamos los jugadores en los que Cd coincide.

En los argumentos que siguen, diremos que D ≡ E si ambos numerostienen el mismo resto al dividir por p.

Debemos observar que la planificacion propuesta agrupa a los jugadoresen conjuntos de p elementos. En los rondas 0 y p eso es claro. Conside-ramos una ronda k con 0 < k < p. Para cada Ci fijado, al mover Cd, elresto de Ci + k · Cd es siempre distinto (si no lo fuera tendrıamos dos

16

Page 256: COMPENDIUM OMEFL - ToomatesXXXV Olimpiada Matemática Española Primera Fase Soluciones de la propuesta de problemas Problema 1 ¿Qué dígitos se han omitido en la siguiente multiplicación?

jugadores CiCd, CiC′d para los que Ci + k ·Cd ≡ Ci + k ·C ′d, y restando

llegarıamos a que k(C ′d −Cd) es multiplo de p, sin que ni k ni C ′d −Cd

lo sean). Por tanto, obtenemos una vez, y solo una, todos los posiblesrestos. Al variar Ci obtenemos p veces cada uno de los restos.

Supongamos que dos jugadores, CiCd y C ′iC′d, se enfrentan dos veces.

Si lo hacen en rondas 0 ≤ j < k ≤ p − 1 tenemos que Ci + k · Cd ≡C ′i +k ·C ′d, Ci + j ·Cd ≡ C ′i + j ·C ′d. Restando, (k− j) ·Cd ≡ (k− j) ·C ′d.Como k − j es primo con p, Cd ≡ C ′d. Luego Cd = C ′d. Llevando estaigualdad a nuestra hipotesis, y restando, obtenemos facilmente Ci ≡ C ′i.Luego Ci = C ′i.

Si los jugadores se enfrentan en rondas k y p con 0 ≤ k < p obtenemosdirectamente que Cd = C ′d, y repetimos el argumento anterior.

17

Page 257: COMPENDIUM OMEFL - ToomatesXXXV Olimpiada Matemática Española Primera Fase Soluciones de la propuesta de problemas Problema 1 ¿Qué dígitos se han omitido en la siguiente multiplicación?

LI Olimpiada Matematica Espanola

Primera Fase

Primera sesion

Viernes manana, 16 de enero de 2015

OlimpiadaMatemáticaEspañola RSME

1. Demuestra que(ax+ by)2 ≤ ax2 + by2

para cualesquiera x, y ∈ R y cualesquiera a, b ∈ R con a + b = 1, a, b ≥ 0.¿En que casos se da la igualdad?

2. Sean r y s dos rectas paralelas, y A un punto fijo a igual distancia de ambasrectas. Para cada punto B de la recta r, sea C el punto de la recta s tal queBAC = 90◦, y sea P el pie de la perpendicular desde A sobre la recta BC.Demuestra que, independientemente de que punto B de la recta r tomemos,el punto P esta sobre una circunferencia fija.

3. Un campeonato de baloncesto se ha jugado por sistema de liga a dos vueltas(cada par de equipos se enfrentan dos veces) y sin empate (si el partidoacaba en empate hay prorrogas hasta que gane uno de los dos). El ganadordel partido obtiene 2 puntos y el perdedor 1 punto. Al final del campeonato,la suma de de los puntos obtenidos por todos los equipos salvo el campeones de 2015 puntos. ¿Cuantos partidos ha ganado el campeon?

No esta permitido el uso de calculadoras.Cada problema se puntua sobre 7 puntos.

El tiempo de cada sesion es de 3 horas y media.

Page 258: COMPENDIUM OMEFL - ToomatesXXXV Olimpiada Matemática Española Primera Fase Soluciones de la propuesta de problemas Problema 1 ¿Qué dígitos se han omitido en la siguiente multiplicación?

LI Olimpiada Matematica Espanola

Primera Fase

Segunda sesion

Viernes tarde, 16 de enero de 2015

OlimpiadaMatemáticaEspañola RSME

4. Los enteros positivos x, y, z cumplen

x+ 2y = z , x2 − 4y2 + z2 = 310

Halla todos los posibles valores del producto xyz.

5. En una recta tenemos cuatro puntos A, B, C y D, en ese orden, de formaque AB = CD. El punto E es un punto fuera de la recta tal que CE = DE.Demuestra que

CED = 2AEB

si y solo si AC = EC.

6. Halla todas las ternas de reales positivos (x, y, z) que cumplan el sistema

⎧⎪⎨⎪⎩

2x√x+ 1− y(y + 1) = 1

2y√y + 1− z(z + 1) = 1

2z√z + 1− x(x+ 1) = 1

No esta permitido el uso de calculadoras.Cada problema se puntua sobre 7 puntos.

El tiempo de cada sesion es de 3 horas y media.

Page 259: COMPENDIUM OMEFL - ToomatesXXXV Olimpiada Matemática Española Primera Fase Soluciones de la propuesta de problemas Problema 1 ¿Qué dígitos se han omitido en la siguiente multiplicación?

LI Olimpiada Matematica Espanola

Primera Fase

Primera sesion

Viernes tarde, 16 de enero de 2015

OlimpiadaMatemáticaEspañola RSME

1. Los enteros positivos x, y, z cumplen

x+ 2y = z , x2 − 4y2 + z2 = 310

Halla todos los posibles valores del producto xyz.

2. En una recta tenemos cuatro puntos A, B, C y D, en ese orden, de formaque AB = CD. El punto E es un punto fuera de la recta tal que CE = DE.Demuestra que

CED = 2AEB

si y solo si AC = EC.

3. Halla todas las ternas de reales positivos (x, y, z) que cumplan el sistema

⎧⎪⎨⎪⎩

2x√x+ 1− y(y + 1) = 1

2y√y + 1− z(z + 1) = 1

2z√z + 1− x(x+ 1) = 1

No esta permitido el uso de calculadoras.Cada problema se puntua sobre 7 puntos.

El tiempo de cada sesion es de 3 horas y media.

Page 260: COMPENDIUM OMEFL - ToomatesXXXV Olimpiada Matemática Española Primera Fase Soluciones de la propuesta de problemas Problema 1 ¿Qué dígitos se han omitido en la siguiente multiplicación?

LI Olimpiada Matematica Espanola

Primera Fase

Segunda sesion

Sabado manana, 17 de enero de 2015

OlimpiadaMatemáticaEspañola RSME

4. Alrededor de una mesa circular estan sentadas seis personas. Cada una llevaun sombrero. Entre cada dos personas hay una mampara de modo que cadauna puede ver los sombreros de las tres que estan enfrente, pero no puede verel de la persona de su izquierda ni el de la de su derecha ni el suyo propio.Todas saben que tres de los sombreros son blancos y tres negros. Tambiensaben que cada una de ellas es capaz de obtener cualquier deduccion logicaque sea factible. Empezamos por una de las seis personas y le preguntamos”¿puedes deducir el color de algun sombrero de los que no ves?”. Una vezque ha respondido (todas oyen la respuesta), pasamos a la persona de suizquierda y le hacemos la misma pregunta, y asi sucesivamente. Demuestraque una de las tres primeras respondera ”Si”.

5. El triangulo �ABC es isosceles en C, y sea Γ su circunferencia circunscrita.Sea M el punto medio del arco BC de Γ que no contiene a A, y sea N elpunto donde la paralela a AB por M vuelve a cortar a Γ. Se sabe que ANes paralela a BC. ¿Cuales son las medidas de los angulos de �ABC?

6. Sean x, y, z reales positivos tales que x + y + z = 3. Halla el valor maximoalcanzado por √

x+√

2y + 2 +√3z + 6.

¿Para que valores de x, y, z se alcanza dicho maximo?

No esta permitido el uso de calculadoras.Cada problema se puntua sobre 7 puntos.

El tiempo de cada sesion es de 3 horas y media.

Page 261: COMPENDIUM OMEFL - ToomatesXXXV Olimpiada Matemática Española Primera Fase Soluciones de la propuesta de problemas Problema 1 ¿Qué dígitos se han omitido en la siguiente multiplicación?

LI Olimpiada Matematica Espanola

Primera Fase

Primera sesion

Sabado manana, 17 de enero de 2015

OlimpiadaMatemáticaEspañola RSME

1. Alrededor de una mesa circular estan sentadas seis personas. Cada una llevaun sombrero. Entre cada dos personas hay una mampara de modo que cadauna puede ver los sombreros de las tres que estan enfrente, pero no puede verel de la persona de su izquierda ni el de la de su derecha ni el suyo propio.Todas saben que tres de los sombreros son blancos y tres negros. Tambiensaben que cada una de ellas es capaz de obtener cualquier deduccion logicaque sea factible. Empezamos por una de las seis personas y le preguntamos”¿puedes deducir el color de algun sombrero de los que no ves?”. Una vezque ha respondido (todas oyen la respuesta), pasamos a la persona de suizquierda y le hacemos la misma pregunta, y asi sucesivamente. Demuestraque una de las tres primeras respondera ”Si”.

2. El triangulo �ABC es isosceles en C, y sea Γ su circunferencia circunscrita.Sea M el punto medio del arco BC de Γ que no contiene a A, y sea N elpunto donde la paralela a AB por M vuelve a cortar a Γ. Se sabe que ANes paralela a BC. ¿Cuales son las medidas de los angulos de �ABC?

3. Sean x, y, z reales positivos tales que x + y + z = 3. Halla el valor maximoalcanzado por √

x+√

2y + 2 +√3z + 6.

¿Para que valores de x, y, z se alcanza dicho maximo?

No esta permitido el uso de calculadoras.Cada problema se puntua sobre 7 puntos.

El tiempo de cada sesion es de 3 horas y media.

Page 262: COMPENDIUM OMEFL - ToomatesXXXV Olimpiada Matemática Española Primera Fase Soluciones de la propuesta de problemas Problema 1 ¿Qué dígitos se han omitido en la siguiente multiplicación?

LI Olimpiada Matematica Espanola

Primera Fase

Segunda sesion

Sabado tarde, 17 de enero de 2015

OlimpiadaMatemáticaEspañola RSME

4. Encuentra todas las aplicaciones f : Z → Z que verifican

f(n) + f(n+ 1) = 2n+ 1

para cualquier entero n y ademas

63∑i=1

f(i) = 2015

5. Sea n ≥ 2 un entero positivo. Tenemos 2n bolas, en cada una de las cualeshay escrito un entero. Se cumple que, siempre que formamos n parejas conlas bolas, dos de estas parejas tienen la misma suma.

1) Demuestra que hay cuatro bolas con el mismo numero.

2) Demuestra que el numero de valores distintos que hay en las bolas es comomucho n− 1.

6. Encuentra todos los enteros positivos n, que verifican

n = 22x−1 − 5x− 3 =(2x−1 − 1

)(2x + 1)

para algun entero positivo x.

No esta permitido el uso de calculadoras.Cada problema se puntua sobre 7 puntos.

El tiempo de cada sesion es de 3 horas y media.

Page 263: COMPENDIUM OMEFL - ToomatesXXXV Olimpiada Matemática Española Primera Fase Soluciones de la propuesta de problemas Problema 1 ¿Qué dígitos se han omitido en la siguiente multiplicación?

Problemas Primera Sesion

1. Demuestra que(ax+ by)2 ≤ ax2 + by2

para cualesquiera x, y ∈ R y cualesquiera a, b ∈ R con a+ b = 1, a, b ≥ 0. ¿En que casosse da la igualdad?

Solucion 1. Notese que

ax2 + by2 − (ax+ by)2 = a(1− a)x2 + b(1− b)y2 − 2abxy = ab(x− y)2,

donde hemos usado que 1−a = b y 1−b = a. Esta expresion es claramente no negativa,siendo nula si y solo si bien ab = 0 (es decir, uno de entre a, b es 0 y el otro es 1), bienx = y.

Solucion 2. Considerense los vectores(√

a,√b)

y(√

ax,√by)

, cuyo producto escalar

es ax+ by, y cuyos modulos son√a+ b = 1 y

√ax2 + by2. La desigualdad propuesta

es equivalente a la desigualdad del producto escalar aplicada a estos vectores, y por lotanto cierta, dandose la igualdad si y solo si ambos vectores son proporcionales, cosaque puede pasar bien si una de sus coordenadas es nula (es decir, si a = 0 o b = 0),bien si ambas coordenadas son proporcionales cuando no son nulas, es decir, x = y.

Solucion 3. La funcion f(z) = z2 es claramente convexa, con lo que por la desigualdadde Jensen, para cualesquiera reales no negativos a, b, y cualesquiera reales x, y, se tiene

(ax+ by)2 = f(ax+ by) ≤ af(x) + bf(y)

a+ b=ax2 + by2

a+ b.

Usando que a+ b = 1, se obtiene el resultado pedido, dandose la igualdad bien si unode los dos puntos ”desaparece” (es decir, a = 0 o b = 0), o en caso contrario si ambospuntos coinciden (es decir, x = y).

2. Sean r y s dos rectas paralelas, y A un punto fijo a igual distancia de ambasrectas. Para cada punto B de la recta r, sea C el punto de la recta s tal que ∠BAC =90◦, y sea P el pie de la perpendicular desde A sobre la recta BC. Demuestra que,independientemente de que punto B de la recta r tomemos, el punto P esta sobre unacircunferencia fija.

Solucion 1. Sea Q el punto de r tal que AQ es perpendicular a r. Sea D el puntodonde AC corta a r. Como A esta a la misma distancia de las rectas r y s, AC = AD.Los triangulos ABC y ABD son ambos rectangulos en A, comparten el lado AB, yel lado AC es igual al lado AD. En consecuencia, ambos triangulos son iguales. Lospies de las alturas desde A en cada triangulo son P y Q, respectivamente, por lo queAP = AQ. Como Q no depende de B, la distancia AP = AQ es fija, y el punto P esta

1

Page 264: COMPENDIUM OMEFL - ToomatesXXXV Olimpiada Matemática Española Primera Fase Soluciones de la propuesta de problemas Problema 1 ¿Qué dígitos se han omitido en la siguiente multiplicación?

sobre la circunferencia fija de centro A, que es tangente simultaneamente a las rectas ry s.

r

s

QD B

A

P

C

r

s

B

A

P

C

M

Solucion 2. Sea M el punto medio del segmento BC. Como el triangulo ABC esrectangulo en A, M es su circuncentro, es decir, AM = MB = MC. Llamando d a ladistancia de A a las rectas r y s, notese que la longitud de las alturas desde B y desdeC sobre AM es d, con lo que las areas de AMB y AMC son ambas iguales a d·AM

2 , y elarea del triangulo ABC es d · AM . Pero BC = MB +MC = 2AM , luego la longitudAP de la altura desde A sobre BC es 2d·AM

2AM = d, que es constante, concluyendo igualque en la solucion anterior.

r

s

QD B

A

P

C

r

s

B

A

P

C

M

Solucion 3. Si h es la distancia entre A y las rectas r y s, podemos tomar un sistemade coordenadas cartesianas tales que A ≡ (0, 0), r ≡ y = h, s ≡ y = −h, y paracualquier punto B ≡ (d, h), la recta AB tiene pendiente h

d , con lo que

AC ≡ y = −dxh, C ≡

(h2

d,−h

), BC ≡ y =

2hdx

d2 − h2−h(d2 + h2

)(d2 − h2)

.

La ecuacion de la recta AP es entonces y = −d2−h2

2hd x, con lo que podemos hallar Pcomo la interseccion de esta recta con la recta BC, resultando finalmente tras algo dealgebra en

P ≡

(2h2d

d2 + h2,−

h(d2 − h2

)d2 + h2

), AP 2 = h2

4h2d2 +(d2 − h2

)2(d2 + h2)2

= h2,

es decir AP = h, concluyendose como en las soluciones anteriores.�

3. Un campeonato de baloncesto se ha jugado por sistema de liga a dos vueltas (cadapar de equipos se enfrentan dos veces) y sin empate (si el partido acaba en empate hay

2

Page 265: COMPENDIUM OMEFL - ToomatesXXXV Olimpiada Matemática Española Primera Fase Soluciones de la propuesta de problemas Problema 1 ¿Qué dígitos se han omitido en la siguiente multiplicación?

prorrogas hasta que gane uno de los dos). El ganador del partido obtiene 2 puntos yel perdedor 1 punto. Al final del campeonato, la suma de de los puntos obtenidos portodos los equipos salvo el campeon es de 2015 puntos. ¿Cuantos partidos ha ganado elcampeon?

Solucion. Supongamos que el numero de equipos es n. Entonces, se juegan un totalde 2

(n2

)= n2−n partidos en el campeonato por ser a doble vuelta. En cada partido se

dan 3 puntos, por lo que 3n2 − 3n es el numero total de puntos dados. Si el campeontiene P puntos, y los otros n− 1 equipos tienen entre todos 2015 puntos, entonces

P = 3n2 − 3n− 2015,

donde ademas P > 2015n−1 para poder ser el campeon. Para que se cumpla esto, ha de ser

3n2 − 3n− 2015 >2015

n− 1, 3n(n− 1) >

2015n

n− 1, n− 1 >

√2015

3.

Como 252 = 625 < 20153 , se tiene que n > 26, o n ≥ 27.

Por otra parte, la puntuacion maxima que ha podido obtener el ganador es 4(n− 1), siha ganado todos sus partidos (2 partidos con cada uno de los otros n− 1 equipos), esdecir,

3n2 − 3n− 2015 ≤ 4(n− 1), (3n− 4)(n− 1) ≤ 2015.

Ahora bien, si n ≥ 28, entonces 3n− 4 ≥ 80, n− 1 ≥ 27, y 80 · 27 = 2160 > 2015, luegoha de ser n ≤ 27.Luego n = 27 es el numero de equipos en el campeonato, el numero de puntos obtenidospor el campeon es 3 · 272 − 3 · 27− 2015 = 91, y como estos puntos se han obtenido en2 · 26 = 52 partidos, el numero de partidos ganados (en los que se obtienen 2 puntos enlugar de 1) es claramente 91− 52 = 39.

4. Los enteros positivos x, y, z cumplen

x+ 2y = z, x2 − 4y2 + z2 = 310.

Halla todos los posibles valores del producto xyz.

Solucion 1. Podemos despejar 2y de la primera ecuacion y sustituir en la segunda,con lo que ha de cumplirse

310 = x2 − (z − x)2 + z2 = 2zx, zx = 155 = 5 · 31.

Luego al ser 5, 31 primos, se tiene que z ha de tomar uno de los valores 155, 31, 5, 1,tomando x respectivamente los valores 1, 5, 31, 155. Como ademas z = x+ 2y > x, losdos ultimos casos quedan descartados. En los dos primeros casos, se tiene que y = z−x

2toma respectivamente los valores 77 y 13, resultando respectivamente en

xyz = 1 · 77 · 155 = 11935, xyz = 5 · 13 · 31 = 2015.

Solucion 2. Como x2 − 4y2 = (x − 2y)(x + 2y) = z(x − 2y), tenemos que z ha dedividir a 310−z2, luego a 310. Ademas, z no puede ser par, pues en ese caso x tambien

3

Page 266: COMPENDIUM OMEFL - ToomatesXXXV Olimpiada Matemática Española Primera Fase Soluciones de la propuesta de problemas Problema 1 ¿Qué dígitos se han omitido en la siguiente multiplicación?

lo serıa, y x2 − 4y2 + z2 serıa multiplo de 4, pero 310 no lo es. Luego z ha de dividir a155 = 5 · 31, es decir, z ha de tomar uno de los valores 1, 5, 31, 155. Como z = x+ 2y,con x, y enteros positivos, es imposible que z = 1, y si z = 5, entonces bien x = 3,y = 1, bien x = 1, y = 2, que obviamente no satisfacen la segunda ecuacion. Se tieneentonces que z = 31 o z = 155, tomando entonces respectivamente 2y − x = z2−310

z losvalores 21 y 153, que junto a 2y + x = z, nos permite hallar los mismos valores de x, yque por el metodo anterior, bastando multiplicarlos para hallar los dos mismos valoresdel producto xyz.

5. En una recta tenemos cuatro puntos A, B, C y D, en ese orden, de forma queAB = CD. E es un punto fuera de la recta tal que CE = DE. Demuestra que∠CED = 2∠AEB si y solo si AC = EC.

Solucion 1. Sea F el punto tal que los triangulos ABF y CDE son iguales. Claramenteun triangulo es el otro desplazado por AC, luego EF = AC y AF = CE = DE = BF .Trazamos la circunferencia de centro F que pasa por A y B, y como ∠AFB = ∠CED,por ser el angulo central el doble del inscrito, ∠AEB = 2∠CED si y solo si E estasobre la circunferencia que acabamos de trazar, es decir, si y solo si EF = AF , y estoes equivalente a AC = EC.

A B C D

EF

A B C D

E

M

P Q

Solucion 2. Sean M el punto medio de CD, P el simetrico de E respecto de C, y Qel simetrico de E respecto de la recta CD.

A B C D

EF

A B C D

E

M

P Q

El triangulo EPQ es el resultado de aplicar a ECM una homotecia de centro E y razon2, con lo que EPQ es claramente rectangulo en Q, con PQ = 2CM = CD = AB, siendoademas PQ paralelo a CD, luego a AB. Se tiene entonces que EP es diametro de lacircunferencia circunscrita a EPQ, que tiene por lo tanto centro en C y radio CE. Almismo tiempo, al ser ABQP paralelogramo por ser AB = PQ paralelos, AP es paralela

4

Page 267: COMPENDIUM OMEFL - ToomatesXXXV Olimpiada Matemática Española Primera Fase Soluciones de la propuesta de problemas Problema 1 ¿Qué dígitos se han omitido en la siguiente multiplicación?

a BQ, que es la simetrica de BE respecto a AD, mientras que AP es la simetrica deAE respecto a AD, luego ∠PAQ = ∠AEB. Se tiene entonces que ∠CED = 2∠AEBy CE = CA son ambos equivalentes a ∠PAQ = ∠PEQ, luego equivalentes entre sı,como querıamos demostrar.

Solucion 3. Notemos en primer lugar que solo es necesario demostrar que si AC = EC,entonces ∠CED = 2∠AEB. Para ello, consideremos que A esta a la izquierda de Dsobre la recta horizontal AD, y AB esta en una posicion tal que ∠AEB es la mitad de∠CED. Si ahora desplazamos E hacia la derecha (equivalente a desplazar AB haciala izquierda), ∠AEB decrece (nos basta con considerar la circunferencia circunscrita aAEB en su posicion inicial, y observar que E ”sale” de la circunferencia). De formaanaloga, si desplazamos E hacia la izquierda (equivalente a desplazar AB hacia laderecha), ∠AEB crece (E ”entra” en la circunferencia circunscrita a AEB). Luegoexiste a lo sumo una posicion de AB sobre la recta AD a la izquierda de CD, tal que∠CED = 2∠AEB, y nos basta con demostrar que cuando AC = EC, AB esta dehecho en tal posicion.Sea entonces un sistema de coordenadas con centro en C y tal que el eje horizontalcoincide con la recta por A,B,C,D. Denotando por R a la distancia EC, y llamando∠CED = 2α (con lo que α es claramente agudo), se tiene que AB = CD = 2R sinα,A ≡ (−R, 0) por ser AC = EC, B ≡ (−R+2R sinα, 0) y E ≡ (R sinα,R cosα). Ahorabien, −→

AE ≡ (R+R sinα,R cosα),−−→BE ≡ (R−R sinα,R cosα),

con lo que −→AE ·

−−→BE = R2 −R2 sin2 α+R2 cos2 α = 2R2 cos2 α,∣∣∣−→AE∣∣∣ =

√R2 + 2R2 sinα+R2 sin2 α+R2 cos2 α = R

√2 + 2 sinα,∣∣∣−→AE∣∣∣ =

√R2 − 2R2 sinα+R2 sin2 α+R2 cos2 α = R

√2− 2 sinα,

y como √2 + 2 sinα ·

√2− 2 sinα = 2

√1− sin2 α = 2 cosα,

tenemos que

cos∠AEB =

−→AE ·

−−→BE∣∣∣−→AE∣∣∣ · ∣∣∣−−→BE∣∣∣ =

2R2 cos2 α

2R2 cosα= cosα,

y queda concluıda la demostracion.Tambien es posible completar la demostracion sin realizar la observacion inicial, tomandoA ≡ (−d, 0), donde hemos de demostrar que d = AC si y solo si cos∠AEB = cosα.La segunda condicion se traduce, tras algo de algebra, en la relacion(

d2 −R2) (d2 +R2 + 2R2 cos(2α)

)sin2 α = 0,

donde el segundo y el tercer factores son claramente positivos (usamos para ello qued > R para que B este a la izquierda de C), con lo que esta relacion es en efectoequivalente a d = R, como querıamos demostrar.

6. Halla todas las ternas de reales positivos (x, y, z) que cumplan el sistema

2x√x+ 1− y(y + 1) = 1,

5

Page 268: COMPENDIUM OMEFL - ToomatesXXXV Olimpiada Matemática Española Primera Fase Soluciones de la propuesta de problemas Problema 1 ¿Qué dígitos se han omitido en la siguiente multiplicación?

2y√y + 1− z(z + 1) = 1,

2z√z + 1− x(x+ 1) = 1.

Solucion 1. Notese que, por la desigualdad entre medias aritmetica y geometrica, setiene que

x2 + x+ 1 ≥ 2√x2(x+ 1) = 2x

√x+ 1,

con igualdad si y solo si x2 = x + 1, es decir si y solo si x es una raız de la ecuacionr2 − r − 1 = 0. Se tiene entonces de la primera ecuacion que

y2 + y + 1 = 2x√x+ 1 ≤ x2 + x+ 1,

y de forma similar para las otras dos, con lo que

x2 + x+ 1 ≥ y2 + y + 1 ≥ z2 + z + 1 ≥ x2 + x+ 1,

con lo que se ha de dar la igualdad en las tres desigualdades, es decir, x, y, z sonsoluciones de la ecuacion r2− r− 1 = 0. El producto de las dos raıces de esta ecuaciones −1, luego exactamente una de ellas es negativa, y x, y, z son iguales entre sı e igualesa la raız positiva, es decir, la unica solucion es

x = y = z =1 +√

5

2.

Solucion 2. Supongamos que x < y, luego de la primera ecuacion obtenemos

2x√x+ 1 = y2 + y + 1 > x2 + x+ 1,

o tras elevar al cuadrado y reagrupar terminos,

0 > x4 − 2x3 − x2 + 2x+ 1 =(x2 − x− 1

)2,

claramente falso, luego x ≥ y, con x = y si y solo si son iguales a la raız positiva der2 − r − 1 = 0. De forma similar, obtenemos de la segunda ecuacion que y ≥ z, y dela tercera que z ≥ x, con analogas condiciones de igualdad. Luego x ≥ y ≥ z ≥ x, conigualdad si y solo si x, y, z son la raız positiva de la ecuacion r2−r−1 = 0, obteniendosela misma unica solucion que por el metodo anterior.

6

Page 269: COMPENDIUM OMEFL - ToomatesXXXV Olimpiada Matemática Española Primera Fase Soluciones de la propuesta de problemas Problema 1 ¿Qué dígitos se han omitido en la siguiente multiplicación?

1 Los enteros positivos x, y, z cumplen

x+ 2y = z, x2 − 4y2 + z2 = 310.

Halla todos los posibles valores del producto xyz.

Solucion 1. Podemos despejar 2y de la primera ecuacion y sustituir en la segunda,con lo que ha de cumplirse

310 = x2 − (z − x)2 + z2 = 2zx, zx = 155 = 5 · 31.

Luego al ser 5, 31 primos, se tiene que z ha de tomar uno de los valores 155, 31, 5, 1,tomando x respectivamente los valores 1, 5, 31, 155. Como ademas z = x+ 2y > x, losdos ultimos casos quedan descartados. En los dos primeros casos, se tiene que y = z−x

2toma respectivamente los valores 77 y 13, resultando respectivamente en

xyz = 1 · 77 · 155 = 11935, xyz = 5 · 13 · 31 = 2015.

Solucion 2. Como x2 − 4y2 = (x − 2y)(x + 2y) = z(x − 2y), tenemos que z ha dedividir a 310−z2, luego a 310. Ademas, z no puede ser par, pues en ese caso x tambien

1

Problemas Primera Sesion

Page 270: COMPENDIUM OMEFL - ToomatesXXXV Olimpiada Matemática Española Primera Fase Soluciones de la propuesta de problemas Problema 1 ¿Qué dígitos se han omitido en la siguiente multiplicación?

lo serıa, y x2 − 4y2 + z2 serıa multiplo de 4, pero 310 no lo es. Luego z ha de dividir a155 = 5 · 31, es decir, z ha de tomar uno de los valores 1, 5, 31, 155. Como z = x+ 2y,con x, y enteros positivos, es imposible que z = 1, y si z = 5, entonces bien x = 3,y = 1, bien x = 1, y = 2, que obviamente no satisfacen la segunda ecuacion. Se tieneentonces que z = 31 o z = 155, tomando entonces respectivamente 2y − x = z2−310

z losvalores 21 y 153, que junto a 2y + x = z, nos permite hallar los mismos valores de x, yque por el metodo anterior, bastando multiplicarlos para hallar los dos mismos valoresdel producto xyz.

2 En una recta tenemos cuatro puntos A, B, C y D, en ese orden, de forma queAB = CD. E es un punto fuera de la recta tal que CE = DE. Demuestra que∠CED = 2∠AEB si y solo si AC = EC.

Solucion 1. Sea F el punto tal que los triangulos ABF y CDE son iguales. Claramenteun triangulo es el otro desplazado por AC, luego EF = AC y AF = CE = DE = BF .Trazamos la circunferencia de centro F que pasa por A y B, y como ∠AFB = ∠CED,por ser el angulo central el doble del inscrito, ∠AEB = 2∠CED si y solo si E estasobre la circunferencia que acabamos de trazar, es decir, si y solo si EF = AF , y estoes equivalente a AC = EC.

A B C D

EF

A B C D

E

M

P Q

Solucion 2. Sean M el punto medio de CD, P el simetrico de E respecto de C, y Qel simetrico de E respecto de la recta CD.

A B C D

EF

A B C D

E

M

P Q

El triangulo EPQ es el resultado de aplicar a ECM una homotecia de centro E y razon2, con lo que EPQ es claramente rectangulo en Q, con PQ = 2CM = CD = AB, siendoademas PQ paralelo a CD, luego a AB. Se tiene entonces que EP es diametro de lacircunferencia circunscrita a EPQ, que tiene por lo tanto centro en C y radio CE. Almismo tiempo, al ser ABQP paralelogramo por ser AB = PQ paralelos, AP es paralela

2

Page 271: COMPENDIUM OMEFL - ToomatesXXXV Olimpiada Matemática Española Primera Fase Soluciones de la propuesta de problemas Problema 1 ¿Qué dígitos se han omitido en la siguiente multiplicación?

a BQ, que es la simetrica de BE respecto a AD, mientras que AP es la simetrica deAE respecto a AD, luego ∠PAQ = ∠AEB. Se tiene entonces que ∠CED = 2∠AEBy CE = CA son ambos equivalentes a ∠PAQ = ∠PEQ, luego equivalentes entre sı,como querıamos demostrar.

Solucion 3. Notemos en primer lugar que solo es necesario demostrar que si AC = EC,entonces ∠CED = 2∠AEB. Para ello, consideremos que A esta a la izquierda de Dsobre la recta horizontal AD, y AB esta en una posicion tal que ∠AEB es la mitad de∠CED. Si ahora desplazamos E hacia la derecha (equivalente a desplazar AB haciala izquierda), ∠AEB decrece (nos basta con considerar la circunferencia circunscrita aAEB en su posicion inicial, y observar que E ”sale” de la circunferencia). De formaanaloga, si desplazamos E hacia la izquierda (equivalente a desplazar AB hacia laderecha), ∠AEB crece (E ”entra” en la circunferencia circunscrita a AEB). Luegoexiste a lo sumo una posicion de AB sobre la recta AD a la izquierda de CD, tal que∠CED = 2∠AEB, y nos basta con demostrar que cuando AC = EC, AB esta dehecho en tal posicion.Sea entonces un sistema de coordenadas con centro en C y tal que el eje horizontalcoincide con la recta por A,B,C,D. Denotando por R a la distancia EC, y llamando∠CED = 2α (con lo que α es claramente agudo), se tiene que AB = CD = 2R sinα,A ≡ (−R, 0) por ser AC = EC, B ≡ (−R+2R sinα, 0) y E ≡ (R sinα,R cosα). Ahorabien, −→

AE ≡ (R+R sinα,R cosα),−−→BE ≡ (R−R sinα,R cosα),

con lo que −→AE ·

−−→BE = R2 −R2 sin2 α+R2 cos2 α = 2R2 cos2 α,∣∣∣−→AE∣∣∣ =

√R2 + 2R2 sinα+R2 sin2 α+R2 cos2 α = R

√2 + 2 sinα,∣∣∣−→AE∣∣∣ =

√R2 − 2R2 sinα+R2 sin2 α+R2 cos2 α = R

√2− 2 sinα,

y como √2 + 2 sinα ·

√2− 2 sinα = 2

√1− sin2 α = 2 cosα,

tenemos que

cos∠AEB =

−→AE ·

−−→BE∣∣∣−→AE∣∣∣ · ∣∣∣−−→BE∣∣∣ =

2R2 cos2 α

2R2 cosα= cosα,

y queda concluıda la demostracion.Tambien es posible completar la demostracion sin realizar la observacion inicial, tomandoA ≡ (−d, 0), donde hemos de demostrar que d = AC si y solo si cos∠AEB = cosα.La segunda condicion se traduce, tras algo de algebra, en la relacion(

d2 −R2) (d2 +R2 + 2R2 cos(2α)

)sin2 α = 0,

donde el segundo y el tercer factores son claramente positivos (usamos para ello qued > R para que B este a la izquierda de C), con lo que esta relacion es en efectoequivalente a d = R, como querıamos demostrar.

3 Halla todas las ternas de reales positivos (x, y, z) que cumplan el sistema

2x√x+ 1− y(y + 1) = 1,

3

Page 272: COMPENDIUM OMEFL - ToomatesXXXV Olimpiada Matemática Española Primera Fase Soluciones de la propuesta de problemas Problema 1 ¿Qué dígitos se han omitido en la siguiente multiplicación?

2y√y + 1− z(z + 1) = 1,

2z√z + 1− x(x+ 1) = 1.

Solucion 1. Notese que, por la desigualdad entre medias aritmetica y geometrica, setiene que

x2 + x+ 1 ≥ 2√x2(x+ 1) = 2x

√x+ 1,

con igualdad si y solo si x2 = x + 1, es decir si y solo si x es una raız de la ecuacionr2 − r − 1 = 0. Se tiene entonces de la primera ecuacion que

y2 + y + 1 = 2x√x+ 1 ≤ x2 + x+ 1,

y de forma similar para las otras dos, con lo que

x2 + x+ 1 ≥ y2 + y + 1 ≥ z2 + z + 1 ≥ x2 + x+ 1,

con lo que se ha de dar la igualdad en las tres desigualdades, es decir, x, y, z sonsoluciones de la ecuacion r2− r− 1 = 0. El producto de las dos raıces de esta ecuaciones −1, luego exactamente una de ellas es negativa, y x, y, z son iguales entre sı e igualesa la raız positiva, es decir, la unica solucion es

x = y = z =1 +√

5

2.

Solucion 2. Supongamos que x < y, luego de la primera ecuacion obtenemos

2x√x+ 1 = y2 + y + 1 > x2 + x+ 1,

o tras elevar al cuadrado y reagrupar terminos,

0 > x4 − 2x3 − x2 + 2x+ 1 =(x2 − x− 1

)2,

claramente falso, luego x ≥ y, con x = y si y solo si son iguales a la raız positiva der2 − r − 1 = 0. De forma similar, obtenemos de la segunda ecuacion que y ≥ z, y dela tercera que z ≥ x, con analogas condiciones de igualdad. Luego x ≥ y ≥ z ≥ x, conigualdad si y solo si x, y, z son la raız positiva de la ecuacion r2−r−1 = 0, obteniendosela misma unica solucion que por el metodo anterior.

4

Page 273: COMPENDIUM OMEFL - ToomatesXXXV Olimpiada Matemática Española Primera Fase Soluciones de la propuesta de problemas Problema 1 ¿Qué dígitos se han omitido en la siguiente multiplicación?

Problemas Segunda Sesion

4. Alrededor de una mesa circular estan sentadas seis personas. Cada una lleva unsombrero. Entre cada dos personas hay una mampara de modo que cada una puedever los sombreros de las tres que estan enfrente, pero no puede ver el de la persona desu izquierda ni el de la de su derecha ni el suyo propio. Todas saben que tres de lossombreros son blancos y tres negros. Tambien saben que cada una de ellas es capazde obtener cualquier deduccion logica que sea factible. Empezamos por una de las seispersonas y le preguntamos ”¿puedes deducir el color de algun sombrero de los que noves?”. Una vez que ha respondido (todas oyen la respuesta), pasamos a la persona desu izquierda y le hacemos la misma pregunta, y ası sucesivamente. Demuestra que unade las tres primeras respondera ”Sı”.

Solucion. Numeramos las personas en el orden en que van respondiendo, con lo quela persona 1 ve los sombreros de las personas 3, 4, 5, la persona 2 los de las personas4, 5, 6, y la persona 3 los de las personas 5, 6, 1.Supongamos que ni la persona 1 ni la persona 2 han podido responder ”Sı”. Lossombreros de las personas 3, 4, 5 no pueden ser todos del mismo color, porque si no lapersona 1 sabrıa que todos los sombreros que no ve son del otro color. Si los sombrerosde las personas 4, 5 fueran del mismo color, entonces la persona 2 sabe que el sombrero3 ha de ser del otro color, con lo que los sombreros 4, 5 han de ser de distinto color.Pero entonces la persona 3 sabe que el color del sombrero 4, que no ve, es distinto aldel sombrero 5, que sı ve. Luego o una de las dos primeras personas contesta ”Sı”, o silas dos primeras contestan ”No”, entonces la tercera contesta ”Sı”.

5 El triangulo ABC es isosceles en C, y sea Γ su circunferencia circunscrita. SeaM el punto medio del arco BC de Γ que no contiene a A, y sea N el punto donde laparalela a AB por M vuelve a cortar a Γ. Se sabe que AN es paralela a BC. ¿Cualesson las medidas de los angulos de ABC?

Solucion. Si AN es paralela a BC, entonces ABCN es un trapecio con circunferenciacircunscrita, y por lo tanto isosceles.

CB

NA

M

5

Page 274: COMPENDIUM OMEFL - ToomatesXXXV Olimpiada Matemática Española Primera Fase Soluciones de la propuesta de problemas Problema 1 ¿Qué dígitos se han omitido en la siguiente multiplicación?

Se tiene entonces que ∠ANC = ∠BAN . Pero ∠NAC = ∠ACB y ∠ANM = ∠ABCpor ser AN y BC paralelas, y ser AB y MN tambien paralelas. Tenemos entoncespor una parte que ∠BAN = ∠A + ∠C. Finalmente, ∠CNM = ∠CAM = 1

2∠A porser M el punto medio del arco BC, siendo entonces AM la bisectriz de ∠A, con lo que∠ANC = ∠B+ 1

2∠A. Igualando ambos angulos, y usando que ∠A = ∠B por ser ABCisosceles en C, se tiene que ∠A = ∠B = 2∠C, luego 180◦ = ∠A + ∠B + ∠C = 5∠C,para ∠C = 36◦, y ∠A = ∠B = 72◦.

6 Sean x, y, z reales positivos tales que x+y+z = 3. Halla el valor maximo alcanzadopor √

x+√

2y + 2 +√

3z + 6.

¿Para que valores de x, y, z se alcanza dicho maximo?

Solucion 1. Consideremos los vectores(√x,√y + 1,

√z + 2

)y(√

1,√

2,√

3), cuyas

coordenadas son todas reales y positivas, cuyos modulos respectivos son√x+ y + z + 3 =√

6 y√

1 + 2 + 3 =√

6, y cuyo producto escalar es la expresion cuyo maximo se pidehallar. Por la desigualdad del producto escalar, el valor maximo es igual al productode modulos de vectores, que es 6, dandose la igualdad cuando ambos vectores son pro-porcionales. En este caso, al tener ambos vectores el mismo modulo, se da la igualdadsi y solo si ambos vectores son iguales, es decir,

√x+

√2y + 2 +

√3z + 6 ≤ 6,

con igualdad si y solo si x = y = z = 1.

Solucion 2. La funcion f(x) =√x es concava, por lo tanto, por la desigualdad de

Jensen, se tiene que

√x+

√2y + 2 +

√3z + 6 = f(x) + 2f

(y + 1

2

)+ 3f

(z + 2

3

)≤

≤ 6f

(x+ (y + 1) + (z + 2)

6

)= 6f(1) = 6,

con igualdad si y solo si x = y+12 = z+2

3 , es decir, y = 2x− 1 y z = 3x− 2, con lo quex+ y + z = 6x− 3 = 3, o x = y = z = 1.

6

Page 275: COMPENDIUM OMEFL - ToomatesXXXV Olimpiada Matemática Española Primera Fase Soluciones de la propuesta de problemas Problema 1 ¿Qué dígitos se han omitido en la siguiente multiplicación?

Problemas Segunda Sesion

1. Alrededor de una mesa circular estan sentadas seis personas. Cada una lleva unsombrero. Entre cada dos personas hay una mampara de modo que cada una puedever los sombreros de las tres que estan enfrente, pero no puede ver el de la persona desu izquierda ni el de la de su derecha ni el suyo propio. Todas saben que tres de lossombreros son blancos y tres negros. Tambien saben que cada una de ellas es capazde obtener cualquier deduccion logica que sea factible. Empezamos por una de las seispersonas y le preguntamos ”¿puedes deducir el color de algun sombrero de los que noves?”. Una vez que ha respondido (todas oyen la respuesta), pasamos a la persona desu izquierda y le hacemos la misma pregunta, y ası sucesivamente. Demuestra que unade las tres primeras respondera ”Sı”.

Solucion. Numeramos las personas en el orden en que van respondiendo, con lo quela persona 1 ve los sombreros de las personas 3, 4, 5, la persona 2 los de las personas4, 5, 6, y la persona 3 los de las personas 5, 6, 1.Supongamos que ni la persona 1 ni la persona 2 han podido responder ”Sı”. Lossombreros de las personas 3, 4, 5 no pueden ser todos del mismo color, porque si no lapersona 1 sabrıa que todos los sombreros que no ve son del otro color. Si los sombrerosde las personas 4, 5 fueran del mismo color, entonces la persona 2 sabe que el sombrero3 ha de ser del otro color, con lo que los sombreros 4, 5 han de ser de distinto color.Pero entonces la persona 3 sabe que el color del sombrero 4, que no ve, es distinto aldel sombrero 5, que sı ve. Luego o una de las dos primeras personas contesta ”Sı”, o silas dos primeras contestan ”No”, entonces la tercera contesta ”Sı”.

2. El triangulo ABC es isosceles en C, y sea Γ su circunferencia circunscrita. SeaM el punto medio del arco BC de Γ que no contiene a A, y sea N el punto donde laparalela a AB por M vuelve a cortar a Γ. Se sabe que AN es paralela a BC. ¿Cualesson las medidas de los angulos de ABC?

Solucion. Si AN es paralela a BC, entonces ABCN es un trapecio con circunferenciacircunscrita, y por lo tanto isosceles.

CB

NA

M

7

Page 276: COMPENDIUM OMEFL - ToomatesXXXV Olimpiada Matemática Española Primera Fase Soluciones de la propuesta de problemas Problema 1 ¿Qué dígitos se han omitido en la siguiente multiplicación?

Se tiene entonces que ∠ANC = ∠BAN . Pero ∠NAC = ∠ACB y ∠ANM = ∠ABCpor ser AN y BC paralelas, y ser AB y MN tambien paralelas. Tenemos entoncespor una parte que ∠BAN = ∠A + ∠C. Finalmente, ∠CNM = ∠CAM = 1

2∠A porser M el punto medio del arco BC, siendo entonces AM la bisectriz de ∠A, con lo que∠ANC = ∠B+ 1

2∠A. Igualando ambos angulos, y usando que ∠A = ∠B por ser ABCisosceles en C, se tiene que ∠A = ∠B = 2∠C, luego 180◦ = ∠A + ∠B + ∠C = 5∠C,para ∠C = 36◦, y ∠A = ∠B = 72◦.

3. Sean x, y, z reales positivos tales que x+y+z = 3. Halla el valor maximo alcanzadopor √

x+√

2y + 2 +√

3z + 6.

¿Para que valores de x, y, z se alcanza dicho maximo?

Solucion 1. Consideremos los vectores(√x,√y + 1,

√z + 2

)y(√

1,√

2,√

3), cuyas

coordenadas son todas reales y positivas, cuyos modulos respectivos son√x+ y + z + 3 =√

6 y√

1 + 2 + 3 =√

6, y cuyo producto escalar es la expresion cuyo maximo se pidehallar. Por la desigualdad del producto escalar, el valor maximo es igual al productode modulos de vectores, que es 6, dandose la igualdad cuando ambos vectores son pro-porcionales. En este caso, al tener ambos vectores el mismo modulo, se da la igualdadsi y solo si ambos vectores son iguales, es decir,

√x+

√2y + 2 +

√3z + 6 ≤ 6,

con igualdad si y solo si x = y = z = 1.

Solucion 2. La funcion f(x) =√x es concava, por lo tanto, por la desigualdad de

Jensen, se tiene que

√x+

√2y + 2 +

√3z + 6 = f(x) + 2f

(y + 1

2

)+ 3f

(z + 2

3

)≤

≤ 6f

(x+ (y + 1) + (z + 2)

6

)= 6f(1) = 6,

con igualdad si y solo si x = y+12 = z+2

3 , es decir, y = 2x− 1 y z = 3x− 2, con lo quex+ y + z = 6x− 3 = 3, o x = y = z = 1.

4. Encuentra todas las aplicaciones f : Z→ Z que verifican f(n) + f(n+ 1) = 2n+ 1para cualquier entero n y ademas

63∑i=1

f(i) = 2015.

Solucion. Notese que f(n+1) = 2n+1−f(n), con lo que podemos hallar sucesivamente

f(1) = 1− f(0), f(2) = 3− f(1) = 2 + f(0), f(3) = 5− f(2) = 3− f(0), . . .

8

Page 277: COMPENDIUM OMEFL - ToomatesXXXV Olimpiada Matemática Española Primera Fase Soluciones de la propuesta de problemas Problema 1 ¿Qué dígitos se han omitido en la siguiente multiplicación?

Esto nos permite conjeturar que f(n) = n + (−1)nf(0) para todo n ≥ 0, cosa quepodemos demostrar por induccion, siendo cierto como ya se ha visto para n = 1, 2, 3, ysi es cierto para n, entonces

f(n+ 1) = 2n+ 1− n− (−1)nf(0) = n+ 1 + (−1)n+1f(0).

Luego es cierto para todo entero positivo n. Induccion hacia atras, usando que f(n) =2n + 1 − f(n + 1) nos permite comprobar igualmente que esta expresion es de hechovalida para todo entero n, tambien negativo.Notese ahora que en {1, 2, 3, . . . , 63} hay exactamente un entero impar mas que enterospares. En la imagen de cada entero par aparece f(0) con signo positivo, y en la de cadaentero impar con signo negativo, luego

2015 =

63∑i=1

f(i) = −f(0) +

63∑i=1

i =63 · 64

2− f(0) = 2016− f(0).

Concluımos que f(0) = 1, con lo que la unica funcion que satisface las condiciones deenunciado es

f(n) = n+ (−1)n.

5. Sea n ≥ 2 un entero positivo. Tenemos 2n bolas, en cada una de las cuales hayescrito un entero. Se cumple que, siempre que formamos n parejas con las bolas, dosde estas parejas tienen la misma suma.

(1) Demuestra que hay cuatro bolas con el mismo numero.

(2) Demuestra que el numero de valores distintos que hay en las bolas es como muchon− 1.

Solucion. (1) Sean los valores de las bolas, en orden no creciente, a1 ≥ a2 ≥ · · · ≥a2n. Formemos la pareja k-esima emparejando la bola a2k−1 con la bola a2k parak = 1, 2, . . . , n, con lo que sus sumas son

s1 = a1 + a2 ≥ s2 = a3 + a4 ≥ · · · ≥ sn = a2n−1 + a2n.

Al estar las sumas en orden no creciente, si dos de ellas son iguales, han de ser igualesdos sumas consecutivas, es decir ha de ser a2k−1 + a2k = a2k+1 + a2k+2, con a2k−1 ≥a2k ≥ a2k+1 ≥ a2k+2, luego obviamente estos cuatro enteros han de ser iguales.

(2) Supongamos que hay al menos n valores distintos, que podemos ordenar en ordendecreciente b1 > b2 > · · · > bn. Ordenamos ahora los valores de las restantes n bolas enorden no creciente, c1 ≥ c2 ≥ · · · ≥ cn. Haciendo las parejas (bi, ci) para i = 1, 2, . . . , nes claro que las parejas i-esima e i+ 1-esimas tienen valores bi + ci > bi+1 + ci+1, con loque las parejas estan ordenadas con valores de suma estrictamente decrecientes, y nopuede haber dos con la misma suma, contradiccion. Luego hay a lo sumo n− 1 valoresdistintos.

6. Encuentra todos los enteros positivos n, que verifican

n = 22x−1 − 5x− 3 =(2x−1 − 1

)(2x + 1)

9

Page 278: COMPENDIUM OMEFL - ToomatesXXXV Olimpiada Matemática Española Primera Fase Soluciones de la propuesta de problemas Problema 1 ¿Qué dígitos se han omitido en la siguiente multiplicación?

para algun entero positivo x.

Solucion 1. Realizando el producto del miembro de la derecha y reorganizandoterminos, la igualdad entre los miembros segundo y tercero se puede escribir como

2x−1 = 5x+ 2.

Se comprueba facilmente que ni x = 1 ni x = 2 son soluciones, mientras que si x ≥ 3,entonces 5x+ 2 ha de ser multiplo de 4, luego x es par pero no multiplo de 4, es decir,x ≥ 6. Para x = 6, se comprueba que 2x−1 = 5x+ 2 = 32, con lo que serıa una posiblesolucion. Notese ahora que si para x ≥ 6 se tiene que 2x−1 > 5x (cosa que es ciertapara x = 6), entonces 2x > 10x > 5(x + 1) + 2, es decir, por induccion 2x−1 siempresera mayor que 5x+ 2 para todo x ≥ 7. Luego el unico valor que puede tomar el enteropositivo x es 6, que a su vez resulta en

n = 211 − 30− 3 = 2048− 33 = 2015.

Solucion 2. Obtenemos igual que en la solucion anterior la ecuacion 2x−1 = 5x+ 2, yencontramos la solucion x = 6, con lo que nos queda solo demostrar que es unica.La funcion y = 2x−1 es convexa en x, luego tiene a lo sumo dos intersecciones concualquier recta, en particular con la recta y = 5x + 2. Como 2x−1 > 5x + 2 tantopara x → −∞ como para x → +∞, siendo 2x−1 < 5x + 2 para x = 0, hay enefecto exactamente dos soluciones de 2x−1 = 5x+ 2, una negativa (que por lo tanto esirrelevante al problema) y otra positiva, que es x = 6, y por lo tanto es la unica parala que x es un entero positivo.

10

Page 279: COMPENDIUM OMEFL - ToomatesXXXV Olimpiada Matemática Española Primera Fase Soluciones de la propuesta de problemas Problema 1 ¿Qué dígitos se han omitido en la siguiente multiplicación?

LII Olimpiada Matematica Espanola

Primera Fase

Primera sesion

Viernes manana, 15 de enero de 2016

OlimpiadaMatemáticaEspañola RSME

1. Con baldosas cuadradas de lado un numero exacto de unidades se ha podidoembaldosar una habitacion de superficie 18144 unidades cuadradas de lasiguiente manera: el primer dıa se puso una baldosa, el segundo dos baldosas,el tercero tres, etc. ¿Cuantas baldosas fueron necesarias?

2. Hemos empezado la Olimpiada Matematica puntualmente a las 9:00, comohe comprobado en mi reloj, que funcionaba en ese momento correctamente.Cuando he terminado, a las 13:00, he vuelto a mirar el reloj y he visto quelas manecillas se habıan desprendido de su eje pero manteniendo la posicionen la que estaban cuando el reloj funcionaba. Curiosamente las manecillasde las horas y de los minutos aparecıan superpuestas exactamente, una sobreotra, formando un angulo (no nulo) menor que 120◦ con el segundero. ¿A quehora se me averio el reloj? (Dar la respuesta en horas, minutos y segundoscon un error maximo de un segundo; se supone que, cuando funcionaba, lasmanecillas del reloj avanzaban de forma continua.

3. Sea ABC un triangulo rectangulo en C no isosceles con catetos b > a.

i) Hallar el lado del cuadrado AXY Z que circunscribe al triangulo ABC (losvertices B y C tienen que estar en lados distintos del cuadrado).

ii) Explicar paso a paso como construir el cuadrado AXY Z con regla ycompas.

No esta permitido el uso de calculadoras.Cada problema se puntua sobre 7 puntos.

El tiempo de cada sesion es de 3 horas y media.

Page 280: COMPENDIUM OMEFL - ToomatesXXXV Olimpiada Matemática Española Primera Fase Soluciones de la propuesta de problemas Problema 1 ¿Qué dígitos se han omitido en la siguiente multiplicación?

LII Olimpiada Matematica Espanola

Primera Fase

Segunda sesion

Viernes tarde, 15 de enero de 2016

OlimpiadaMatemáticaEspañola RSME

4. Las tres raıces del polinomio x3−14x2+Bx−84 son los lados de un triangulorectangulo. Hallar B.

5. En un triangulo ABC la bisectriz por A, la mediana por B y la altura porC son concurrentes y ademas la bisectriz por A y la mediana por B sonperpendiculares. Si el lado AB mide una unidad, hallar cuanto miden losotros dos lados.

6. De cuantas formas se pueden colorear los vertices de un poligono con n ≥ 3lados usando tres colores de forma que haya exactamente m lados, 2 ≤ m ≤n, con los extremos de colores diferentes?

No esta permitido el uso de calculadoras.Cada problema se puntua sobre 7 puntos.

El tiempo de cada sesion es de 3 horas y media.

Page 281: COMPENDIUM OMEFL - ToomatesXXXV Olimpiada Matemática Española Primera Fase Soluciones de la propuesta de problemas Problema 1 ¿Qué dígitos se han omitido en la siguiente multiplicación?

LII Olimpiada Matematica Espanola

Primera Fase

Primera sesion

Viernes tarde, 15 de enero de 2016

OlimpiadaMatemáticaEspañola RSME

1. En la primera fila de un tablero 5×5 se colocan 5 fichas que tienen una carablanca y otra negra, mostrando todas la cara blanca. Cada ficha se puedemover de una casilla a cualquiera de las contiguas (horizontal o vertical-mente) dandole la vuelta en cada movimiento. Ademas, varias fichas puedenocupar una misma casilla. ¿Se puede conseguir mediante una secuencia demovimientos que las 5 fichas queden en la ultima fila, en casillas distintas yque todas ellas muestren la cara negra?

2. Cada 20 minutos durante una semana se travasa una cantidad exacta delitros de agua (siempre la misma cantidad) desde un tanque con 25000 litrosa otro deposito inicialmente vacıo. Desde este segundo deposito, a intervalosregulares de tiempo, se extrae primero 1 litro, luego 2, luego 3, etc. Justoal final de la semana coinciden el ultimo travase y la ultima extraccion,quedando en ese momento vacıo el segundo deposito. Determinar cuantaagua se ha extraıdo en total durante la semana, en caso de que los datosdel problema lo permitan. (Se supone que los trasvases y las extracciones serealizan instantaneamente. El primer trasvase se hace pasados los primeros20 minutos y la primera extraccion, pasado el primer intervalo de tiempo.)

3. Sea n ≥ 1 y P (x) un polinomio con coeficientes enteros que cumple quelos numeros P (1), P (2), . . . , P (n) son 1, 2, . . . , n (no necesariamente en esteorden). Demostrar que uno de los numeros P (0) o P (n+ 1) es multiplo den!.

No esta permitido el uso de calculadoras.Cada problema se puntua sobre 7 puntos.

El tiempo de cada sesion es de 3 horas y media.

Page 282: COMPENDIUM OMEFL - ToomatesXXXV Olimpiada Matemática Española Primera Fase Soluciones de la propuesta de problemas Problema 1 ¿Qué dígitos se han omitido en la siguiente multiplicación?

LII Olimpiada Matematica Espanola

Primera Fase

Segunda sesion

Sabado manana, 16 de enero de 2016

OlimpiadaMatemáticaEspañola RSME

4. Para pertenecer a un club cada nuevo socio debe pagar como cuota de in-scripcion a cada miembro del club la misma cantidad que el tuvo que pagaren total cuando ingreso mas un euro. Si el primer socio pago un euro, ¿cuantodebera pagar en total el n-esimo socio?

5. Dos circunferencias C y C ′ son secantes en dos puntos P y Q. La recta queune los centros corta a C en R y a C ′ en R′, la que une P y R′ corta a C enX �= P y la que une P y R corta a C ′ en X ′ �= P . Si los tres puntos X , Q,X ′ estan alineados se pide:

i) Hallar el angulo � XPX ′.ii) Demostrar que (d+ r− r′)(d− r+ r′) = rr′, donde d es la distancia entrelos centros de las circunferencias y r y r′ sus radios.

6. Encontrar cuantas soluciones enteras tiene la ecuacion

|5− x1 − x2|+ |5 + x1 − x2|+ |5 + x2 + x3|+ |5 + x2 − x3| = 20

No esta permitido el uso de calculadoras.Cada problema se puntua sobre 7 puntos.

El tiempo de cada sesion es de 3 horas y media.

Page 283: COMPENDIUM OMEFL - ToomatesXXXV Olimpiada Matemática Española Primera Fase Soluciones de la propuesta de problemas Problema 1 ¿Qué dígitos se han omitido en la siguiente multiplicación?

LII Olimpiada Matematica Espanola

Primera Fase

Primera sesion

Sabado manana, 16 de enero de 2016

OlimpiadaMatemáticaEspañola RSME

1. Para pertenecer a un club cada nuevo socio debe pagar como cuota de in-scripcion a cada miembro del club la misma cantidad que el tuvo que pagaren total cuando ingreso mas un euro. Si el primer socio pago un euro, ¿cuantodebera pagar en total el n-esimo socio?

2. Dos circunferencias C y C ′ son secantes en dos puntos P y Q. La recta queune los centros corta a C en R y a C ′ en R′, la que une P y R′ corta a C enX �= P y la que une P y R corta a C ′ en X ′ �= P . Si los tres puntos X , Q,X ′ estan alineados se pide:

i) Hallar el angulo � XPX ′.ii) Demostrar que (d+ r− r′)(d− r+ r′) = rr′, donde d es la distancia entrelos centros de las circunferencias y r y r′ sus radios.

3. Encontrar cuantas soluciones enteras tiene la ecuacion

|5− x1 − x2|+ |5 + x1 − x2|+ |5 + x2 + x3|+ |5 + x2 − x3| = 20

No esta permitido el uso de calculadoras.Cada problema se puntua sobre 7 puntos.

El tiempo de cada sesion es de 3 horas y media.

Page 284: COMPENDIUM OMEFL - ToomatesXXXV Olimpiada Matemática Española Primera Fase Soluciones de la propuesta de problemas Problema 1 ¿Qué dígitos se han omitido en la siguiente multiplicación?

LII Olimpiada Matematica Espanola

Primera Fase

Segunda sesion

Sabado tarde, 16 de enero de 2016

OlimpiadaMatemáticaEspañola RSME

4. Encontrar la solucion entera mas pequena de la ecuacion

⌊x8

⌋−

⌊ x

40

⌋+

⌊ x

240

⌋= 210

(Si x es un numero real, �x� es la parte entera de x, esto es, el mayor numeroentero menor o igual que x.)

5. Sean C y C ′ dos circunferencias tangentes exteriores con centros O y O′

y radios 1 y 2, respectivamente. Desde O se traza una tangente a C ′ conpunto de tangencia en P ′ y desde O′ se traza la tangente a C con punto detangencia en P en el mismo semiplano que P ′ respecto de la recta que pasapor O y O′. Hallar el area del triangulo OXO′, donde X es el punto de cortede O′P y OP ′.

6. Si n es un numero natural, el n-esimo numero triangular es Tn = 1+2+· · ·+n.Hallar todos los valores de n para los que el producto de los 16 numerostriangulares consecutivos TnTn+1 · · ·Tn+15 es un cuadrado perfecto.

No esta permitido el uso de calculadoras.Cada problema se puntua sobre 7 puntos.

El tiempo de cada sesion es de 3 horas y media.

Page 285: COMPENDIUM OMEFL - ToomatesXXXV Olimpiada Matemática Española Primera Fase Soluciones de la propuesta de problemas Problema 1 ¿Qué dígitos se han omitido en la siguiente multiplicación?

Soluciones

Viernes manana, primera sesion

1. Con baldosas cuadradas de lado un numero exacto de unidades se ha

podido embaldosar una habitacion de superficie 18144 unidades cuadradas

de la siguiente manera: el primer dıa se puso una baldosa, el segundo, dos

baldosas, el tercero tres, etc. ¿Cuantas baldosas fueron necesarias?

Solucion. Supongamos que fueron necesarias n baldosas y que su tamano

es k × k. Entonces nk2 = 18144 = 25 × 34 × 7. Hay nueve casos posibles

para n, a saber, 2 × 7, 23 × 7, 25 × 7, 2 × 32 × 7, 23 × 32 × 7, 25 × 32 × 7,

2× 34 × 7, 23 × 34 × 7, 25 × 34 × 7. Ademas este numero tiene que poderse

expresar en la forma 1 + 2 + 3 + · · ·+N = N(N + 1)/2 y esto solo es posible

en el caso sexto: 25 × 32 × 7 = 63 × 64/2 = 2016. Para descartar los otros

casos rapidamente observamos que N y N + 1 son numeros primos entre sı.

Si por ejemplo N(N + 1)/2 = 23 × 7, tendrıa que ser N + 1 = 24 y N = 7,

que es imposible, etc. Por tanto, se necesitaron 2016 baldosas.

2. Hemos empezado la Olimpiada Matematica puntualmente a las 9:00,

como he comprobado en mi reloj, que funcionaba en ese momento correcta-

mente. Cuando he terminado, a las 13:00, he vuelto a mirar el reloj y he

visto que las manecillas se habıan desprendido de su eje pero manteniendo

la posicion en la que estaban cuando el reloj funcionaba. Curiosamente

las manecillas de las horas y de los minutos aparecıan superpuestas exacta-

mente, una sobre otra, formando un angulo (no nulo) menor que 120◦ con

el segundero. ¿A que hora se me averio el reloj? (Dar la respuesta en horas,

minutos y segundos con un error maximo de un segundo; se supone que,

cuando funcionaba, las manecillas del reloj avanzaban de forma continua.)

Solucion. Si medimos el tiempo t en segundos a partir de las 00:00 y los

angulos en grados, en sentido horario y a partir de la posicion de las maneci-

llas a las 00:00, tenemos que el angulo barrido por la manecilla de las horas

4

Page 286: COMPENDIUM OMEFL - ToomatesXXXV Olimpiada Matemática Española Primera Fase Soluciones de la propuesta de problemas Problema 1 ¿Qué dígitos se han omitido en la siguiente multiplicación?

en el instante t es αhor(t) = t/120 y barrido por el minutero, αmin(t) = t/10.

Como ambas manecillas han aparecido superpuestas, los dos angulos han de

coincidir en el momento t en que el reloj se ha averiado. El minutero ha

podido dar alguna vuelta completa, por tanto debe tenerse

t

10=

t

120+ 360k,

con k ≥ 0 un numero entero, es decir, t = 360×12011 k. Como la averıa ha

sido entre las 9:00 y las 13:00, tiene que ser 9 ≤ k ≤ 12. El angulo para el

segundero es αseg(t) = 6t, por tanto la diferencia

6t− t

120=

360× 719

11k = (360× 65 +

360× 4

11)k

debe ser, salvo multiplos de 360, un numero β entre −120 y 120. Si k = 9,

β = (360 × 3)/11, que efectivamente esta en este rango. Sin embargo, si

k = 10 o 12, β = ±(360 × 4)/11, que esta fuera de este intervalo. El caso

k = 11 tambien se excluye puesto que se tendrıa β = 0 y las tres manecillas

no estan superpuestas. Por lo tanto el unico caso posible es k = 9, que

corresponde al momento

t =360× 120× 9

11= 3600× 9 + 60× 49 + 5 +

5

11,

lo que significa que el reloj se averio a las 9:49:05.

3. Sea ABC un triangulo rectangulo en C no isosceles con catetos b > a.

(i) Hallar el lado del cuadrado AXY Z que circunscribe al triangulo ABC

(los vertices B y C tienen que estar en lados distintos del cuadrado).

(ii) Explicar paso a paso como construir el cuadrado AXY Z con regla y

compas.

Solucion. (i) Sea l la longitud del cuadrado y x la longitud del segmento

XC. Los triangulos rectangulos AXC y BY C son semejantes (puesto que

∠BCY = π/2 − ∠ACX = ∠CAX), de donde l/b = (l − x)/a, es decir,

x/l = (b− a)/b. Entonces, aplicando el Teorema de Pitagoras,

b2 = l2 + x2 = l2(1 + (x

l)2) = l2(1 +

(b− a)2

b2),

de donde l = b2/√

(b− a)2 + b2.

5

Page 287: COMPENDIUM OMEFL - ToomatesXXXV Olimpiada Matemática Española Primera Fase Soluciones de la propuesta de problemas Problema 1 ¿Qué dígitos se han omitido en la siguiente multiplicación?

(ii) Para construir el cuadrado observamos que, por (i), se tiene que la

tangente del angulo α = ∠CAX es x/l = (b − a)/b. Prolongamos el lado

BC del triangulo hasta un punto B′ de modo que BB′ mida b unidades.

Ası, CB′ mide b− a y el angulo ∠CAB′ tiene tangente (b− a)/b. El vertice

X del cuadrado que buscamos tiene que estar entonces sobre la recta que

contiene a A y B′. Por otro lado, el angulo ∠AXC tiene que ser de 90◦,

ası que X tiene que estar sobre la circunferencia con diametro AC. Por

lo tanto, basta con trazar esta circunferencia y su interseccion con la recta

por A y B′ sera el punto X buscado. Los puntos Y y Z que completan el

cuadrado se obtienen ahora facilmente.

Viernes tarde, segunda sesion

4. Las tres raıces del polinomio x3 − 14x2 + Bx − 84 son los lados de un

triangulo rectangulo. Hallar B.

Solucion. Sean u, v y w las tres raıces y supongamos que w2 = u2+v2. Por

las relaciones de Cardano, u+v+w = 14, uv+uw+vw = B y uvw = 84. Si

s = u+v y p = uv, se tiene entonces que s+w = 14, pw = 84 y s2 = w2+2p.

Sustituyendo en esta ultima ecuacion los valores de s y p en funcion de w

y operando, queda w2 − 7w + 6 = 0, luego w = 1 o 6. Si fuera w = 1,

tendrıamos s = 13, p = 84 y u y v serıan raıces de x2−13x+ 84 = 0, que no

tiene soluciones reales. Por tanto, w = 6, s = 8, p = 14 y B = p+ ws = 62.

(Efectivamente, las tres raıces de x3−14x2+62x−84 son 6, 4+√

2 y 4−√

2

y 62 = (4 +√

2)2 + (4−√

2)2.)

6

Page 288: COMPENDIUM OMEFL - ToomatesXXXV Olimpiada Matemática Española Primera Fase Soluciones de la propuesta de problemas Problema 1 ¿Qué dígitos se han omitido en la siguiente multiplicación?

5. En un triangulo ABC la bisectriz por A, la mediana por B y la altura

por C son concurrentes y ademas la bisectriz por A y la mediana por B son

perpendiculares. Si el lado AB mide una unidad, hallar cuanto miden los

otros dos lados.

Solucion. Damos dos soluciones diferentes.

Solucion 1. Sean P , M y Q los pies de la bisectriz por A, la mediana por

B y la altura por C, respectivamente, que se cortan en el punto X. En el

triangulo ABM la bisectriz por A, AX, es perpendicular a BM (puesto que

por hipotesis la mediana y la bisectriz de ABC son perpendiculares), por

tanto ∠ABX = ∠AMX, esto es ABM es isosceles y AM = AB = 1, con lo

cual AC = 2.

Sea BP = x y BQ = y. Por el Teorema de la bisectriz, BP/AB =

PC/AC, esto es PC = 2x. Ahora, por el Teorema de Ceva,

1 =MC

CP· PBBQ· QAAM

=1

2x· xy· 1− y

1,

de donde y = 1/3. Trazamos las perpendiculares a AC por B y X, respec-

tivamente con pies R y S. Los triangulos XQB y XSM son congruentes

(son rectangulos, XB = XM por ser AX la altura del triangulo isosceles

ABM y XQ = XS por ser perpendiculares a los lados AB y AC desde un

punto de la bisectriz), por tanto SM = BQ = y = 1/3. Por otro lado, por

el Teorema de Thales, BX = XM implica RS = SM , luego RS = 1/3 y

AR = AM −RS − SM = 1/3. Finalmente, por el Teorema de Pitagoras,

BC2 = BR2 +RC2 = AB2 −AR2 +RC2 = 1− 1

9+ (2− 1

3)2 =

11

3.

Ası pues, los otros dos lados miden 2 y√333 .

7

Page 289: COMPENDIUM OMEFL - ToomatesXXXV Olimpiada Matemática Española Primera Fase Soluciones de la propuesta de problemas Problema 1 ¿Qué dígitos se han omitido en la siguiente multiplicación?

Solucion 2. Igual que antes, AC = 2. Fijamos un sistema de coordenadas

con origen en A de forma que ~AC = (2, 0). Sea ~AB = (x, y). Entonces

~CX = ~CA+ ~AX = (−2, 0) + (x+ 1

2,y

2) = (

x− 3

2,y

2)

(notar que X es el punto medio de BM). Pero los vectores ~AB y ~CX

son ortogonales, luego x(x − 3) + y2 = 0. Como x2 + y2 = 1, x = 1/3 e

y =√

8/3. Por el Teorema de Pitagoras, BC2 = y2 + (2− x)2 = 11/3, esto

es BC =√

33/3.

6. ¿De cuantas formas se pueden colorear los vertices de un polıgono con

n ≥ 3 lados usando tres colores de forma que haya exactamente m lados,

2 ≤ m ≤ n, con los extremos de colores diferentes?

Solucion. En el polıgono senalamos los puntos medios de los m lados cuyos

extremos deben colorearse con colores diferentes. Esto puede hacerse de(nm

)formas. Los n vertices del polıgono quedan divididos ası en m grupos de

vertices consecutivos en los que todos ellos tienen el mismo color pero los

vertices de grupos adyacentes tienen colores diferentes. Hay entonces tantas

formas de colorear estos grupos como formas de colorear un polıgono de m

lados sin que haya ningun lado con los extremos del mismo color (que es en

realidad el caso particular m = n del problema; esta interpretacion tambien

es valida si m = 2 considerando el “polıgono” con 2 vertices unidos por un

doble lado). La solucion sera entonces(nm

)Cm, donde Cm es este numero.

Obtendremos Cm encontrando una relacion de recurrencia para estos

numeros. Obviamente, C2 = C3 = 6. Supongamos entonces m ≥ 4 y

fijemos tres vertices consecutivos P1, P2, P3. Si P1 y P3 van coloreados de

forma distinta, simplemente podemos unirlos directamente eliminando P2

y resultara un polıgono con m − 1 lados a colorear de la forma indicada.

Recıprocamente, cada uno de estos polıgonos coloreados dara origen a uno

con m lados (el color de P2 quedara determinado por los de P1 y P3), por

tanto contribuyen a Cm con Cm−1 posibilidades. Por otro lado, si P1 y

P3 tienen el mismo color, los podemos pegar eliminando P2, resultando un

polıgono con m − 2 lados que tendremos que colorear. Cada una de estas

coloraciones dara origen ahora a 2 coloraciones para el polıgono original

(pues habra 2 posibilidades para P2) y por tanto, la contribucion a Cm de

este caso es 2Cm−2. Encontramos ası la relacion de recurrencia buscada:

Cm = Cm−1 + 2Cm−2.

De la relacion anterior obtenemos facilmente los primeros valores de Cm,

m ≥ 2: 6, 6, 18, 30, 66, 126,. . . Si comparamos esta sucesion con la de las

8

Page 290: COMPENDIUM OMEFL - ToomatesXXXV Olimpiada Matemática Española Primera Fase Soluciones de la propuesta de problemas Problema 1 ¿Qué dígitos se han omitido en la siguiente multiplicación?

potencias de 2: 4, 8, 16, 32, 64, 128,. . . , parece obvio que Cm = 2m+(−1)m2,

formula que se puede confirmar facilmente por induccion. En efecto, la

formula es correcta para C2 y C3 y, si m ≥ 4,

Cm = Cm−1+2Cm−2 = 2m−1+(−1)m−12+2(2m−2+(−1)m−22) = 2m+(−1)m2.

Por tanto la solucion es(nm

)(2m + (−1)m2).

Page 291: COMPENDIUM OMEFL - ToomatesXXXV Olimpiada Matemática Española Primera Fase Soluciones de la propuesta de problemas Problema 1 ¿Qué dígitos se han omitido en la siguiente multiplicación?

Viernes tarde, primera sesion

1. En la primera fila de un tablero 5 × 5 se colocan 5 fichas que tienen

una cara blanca y otra negra, mostrando todas la cara blanca. Cada ficha

se puede mover de una casilla a cualquiera de las contiguas (horizontal o

verticalmente) dandole la vuelta en cada movimiento. Ademas, varias fichas

pueden ocupar una misma casilla. ¿Se puede conseguir mediante una se-

cuencia de movimientos que las 5 fichas queden en la ultima fila, en casillas

distintas y que todas ellas muestren la cara negra?

Solucion. Si pintamos las casillas del tablero alternativamente de blanco

y negro como en un tablero de ajedrez, sucede que una ficha cuyo color

visible coincida con el de la casilla, al moverse seguira teniendo el mismo

color que la nueva casilla (puesto que tanto el color de la ficha como el de la

casilla cambian). Supuesto que la casilla superior izquierda la hemos dejado

blanca, en el inicio hay 3 fichas cuyo color (blanco) coincide con el de la

casilla. En todo momento debera suceder que el color de tres fichas es el

mismo que el de la casilla que ocupen (y el de las otras dos, diferente). Sin

embargo, colocando las fichas con la cara negra en la ultima fila, resulta que

solo dos fichas tendran el color (negro) de su casilla. Por lo tanto, no es

posible colocar las fichas de esta manera.

2. Cada 20 minutos durante una semana se travasa una cantidad exacta de

litros de agua (siempre la misma cantidad) desde un tanque con 25000 litros

a otro deposito inicialmente vacıo. Desde este segundo deposito, a intervalos

regulares de tiempo, se extrae primero 1 litro, luego 2, luego 3, etc. Justo

al final de la semana coinciden el ultimo travase y la ultima extraccion,

quedando en ese momento vacıo el segundo deposito. Determinar cuanta

agua se ha extraıdo en total durante la semana, en caso de que los datos

del problema lo permitan. (Se supone que los trasvases y las extracciones se

realizan instantaneamente. El primer trasvase se hace pasados los primeros

20 minutos y la primera extraccion, pasado el primer intervalo de tiempo.)

9

Page 292: COMPENDIUM OMEFL - ToomatesXXXV Olimpiada Matemática Española Primera Fase Soluciones de la propuesta de problemas Problema 1 ¿Qué dígitos se han omitido en la siguiente multiplicación?

Solucion. Sea n el numero de extracciones de agua realizadas durante la

semana. En total habran extraıdo Tn = 1+2+· · ·+n = n(n+1)/2 litros. Por

otro lado si el caudal que se trasvasa cada 20 minutos al segundo deposito es

de k litros, el total de litros que ha entrado es 7×24×3×k = 23×32×7×k,

ası que 23 × 32 × 7× k = n(n+ 1)/2 y esta cantidad tiene que ser ≤ 25000,

por tanto 24 × 32 × 7 × k = n(n + 1) ≤ 50000. Por la ultima desigualdad,

n ≤ 223. Ahora, los numeros n y n + 1 son primos entre sı luego cada

potencia 24, 32, 7 divide a n o a n+ 1. Ciertamente 24 × 32 × 7 = 1008 no

puede dividir a n ni a n + 1, dado que n ≤ 223. Supongamos que n = 16c

es multiplo de 16. Entonces n + 1 es multiplo de 9 o 7. En el primer caso

se tendrıa n + 1 = 16c + 1 ≡ 0 (mod 9), es decir, c ≡ 5 (mod 9). Pero si

c = 5, n = 80 y 7 no divide a 80×81 y, si c ≥ 5+9 = 14, n ≥ 16×14 = 224.

Por otro lado, en el segundo caso, n + 1 = 16c + 1 ≡ 0 (mod 7), de donde

c ≡ 3 (mod 7). Pero 9 no divide al producto n(n + 1) si c = 3 o 10, y si

c ≥ 17, n > 223. Concluimos que n no es multiplo de 16 y n+ 1 sı. Si n es

multiplo de 9 y n+ 1 de 16× 7, tendrıamos n = 16× 7× c− 1 ≡ 0 (mod 9),

es decir, c ≡ 7 (mod 9) y entonces c ≥ 7 y n > 223. Similarmente, si n es

multiplo de 7 y n + 1 de 16 × 9, n = 16 × 9 × c − 1 ≡ 0 (mod 7), es decir,

c ≡ 2 (mod 7) y entonces c ≥ 2 y n > 223. El unico caso que queda es que

n sea multiplo de 9 × 7 = 63 y n + 1 de 16. Entonces n + 1 = 63c + 1 ≡ 0

(mod 16) y c ≡ 1 (mod 16) y necesariamente c = 1 (si no n > 223). Por lo

tanto solo hay una solucion posible, a saber, n = 63, lo que da un volumen

total extraıdo de T63 = 63× 64/2 = 2016 litros.

3. Sea n ≥ 1 y P (x) un polinomio con coeficientes enteros que cumple que

los numeros P (1), P (2), . . . , P (n) son 1, 2, . . . , n (no necesariamente en este

orden). Demostrar que uno de los numeros P (0) o P (n+ 1) es multiplo de

n!.

Solucion. Si i y j son dos numeros enteros, se tiene que ik − jk = (i −j)(ik−1 + ik−2j + · · · + ijk−2 + jk−1) es multiplo de i − j. Entonces, si

P (x) = amxm + · · ·+ a2x

2 + a1x+ a0,

P (i)− P (j) = am(im − jm) + · · ·+ a2(i2 − j2) + a1(i− j)

tambien es multiplo de i − j. En particular, n − 1 divide a P (n) − P (1).

Como P (1) y P (n) son enteros distintos entre 1 y n tiene que ser P (1) = 1

y P (n) = n o al reves, P (1) = n y P (n) = 1. En el primer caso, n − 2 =

(n− 1)− 1 divide a P (n− 1)−P (1) = P (n− 1)− 1 y 2 ≤ P (n− 1) ≤ n− 1,

luego tiene que ser P (n− 1) = n− 1 y, similarmente P (n− 2) = n− 2, etc.

10

Page 293: COMPENDIUM OMEFL - ToomatesXXXV Olimpiada Matemática Española Primera Fase Soluciones de la propuesta de problemas Problema 1 ¿Qué dígitos se han omitido en la siguiente multiplicación?

De forma parecida se ve que en el segundo caso P (n− 1) = 2, P (n− 2) = 3,

etc.

Si P (i) = i para todo 1 ≤ i ≤ n, todos estos numeros son raıces de

P (x)− x, luego

P (x) = c(x)(x− 1)(x− 2) . . . (x− n) + x

para algun polinomio con coeficientes enteros c(x). Por otro lado, si P (i) =

n− i+ 1 para todo 1 ≤ i ≤ n, se tiene que todos los enteros 1 ≤ i ≤ n son

raıces de P (x)− n+ x− 1, luego

P (x) = c(x)(x− 1)(x− 2) . . . (x− n) + n− x+ 1

para algun polinomio con coeficientes enteros c(x). En el primer caso P (0) =

(−1)nc(0)n! y, en el segundo, P (n+ 1) = c(n+ 1)n!, luego efectivamente n!

divide a P (0) o a P (n+ 1).

S´abado ma˜nana, segunda sesi´on

4, 1. Para pertenecer a un club cada nuevo socio debe pagar como cuota

de inscripcion a cada miembro del club la misma cantidad que el tuvo que

pagar en total cuando ingreso mas un euro. Si el primer socio pago un euro,

¿cuanto debera pagar en total el n-esimo socio?

Solucion. Sea an la cuota total del socio n-esimo y sea sn = a1 + · · ·+ an.

El n-esimo (n ≥ 2) socio tiene que pagar en total (a1 + 1) + (a2 + 1) + · · ·+(an−1 + 1) = sn−1 + n− 1 euros, luego an = sn−1 + n− 1 y

sn = sn−1 + an = sn−1 + sn−1 + (n− 1) = 2sn−1 + n− 1.

Iterando esta relacion queda sn = 2n−1 + 2n−2×1 + 2n−3×2 + · · ·+ 2× (n−2) + (n− 1), de donde sn = 2sn− sn = 2n + 2n−2 + 2n−3 + · · ·+ 2− n+ 1 =

2n + 2n−1 − 1− n y entonces, para n ≥ 2,

an = sn − sn−1 = 2n − 2n−2 − 1 = 3× 2n−2 − 1.

5, 2. Dos circunferencias C y C ′ son secantes en dos puntos P y Q. La

recta que une los centros corta a C en R y a C ′ en R′, la que une P y R′

corta a C en X 6= P y la que une P y R corta a C ′ en X ′ 6= P . Si los tres

puntos X, Q, X ′ estan alineados se pide:

(i) hallar el angulo ∠XPX ′.

11

Page 294: COMPENDIUM OMEFL - ToomatesXXXV Olimpiada Matemática Española Primera Fase Soluciones de la propuesta de problemas Problema 1 ¿Qué dígitos se han omitido en la siguiente multiplicación?

(ii) demostrar que (d + r − r′)(d − r + r′) = rr′, donde d es la distancia

entre los centros de las circunferencias y r y r′ sus radios.

Solucion. (i) Sean F y F ′ los puntos diametralmente opuestos a R y

R′ en C y C ′, respectivamente. Por el Teorema del angulo inscrito se

tiene que ∠PFQ = ∠PXQ = α que, por simetrıa, es el doble de ∠PFR,

luego ∠PFR = α/2. Como el triangulo PFR es rectangulo en P (al ser

FR diametro de C), deducimos que ∠PRF = π/2 − α/2. Similarmente,

∠PR′F ′ = π/2 − β/2, donde β = ∠PX ′Q. Por otro lado, considerando

el triangulo XPX ′, ∠XPX ′ = π − α − β, luego sumando los angulos del

triangulo PRR′,

2− α

2) + (

π

2− β

2) + (π − α− β) = π,

es decir α+ β = 2π/3 y ∠XPX ′ = π/3.

(ii) Consideramos el triangulo OPO′, donde O y O′ son los centros de C y C ′,

respectivamente. Nuevamente, por el Teorema del angulo inscrito, el angulo

central ∠POR es 2∠PFR = α y similarmente ∠PO′R′ = 2∠PF ′R′ = β,

luego ∠OPO′ = π/3. Los lados del triangulo OPO′ son los radios r y r′ y

la distancia d entre los centros, por tanto el resultado se sigue directamente

del Teorema del coseno: d2 = r2 + r′2 − rr′, que es equivalente a la relacion

dada en el enunciado.

12

Page 295: COMPENDIUM OMEFL - ToomatesXXXV Olimpiada Matemática Española Primera Fase Soluciones de la propuesta de problemas Problema 1 ¿Qué dígitos se han omitido en la siguiente multiplicación?

6, 3. Encontrar cuantas soluciones enteras tiene la ecuacion

|5− x1 − x2|+ |5 + x1 − x2|+ |5 + x2 + x3|+ |5 + x2 − x3| = 20.

Solucion. Podemos reescribir la ecuacion en la forma

|y1|+ |y2 − y1|+ |y3 − y2|+ |20− y3| = 20, (1)

donde y1 = 5 − x1 − x2, y2 = 10 − 2x2 y y3 = 15 − x2 + x3, por tanto

toda solucion entera de la ecuacion original da una solucion entera de (??)

con y2 un numero par. Recıprocamente, es inmediato comprobar que toda

solucion de (??) con y2 par, da una solucion de la ecuacion del enunciado.

Observamos que (??) se puede escribir como

d(0, y1) + d(y1, y2) + d(y2, y3) + d(y3, 20) = d(0, 20),

donde d(x, y) = |x − y| es la distancia entre los numeros reales x e y. Solo

puede darse esta situacion si 0 ≤ y1 ≤ y2 ≤ y3 ≤ 20 (podemos imaginar

una regla de carpintero con cuatro segmentos de longitudes que suman 20

unidades y que tienen que cubrir desde 0 a 20, que es una distancia de 20

unidades. La unica posibilidad es que la regla este completamente estirada).

Se trata entonces de contar las ternas de numeros enteros 0 ≤ y1 ≤ y2 ≤ y3 ≤20 con y2 par. Si escribimos y2 = 2k con 0 ≤ k ≤ 10, hay 2k+1 posibilidades

para y1 y 21− 2k para y3, luego el numero de soluciones buscado es

10∑k=0

(2k+1)(21−2k) =10∑k=0

(21+40k−4k2) = 21×11+40×55−4×385 = 891.

Por tanto el numero de soluciones enteras de la ecuacion dada es 891.

Page 296: COMPENDIUM OMEFL - ToomatesXXXV Olimpiada Matemática Española Primera Fase Soluciones de la propuesta de problemas Problema 1 ¿Qué dígitos se han omitido en la siguiente multiplicación?

S´abado ma˜nana, primera sesi´on

4, 1. Para pertenecer a un club cada nuevo socio debe pagar como cuota

de inscripcion a cada miembro del club la misma cantidad que el tuvo que

pagar en total cuando ingreso mas un euro. Si el primer socio pago un euro,

¿cuanto debera pagar en total el n-esimo socio?

Solucion. Sea an la cuota total del socio n-esimo y sea sn = a1 + · · ·+ an.

El n-esimo (n ≥ 2) socio tiene que pagar en total (a1 + 1) + (a2 + 1) + · · ·+(an−1 + 1) = sn−1 + n− 1 euros, luego an = sn−1 + n− 1 y

sn = sn−1 + an = sn−1 + sn−1 + (n− 1) = 2sn−1 + n− 1.

Iterando esta relacion queda sn = 2n−1 + 2n−2×1 + 2n−3×2 + · · ·+ 2× (n−2) + (n− 1), de donde sn = 2sn− sn = 2n + 2n−2 + 2n−3 + · · ·+ 2− n+ 1 =

2n + 2n−1 − 1− n y entonces, para n ≥ 2,

an = sn − sn−1 = 2n − 2n−2 − 1 = 3× 2n−2 − 1.

5, 2. Dos circunferencias C y C ′ son secantes en dos puntos P y Q. La

recta que une los centros corta a C en R y a C ′ en R′, la que une P y R′

corta a C en X 6= P y la que une P y R corta a C ′ en X ′ 6= P . Si los tres

puntos X, Q, X ′ estan alineados se pide:

(i) hallar el angulo ∠XPX ′.

11

Page 297: COMPENDIUM OMEFL - ToomatesXXXV Olimpiada Matemática Española Primera Fase Soluciones de la propuesta de problemas Problema 1 ¿Qué dígitos se han omitido en la siguiente multiplicación?

(ii) demostrar que (d + r − r′)(d − r + r′) = rr′, donde d es la distancia

entre los centros de las circunferencias y r y r′ sus radios.

Solucion. (i) Sean F y F ′ los puntos diametralmente opuestos a R y

R′ en C y C ′, respectivamente. Por el Teorema del angulo inscrito se

tiene que ∠PFQ = ∠PXQ = α que, por simetrıa, es el doble de ∠PFR,

luego ∠PFR = α/2. Como el triangulo PFR es rectangulo en P (al ser

FR diametro de C), deducimos que ∠PRF = π/2 − α/2. Similarmente,

∠PR′F ′ = π/2 − β/2, donde β = ∠PX ′Q. Por otro lado, considerando

el triangulo XPX ′, ∠XPX ′ = π − α − β, luego sumando los angulos del

triangulo PRR′,

2− α

2) + (

π

2− β

2) + (π − α− β) = π,

es decir α+ β = 2π/3 y ∠XPX ′ = π/3.

(ii) Consideramos el triangulo OPO′, donde O y O′ son los centros de C y C ′,

respectivamente. Nuevamente, por el Teorema del angulo inscrito, el angulo

central ∠POR es 2∠PFR = α y similarmente ∠PO′R′ = 2∠PF ′R′ = β,

luego ∠OPO′ = π/3. Los lados del triangulo OPO′ son los radios r y r′ y

la distancia d entre los centros, por tanto el resultado se sigue directamente

del Teorema del coseno: d2 = r2 + r′2 − rr′, que es equivalente a la relacion

dada en el enunciado.

12

Page 298: COMPENDIUM OMEFL - ToomatesXXXV Olimpiada Matemática Española Primera Fase Soluciones de la propuesta de problemas Problema 1 ¿Qué dígitos se han omitido en la siguiente multiplicación?

6, 3. Encontrar cuantas soluciones enteras tiene la ecuacion

|5− x1 − x2|+ |5 + x1 − x2|+ |5 + x2 + x3|+ |5 + x2 − x3| = 20.

Solucion. Podemos reescribir la ecuacion en la forma

|y1|+ |y2 − y1|+ |y3 − y2|+ |20− y3| = 20, (1)

donde y1 = 5 − x1 − x2, y2 = 10 − 2x2 y y3 = 15 − x2 + x3, por tanto

toda solucion entera de la ecuacion original da una solucion entera de (??)

con y2 un numero par. Recıprocamente, es inmediato comprobar que toda

solucion de (??) con y2 par, da una solucion de la ecuacion del enunciado.

Observamos que (??) se puede escribir como

d(0, y1) + d(y1, y2) + d(y2, y3) + d(y3, 20) = d(0, 20),

donde d(x, y) = |x − y| es la distancia entre los numeros reales x e y. Solo

puede darse esta situacion si 0 ≤ y1 ≤ y2 ≤ y3 ≤ 20 (podemos imaginar

una regla de carpintero con cuatro segmentos de longitudes que suman 20

unidades y que tienen que cubrir desde 0 a 20, que es una distancia de 20

unidades. La unica posibilidad es que la regla este completamente estirada).

Se trata entonces de contar las ternas de numeros enteros 0 ≤ y1 ≤ y2 ≤ y3 ≤20 con y2 par. Si escribimos y2 = 2k con 0 ≤ k ≤ 10, hay 2k+1 posibilidades

para y1 y 21− 2k para y3, luego el numero de soluciones buscado es

10∑k=0

(2k+1)(21−2k) =10∑k=0

(21+40k−4k2) = 21×11+40×55−4×385 = 891.

Por tanto el numero de soluciones enteras de la ecuacion dada es 891.

Sabado tarde, segunda sesion

4. Encontrar la solucion entera mas pequena de la ecuacion

bx8c − b x

40c+ b x

240c = 210.

(Si x es un numero real, bxc es la parte entera de x, esto es, el mayor numero

entero menor o igual que x.)

13

Page 299: COMPENDIUM OMEFL - ToomatesXXXV Olimpiada Matemática Española Primera Fase Soluciones de la propuesta de problemas Problema 1 ¿Qué dígitos se han omitido en la siguiente multiplicación?

Solucion. Sea x una solucion entera de la ecuacion. Dividiendo, primero

por 240, luego el resto r1 por 40 y el nuevo resto r2 por 8, resulta

x = 240c1 + r1 = 240c1 + 40c2 + r2 = 240c1 + 40c2 + 8c3 + r3,

donde 0 ≤ c2 < 6, 0 ≤ c3 < 5 y 0 ≤ r3 < 8 (las desigualdades para c2 y c3se obtienen de 0 ≤ r1 < 240 y 0 ≤ r2 < 40). Entonces

210 = bx8c−b x

40c+b x

240c = (30c1+5c2+c3)−(6c1+c2)+c1 = 25c1+4c2+c3,

y el unico caso posible es c1 = 8, c2 = 2, c3 = 2 (reduciendo modulo 5 queda

c2 ≡ c3 (mod 5) y necesariamente c2 = c3; entonces 42 = 5c1 + c2 y tiene

que ser c2 = 2, c1 = 8), con lo que

x = 240× 8 + 40× 2 + 8× 2 + r3 = 2016 + r3

y la menor solucion entera de la ecuacion dada es x = 2016 (las otras son

2017, . . . , 2023).

5. Sean C y C ′ dos circunferencias tangentes exteriores con centros O y O′

y radios 1 y 2, respectivamente. Desde O se traza una tangente a C ′ con

punto de tangencia en P ′ y desde O′ se traza la tangente a C con punto

de tangencia en P en el mismo semiplano que P ′ respecto de la recta que

pasa por O y O′. Hallar el area del triangulo OXO′, donde X es el punto

de corte de O′P y OP ′.

Solucion. Los triangulos OPO′ y OP ′O′ son rectangulos en P y P ′, res-

pectivamente y ∠PXO = ∠P ′XO′, luego los triangulos PXO y P ′XO′ son

semejantes con razon de semejanza O′P ′/OP = 2. La razon entre sus areas

S′ y S es entonces S′/S = 4. Por el Teorema de Pitagoras OP ′ =√

5 y

O′P = 2√

2, luego si A es el area pedida se tiene que

A+ S′ =1

2O′P ′ ·OP ′ =

√5; A+ S =

1

2OP ·O′P =

√2.

De las relaciones anteriores se obtiene facilmente que A = 4√2−√5

3 .

14

Page 300: COMPENDIUM OMEFL - ToomatesXXXV Olimpiada Matemática Española Primera Fase Soluciones de la propuesta de problemas Problema 1 ¿Qué dígitos se han omitido en la siguiente multiplicación?

6. Si n es un numero natural, el n-esimo numero triangular es Tn = 1 +

2 + · · ·+ n. Hallar todos los valores de n para los que el producto de los 16

numeros triangulares consecutivos TnTn+1 · · ·Tn+15 es un cuadrado perfecto.

Solucion. Como Tn = n(n + 1)/2, el producto de los 16 numeros trian-

gulares es Pn = nCn(n + 16)/216, donde Cn = (n + 1)2 . . . (n + 15)2 es un

cuadrado perfecto. Entonces Pn es un cuadrado perfecto si y solo si lo es

n(n+ 16). Como n y n+ 16 no tienen divisores impares comunes, para que

n(n+ 16) sea un cuadrado perfecto los primos impares que dividen a n y a

n+ 16 deben aparecer elevados a un exponente par, de modo que podemos

escribir n = 2am2 y n + 16 = 2bt2 con a, b = 0 o 1. Mas aun, n y n + 16

tienen la misma paridad, por tanto a = b = 0 o a = b = 1. En el primer

caso t2 = m2 + 16 y solo puede ser m = 3, t = 5 (con m ≥ 7 no puede haber

soluciones porque la diferencia de dos cuadrados distintos ≥ 49 es mayor

que 16 y, con m ≤ 6, solo hay la solucion indicada), por lo que n = 9 y

n(n + 16) = 9 × 25, que es un cuadrado. En el caso a = b = 1, resulta

t2 = m2 + 8, cuya unica solucion es m = 1, t = 3, que corresponde a n = 2,

valor para el que n(n+ 16) = 36 es un cuadrado. La respuesta es por tanto

n = 2 y 9.

15

Page 301: COMPENDIUM OMEFL - ToomatesXXXV Olimpiada Matemática Española Primera Fase Soluciones de la propuesta de problemas Problema 1 ¿Qué dígitos se han omitido en la siguiente multiplicación?

LIII Olimpiada Matematica Espanola

Primera Fase

Primera sesion

Viernes tarde, 13 de enero de 2017

OlimpiadaMatemáticaEspañola RSME

1. Describir todas las soluciones enteras positivas (m,n) de la ecuacion

8m− 7 = n2

y dar el primer valor de m (si existe) mayor que 1959.

2. Se colorean los numeros 1, 2, . . . , n de dos colores, azul y rojo. Probar que sin = 2017 existe una coloracion tal que la ecuacion

8(x+ y) = z

no tiene soluciones monocromaticas. Determinar el menor n para el quenunca es posible colorear los numeros de forma tal que no haya solucionesmonocromaticas.

3. Calcular el numero maximo de raıces reales distintas que puede tener unpolinomio P que verifique la siguiente propiedad: el producto de dos raıcesdistintas de P sigue siendo una raız de P .

No esta permitido el uso de calculadoras.Cada problema se puntua sobre 7 puntos.

El tiempo de cada sesion es de 3 horas y media.

Page 302: COMPENDIUM OMEFL - ToomatesXXXV Olimpiada Matemática Española Primera Fase Soluciones de la propuesta de problemas Problema 1 ¿Qué dígitos se han omitido en la siguiente multiplicación?

LIII Olimpiada Matematica Espanola

Primera Fase

Segunda sesion

Sabado manana, 14 de enero de 2017

OlimpiadaMatemáticaEspañola RSME

4. Encontrar todas las soluciones enteras positivas de

1

a+ b+

1

b+ c+

1

c+ a+

1

a+ b+ c− 2= 1

5. Probar que hay infinitos numeros primos cuyo resto al dividirlos entre 3 es2.

6. En un triangulo acutangulo ABC consideramos su ortocentro, H. SeanA′, B′ y C′ los simetricos de H con respecto a los lados BC, CA y AB,respectivamente. Probar que si los triangulos ABC y A′B′C′ tienen unangulo igual, entonces tambien tienen un lado igual. ¿Es cierto el recıproco?

No esta permitido el uso de calculadoras.Cada problema se puntua sobre 7 puntos.

El tiempo de cada sesion es de 3 horas y media.

Page 303: COMPENDIUM OMEFL - ToomatesXXXV Olimpiada Matemática Española Primera Fase Soluciones de la propuesta de problemas Problema 1 ¿Qué dígitos se han omitido en la siguiente multiplicación?

LIII Olimpiada Matematica Espanola

Primera Fase

Primera sesion

Viernes tarde, 13 de enero de 2017

OlimpiadaMatemáticaEspañola RSME

1. Describir todas las soluciones enteras positivas (m,n) de la ecuacion

8m− 7 = n2

y dar el primer valor de m (si existe) mayor que 1959.

2. Se colorean los numeros 1, 2, . . . , n de dos colores, azul y rojo. Probar que sin = 2017 existe una coloracion tal que la ecuacion

8(x+ y) = z

no tiene soluciones monocromaticas. Determinar el menor n para el quenunca es posible colorear los numeros de forma tal que no haya solucionesmonocromaticas.

3. Calcular el numero maximo de raıces reales distintas que puede tener unpolinomio P que verifique la siguiente propiedad: el producto de dos raıcesdistintas de P sigue siendo una raız de P .

No esta permitido el uso de calculadoras.Cada problema se puntua sobre 7 puntos.

El tiempo de cada sesion es de 3 horas y media.

Page 304: COMPENDIUM OMEFL - ToomatesXXXV Olimpiada Matemática Española Primera Fase Soluciones de la propuesta de problemas Problema 1 ¿Qué dígitos se han omitido en la siguiente multiplicación?

LIII Olimpiada Matematica Espanola

Primera Fase

Segunda sesion

Sabado manana, 14 de enero de 2017

OlimpiadaMatemáticaEspañola RSME

4. Encontrar todas las soluciones enteras positivas de

1

a+ b+

1

b+ c+

1

c+ a+

1

a+ b+ c− 2= 1

5. Probar que hay infinitos numeros primos cuyo resto al dividirlos entre 3 es2.

6. En un triangulo acutangulo ABC consideramos su ortocentro, H. SeanA′, B′ y C′ los simetricos de H con respecto a los lados BC, CA y AB,respectivamente. Probar que si los triangulos ABC y A′B′C′ tienen unangulo igual, entonces tambien tienen un lado igual. ¿Es cierto el recıproco?

No esta permitido el uso de calculadoras.Cada problema se puntua sobre 7 puntos.

El tiempo de cada sesion es de 3 horas y media.

Page 305: COMPENDIUM OMEFL - ToomatesXXXV Olimpiada Matemática Española Primera Fase Soluciones de la propuesta de problemas Problema 1 ¿Qué dígitos se han omitido en la siguiente multiplicación?

Soluciones

Problema 1. Sea E una elipse y consideremos tres rectas paralelas r1, r2 y r3, cada unade las cuales corta a E en dos puntos distintos. Sean estos puntos A1, B1, A2, B2 y A3, B3,respectivamente. Probar que los puntos medios de los segmentos A1B1, A2B2 y A3B3

estan alineados.

Solucion 1. El resultado es inmediato en el caso de que la elipse sea una circunferencia.Esta tres rectas determinan tres cuerdas y sus puntos medios son los puntos de cortede las rectas con un diametro perpendicular a todas ellas. En otro caso, pensandoen la elipse como la interseccion de un cono con un plano (el cono de Apolonio), laproyeccion sobre un plano perpendicular al eje del cono nos da una circunferencia y lasrectas paralelas se proyectan en rectas paralelas.

Solucion 2. Sin perdida de generalidad, podemos suponer que la ecuacion de nuestra

elipse esx2

a2+y2

b2= 1. Entre todas las rectas de pendiente dada m consideraremos dos

particulares: en primer lugar la que pasa por el origen, O, y = mx. Por simetrıa, elorigen sera precisamente el punto medio de los dos puntos en que esta recta corta a laelipse; a continuacion, consideramos una de las dos rectas tangentes a la elipse con estapendiente m en el punto que llamamos P . Si nuestras rectas r1, r2 y r3 tienen pendientem y los puntos medios de los puntos de corte con la elipse van a estar alineados, estospuntos medios deben estar sobre la recta que pasa por O y P .Tomemos una recta con ecuacion y = mx + c y determinemos sus intersecciones conla elipse, A y B, con coordenadas respectivas, (xA, yA) y (xB, yB). xA y xB seran lassoluciones para la ecuacion

x2

a2+

(mx+ c)2

b2= 1,

es decir,(b2 +m2a2)x2 + (2ma2c)x+ a2c2 − a2b2 = 0,

que vienen dadas por

−2ma2c±√

(2ma2c)2 − 4(b2 +m2a2)(a2c2 − a2b2)2(b2 +m2a2)

,

una para cada signo. Por lo tanto, si el punto medio tiene por coordenadas (xM , yM ),

entonces xM =xA + xB

2=−ma2c

b2 +m2a2e yM = mxM + c =

b2c

b2 +m2a2, que estan sobre

la recta y =yMxM

x =−b2

ma2x, que es independiente del valor de c de la recta particular

elegida.

O

P

A

B

y = mx

y = mx+ c

y =−b2

ma2x

Page 306: COMPENDIUM OMEFL - ToomatesXXXV Olimpiada Matemática Española Primera Fase Soluciones de la propuesta de problemas Problema 1 ¿Qué dígitos se han omitido en la siguiente multiplicación?

Solucion 3. Sin perdida de generalidad, podemos suponer que la ecuacion de nuestra

elipse esx2

a2+y2

b2= 1 y que la pendiente comun de r1, r2 y r3 es m. La transformacion,

f , definida por

(x′, y′) = f(x, y) = (x

a,y

b)

convierte la elipse en una circunferencia de ecuacion x′2 + y′2 = 1, y una recta de

ecuacion y = mx + c en una recta de ecuacion by′ = max′ + c, es decir, y′ =ma

bx +

c

b. Por tanto, las imagenes por f de r1, r2 y r3 seran tres rectas que cortan a la

circunferencia formando tres cuerdas paralelas entre sı, y sus puntos medios estaranentonces sobre el diametro ortogonal a todas ellas, es decir, sobre la recta de ecuacion

y′ = − b

max′. Como f conserva los puntos medios, por linealidad, los puntos medios

de A1B1, A2B2 y A3B3 estaran sobre la rectay

b= − b

ma

x

a, es decir, y = − b2

ma2x, el

diametro conjugado de y = mx.

Problema 2. Sea T un triangulo de angulos α, β y γ. ¿Para que valores de α, β y γel triangulo T se puede dividir en tres triangulos congruentes entre sı?

Solucion. Si α = β = γ, el triangulo es equilatero y siendo O el centro de T , lostriangulos que se obtienen uniendo O con cualquiera par de vertices son congruentes.Veremos que solo en este caso se puede obtener una division con tres triangulos con-gruentes, con un vertice comun en el interior de T .

A

BCP

Denotemos por r, s y t los angulos de estos tres triangulos congruentes y supongamos,por ejemplo, que r es uno de los angulos ∠APB, ∠BPC o ∠CPA. Dado que cualquierade estos angulos es menor que π, r sumado a cualquiera de ellos ha de ser mayor queπ, por lo que no pueden ser ni s ni t, pues r+s+ t = π, luego los tres deben ser igualesa r, y T es equilatero.Analicemos ahora el caso en que los tres triangulos congruentes tengan un verticecomun, P sobre uno de los lados, por ejemplo el lado BC, y supongamos que lostriangulos son 4CPA, 4APQ y 4QPB, para algun punto Q sobre el lado AB.Notese que ahora hay analogıa entre los angulos ∠QPA y ∠BPQ, pero no entre estosy ∠APC.

Page 307: COMPENDIUM OMEFL - ToomatesXXXV Olimpiada Matemática Española Primera Fase Soluciones de la propuesta de problemas Problema 1 ¿Qué dígitos se han omitido en la siguiente multiplicación?

A

BCP

Q

Como antes, siendo r, s y t los angulos de los tres triangulos congruentes, comenzamosanalizando el caso en que ∠QPA y ∠BPQ, digamos rPor congruencia se tiene que AQ = QS, por lo que tambien AP = PB. Dado que4APB es isosceles y Q es el punto medio de AB, ∠PQA = ∠PQB = π/2. Luego,los triangulos congruentes son rectangulos y los angulos son r = π/3, s = π/2, t =∠ABC = π/6.Si los angulos ∠APQ y ∠QPB no son el mismo, digamos que son r y s, entonces∠APC = t y dado que tambien r+s+∠PAB+∠PBA, se tendra t = ∠PAB+∠PBA,por lo ninguno de ellos puede ser t. Entonces, ∠PQA = ∠PQB = π/2. Si ademas deno ser el mismo angulo, fuese r 6= s, se tendrıa AQ 6= QB por lo que, por la congruenciade 4AQP y 4QBP , debera tenerse AQ = PB. Pero dos triangulos rectangulos enque la hipotenusa de uno es igual a un cateto del otro, no pueden ser congruentes. Porlo tanto r = s, y estamos en el supuesto anterior.Por tanto, solo existen dos tipos de triangulos que se pueden descomponer en trestriangulos congruentes: los equilateros y los rectangulos (30, 60, 90).

Problema 3. Se considera la funcion f : N→ Z definida como sigue:

f(n) =

{−f(n2 ) si n es par

f(n− 1) + 1 si n es impar,

para n ≥ 0. Demostrar que f(n) es multiplo de 3 si, y solo si, n es multiplo de 3, yhallar el menor numero n que cumple f(n) = 2017.

Solucion 1. De la definicion de f se sigue que f(2a) = (−1)a, para todo a ≥ 0. Siendoa > b ≥ 0, entonces

f(2a + 2b) = f(2b(2a−b + 1)) = (−1)b[(−1)a−b + 1] = (−1)a + (−1)b,

y, en general, si a1 > · · · > ak ≥ 0, se tiene que

f(2a1 + · · ·+ 2ak) = (−1)a1 + · · ·+ (−1)ak .

Los restos de 2n al dividir entre 3 son 1 o 2, dependiendo de si n es par o impar, y dadoque 2 y (−1) se diferencian en 3, se tiene que 2a1 + · · · + 2ak y (−1)a1 + · · · + (−1)ak

dan el mismo resto al dividir entre 3.Para la segunda parte, claramente el menor numero buscado se conseguira sumando lasmenores potencias pares de 2 hasta completar 2017, es decir,

20 + 22 + 22×2 + · · ·+ 22×2016,

progresion geometrica de razon 4 cuya suma es22×2017 − 1

3

Page 308: COMPENDIUM OMEFL - ToomatesXXXV Olimpiada Matemática Española Primera Fase Soluciones de la propuesta de problemas Problema 1 ¿Qué dígitos se han omitido en la siguiente multiplicación?

Solucion 2. Se prueba por induccion en el numero de cifras binarias de n, que f(n) esel numero de unos que tiene la expresion binaria de n en posiciones impares menos elnumero de unos que tiene en posiciones pares (considerando que contamos de derechaa izquierda y que empezamos por la posicion uno). Si

n = (ak . . . a2a1)(2),

entonces usando que 22n+1 ≡ 2 (mod 3) y 22n ≡ 1 (mod 3), resultara que

n ≡ ak(−1)k + · · ·+ a2 − a1,

que es multiplo de 3 si y solo si f(n) = ak(−1)k+1 + · · · − a2 + a1 es multiplo de 3. Dela descripcion de f es obvio que el menor que cumple que f(n) = 2017 es

40 + 41 + · · ·+ 42016 =42017 − 1

3.

Problema 4. Describir todas las soluciones enteras positivas (m,n) de la ecuacion8m− 7 = n2 y dar el primer valor de m (si existe) mayor que 1959.

Solucion. Sea m = k+1, tenemos 8k+1 = n2, o equivalentemente, 8k = (n−1)(n+1).

Evidentemente, n es impar y si consideramos el entero s =n− 1

2, deducimos que

k =1 + s

2s es un numero triangular. De hecho si k es triangular (i.e. m es el numero

posterior a un triangular) siempre hay solucion y por ello las soluciones son (m,n) =

(1 +1 + s

2s,√

1 + 4s(s+ 1)) usando que s es entero no negativo. Asimismo, es facil

verificar que el primer entero triangular superior a 1959 es 2016.

Problema 5. Se colorean los numeros 1, 2, . . . , n de dos colores, azul y rojo. Probarque si n = 2017 existe una coloracion tal que la ecuacion

8(x+ y) = z

no tiene soluciones monocromaticas. Determinar el menor n para el que nunca esposible colorear los numeros de forma tal que no haya soluciones monocromaticas.

Solucion. Para la segunda pregunta, observamos que si el 1 es azul, entonces 8(1+1) =16 es rojo, y por tanto 8(16 + 16) = 256 azul y 8(256 + 1) = 2056 es rojo. En general,si un numero i es azul 8(i+ 1) es rojo, y por consiguiente i− 15 no puede ser rojo yaque en ese caso tendrıamos 8((i− 15) + 16) = 8(i+ 1). Por tanto, si i es azul, i− 15tambien. Ahora bien, 256 es azul y por tanto 256− 15 · 16 = 256− 240 = 16 es azul, locual es una contradiccion. Por tanto, necesariamente ha de ocurrir que n < 2056. Sieso sucede es posible no tener soluciones monocromaticas: pintamos el [1, 15] de azul,el [16, 255] de rojo y el [256, 2055] nuevamente de azul. Por tanto, no hay solucionesmonocromaticas, ya que si x, y fuesen ambos rojos, 8(x + y) ≥ 8(16 + 16) = 256 yya no tenemos numeros rojos tan grandes. Si ambos fuesen azules y ≤ 15 entonces8(x + y) esta entre 16 y 240, con lo que no hay problemas; por su parte, si uno es≥ 256, entonces 8(x + y) ≥ 8 · 257 = 2056, numero que ya no estamos considerando.En el caso 2017 basta considerar esta misma coloracion pintando azul el [256, 2017].

Page 309: COMPENDIUM OMEFL - ToomatesXXXV Olimpiada Matemática Española Primera Fase Soluciones de la propuesta de problemas Problema 1 ¿Qué dígitos se han omitido en la siguiente multiplicación?

Problema 6. Calcular el numero maximo de raıces reales distintas que puede tenerun polinomio P que verifique la siguiente propiedad: el producto de dos raıces distintasde P sigue siendo una raız de P .

Solucion. La respuesta es 4. El polinomio x(x− 1/2)(x− 1)(x− 2), con raıces 0, 1/2,1 y 2, cumple dicha cota. Supongamos que hubiese un polinomio con al menos 5 raıcesdistintas. Es importante resenar que un polinomio tiene una cantidad finita de raıcesy es lo que nos permite tomar maximos y mınimos.Si un polinomio tiene 5 raıces, al menos 4 serıan no nulas y distinguimos dos casos.

1. Si -1 es raız, hay al menos una raız (de hecho, al menos dos) con valor absolutono nulo distinto de 1. Sea A una raız con el mayor valor absoluto entre todasla raıces y supongamos que |A| > 1. Como −1 y A son raıces, −A es raız; peroentonces tambien lo serıa −A · A = −A2 y

∣∣A2∣∣ > |A|, lo que contradirıa la

maximalidad del valor absoluto de A.

Si no existen raıces con valor absoluto > 1, tomemos como A una de las raıces conel menor valor absoluto entre todas las raıces distintas de 0. Se tendra entoncesque |A| < 1, y como −1 y A son raıces, −A es raız y tambien lo sera −A · A =−A2. Pero entonces,

∣∣A2∣∣ < |A|, lo que contradirıa la minimalidad del valor

absoluto de A entre las raıces no nulas.

2. Si −1 no es raız, hay al menos tres raıces no nulas y de modulo distinto de 1.Hay entonces dos raıces A,B con los dos valores absolutos mayores (mayores que1) o bien dos raıces con los dos valores absolutos menores A,B (menores que 1y no nulos). En cualquier caso multiplicando A ·B llegamos a una contradiccioncon la maximalidad o minimalidad.

Problema 7. Encontrar todas las soluciones enteras positivas de

1

a+ b+

1

b+ c+

1

c+ a+

1

a+ b+ c− 2= 1

Solucion. Si a, b, c ≥ 2, entonces la expresion es ≤ 1, con igualdad si y solo si a =b = c = 2. Ası que suponemos ahora que uno de ellos es 1, pongamos sin perdida degeneralidad a = 1. Si b, c ≥ 3 se llega a que la expresion es nuevamente menor que 1,ası que unicamente hay que analizar ahora los casos b = 1 y b = 2, que no dan massoluciones.

Problema 8. Probar que hay infinitos numeros primos cuyo resto al dividirlos entre3 es 2.

Solucion. Evidentemente hay al menos un primo, pues 2 es un ejemplo. Supongamosque solo hubiese un numero finito de ellos, p1, . . . , pk, y sea n el producto de todos ellos.El producto de dos de esos primos deja resto 1 al dividir entre 3, por lo que, dependiendode si k es par o es impar, el resto de dividir n entre 3 sera 1 o 2, respectivamente.Si n deja resto 1 al dividir entre 3, entonces n + 1, que no es divisible por ningunode los primos p1, . . . , pk, deja resto 2. Pero si n no es divisible por ningun primo quedeje resto 2 al dividir entre 3, como no es multiplo de 3, quiere decir n es producto

Page 310: COMPENDIUM OMEFL - ToomatesXXXV Olimpiada Matemática Española Primera Fase Soluciones de la propuesta de problemas Problema 1 ¿Qué dígitos se han omitido en la siguiente multiplicación?

de primos que dejan resto 1 al dividir entre 3, por lo que n tambien deja resto 1, encontradiccion con que deja resto 2.Si n deja resto 2 al dividir entre 3, razonamos con n+3 en vez de con n+1 y obtendremosde nuevo una contradiccion, por lo que no es posible que haya solo un numero finito deprimos con la condicion pedida.

Problema 9. En un triangulo acutangulo ABC consideramos su ortocentro, H. SeanA′, B′ y C ′ los simetricos de H con respecto a los lados BC,CA y AB, respectivamente.Probar que si los triangulos ABC y A′B′C ′ tienen un angulo igual, entonces tambientiene un lado igual. ¿Es cierto el recıproco?

Solucion. Por perpendicularidad de sus lados, ∠CAH = ∠HBC. Por simetrıa con re-specto a CB, ∠CBA′ = ∠HBC, por lo que A′ esta sobre la circunferencia circunscritaa ABC, y analogamente B′ y C ′.Por el teorema del seno, siendo a el lado opuesto al angulo α y R el cincunradio, setiene que a = 2R sinα. Como ambos triangulos comparten circuncırculo, si tiene dosangulos de igual medida tendran dos lados de igual medida.

A

BC

H

A′

El recıproco no es cierto, y el mismo teorema del seno nos sirve para demostrarlo. Bastatener en cuenta que A′B′C ′ no es necesariamente acutangulo, de hecho sus angulosmiden, respectivamente 180 − 2α, 180 − 2β y 180 − 2γ, siendo α, β y γ los angulos deABC, y tomar angulos cuya media sea 90. Por ejemplo, si ABC tiene un angulo de80 y A′B′C ′ tiene uno de 100, ya tendremos garantizado que ambos triangulos tienensendos lados que mide lo mismo. Como 100 = 180 − 2α, se tendra que los angulos deABC son 40,60 y 80, mientras que los de A′B′C ′ seran 100, 60 y 20. No hay dos conel mismo valor.

Problema 10. Probar que dados 4n puntos en el espacio tridimensional, tales queno hay cuatro de ellos coplanarios, siempre se pueden formar n piramides de basetriangular de modo que no hay intersecciones entre ellas.

Solucion. Evidentemente por hipotesis no hay 3 puntos alineados, pues en ese casoanadiendo un cuarto punto cualquiera a la terna violarıamos la hipotesis sobre elcaracter no coplanario.

Page 311: COMPENDIUM OMEFL - ToomatesXXXV Olimpiada Matemática Española Primera Fase Soluciones de la propuesta de problemas Problema 1 ¿Qué dígitos se han omitido en la siguiente multiplicación?

Consideremos todas las posibles ternas de puntos y el plano que determina (unıvocamente,al no haber 3 puntos alineados) cada terna. Hay una cantidad finita de estos planos y,por ello, es posible elegir una familia de planos paralelos que recubran el espacio queno sean paralelos a ninguno de este conjunto finito. Todo plano de dicha familia con-tiene, a lo sumo, dos puntos del conjunto de partida, por construccion. Sin perdida degeneralidad podemos suponer que dicha familia de planos es, por ejemplo, la de planoshorizontales.A continuacion vamos a elegir planos horizontales de dicha familia A1, A2, . . . , An talesque no contengan ningun punto del conjunto inicial y verifiquen la siguiente propiedad.Elegimos A1 para que divida al espacio en dos zonas, superior e inferior, y que la su-perior tenga 4 puntos (o 5 en el peor caso, si resulta que el cuarto y el quinto estana la misma altura). Realizamos una piramide con los cuatro puntos (si hubiese cinco,evitamos utilizar uno de los dos inferiores). Construimos un plano A2 paralelo a A1 demodo que por encima de A2 haya solo 4 o 5 puntos (excluyendo los que ya se han uti-lizado para construir la piramide). Realizamos una piramide con los cuatro puntos (sihubiese cinco, evitamos utilizar uno de los dos inferiores). . . . Construimos un planoAk+1 paralelo a Ak de modo que por encima de Ak haya solo 4 o 5 puntos (excluyendolos que ya se han utilizado para construir piramides). Realizamos una piramide con loscuatro puntos (si hubiese cinco, evitamos utilizar uno de los dos inferiores). Tras con-struir el plano An−1 solo quedaran 4 puntos libres (de los 4n iniciales) que unimos paraformar la ultima piramide. Por el metodo de construccion es claro que las piramidesno se cortan, pues los planos las separan en regiones disjuntas. El unico problemapodrıa ser que dos piramides de regiones contiguas se cortasen en uno de esos planosque la separan, pero eso es imposible pues cada piramide comparte a lo sumo uno desus vertices con el plano (y evidentemente, por construccion, no hay dos piramides quecompartan vertice).

Problema 11. Hallar los valores enteros positivos de m para los que existe una funcionf del conjunto de los numeros enteros en sı mismo tal que f (m)(n) = n+ 2017, dondef (m) consiste en aplicar la funcion f m veces.

Solucion. La primera observacion es que f debe ser biyectiva.

Lema 1. Si f (a)(n) = f (b)(n), entonces tenemos a = b.

Proof. Demostracion: En otro caso, sin perdida de generalidad asumamos a > b. Te-nemos f (a−b)(n) = n, lo que implica que f (m(a−b))(n) = n, que es imposible, pues serıatanto como decir n = n+ 2017(a− b) .

Para cada n fijo consideremos el conjunto An = {f (s)(n) | s es entero }. La primeraobservacion es que si hay un cierto entero en dicho conjunto, entonces todos los enteroscongruentes modulo 2017 con el estan tambien en el conjunto, pues x = f (t)(n) implicax + 2017k = f(t+ km)(n) para cualquier valor entero de k. Ademas, en el conjuntoestan todos los representantes de exactamente m clases distintas de resto modulo 2017.Es imposible que haya mas, pues independientemente de t sucede que f (t)(n) y f (t+m)(n)tienen igual resto modulo 2017, dejando a lo sumo m posibilidades. Asimismo no puedehaber menos, pues por el principio del palomar ello implicarıa que existirıan c,d enterosentre 0 y m− 1 con f (c)(n)− f (d)(n) = 2017k implicando f (c)(n)− f (d+km)(n) = 0, locual contradice al Lema. Finalmente, basta notar que, por construccion, dos conjuntos

Page 312: COMPENDIUM OMEFL - ToomatesXXXV Olimpiada Matemática Española Primera Fase Soluciones de la propuesta de problemas Problema 1 ¿Qué dígitos se han omitido en la siguiente multiplicación?

Ai y Aj son necesariamente iguales o disjuntos. Como entre A0, . . . , A2016 se englobantodos los restos y cada uno aporta m, se deduce que m debe dividir a 2017. Entoncessolo puede ser m = 1 o m = 2017 y, para acabar, basta poner un ejemplo para cadacaso. Claramente, f(n) = n + 1 (caso m = 2017) y f(n) = n + 2017 (caso m = 1)cumplen la propiedad requerida.

Problema 12. Determinar todos los numeros naturales n para los que existe algunnumero natural m con las siguientes propiedades

• m tiene al menos dos cifras (en base 10), todas son distintas y ninguna es 0.

• m es multiplo de n y cualquier reordenacion de sus cifras da lugar a un multiplode n.

Solucion. Notemos que n = 1, 3, 9 funcionan porque en estos casos el criterio dedivisibilidad es que la suma de las cifras sea multiplo de n, y tomando m = 18 ya esta.Para cualquier otro caso, consideramos que m existe y que tiene como cifra mas a laderecha x y luego una y.Entonces, permutando esas dos cifras, nos quedara que como se conserva la congruenciamodulo n, 9(x − y) tiene que ser multiplo de n. Eso quiere decir que x es congruentecon y modulo n/mcd(9, n). Si n no es multiplo de 9, x es congruente con y y ha deser n ≤ 8. Para n = 2, 4 tomamos m = 48. Si n = 8, todas sus cifras tienen que serpares pero eso es incompatible con x− y multiplo de 8 y 1 ≤ x, y ≤ 9.Supongamos ahora que n es multiplo de 3 pero no de 9. Si n = 6, todas las cifras sontambien pares y eso deja como unica opcion {x, y} = {2, 8}. Pero 28 no es multiplo de6 con lo que tendrıa que haber mas cifras, que solo se pueden tomar entre 4 y 6; peroninguno de estos numeros es multiplo de 6: 2 + 8 + 4, 2 + 8 + 4, 2 + 8 + 6, 2 + 8 + 4 + 6.Por tanto n tendrıa que ser impar y como x es congruente con y modulo n/3, tendrıaque pasar que n ≤ 24, y las unicas opciones que no hemos visto todavıa son n = 15 yn = 21. n = 15 no puede ser pues todas las cifras serıan multiplo de 5. Si n = 21,{x, y} = {1, 8} o {x, y} = {2, 9}, con lo que el numero tiene una tercera cifra z yrepitiendo el analisis sucede que que como 90 no es multiplo de 7, y − z es multiplo de7, con lo que x serıa igual a z.Solo nos queda el caso multiplo de 9, y en ese caso x, y son congruentes modulo n/9;entonces sucede que n ≤ 81 y n es impar por lo dicho antes; tampoco puede ser multiplode 5 ni de 7; ası pues, nos quedan los casos n = 27, 81. Si descartamos n = 27 habremosacabado. x, y son congruentes modulo 3; en general, todas las cifras son congruentesmodulo 3 ası que solo podemos tener 3 cifras; para ser multiplo de 9 tenemos que lastres cifras son multiplo de 3 ası que m tendrıa que ser alguna reordenacion de 369,pero simplemente observamos que 369 no es multiplo de 27. Por tanto n puede ser1, 2, 3, 4, 9.

Page 313: COMPENDIUM OMEFL - ToomatesXXXV Olimpiada Matemática Española Primera Fase Soluciones de la propuesta de problemas Problema 1 ¿Qué dígitos se han omitido en la siguiente multiplicación?

LIV Olimpiada Matematica Espanola

Primera Fase

Primera sesion

Viernes manana, 19 de enero de 2018

OlimpiadaMatemáticaEspañola RSME

1. Sean a ≥ 1, b ≥ 1 numeros naturales cuyo maximo comun divisor y mınimocomun multiplo designamos por D y M , respectivamente.

Demostrar queD2 +M2 ≥ a2 + b2.

2. ¿De cuantas maneras se puede escribir 111 como suma de tres numeros en-teros en progresion geometrica?

3. Encontrar las funciones reales f , de variable real, que satisfacen la ecuacionfuncional

f (x+ f (x+ y)) = f (2x) + y

cualesquiera sean x, y reales.

No esta permitido el uso de calculadoras.Cada problema se puntua sobre 7 puntos.

El tiempo de cada sesion es de 3 horas y media.

Page 314: COMPENDIUM OMEFL - ToomatesXXXV Olimpiada Matemática Española Primera Fase Soluciones de la propuesta de problemas Problema 1 ¿Qué dígitos se han omitido en la siguiente multiplicación?

LIV Olimpiada Matematica Espanola

Primera Fase

Segunda sesion

Viernes tarde, 19 de enero de 2018

OlimpiadaMatemáticaEspañola RSME

4. Determinar los numeros reales x > 1 para los cuales existe un triangulo cuyoslados tienen longitudes

x4 + x3 + 2x2 + x+ 1, 2x3 + x2 + 2x+ 1, x4 − 1

5. Sea n un numero natural. Probar que si la ultima cifra de 7n es 3, lapenultima es 4.

6. Sea AD la mediana de un triangulo ABC tal que � ADB = 45◦ y � ACB =30◦. Determinar el valor de � BAD.

No esta permitido el uso de calculadoras.Cada problema se puntua sobre 7 puntos.

El tiempo de cada sesion es de 3 horas y media.

Page 315: COMPENDIUM OMEFL - ToomatesXXXV Olimpiada Matemática Española Primera Fase Soluciones de la propuesta de problemas Problema 1 ¿Qué dígitos se han omitido en la siguiente multiplicación?

LIV Olimpiada Matematica Espanola

Primera Fase

Primera sesion

Viernes tarde, 19 de enero de 2018

OlimpiadaMatemáticaEspañola RSME

1. Determinar los numeros reales x > 1 para los cuales existe un triangulo cuyoslados tienen longitudes

x4 + x3 + 2x2 + x+ 1, 2x3 + x2 + 2x+ 1, x4 − 1

2. Sea n un numero natural. Probar que si la ultima cifra de 7n es 3, lapenultima es 4.

3. Sea AD la mediana de un triangulo ABC tal que � ADB = 45◦ y � ACB =30◦. Determinar el valor de � BAD.

No esta permitido el uso de calculadoras.Cada problema se puntua sobre 7 puntos.

El tiempo de cada sesion es de 3 horas y media.

Page 316: COMPENDIUM OMEFL - ToomatesXXXV Olimpiada Matemática Española Primera Fase Soluciones de la propuesta de problemas Problema 1 ¿Qué dígitos se han omitido en la siguiente multiplicación?

LIV Olimpiada Matematica Espanola

Primera Fase

Segunda sesion

Sabado manana, 20 de enero de 2018

OlimpiadaMatemáticaEspañola RSME

4. Probar que:

1. La suma de las distancias desde un punto de la superficie de la esfera inscritaen un cubo de R

3 a todas las caras del mismo no depende del punto elegido.

2. Misma cuestion anterior para la suma de los cuadrados de las distancias.

3. Misma cuestion que las anteriores para la suma de los cubos de las distancias.

5. Sean a, b, c numeros naturales primos, distintos dos a dos. Demostrar queel numero

(ab)c−1

+ (bc)a−1

+ (ca)b−1 − 1

es un multiplo del producto abc.

6. Se han coloreado 46 cuadrados unitarios de una cuadrıcula 9 × 9. ¿Hay, enla cuadrıcula, alguna figura del tipo

(no necesariamente con la orientacion que muestra el dibujo) con las trescasillas coloreadas?

No esta permitido el uso de calculadoras.Cada problema se puntua sobre 7 puntos.

El tiempo de cada sesion es de 3 horas y media.

Page 317: COMPENDIUM OMEFL - ToomatesXXXV Olimpiada Matemática Española Primera Fase Soluciones de la propuesta de problemas Problema 1 ¿Qué dígitos se han omitido en la siguiente multiplicación?

Soluciones

Viernes mananaProblema 1. Sean a ≥ 1, b ≥ 1 numeros naturales cuyo maximo comun divisor y mınimo

comun multiplo designamos por D y M , respectivamente.

Solucion.Si a′ y b′ son los respectivos cocientes obtenidos al dividir a y b entre D, se tiene:

a = Da′, b = Db′, a′ ≥ 1, b′ ≥ 1, m.c.d.(a′, b′

)= 1, M =

ab

D= Da′b′.

Por consiguiente, la desigualdad propuesta en el enunciado se puede escribir como(Da′b′

)2+D2 ≥

(Da′

)2+(Db′)2,

o, equivalentemente,a′

2b′2

+ 1 ≥ a′2 + b′2,

equivalente, a su vez, a la desigualdad(a′

2 − 1)(

b′2 − 1

)≥ 0,

que es obviamente cierta habida cuenta de que a′ ≥ 1 y b′ ≥ 1 son numeros naturales, con loque concluimos.

Se verifica la igualdad solo cuando a|b o b|a.

Problema 2. ¿De cuantas maneras se puede escribir 111 como suma de tres numerosenteros en progresion geometrica?

Solucion.Sean a, ar, ar2 numeros enteros que satisfacen la ecuacion

a+ ar + ar2 = 111, (1)

siendo r un numero racional.Habida cuenta de que 1 + r+ r2 =

(r + 1

2

)2+ 3

4 > 0, existen numeros naturales p y q, primosentre sı, tales que

1 + r + r2 =p

q(2)

y (1) puede escribirse en la forma a = 111qp , de donde se sigue que p ha de ser un divisor de 111;

a saber, p ∈ {1, 3, 37, 111}.Escribimos ahora (2) como una cuadratica en r,

qr2 + qr + (q − p) = 0, (3)

Page 318: COMPENDIUM OMEFL - ToomatesXXXV Olimpiada Matemática Española Primera Fase Soluciones de la propuesta de problemas Problema 1 ¿Qué dígitos se han omitido en la siguiente multiplicación?

e imponemos que su discriminante, q (4p− 3q), sea igual o mayor que cero. Al ser positivo elfactor de la izquierda (pues q es un numero natural), debera cumplirse que

4p− 3q ≥ 0. (4)

Tenemos, pues:Caso p = 1. El unico valor admisible para q es 1. Al sustituir p y q por 1 en (3), esta

ecuacion se escribe r (r + 1) = 0, obteniendose r = −1 y r = 0.Caso p = 3. Al ser q ≤ 4 (por (4)) y m.c.d.(p, q) = 1 (porque la fraccion p

q es irreducible porhipotesis), los valores admisibles para q son q = 1, q = 2 y q = 4.

Si q = 1, la ecuacion (3) se escribe r2 + r − 2 = 0 y obtenemos r = −2 y r = 1.Si q = 2, la correspondiente cuadratica es 2r2 + 2r − 1 = 0 que no tiene raıces racionales.Si p = 4 se tiene 4r2 + 4r + 1 = 0 y r = −1

2 .Los casos p = 37 y p = 111 pueden discutirse con un razonamiento analogo al anterior.La tabla siguiente muestra las ternas (p, q, r) validas:

p q r

1 1 -1, 0

3 1 -2, 1

3 4 −1/2

37 9 4/3, −7/3

37 16 3/4, −7/4

37 49 −4/7, −3/7

111 1 -11, 10

111 100 −11/10, 1/10

111 121 −10/11, −1/11

correspondiendo a las cuales se obtienen las diecisiete soluciones que resuelven el problema:

r = 0 111 + 0 + 0

r = 1 37 + 37 + 37

r = −1 111− 111 + 111

r = −2 37− 74 + 148

r = −1/2 148− 74 + 37

r = −3/4 48 + 36 + 27

r = 4/3 27 + 36 + 48

r = −7/3 27− 63 + 147

r = −3/7 147− 63 + 27

r = −7/4 48− 84 + 147

r = −4/7 147− 84 + 48

r = 1/10 100 + 10 + 1

r = 10 1 + 10 + 100

r = −11 1− 11 + 121

r = −1/11 121− 11 + 1

r = −10/11 121− 110 + 100

r = −11/10 100− 110 + 121

Problema 3. Encontrar las funciones reales f , de variable real, que satisfacen la ecuacionfuncional

f (x+ f (x+ y)) = f (2x) + y ,

Page 319: COMPENDIUM OMEFL - ToomatesXXXV Olimpiada Matemática Española Primera Fase Soluciones de la propuesta de problemas Problema 1 ¿Qué dígitos se han omitido en la siguiente multiplicación?

cualesquiera sean x, y reales.

Solucion.Haciendo las sustituciones x = f (0) y y = −f (0) obtenemos f (f (0) + f (0)) = f (2 · f (0))+

f (0), de donde f (0) = 0. Sustituyendo ahora x = 0 en la ecuacion dada, dejando la variable yarbitraria, se tiene f (0 + f (y)) = f (0) + y, esto es,

f (f (y)) = y. (5)

Al sustituir y = 0 en la ecuacion del enunciado sera f (x+ f (x)) = f (2x), de donde se sigueque f (f (x+ f (x))) = f (f (2x)) y, por (1), x+ f (x) = 2x, es decir,

f (x) = x

Es inmediato comprobar que esta funcion es solucion cualesquiera sean los valores de x, y,con lo que concluimos.

Viernes tardeProblema 4 o 1 Determinar los numeros reales x > 1 para los cuales existe un triangulo

cuyos lados tienen longitudes

x4 + x3 + 2x2 + x+ 1, 2x3 + x2 + 2x+ 1, x4 − 1.

Solucion.Justificaremos que para todo numero real x > 1 existe un tal triangulo probando que el lado

mayor es menor que la suma de los otros dos. En efecto, para cualquier real x > 1, tenemos:

(i) x4 + x3 + 2x2 + x+ 1 > 0, 2x3 + x2 + 2x+ 1 > 0, x4 − 1 > 0.

(ii) x4 + x3 + 2x2 + x+ 1 > 2x3 + x2 + 2x+ 1, ya que

x4 + x3 + 2x2 + x+ 1−(2x3 + x2 + 2x+ 1

)= x4 − x3 + x2 − x = x (x− 1)

(x2 + 1

)> 0.

(iii) x4 + x3 + 2x2 + x+ 1 > x4 − 1.

(iv) x4 + x3 + 2x2 + x+ 1 <(2x3 + x2 + 2x+ 1

)+(x4 − 1

), puesto que

x4 + x3 + 2x2 + x+ 1−(2x3 + x2 + 2x+ 1

)−(x4 − 1

)= −x3 + x2 − x+ 1= − (x− 1)

(x2 + 1

)< 0

y concluimos.

Problema 5 o 2 Sea n un numero natural. Probar que si la ultima cifra de 7n es 3, lapenultima es 4. Solucion.

Los restos potenciales de 7 respecto del modulo 10 coinciden con la ultima cifra, o cifra delas unidades, de las potencias de 7 y siendo

70 ≡ 1 (mod. 10)71 ≡ 7 (mod. 10)72 ≡ 9 (mod. 10)73 ≡ 3 (mod. 10)74 ≡ 1 (mod. 10) ,

Page 320: COMPENDIUM OMEFL - ToomatesXXXV Olimpiada Matemática Española Primera Fase Soluciones de la propuesta de problemas Problema 1 ¿Qué dígitos se han omitido en la siguiente multiplicación?

resulta que 7n acaba en 3 solo cuando n = 4q + 3, q un numero natural: en efecto, al ser n dela forma n = 4q + r, q y r naturales, r ∈ {0, 1, 2, 3}, tenemos

7n = 74q+r =(74)q · 7r

≡ 1 · 7r (mod. 10)≡ 3 (mod. 10) solo si r = 3 pues 0 ≤ r ≤ 3.

Por consiguiente, si n acaba en 3, sera n = 4q + 3 y

7n = 74q+3 =(74)q · 73

= 2401q · 343= (2400 + 1)q · 343

=

(•

100 + 1

)q

· 343

=

(•

100 + 1

)· 343 (†)

=•

100 + 343

=•

100 + (300 + 43)

=•

100 + 43 (‡)= a1 a2 a3 . . . an00 + 43= a1 a2 a3 . . . an43,

con lo que la cifra de las decenas es 4, como se querıa.

(†)(•

100 + 1

)q

=∑q

j=0

(qj

)100q−j · 1j =

q−1∑j=0

(q

j

)100q−j︸ ︷︷ ︸

=•

100

+

(q

q

)1000︸ ︷︷ ︸

=1

=•

100 + 1.

(‡)•

100 + (300 + 43) =

(•

100 + 300

)︸ ︷︷ ︸

=•

100

+43 =•

100 + 43.

Problema 6 o 3. Sea AD la mediana de un triangulo ABC tal que ∠ADB = 45◦ y∠ACB = 30◦. Determinar el valor de ∠BAD.

Solucion 1.En 4ABC, sea E el pie de la perpendicular trazada desde B. Entonces el triangulo EBC

es rectangulo en E, el punto D es - por hipotesis - el punto medio de su hipotenusa BC y, portanto, el circuncentro de dicho triangulo en cuya circunferencia circunscrita el angulo BDE esel central correspondiente al inscrito ∠BCE. Por consiguiente,

∠BDE = 2 · ∠BCE= 2 · ∠BCA= 2× 30◦

= 60◦.

Se sigue que 4EBD es equilatero (pues es isosceles y tiene un angulo de 60◦) y, a su vez,

∠ADE = ∠BDE − ∠ADB= 60◦ − 45◦

= 15◦

= ∠DAE (por el teorema del angulo exterior aplicado a 4ADC en D).

Page 321: COMPENDIUM OMEFL - ToomatesXXXV Olimpiada Matemática Española Primera Fase Soluciones de la propuesta de problemas Problema 1 ¿Qué dígitos se han omitido en la siguiente multiplicación?

D CB

A

E

30◦45◦

Ası, 4EAD es isosceles con AE = ED. Mas ED = EB, pues 4EBD es equilatero, segunhemos visto.

Por tanto, AE = EB y 4AEB es rectangulo (en E) e isosceles. En consecuencia, ∠BAE =45◦ y

∠BAD = ∠BAE − ∠DAE = 45◦ − 15◦ = 30◦.

Solucion 2.Por el teorema del angulo exterior aplicado a ADC en D,

∠CAD = ∠BDA− ∠DCA = 45◦ − 30◦ = 15◦

= 12∠DCA

y∠DCA = ∠2 · ∠CAD.

La igualdad anterior se puede escribir, de una manera equivalente1 como

AD2 = DC (DC + CA) ,

esto es,

m2a =

a

2

(a2

+ b)

y, por el teorema de la mediana aplicado a ABC, se obtiene

1

4

(2b2 + 2c2 − a2

)=a

2

(a2

+ b)

o, equivalentemente,b2 + c2 − a2 = ab (6)

Consideremos en lo que sigue el triangulo ABC y sus elementos.Sustituimos el primer miembro de (1) por su igual, 2bc cosA (por el teorema del coseno).

Simplificamos y obtenemos2c cosA = a,

1Si x, y, z son las longitudes de los lados de un triangulo XY Z respectivamente opuestos a X, Y , Z, entonces

∠X = 2 · ∠Y ⇔ x2 = y (y + z) .

Page 322: COMPENDIUM OMEFL - ToomatesXXXV Olimpiada Matemática Española Primera Fase Soluciones de la propuesta de problemas Problema 1 ¿Qué dígitos se han omitido en la siguiente multiplicación?

equivalente a2 sinC cosA = sinA.

Dividiendo por cosA y habida cuenta de que ∠C = 30◦, resulta

tanA = 1,

y∠A = 45◦.

Finalmente,∠BAD = ∠BAC − ∠DAC = 45◦ − 15◦ = 30◦.

Sabado mananaProblema 4 Probar que:

1. La suma de las distancias desde un punto de la superficie de la esfera inscrita en un cubode R3 a todas las caras del mismo no depende del punto elegido.

2. Misma cuestion anterior para la suma de los cuadrados de las distancias.

3. Misma cuestion que las anteriores para la suma de los cubos de las distancias.

Solucion.Designaremos por P el punto elegido sobre la esfera inscrita en el cubo y por r el radio de

la misma.

1. Habida cuenta de que la arista del cubo es igual a 2r, su volumen es 8r3, y, al ser P interioral cubo, tambien es igual a la suma de los volumenes de las seis piramides que tienen porbases las caras del hexaedro, vertice comun en P y cuyas alturas son, precisamente, lasdistancias de P a las caras del cubo, esto es, 1

3 (2r)2 σ, por la sabida formula del volumende una piramide y donde σ designa la suma de las distancias de P a todas las caras delcubo.

Ası, pues,

8r3 =1

3(2r)2 σ

yσ = 6r,

que no depende del punto P .

2. Sean x, y, z las respectivas distancias de P a tres de las caras del cubo que tengan unvertice, llamemosle A, comun. Por el teorema de Pitagoras (aplicado dos veces),

x2 + y2 + z2 = PA2. (7)

De manera analoga, al ser 2r − x, 2r − y, 2r − z las tres distancias restantes, si B es elvertice del cubo diagonalmente opuesto al A,

(2r − x)2 + (2r − y)2 + (2r − z)2 = PB2. (8)

De (1) y (2) se sigue que la suma de los cuadrados de las distancias considerada es iguala PA2 + PB2.

Page 323: COMPENDIUM OMEFL - ToomatesXXXV Olimpiada Matemática Española Primera Fase Soluciones de la propuesta de problemas Problema 1 ¿Qué dígitos se han omitido en la siguiente multiplicación?

Por el teorema de la mediana, aplicado al triangulo PAB,

PA2 + PB2 = 2 · PO2 +AB2

2(9)

en donde O designa el punto medio de la diagonal AB.

2r

2r

r

2r

A

O

P

Notese que O es, a su vez, el centro de la esfera inscrita en el cubo (vease la figura); porconsiguiente,

PO = r

y, por (3), teniendo presente que la longitud de la diagonal de un cubo es igual a√

3 vecesla de la arista,

PA2 + PB2 = 2r2 +12r2

2= 8r2,

que no depende del punto P .

3. Sean u, v las respectivas distancias de P a dos caras paralelas del cubo. Entonces,

u3 + v3 = (u+ v)3 − 3uv (u+ v) = (2r)3 − 3uv · (2r) = 8r3 − 6ruv. (10)

y2uv = (u+ v)2 −

(u2 + v2

)= (2r)2 −

(u2 + v2

)= 4r2 −

(u2 + v2

). (11)

Sustituimos en (4) el valor de uv deducido de (5) y obtenemos

u3 + v3 = 8r3 − 3r(4r2 −

(u2 + v2

))= 3r

(u2 + v2

)− 4r3. (12)

Con (6) y sendas expresiones analogas para los dos pares de caras paralelas restantes seobtiene:

sumade los cubos

de las distancias= 3r · (suma de los cuadrados de las distancias) − 3 · 4r3

= 3r · 8r2 − 12r3

= 12r3,

que no depende del punto P elegido, con lo que concluimos.

Page 324: COMPENDIUM OMEFL - ToomatesXXXV Olimpiada Matemática Española Primera Fase Soluciones de la propuesta de problemas Problema 1 ¿Qué dígitos se han omitido en la siguiente multiplicación?

Problema 5. Sean a, b, c numeros naturales primos, distintos dos a dos. Demostrar queel numero

(ab)c−1 + (bc)a−1 + (ca)b−1 − 1

es un multiplo del producto abc.

Solucion.Al ser a, b, c primos, el producto ab no es divisible por c (si c|ab, serıa c|a o c|b; habida cuenta

de que c > 1, a 6= c, b 6= c, tendrıamos una contradiccion con el hecho de ser a y b primos).Entonces, por la congruencia de Fermat (†),

(ab)c−1 ≡ 1 (mod. c)

y

(ab)c−1 − 1 =•c.

Analogamente,

(bc)a−1 − 1 =•a, (ca)b−1 − 1 =

•b.

Multiplicando miembro a miembro las igualdades anteriores, se obtiene

ab+c−2bc+a−2ca+b−2 − ab+c−2bc−1cb−1 − ac−1bc+a−2ca−1 − ab−1ba−1ca+b−2

+ (ab)c−1 + (bc)a−1 + (ca)− 1

=•abc

y

(ab)c−1 + (bc)a−1 + (ca)− 1 =•abc− ab+c−2bc+a−2ca+b−2 + ab+c−2bc−1cb−1

+ac−1bc+a−2ca−1 + ab−1ba−1ca+b−2,(13)

cuyo segundo miembro es un multiplo de abc por ser suma de multiplos de abc: en efecto, todoslos sumandos en el segundo miembro de (3) contienen el factor abc pues los exponentes quefiguran son iguales o mayores que 1 al ser, por hipotesis, a > 1, b > 1, c > 1. Por consiguiente,tambien el primer miembro de (3) es multiplo de abc:

(ab)c−1 + (bc)a−1 + (ca)b−1 − 1 =•abc,

como se querıa.

(†) Lema. Sean p y h numeros naturales, p primo, 1 ≤ h ≤ p − 1. Entonces(ph

)es un

multiplo de p.En efecto, por la teorıa combinatoria,

(ph

)= p!

h!(p−h)! es un numero natural . El exponentede p en la descomposicion de p! en producto de factores primos es 1: supongamos, para llegara una contradiccion, que p! = p2m, m un numero natural. Dividiendo por p, (p− 1)! = pm y pserıa un divisor de (p− 1)!; por consiguiente, al ser p primo, p dividirıa a alguno de los factoresdel producto (p− 1) × (p− 2) × · · · × 2 × 1, lo que constituye la contradiccion buscada ya quecada uno de dichos factores es menor que p.

Por un razonamiento analogo, el exponente de p en la descomposicion en factores primosde h! y de (p− h)! es 0. Ası, pues, p figura con exponente 1 en la descomposicion en factoresprimos del numero natural p!

h!(p−h)! y(ph

)es un multiplo de p.

Corolario 1. Si a es un numero entero y p un numero natural primo, entonces

(a+ 1)p ≡ ap + 1 (mod. p) (14)

Page 325: COMPENDIUM OMEFL - ToomatesXXXV Olimpiada Matemática Española Primera Fase Soluciones de la propuesta de problemas Problema 1 ¿Qué dígitos se han omitido en la siguiente multiplicación?

Solo hay que observar que la suma en el segundo miembro de la identidad

(a+ 1)p − (ap + 1) =

(p

1

)ap−1 +

(p

2

)ap−2 + · · ·+

(p

p− 1

)a

es un multiplo de p por serlo cada uno de los sumandos(ph

)ap−h.

Corolario 2. Si a es un numero entero y p un numero natural primo, entonces

ap ≡ a (mod. p)

Es suficiente una demostracion para a un numero natural. El corolario es trivialmente ciertopara a = 0 y para a = 1. Supongamos

ap ≡ a (mod. p), a ∈ N,

y probemos que(a+ 1)p ≡ a+ 1 (mod. p) (15)

A tal fin, restamos a+ 1 a los dos miembros de (1) y obtenemos la congruencia siguiente:

(a+ 1)p − (a+ 1) ≡ ap − a (mod. p)

cuyo segundo miembro es multiplo de p por hipotesis. En consecuencia, tambien el primermiembro sera un multiplo de p:

(a+ 1)p − (a+ 1) =•p

o, equivalentemente,(a+ 1)p ≡ a+ 1 (mod. p),

como se querıa.Proposicion (Congruencia de Fermat). Si p es un numero natural primo y a es un numero

entero no divisible por p,ap−1 ≡ 1 (mod. p).

Demostracion. Tenemos p|a(ap−1 − 1

)(por el corolario 2) y m.c.d.(a, p) = 1 (pues p - a, por

hipotesis). El teorema fundamental de la aritmetica da inmediatamente

p|(ap−1 − 1

),

equivalente aap−1 ≡ 1 (mod. p),

con lo que concluimos.

Problema 6. Se han coloreado 46 cuadrados unitarios de una cuadrıcula 9×9. ¿Hay, enla cuadrıcula, alguna figura del tipo

(no necesariamente con la orientacion que muestra el dibujo) con las tres casillas coloreadas?

Solucion.La respuesta es afirmativa. En efecto, probaremos que si coloreamos como indica el enunci-

ado, se puede encontrar siempre una figura del tipo dado con las tres casillas coloreadas.A tal fin, dividamos la cuadrıcula 9×9 en dieciseis cuadrados 2×2, tres figuras como la

siguiente:

Page 326: COMPENDIUM OMEFL - ToomatesXXXV Olimpiada Matemática Española Primera Fase Soluciones de la propuesta de problemas Problema 1 ¿Qué dígitos se han omitido en la siguiente multiplicación?

(16)

y una mas como la que sigue:

(17)

tal como se muestra en el esquema siguiente:

Si no hubiera ninguna figura del tipo

con las tres casillas coloreadas, entonces cada uno de los dieciseis cuadrados tendrıa, a lo mas,2 casillas coloreadas; cada una de las tres figuras (1) tendrıa, a lo mas, 3 casillas coloreadas y lafigura (2), 4 casillas coloreadas, como maximo.

Ası, pues, se habrıan coloreado, como maximo, 2×16+3×3+4 = 45 casillas de la cuadrıcula,lo que es contrario a la hipotesis.

Page 327: COMPENDIUM OMEFL - ToomatesXXXV Olimpiada Matemática Española Primera Fase Soluciones de la propuesta de problemas Problema 1 ¿Qué dígitos se han omitido en la siguiente multiplicación?

LV Olimpiada Matematica Espanola

Primera Fase

Primera sesion

Viernes manana, 18 de enero de 2019

OlimpiadaMatemáticaEspañola RSME

1. Para cada numero de cuatro cifras abcd, denotamos por S al numero S =abcd − dcba. Demuestra que S es multiplo de 37 si y solo si las dos cifrascentrales del numero abcd son iguales.

2. Demuestra que para todo n ≥ 2 podemos encontrar n numeros reales

x1, x2, . . . , xn �= 1

de manera que los productos

x1 · x2 · . . . · xn y1

1− x1· 1

1− x2· . . . · 1

1− xn

son iguales.

3. El trapecio isosceles ABCD tiene lados paralelos AB y CD. Sabemos queAB = 6, AD = 5 y � DAB = 60◦. Se lanza un rayo de luz desde A querebota en CB en el punto E e interseca en AD en el punto F . Si AF = 3,calcula el area del triangulo AFE.

No esta permitido el uso de calculadoras.Cada problema se puntua sobre 7 puntos.

El tiempo de cada sesion es de 3 horas y media.

Page 328: COMPENDIUM OMEFL - ToomatesXXXV Olimpiada Matemática Española Primera Fase Soluciones de la propuesta de problemas Problema 1 ¿Qué dígitos se han omitido en la siguiente multiplicación?

LV Olimpiada Matematica Espanola

Primera Fase

Segunda sesion

Viernes tarde, 18 de enero de 2019

OlimpiadaMatemáticaEspañola RSME

4. Sea p ≥ 3 un numero primo y consideramos el triangulo rectangulo de catetomayor p2 − 1 y cateto menor 2p. Inscribimos en el triangulo un semicırculocuyo diametro se apoya en el cateto mayor del triangulo y que es tangentea la hipotenusa y al cateto menor del triangulo. Encuentra los valores de ppara los cuales el radio del semicırculo es un numero entero.

5. ¿Existen m,n numeros naturales de forma que

n2 + 2018mn+ 2019m+ n− 2019m2

es un numero primo?

6. Fijamos un numero natural k ≥ 1. Encuentra todos los polinomios P (x) quecumplan

P (xk)− P (kx) = xkP (x)

para todo valor de x ∈ R.

No esta permitido el uso de calculadoras.Cada problema se puntua sobre 7 puntos.

El tiempo de cada sesion es de 3 horas y media.

Page 329: COMPENDIUM OMEFL - ToomatesXXXV Olimpiada Matemática Española Primera Fase Soluciones de la propuesta de problemas Problema 1 ¿Qué dígitos se han omitido en la siguiente multiplicación?

LV Olimpiada Matematica Espanola

Primera Fase

Primera sesion

Viernes tarde, 18 de enero de 2019

OlimpiadaMatemáticaEspañola RSME

1. Sea p ≥ 3 un numero primo y consideramos el triangulo rectangulo de catetomayor p2 − 1 y cateto menor 2p. Inscribimos en el triangulo un semicırculocuyo diametro se apoya en el cateto mayor del triangulo y que es tangentea la hipotenusa y al cateto menor del triangulo. Encuentra los valores de ppara los cuales el radio del semicırculo es un numero entero.

2. ¿Existen m,n numeros naturales de forma que

n2 + 2018mn+ 2019m+ n− 2019m2

es un numero primo?

3. Fijamos un numero natural k ≥ 1. Encuentra todos los polinomios P (x) quecumplan

P (xk)− P (kx) = xkP (x)

para todo valor de x ∈ R.

No esta permitido el uso de calculadoras.Cada problema se puntua sobre 7 puntos.

El tiempo de cada sesion es de 3 horas y media.

Page 330: COMPENDIUM OMEFL - ToomatesXXXV Olimpiada Matemática Española Primera Fase Soluciones de la propuesta de problemas Problema 1 ¿Qué dígitos se han omitido en la siguiente multiplicación?

LV Olimpiada Matematica Espanola

Primera Fase

Segunda sesion

Sabado manana, 19 de enero de 2019

OlimpiadaMatemáticaEspañola RSME

4. Considera el conjunto de numeros enteros positivos n cumpliendo que 1 ≤n ≤ 1000000. En ese conjunto, indica si es mayor la cantidad de numerosque pueden expresarse de la forma a3+mb2, con a, b ∈ N y m ∈ {0, 2, 4, 6, 8}o la cantidad de numeros que no pueden expresarse de esa manera.

5. Prueba que para todo a, b, c > 0 se cumple que

a2

b3c− a

b2≥ c

b− c2

a

¿En que caso se cumple la igualdad?

6. Consideramos un triangulo ABC y un punto D en el lado AC. Si AB =DC = 1, � DBC = 30◦ y � ABD = 90◦, calcula el valor de AD.

No esta permitido el uso de calculadoras.Cada problema se puntua sobre 7 puntos.

El tiempo de cada sesion es de 3 horas y media.

Page 331: COMPENDIUM OMEFL - ToomatesXXXV Olimpiada Matemática Española Primera Fase Soluciones de la propuesta de problemas Problema 1 ¿Qué dígitos se han omitido en la siguiente multiplicación?

Soluciones fase local OME curso 2018/19Alejandro Miralles Montolıo

PRIMER DIA

1. Para cada n´umero de cuatro cifras abcd, denotamos por S al n´umero S = abcd − dcba. Demuestra que S es m´ultiplo de 37 si y s´olo si las dos cifras centrales del n´umero abcd son iguales.

Solucion. Escribimos el numero como abcd como 1000a+100b+10c+dy el numero dcba como 1000d+ 100c+ 10b+ a. Por tanto,

S = abcd− dcba = 1000a+ 100b+ 10c+ d− (1000d+ 100c+ 10b+ a) =

999(a− d) + 90(b− c) = 37 · 33(a− d) + 2 · 5 · 32(b− c).

El primer sumando es obviamente multiplo de 37. El segundo sumandono tiene el factor 37 ya que este es un numero primo y |b− c| ≤ 9. Portanto, S sera multiplo de 37 si y solo si b− c = 0, es decir, si y solo sib = c.

2. Demuestra que para todo n ≥ 2 podemos encontrar n numeros realesx1, x2, · · · , xn 6= 1 de manera que los productos

x1 · x2 · . . . · xn y1

1− x1· 1

1− x2· . . . · 1

1− xn

son iguales.

Solucion. Dado x 6= 1, notemos que la ecuacion

x =1

1− x⇐⇒ x2 − x+ 1 = 0

no tiene soluciones reales. Sin embargo, dados x, y 6= 1,

x · y =1

1− x· 1

1− y

1

Page 332: COMPENDIUM OMEFL - ToomatesXXXV Olimpiada Matemática Española Primera Fase Soluciones de la propuesta de problemas Problema 1 ¿Qué dígitos se han omitido en la siguiente multiplicación?

tiene una solucion sencilla ya que

x · y =1

1− x· 1

1− y=

1

x− 1· 1

y − 1

y las ecuaciones

x =1

x− 1e y =

1

y − 1⇐⇒ x2 − x− 1 = 0 e y2 − y − 1 = 0

sı tienen solucion

x = y =1±√

5

2.

Por tanto, si n es un numero par, podemos agrupar de dos en doscada termino de cada producto y utilizar lo anterior, encontrando lassoluciones

x1 = x2 = · · · = xn =1±√

5

2.

Si n = 3, consideramos, por ejemplo, x1 = x2 = 2. La igualdad delenunciado nos lleva a la ecuacion

4x3 =

(1

1− 2

)21

1− x3=

1

1− x3⇐⇒ 4x23 − 4x3 + 1 = 0

que da la solucion x3 = 12. Ası, obtenemos (x1, x2, x3) = (2, 2, 1

2). Si n

es cualquier impar mayor que 3, basta con completar estos tres valorescon un numero par de valores de los xk utilizando el caso par anterior.

3. El trapecio isosceles ABCD tiene lados paralelos AB y CD. Sabemos

que AB = 6, AD = 5 y ∠DAB = 60o. Se lanza un rayo de luz desde A que rebota en CB en el punto E e interseca en AD en el punto F . Si AF = 3, calcula el ´area del tri´angulo AFE.

Solucion. Puesto que el trapecio es isosceles y ∠DAB = 60o, podemosalargar los lados AD y BC que intersectan en G, formando ası untriangulo equilatero ABG.

2

Page 333: COMPENDIUM OMEFL - ToomatesXXXV Olimpiada Matemática Española Primera Fase Soluciones de la propuesta de problemas Problema 1 ¿Qué dígitos se han omitido en la siguiente multiplicación?

Llamando α = ∠EAB, tendremos que ∠AEB = 120−α. Como el rayosale simetricamente del lado BC, tendremos que ∠GEF = 120 − α y,por tanto,

∠FEA = 180− 2(120− α) = 2α− 60.

Como ∠FAE = 60 − α, tendremos que ∠AFE = 180 − α y enton-ces ∠GFE = α. Esto demuestra que los triangulos GFE y BAE sonsemejantes. Llamando x = GE, tendremos que EB = 6 − x. ComoFG = AG−AF = 6−3 = 3, tendremos por la semejanza de triangulosque

3

x=

6

6− x⇐⇒ 18− 3x = 6x⇐⇒ 9x = 18

de donde obtenemos que x = 2 y, por tanto, GE = 2 y EB = 4.Tendremos

AB/GF = 6/3 = 2 =⇒ Area(BAE)/Area(GFE) = 22 = 4.

El area del triangulo AFE se puede calcular, por ejemplo,

Area(AFE) = Area(AGB)− Area(GFE)− Area(AEB) =

Area(AGB)− 5Area(GFE) =1

2· 6 · 6 · sen 60− 5 · 1

2· 2 · 3 · sen 60 =

(18− 15)

√3

2=

3√

3

2.

3

Page 334: COMPENDIUM OMEFL - ToomatesXXXV Olimpiada Matemática Española Primera Fase Soluciones de la propuesta de problemas Problema 1 ¿Qué dígitos se han omitido en la siguiente multiplicación?

SEGUNDO DIA

4. Sea p ≥ 3 un numero primo y consideramos el triangulo rectangulo de cateto mayor p2 − 1 y cateto menor 2p. Inscribimos en el triangulo un semic´ırculo cuyo di´ametro se apoya en el cateto mayor del tri´angulo y que es tangente a la hipotenusa y al cateto menor del tri´angulo. Encuentra los valores de p para los cuales el radio del semic´ırculo es un n´umero entero.Solucion. En el triangulo rectangulo ABC, consideramos AB = p2−1,AC = 2p.

Por el Teorema de Pitagoras, tendremos que BC2

= AC2

+ AB2, ası

que

BC2

= (2p)2 + (p2− 1)2 = 4p2 + p4− 2p2 + 1 = p4 + 2p2 + 1 = (p2 + 1)2

de donde obtenemos que BC = p2 + 1.

4

Page 335: COMPENDIUM OMEFL - ToomatesXXXV Olimpiada Matemática Española Primera Fase Soluciones de la propuesta de problemas Problema 1 ¿Qué dígitos se han omitido en la siguiente multiplicación?

Llamando r al radio del semicırculo, E el centro del cırculo que esta enel lado AB y F el punto de tangencia del semicırculo con el lado BC,tendremos que AE = EF = r. Por una parte, el area del trianguloABC viene dada por

Area(ABC) =1

2AB · AC = (p2 − 1)p.

Por otra,

Area(ABC) = Area(AEC) + Area(ECB) =1

2AE ·AC +

1

2BC ·EF =

1

2r · 2p+

1

2(p2 + 1) · r =

r

2(p2 + 2p+ 1) =

r

2(p+ 1)2.

Igualando las dos expresiones para el area del triangulo ABC, obtene-mos que

r

2(p+ 1)2 = (p2 − 1)p

de donde

r =(p2 − 1)2p

(p+ 1)2=

2p(p− 1)

p+ 1.

Es un calculo sencillo comprobar que

2p− 4 <2p(p− 1)

p+ 1< 2p

ası que las unicas posibilidades para r son 2p− 1, 2p− 2 y 2p− 3.

Si r = 2p− 1, entonces

2p− 1 =2p(p− 1)

p+ 1=⇒ 2p2 + 2p− p− 1 = 2p2 − 2p =⇒ p = 1/3

Si r = 2p− 2 = 2(p− 1), entonces

2(p− 1) =2p(p− 1)

p+ 1=⇒ 1 =

2p

p+ 1=⇒ p = 1

Si r = 2p− 3, entonces

2p− 3 =2p(p− 1)

p+ 1=⇒ 2p2 + 2p− 3p− 3 = 2p2 − 2p =⇒ p = 3

5

Page 336: COMPENDIUM OMEFL - ToomatesXXXV Olimpiada Matemática Española Primera Fase Soluciones de la propuesta de problemas Problema 1 ¿Qué dígitos se han omitido en la siguiente multiplicación?

Por tanto, la unica solucion valida es p = 3, lo que nos da el valor der = 3.

5. ¿Existen m,n numeros naturales de forma que

n2 + 2018mn+ 2019m+ n− 2019m2

es un numero primo?

Solucion. Tratamos de factorizar la expresion del enunciado. Igua-lando esta expresion a 0, tendremos

n2 + (2018m+ 1)n+ 2019m− 2019m2 = 0

que podemos tratar como una ecuacion en la variable n, obteniendoque

n =−(2018m+ 1)±

√(2018m+ 1)2 − 4(2019m− 2019m2)

2.

La expresion dentro de la raız viene dada por

(2018m+ 1)2 − 4(2019m− 2019m2) =

20182m2 + 2 · 2018m+ 1− 4 · 2019m+ 4 · 2019m2 =

(20182 + 4 · 2018 + 4)m2 − 2m+ 1 = 20202m2 − 2m+ 1 = (2020m− 1)2

ası que

n =−(2018m+ 1)± (2020m− 1)

2

y, entonces, las soluciones son

n1 =2m− 2

2= m− 1

y

n2 =−4038m

2= −2019m.

Por tanto, podemos factorizar la expresion del enunciado como

n2 + 2018mn+ 2019m+ n− 2019m2 = (n+ 2019m)(n−m+ 1).

6

Page 337: COMPENDIUM OMEFL - ToomatesXXXV Olimpiada Matemática Española Primera Fase Soluciones de la propuesta de problemas Problema 1 ¿Qué dígitos se han omitido en la siguiente multiplicación?

En este producto, el primer factor es obviamente mayor que 1. Unacondicion necesaria para que esta expresion sea un numero primo esque n − m + 1 = 1, es decir, n = m. En este caso, el primer factorqueda de la forma n + 2019m = 2020n, que es un numero compuestoya que 2020 lo es. Por tanto, la expresion del enunciado no sera unnumero primo para ningun valor de n y de m naturales.

6. Fijamos un numero natural k ≥ 1. Encuentra todos los polinomiosP (x) que cumplan

P (xk)− P (kx) = xkP (x)

para todo valor de x ∈ R.

Solucion. Fijemonos que una solucion trivial es P (x) = 0 para cual-quier valor de k ≥ 1.

Para encontrar otras soluciones, fijamos primero k = 1. En ese caso, laecuacion queda

P (x)− P (x) = xP (x),

por lo que P (x) = 0, que es la solucion anterior.

Si k ≥ 2 y P (x) es una constante c, tendremos que c − c = xkc y, portanto, cxk = 0, imposible a menos que c = 0, que nos da el polinomiotrivial P (x) = 0 de nuevo.

Supongamos pues que k ≥ 2 y que el grado del polinomio es n ≥ 1.Es obvio que P (xk) tendra grado nk y P (2x) tendra grado n, ası queel termino de la izquierda de la igualdad sera un polinomio de gradonk ya que k ≥ 1. El termino de la derecha sera un polinomio de gradon+ k, ası que tendremos que nk = n+ k, de donde k(n− 1) = n y, portanto,

k =n

n− 1= 1 +

1

n− 1.

Como k es un numero natural, tendremos que necesariamente n = 2 y,por tanto, k = 2. Escribimos pues P (x) = ax2 + bx+ c y, sustituyendoen la ecuacion, obtenemos:

ax4 + bx2 + c− (4ax2 + 2bx+ c) = ax4 + bx3 + cx2

7

Page 338: COMPENDIUM OMEFL - ToomatesXXXV Olimpiada Matemática Española Primera Fase Soluciones de la propuesta de problemas Problema 1 ¿Qué dígitos se han omitido en la siguiente multiplicación?

y simplificando obtenemos (b − 4a)x2 = bx3 + cx2, de donde necesa-riamente obtenemos que b = 0 y c = −4a. Ası pues, los polinomioscumpliendo la propiedad del enunciado seran todos los de la formaP (x) = a(x2 − 4) para todo a ∈ R.

En resumen, para todo k ≥ 1, una solucion es P (x) = 0. En el casok = 2, los polinomios de la forma P (x) = a(x2 − 4) para cualquiera ∈ R tambien son solucion.

TERCER DIA

7. Considera el conjunto de n´umeros enteros positivos n cumpliendo que 1 ≤ n ≤ 1000000. En ese conjunto, indica si es mayor la cantidad de n´umeros que pueden expresarse de la forma a3+mb2, con a, b ∈ N y m ∈ {0, 2, 4, 6, 8}o la cantidad de numeros que no pueden expresarse de esa manera.

Solucion. Como 0 ≤ a3, b2 ≤ a3 + mb2 ≤ 1000000, tendremos que0 ≤ a ≤ 100 y 0 ≤ b ≤ 1000. Para m = 0, tenemos que a3 +mb2 = a3 yla cantidad de numeros de esa forma sera 100. Para cada m = 2, 4, 6, 8,la cantidad de numeros sera menor o igual que 100 · 1000 = 100000.Por tanto, habra, a lo sumo, 100 + 4 · 100000 = 400100 numeros dela forma a3 +mb2 con las condiciones del enunciado. Por tanto, habramas numeros que no pueden expresarse de esa manera.

8. Prueba que para todo a, b, c > 0 se cumple que

a2

b3c− a

b2≥ c

b− c2

a.

¿En que caso se cumple la igualdad?

Solucion. Multiplicando por ab3c toda la desigualdad para eliminarlos denominadores, tendremos que

a3 − a2bc ≥ ac2b2 − c3b3 ⇐⇒ a3 − ac2b2 ≥ a2bc− c3b3 ⇐⇒

a2(a− bc) ≥ c2b2(a− bc)⇐⇒ (a2 − b2c2)(a− bc) ≥ 0.

Como la funcion f(x) = x2 es creciente, tendremos que los dos terminosdel producto de la izquierda deben tener el mismo signo, ası que la

8

Page 339: COMPENDIUM OMEFL - ToomatesXXXV Olimpiada Matemática Española Primera Fase Soluciones de la propuesta de problemas Problema 1 ¿Qué dígitos se han omitido en la siguiente multiplicación?

desigualdad es cierta. La igualdad se dara si y solo si a − bc = 0, esdecir, si a = bc.

9. Consideramos un triangulo ABC y un punto D en el lado AC. SiAB = DC = 1, ∠DBC = 30o y ∠ABD = 90o, calcula el valor de AD.

Solucion. Llamando ∠ADB = α, tendremos que ∠BDC = 180− α.Utilizando el teorema de los senos en el triangulo ADB tenemos que

x

1=

1

sinα=

BD

sin(90− α)

y en el triangulo DBC tendremos que

1

sin 30=

BD

sin(α− 30)=

BC

sin(180− α)

De las primeras igualdades deducimos que BD = cosαsinα

y de la segundaque

BD =sin(α− 30)

sin 30= 2 sin(α− 30) = 2(sinα cos 30− cosα sin 30)

y ası, BD =√

3 sinα− cosα. Igualando tenemos

cosα

sinα=√

3 sinα− cosα ⇒ cosα =√

3 sin2 α− sinα cosα

de donde cosα(1 + sinα) =√

3 sin2 α y, elevando al cuadrado y deno-tando por t = sinα, (1−t2)(1+t)2 = 3t4, de donde 4t4+2t3−2t−1 = 0y, factorizando, 2t3(2t+1)−(2t+1) = 0. Por tanto, (2t3−1)(2t+1) = 0y, por tanto, tenemos dos opciones: Si 2t + 1 = 0, entonces t = −1

2y

x = 1/t = −2, imposible. Por tanto, 2t3 − 1 = 0, ası que t = 13√2

y

deducimos que x = 3√

2.

9

Page 340: COMPENDIUM OMEFL - ToomatesXXXV Olimpiada Matemática Española Primera Fase Soluciones de la propuesta de problemas Problema 1 ¿Qué dígitos se han omitido en la siguiente multiplicación?

LVI Olimpiada Matematica Espanola

Primera Fase

Primera sesion

Viernes manana, 17 de enero de 2020

OlimpiadaMatemáticaEspañola RSME

1. Dado un numero natural n > 1, realizamos la siguiente operacion: si n es par,lo dividimos entre dos; si n es impar, le sumamos 5. Si el numero obtenidotras esta operacion es 1, paramos el proceso; en caso contrario, volvemosa aplicar la misma operacion, y ası sucesivamente. Determinar todos losvalores de n para los cuales este proceso es finito, es decir, se llega a 1 enalgun momento.

2. Sean a1, a2, . . . , a2020 2020 numeros reales de manera que la suma de 1009 deellos cualesquiera es positiva. Demostrar que la suma de los 2020 numerostambien es positiva.

3. Determinar todos los valores reales de (x, y, z) para los cuales

x+ y + z = 1

x2y + y2z + z2x = xy2 + yz2 + zx2

x3 + y2 + z = y3 + z2 + x

No esta permitido el uso de calculadoras.Cada problema se puntua sobre 7 puntos.

El tiempo de cada sesion es de 3 horas y media.

Page 341: COMPENDIUM OMEFL - ToomatesXXXV Olimpiada Matemática Española Primera Fase Soluciones de la propuesta de problemas Problema 1 ¿Qué dígitos se han omitido en la siguiente multiplicación?

LVI Olimpiada Matematica Espanola

Primera Fase

Segunda sesion

Viernes tarde, 17 de enero de 2020

OlimpiadaMatemáticaEspañola RSME

4. Consideramos el polinomio

p(x) = (x− a)(x− b) + (x− b)(x− c) + (x− c)(x− a)

Demostrar que p(x) ≥ 0 para todo x ∈ R si, y solamente si, a = b = c.

5. Sea ABC un triangulo con AB < AC y sea I su incentro. El incırculo estangente al lado BC en el punto D. Sea E el unico punto que satisface queD es el punto medio del segmento BE. La l’ınea perpendicular a BC quepasa por E corta a CI en el punto P . Demostrar que BP es perpendiculara AD.

Observacion. El incırculo de ABC es el cırculo que es tangente a los treslados del triangulo. El incentro es el centro de dicho cırculo.

6. Sea n un entero positivo. En una cuadrıcula de tamano n×n, algunas casillastienen un espejo de doble cara a lo largo de una de sus diagonales. En elexterior de cada casilla de los lados izquierdo y derecho de la cuadrıcula seencuentra un puntero laser, que apunta horizontalmente hacia la cuadrıcula.Los laseres se numeran de 1 a n en cada lado, en ambos casos de arriba haciaabajo. Un laser es rojo cuando sale de la cuadrıcula por el borde superiory es verde si sale de la cuadrıcula por el borde inferior. Si cada laser saleo bien por el borde inferior o por el superior, demostrar que la suma de loslaseres rojos es menor o igual que la suma de los laseres verdes.

No esta permitido el uso de calculadoras.Cada problema se puntua sobre 7 puntos.

El tiempo de cada sesion es de 3 horas y media.

Page 342: COMPENDIUM OMEFL - ToomatesXXXV Olimpiada Matemática Española Primera Fase Soluciones de la propuesta de problemas Problema 1 ¿Qué dígitos se han omitido en la siguiente multiplicación?

LVI Olimpiada Matematica Espanola

Primera Fase

Primera sesion

Viernes tarde, 17 de enero de 2020

OlimpiadaMatemáticaEspañola RSME

1. Consideramos el polinomio

p(x) = (x− a)(x− b) + (x− b)(x− c) + (x− c)(x− a)

Demostrar que p(x) ≥ 0 para todo x ∈ R si, y solamente si, a = b = c.

2. Sea ABC un triangulo con AB < AC y sea I su incentro. El incırculo estangente al lado BC en el punto D. Sea E el unico punto que satisface queD es el punto medio del segmento BE. La l’ınea perpendicular a BC quepasa por E corta a CI en el punto P . Demostrar que BP es perpendiculara AD.

Observacion. El incırculo de ABC es el cırculo que es tangente a los treslados del triangulo. El incentro es el centro de dicho cırculo.

3. Sea n un entero positivo. En una cuadrıcula de tamano n×n, algunas casillastienen un espejo de doble cara a lo largo de una de sus diagonales. En elexterior de cada casilla de los lados izquierdo y derecho de la cuadrıcula seencuentra un puntero laser, que apunta horizontalmente hacia la cuadrıcula.Los laseres se numeran de 1 a n en cada lado, en ambos casos de arriba haciaabajo. Un laser es rojo cuando sale de la cuadrıcula por el borde superiory es verde si sale de la cuadrıcula por el borde inferior. Si cada laser saleo bien por el borde inferior o por el superior, demostrar que la suma de loslaseres rojos es menor o igual que la suma de los laseres verdes.

No esta permitido el uso de calculadoras.Cada problema se puntua sobre 7 puntos.

El tiempo de cada sesion es de 3 horas y media.

Page 343: COMPENDIUM OMEFL - ToomatesXXXV Olimpiada Matemática Española Primera Fase Soluciones de la propuesta de problemas Problema 1 ¿Qué dígitos se han omitido en la siguiente multiplicación?

LVI Olimpiada Matematica Espanola

Primera Fase

Segunda sesion

Sabado manana, 18 de enero de 2020

OlimpiadaMatemáticaEspañola RSME

4. Ana y Bernardo juegan al siguiente juego. Se empieza con una bolsa quecontiene n ≥ 1 piedras. En turnos sucesivos y empezando por Ana, cadajugador puede hacer los siguientes movimientos: si el numero de piedras enla bolsa es par, el jugador puede coger una sola piedra o la mitad de laspiedras. Si el numero de piedras en la bolsa es impar, tiene que coger unasola piedra. El objetivo del juego es coger la ultima piedra. Determinar paraque valores de n Ana tiene una estrategia ganadora.

5. Determinar para que valores de n existe un polıgono convexo de n ladoscuyos angulos internos, expresados en grados, son todos enteros, estan enprogresion aritmetica y no son todos iguales.

6. Sea O un punto interior del triangulo ABC y sean M , N y P las inter-secciones de AO con BC, BO con CA y CO con AB, respectivamente.Demostrar que de entre los seis triangulos que se forman, hay al menos doscuya area es menor o igual que [ABC]/6.

Observacion. [ABC] denota el area del triangulo ABC.

No esta permitido el uso de calculadoras.Cada problema se puntua sobre 7 puntos.

El tiempo de cada sesion es de 3 horas y media.

Page 344: COMPENDIUM OMEFL - ToomatesXXXV Olimpiada Matemática Española Primera Fase Soluciones de la propuesta de problemas Problema 1 ¿Qué dígitos se han omitido en la siguiente multiplicación?

FASE LOCAL DE LA OLIMPIADA MATEMATICA ESPANOLA.Curso 2019-2020.

Propuesta de problemas (con soluciones).

Sesion viernes manana.

Problema 1. Dado un numero natural n > 1, realizamos la siguiente operacion: si nes par, lo dividimos entre dos; si n es impar, le sumamos 5. Si el numero obtenido trasesta operacion es 1, paramos el proceso; en caso contrario, volvemos a aplicar la mismaoperacion, y ası sucesivamente. Determinar todos los valores de n para los cuales esteproceso es finito, es decir, se llega a 1 en algun momento.

Solucion. En primer lugar, es inmediato comprobar que siempre que empezamos por2, 3 o 4 el proceso termina y que si empezamos por 5 entramos en el bucle (5, 10, 5,10, . . . ) y nunca acabamos.Si el numero por el que se empieza es mayor que 5, en uno o dos pasos siempre pasamosa un numero mas pequeno. Para comprobar esto, observemos que si n es par, resultaevidente; si es impar, esto se sigue de la desigualdad n+5

2 < n, ya que n+52 es el numero

obtenido tras dos iteraciones. Por tanto, siempre acabamos llegando a un numero menoro igual que 5 y unicamente no terminaremos en aquellos casos para los que se vaya aparar al 5 despues de un numero cualquiera de iteraciones.Ahora bien, aplicando sucesivamente las operaciones sumar cinco o dividir entre 2 siem-pre mantenemos la propiedad de ser multiplo de 5, con lo cual tenemos simultaneamentelos dos resultados que queremos: (a) Para aquellos numeros que no son multiplos de 5,aplicando sucesivamente las operaciones, siempre se acaba llegando a 1, 2, 3 o 4 y porlo tanto no hay ciclo. (b) De la misma manera, cualquier multiplo de 5 acaba llegandoal 5 y provocando una repeticion infinita que nunca llega al numero 1. Por ende, elproceso es finito si y solo si n no es multiplo de 5.

Problema 2. Sean a1, a2, . . . , a2020 2020 numeros reales de manera que la suma de1009 de ellos cualesquiera es positiva. Demostrar que la suma de los 2020 numerostambien es positiva.

Solucion 1. Sea S la suma de los 2020 numeros dados. Podemos escribir el numero1009S como la suma de 1009 veces S. Esto da como resultado

1009S = a1 + a2 + a3 + . . .+ a1008 + a1009 + a1010 + . . .+ a2018 + a2019 + a2020

+ a1 + a2 + a3 + . . .+ a1008 + a1009 + a1010 + . . .+ a2018 + a2019 + a2020

+ a1 + a2 + a3 + . . .+ a1008 + a1009 + a1010 + . . .+ a2018 + a2019 + a2020

+ . . . . . . . . . . . .

+ a1 + a2 + a3 + . . .+ a1008 + a1009 + a1010 + . . .+ a2018 + a2019 + a2020

+ a1 + a2 + a3 + . . .+ a1008 + a1009 + a1010 + . . .+ a2018 + a2019 + a2020

= (a1 + a2 + . . .+ a1008 + a1009) + (a2 + a3 + . . .+ a1009 + a1010)

+ . . .+ (a2020 + a1 + a2 + . . .+ a1008).

Como todas las sumas de 1009 numeros (se han coloreado 3 de esas sumas) son positivas,entonces 1009S es positivo y la suma S de los numeros dados tiene que ser positiva,como se pedıa.

Page 345: COMPENDIUM OMEFL - ToomatesXXXV Olimpiada Matemática Española Primera Fase Soluciones de la propuesta de problemas Problema 1 ¿Qué dígitos se han omitido en la siguiente multiplicación?

Solucion 2. Sea S la suma de los 2020 numeros dados. Sin perdida de generalidad,asumimos que a1 ≤ a2 ≤ · · · ≤ a2020. Por la condicion del enunciado, la suma a1 +a2 +· · ·+ a1009 es positiva. Esto quiere decir que al menos uno de los sumandos es positivo,en particular a1009 lo es por ser el mayor de ellos. Por lo tanto, ai es positivo para todo1010 ≤ i ≤ 2020. Si escribimos

S = (a1 + a2 + · · ·+ a1009) + a1010 + a1011 + · · ·+ a2020,

observamos que hemos escrito S como la suma de terminos positivos, con lo cual suvalor es positivo.

Problema 3. Determinar todos los valores reales de (x, y, z) para los cuales

x+ y + z = 1x2y + y2z + z2x = xy2 + yz2 + zx2

x3 + y2 + z = y3 + z2 + x.

Solucion. La segunda ecuacion la podemos reescribir como

(x− y)(y − z)(z − x) = 0.

Ahora, dado que la tercera ecuacion no es simetrica, vamos a distinguir 3 casos dife-rentes:

(a) Si x = y, la tercera ecuacion queda

x2 + z = z2 + x,

o alternativamente(x− z)(x+ z − 1) = 0.

Por tanto, de las dos ultimas tenemos que las opciones son (λ, λ, λ) o (λ, λ,−λ+1).Sustituyendo en la primera tenemos directamente dos soluciones del sistema, queson (1/3, 1/3, 1/3) y (0, 0, 1).

(b) Si x = z, la tercera ecuacion queda

x3 + y2 = y3 + x2,

o alternativamente

(x− y)(x2 + y2 + xy − x− y) = 0.

De aquı obtenemos, sustituyendo en la primera ecuacion, dos nuevas soluciones(ademas de la correspondiente a x = y = z), que son (0, 1, 0) y (2/3,−1/3, 2/3).

(c) Si y = z, la tercera ecuacion queda

x3 + y = y3 + x,

o alternativamente(x− y)(x2 + xy + y2 − 1) = 0.

De aquı obtenemos dos nuevas soluciones, (1, 0, 0) y (−1, 1, 1)

Por tanto, el sistema tiene las seis soluciones que hemos hallado.

Page 346: COMPENDIUM OMEFL - ToomatesXXXV Olimpiada Matemática Española Primera Fase Soluciones de la propuesta de problemas Problema 1 ¿Qué dígitos se han omitido en la siguiente multiplicación?

Sesion viernes tarde.

Problema 1. Consideramos el polinomio

p(x) = (x− a)(x− b) + (x− b)(x− c) + (x− c)(x− a).

Demostrar que p(x) ≥ 0 para todo x ∈ R si y solamente si a = b = c.

Solucion 1. En primer lugar, observamos que cuando a = b = c se tiene que p(x) =3(x− a)2, que claramente satisface p(x) ≥ 0 para todo x ∈ R.Supongamos ahora que p(x) ≥ 0 para todo x ∈ R. Desarrollando la expresion como unpolinomio cuadratico, obtenemos que

p(x) = 3x2 − 2(a+ b+ c)x+ ab+ bc+ ca ≥ 0.

En particular, esto quiere decir que el discriminante del polinomio, ∆, es menor o igualque 0. Dicho discriminante se puede obtener como

∆ = 4(a+ b+ c)2 − 12(ab+ bc+ ca) = 4(a2 + b2 + c2 − ab− bc− ca).

Por lo tanto, tenemos que

0 ≥ 2(a2 + b2 + c2 − ab− bc− ca) = (a− b)2 + (b− c)2 + (c− a)2,

y de aquı se sigue que necesariamente a = b = c.Solucion 2. En primer lugar, observamos que cuando a = b = c se tiene que p(x) =3(x− a)2, que claramente satisface p(x) ≥ 0 para todo x ∈ R.Supongamos ahora que a, b y c no son todos iguales. Supongamos primero que lostres son distintos, sin perdida de generalidad a < b < c. Entonces se tiene p(b) =(b− c)(b− a) < 0, con lo que es falso que p(x) sea siempre no negativo.Supongamos entonces que exactamente dos de los valores a, b y c son iguales, sin perdidade generalidad a = b. Podemos sacar x− a como factor comun en la expresion de p(x),y obtenemos

p(x) = (x− a)(3x− a− 2c),

de lo que se deduce que a y a+2c3 son las dos raıces de p(x), que son distintas porque

a 6= c. Concluimos que p(x) es negativo en el intervalo abierto entre ambas raıces.

Problema 2. Sea ABC un triangulo con AB < AC y sea I su incentro. El incırculoes tangente al lado BC en el punto D. Sea E el unico punto que satisface que D es elpunto medio del segmento BE. La lınea perpendicular a BC que pasa por E corta aCI en el punto P . Demostrar que BP es perpendicular a AD.Observacion. El incırculo de ABC es el cırculo que es tangente a los tres lados deltriangulo. El incentro es el centro de dicho cırculo.Solucion 1. Como P esta en la bisectriz de ∠ACB, el cırculo γ de centro P y radio PEes tangente al lado AC en un punto J . De esta manera AJ = AC −CJ = AC −CE =AC−BC+2BD = AC−BC+(AB+BC−AC) = AB. Esto implica que el cuadrilateroABDP tiene diagonales perpendiculares, ya que

AB2 +DP 2 = AB2 + PE2 +DE2 = AJ2 + PJ2 +BD2 = AJ2 +BD2,

donde hemos usado que los triangulos AJP y BEP son rectangulos.

Page 347: COMPENDIUM OMEFL - ToomatesXXXV Olimpiada Matemática Española Primera Fase Soluciones de la propuesta de problemas Problema 1 ¿Qué dígitos se han omitido en la siguiente multiplicación?

γPJ

ED CB

A

Figura 1: Esquema para resolver el Problema 2

Solucion 2. Vamos a considerar un sistema de coordenadas centrado en B y de maneraque BC es coincidente con el eje de abscisas. Como es habitual, llamaremos a, b y c a laslongitudes de los ladosBC, CA yAB, respectivamente; p designara al semiperımetro deltriangulo; r al radio del incırculo; y β = ∠ABC. De esta manera, no resulta complicadoidentificar las coordenadas de los puntos B, P , A y D. En primer lugar, B = (0, 0) yA = (c cosβ, 2pr/a), dado que la altura sobre el lado a, ha, cumple que

ha =2S

a=

2pr

a,

siendo S el area del triangulo. Por otra parte, D = (p − b, 0). Finalmente, aplicandoel teorema de Tales a los triangulos CEP y CDI, obtenemos que P = (a+ c− b, (b−c)r/(p− c)).Solo queda por comprobar que el producto escalar de los vectores es cero para obtenerası la perpendicularidad buscada. Aplicando el teorema del coseno para expresar cosβen funcion de los lados, obtenemos entonces que el resultado a probar es equivalente a

(c− b)(p− a)

a· 2(p− b) +

2pr

a· (b− c)rp− c

= 0.

Si dividimos por b− c (dado que c < b por la condicion del enunciado), nos queda

r2 =(p− a)(p− b)(p− c)

p.

Ahora bien, esta expresion es claramente cierta como resultado de combinar la igualdadr = S/p con la formula de Heron, y por tanto hemos concluido.

Problema 3. Sea n un entero positivo. En una cuadrıcula de tamano n × n, algunascasillas tienen un espejo de doble cara a lo largo de una de sus diagonales. En el exteriorde cada casilla de los lados izquierdo y derecho de la cuadrıcula se encuentra un punterolaser, que apunta horizontalmente hacia la cuadrıcula. Los laseres se numeran de 1 a nen cada lado, en ambos casos de arriba hacia abajo. Un laser es rojo cuando sale de lacuadrıcula por el borde superior y es verde si sale de la cuadrıcula por el borde inferior.Si cada laser sale o bien por el borde inferior o por el superior, demostrar que la sumade los laseres rojos es menor o igual que la suma de los laseres verdes.Solucion. Consideremos la union S de las lıneas de centros de cada fila y columna.Como cada espejo forma un angulo de 45 grados con las direcciones de la cuadrıcula,

Page 348: COMPENDIUM OMEFL - ToomatesXXXV Olimpiada Matemática Española Primera Fase Soluciones de la propuesta de problemas Problema 1 ¿Qué dígitos se han omitido en la siguiente multiplicación?

1

3 3

2 2

1

Figura 2: Un ejemplo de una configuracion para el Problema 3, donde se muestran losrecorridos de dos laseres.

los laseres se mueven a lo largo de S. Ademas, ningun segmento puede ser usado pordos laseres distintos. En efecto, si fuesen en la misma direccion, serıa posible rehacerla pista de los laseres hasta su punto de partida, que deberıa ser el mismo para ambos.Si fuesen en direcciones contrarias, cada uno acabarıa en el origen del otro, lo cual esimposible porque sabemos que ambos acaban en el borden superior o en el inferior, ynunca en el lateral. Hay 2n laseres y 2n puntos por los que un laser puede abandonarla cuadrıcula, con lo que hay una biyeccion entre esos dos conjuntos. En particular,hay n laseres rojos que podemos numerar r1, . . . , rn y n laseres verdes, con numerosv1, . . . , vn.Un laser solo se puede mover verticalmente a traves de los segmentos verticales deS, cuya longitud total es de n2. Un laser rojo con numero ri necesita atravesar unadistancia de al menos ri−1/2 usando segmentos verticales, mientras que uno verde conel numero vi necesita por lo menos n− vi + 1/2. Se tiene por tanto la desigualdad

n∑i=1

(ri −

1

2

)+

n∑i=1

(n− vi +

1

2

)≤ n2,

de donde se deduce directamente que∑ri ≤

∑vi, como querıamos demostrar.

Page 349: COMPENDIUM OMEFL - ToomatesXXXV Olimpiada Matemática Española Primera Fase Soluciones de la propuesta de problemas Problema 1 ¿Qué dígitos se han omitido en la siguiente multiplicación?

Sesion sabado manana.

Problema 1. Ana y Bernardo juegan al siguiente juego. Se empieza con una bolsa quecontiene n ≥ 1 piedras. En turnos sucesivos y empezando por Ana, cada jugador puedehacer los siguientes movimientos: si el numero de piedras en la bolsa es par, el jugadorpuede coger una sola piedra o la mitad de las piedras. Si el numero de piedras en labolsa es impar, tiene que coger una sola piedra. El objetivo del juego es coger la ultimapiedra. Determinar para que valores de n Ana tiene una estrategia ganadora.

Solucion. Ana tiene una estrategia ganadora para n = 1, 3 y para todos los numerospares mayores que 3. En primer lugar, observamos que en los casos n = 1, 2, 3 todoslos movimientos estan determinados y Ana gana cuando n = 1 o n = 3. Si el numeroinicial de piedras es par y mayor que 4, Ana puede coger una sola piedra, lo cual obligaa Bernardo a coger solamente otra. De esta manera, Ana puede forzar a que se llegue auna situacion con 4 piedras en la bolsa en el momento de su turno. En este caso, Anacoge 2 piedras, lo cual deja a Bernardo en una posicion perdedora y por lo tanto Anagana el juego.De forma analoga, si el juego empieza con un numero impar y mayor que 4 de piedras,Ana tiene que coger necesariamente una sola piedra, lo cual deja a Bernardo en unaposicion ganadora. Por consiguiente, Ana unicamente puede ganar si n = 1, 3 o elnumero es par y mayor que 3.

Problema 2. Determinar para que valores de n existe un polıgono convexo de n ladoscuyos angulos internos, expresados en grados, son todos enteros, estan en progresionaritmetica y no son todos iguales.

Solucion. Sea x la medida del angulo mayor y d la diferencia de la progresion aritmeti-ca. De esta manera,

nx− dn(n− 1)

2= 180(n− 2),

o alternativamente720 = n(360− 2x+ d(n− 1)). (1)

Como 360− 2x > 0 por tratarse de un polıgono convexo, necesariamente sucede que

n(n− 1) < dn(n− 1) < 720. (2)

En particular, resolviendo la ecuacion cuadratica, tenemos que n ≤ 27. Ahora bien, sianalizamos la ecuacion (1), resulta que n divide a 720, por lo que basta con cenirse a losdivisores de 720 que esten entre 3 y 27, esto es: {3, 4, 5, 6, 8, 9, 10, 12, 15, 16, 18, 20, 24}.Vamos a analizar cuando hay una solucion para d = 1. En ese caso, ha de ser

x =n− 1 + 360 + 720/n

2.

Si n es impar, n− 1 sera par y para tener un numero entero tenemos que imponer que720/n sea par, lo cual esta claro si n es impar. Eso nos da soluciones para {3, 5, 9, 15}.Si n es par, 720/n ha de ser impar, con lo que n tiene que ser multiplo de 16 y soloproporciona una solucion para n = 16.Para el resto de casos, podemos asumir ahora que d ≥ 2, y volviendo a (2) tenemos quen ≤ 18, lo que descarta n = 20 y n = 24 y deja 6 casos para analizar con d = 2, donde

Page 350: COMPENDIUM OMEFL - ToomatesXXXV Olimpiada Matemática Española Primera Fase Soluciones de la propuesta de problemas Problema 1 ¿Qué dígitos se han omitido en la siguiente multiplicación?

se tiene que

x =2(n− 1) + 360 + 720/n

2.

Esto hace que solo tengamos que imponer que 720/n sea par, lo que se logra siempre quen no sea multiplo de 16 y funciona por tanto para n ∈ {4, 6, 8, 10, 12, 18}. Concluimosentonces que lo pedido en el enunciado es posible para

n ∈ {3, 4, 5, 6, 8, 9, 10, 12, 15, 16, 18}.

Problema 3. Sea O un punto interior del triangulo ABC y sean M , N y P lasintersecciones de AO con BC, BO con CA y CO con AB, respectivamente. Demostrarque de entre los seis triangulos que se forman, hay al menos dos cuya area es menor oigual que [ABC]/6.Observacion. [ABC] denota el area del triangulo ABC.

A6

A5 A4

A3A2A1

OP

N

M CB

A

Solucion. Sea S = [ABC], A1 = [BOM ], A2 = [MOC], A3 = [NOC], A4 = [AON ],A5 = [AOP ] y A6 = [BOP ]. Usando el teorema de Ceva obtenemos que

BM

MC· CNNA

· APPB

=A1

A2· A3

A4· A5

A6= 1,

y por lo tantoA1 ·A3 ·A5 = A2 ·A4 ·A6.

Usando la desigualdad entre las medias aritmetica y geometrica resultara que(S6

)6=(A1 +A2 +A3 +A4 +A5 +A6

6

)6≥ A1A2A3A4A5A6.

De esta manera, tenemos que(S6

)3≥ A1A3A5,

(S6

)3≥ A2A4A6,

y de aquı se sigue inmediatamente que

mın{A1, A3, A5} ≤S

6, mın{A2, A4, A6} ≤

S

6.

Queda por tanto demostrado que al menos dos de las seis cantidades son menores oiguales que S/6, como querıamos.

Page 351: COMPENDIUM OMEFL - ToomatesXXXV Olimpiada Matemática Española Primera Fase Soluciones de la propuesta de problemas Problema 1 ¿Qué dígitos se han omitido en la siguiente multiplicación?

Sesion sabado tarde.

Problema 1. Demostrar que la suma de los divisores positivos de un numero de laforma 3k + 2 siempre es un multiplo de 3.

Solucion 1. Sea n el numero en cuestion y consideremos un divisor suyo cualquiera,d. Como n = 3k + 2, d no puede ser multiplo de 3. Si d = 3` + 1, entonces n/d, quees otro divisor distinto de n (dado que n no es cuadrado perfecto), sera de la forma3`′+2, y viceversa. Por tanto, podemos agrupar a los divisores de n en parejas {d, n/d}de manera que la suma es multiplo de 3, y por lo tanto la suma de todos los divisorestambien es multiplo de 3.

Solucion 2. Sea n = pa11 · · · parr la descomposicion de n en factores primos. Por ser n dela forma 3k+2, uno de los factores primos ha de ser de la forma 3`+2 y estar elevado aexponente impar. Sin perdida de generalidad, podemos suponer que dicho factor primoes p1. La suma de los divisores de n se expresa por la formula

pa1+11 − 1

p1 − 1· · · p

ar+1r − 1

pr − 1.

En esta expresion, el primero de los factores es multiplo de 3, al serlo pa1+11 − 1 pero

no p1 − 1. Esto es ası ya que resulta inmediato comprobar que cualquier numero de laforma 3a + 2 elevado a un exponente par da como resultado un numero de la forma3b+ 1.

Problema 2. Sea ABC un triangulo acutangulo. Sea D el pie de la altura correspon-diente al lado BC; M el punto medio del lado BC y F el punto de corte de la bisectrizinterior del angulo ∠BAC con el lado BC. Determinar todos los triangulos para loscuales F es el punto medio del segmento DM .Observacion. Se entiende que la determinacion de un triangulo consiste en dar lalongitud de sus lados, en terminos de algun parametro si fuese menester.

Solucion. La condicion DF = FM se puede reescribir como

2BF = BM +BD. (3)

Si llamamos a, b y c a las longitudes de los lados BC, CA y AB, respectivamente, yaplicamos el teorema de la bisectriz tendremos que

BF

c=FC

b=

a

b+ c.

Por consiguiente,

BF =ac

b+ c. (4)

Por otra parte, aplicando primero trigonometrıa en el triangulo ABD y luego el teoremadel coseno en el triangulo ABC,

BD = c cosβ =c2 + a2 − b2

2a. (5)

Sustituyendo los valores de (4) y (5) en (3), obtenemos que

2ac

b+ c=a

2+c2 + a2 − b2

2a.

Page 352: COMPENDIUM OMEFL - ToomatesXXXV Olimpiada Matemática Española Primera Fase Soluciones de la propuesta de problemas Problema 1 ¿Qué dígitos se han omitido en la siguiente multiplicación?

Operando, se llega a

2a2b− 2a2c+ c3 − b2c+ c2b− b3 = (b− c)(2a2 − c2 − bc− bc− b2)

= (b− c)(2a2 − (b+ c)2) = (b− c)(√

2a− b− c)(√

2a+ b+ c) = 0.

De los tres factores, el ultimo claramente no puede ser cero, lo que deja como opcionesposibles

b = c o b+ c =√

2a.

Es decir, o el triangulo es isosceles en A o el lado a viene dado por la media aritmeticade b y c multiplicada por el factor

√2.

Finalmente, observemos que un triangulo del tipo (a, b, b) es acutangulo si y solo sia <√

2b. Por otra parte, uno del tipo ((b+ c)/√

2, b, c) lo es si y solo si c/3 < b < c.

Problema 3. Sea n un entero positivo. Calcular la siguiente suma:

3

1 · 2 · 4 · 5+

4

2 · 3 · 5 · 6+

5

3 · 4 · 6 · 7+ . . .+

n+ 2

n · (n+ 1) · (n+ 3) · (n+ 4).

Solucion. Denotemos por S la suma buscada. En lugar de S vamos a calcular 2S, quese escribe como

2S =

n∑k=1

2k + 4

k(k + 1)(k + 3)(k + 4)=

n∑k=1

(1

k(k + 3)− 1

(k + 1)(k + 4)

)

=

(1

4− 1

10

)+

(1

10− 1

18

)+ . . .+

(1

n(n+ 3)− 1

(n+ 1)(n+ 4)

)=

1

4− 1

(n+ 1)(n+ 4)=

n(n+ 5)

4(n+ 1)(n+ 4).

De aquı se concluye inmediatamente que

S =n(n+ 5)

8(n+ 1)(n+ 4).

Comentario. Se puede proceder de la manera tradicional considerando la descompo-sicion en fracciones simples. De esta forma,

k + 2

k(k + 1)(k + 3)(k + 4)=

1/6

k− 1/6

k + 1− 1/6

k + 3+

1/6

k + 4,

y podemos proceder de la misma manera que antes considerando la cancelacion te-lescopica.

Page 353: COMPENDIUM OMEFL - ToomatesXXXV Olimpiada Matemática Española Primera Fase Soluciones de la propuesta de problemas Problema 1 ¿Qué dígitos se han omitido en la siguiente multiplicación?

Olimpiada Matemática Española, Fase Local 2021

Primer día.

1

Sea ABC un triángulo rectángulo con el ángulo recto en el vértice C. Sea P el punto de

corte de la bisectriz del ángulo BAC con el segmento BC, y Q el punto de corte de la

bisectriz del ángulo ABC con el segmento AC. Sean M y N los puntos de corte con el

segmento AB de las rectas perpendiculares al mismo y que pasan por P y Q

respectivamente.

¿Cuánto vale el ángulo MCN ?

2

Considera el siguiente par de números naturales de 4 cifras:

(m, n) = (2601, 1600) (es decir, m = 2601, n = 1600).

Fíjate que m y n verifican las siguientes propiedades:

1. Son números de 4 cifras (esto es, entre 1000 y 9999).

2. Son cuadrados perfectos.

3.Tienen las mismas cifras en exactamente dos de las cuatro posiciones. (En

nuestro ejemplo, en la segunda y tercera posiciones.)

4. En las dos posiciones en las que las cifras son distintas, la cifra que aparece en

m es igual a la que aparece en n más 1.

Encuentra todos los pares de números naturales que verifican estas cuatro propiedades.

Segundo día

3

Determinar todos los números de cuatro cifras abcdn tales que al insertar un dígito 0

en cualquier posición se obtiene un múltiplo de 7.

4

Determinar todas las parejas de enteros positivos ( m, n ) para los cuales es posible

colocar algunas piedras en las casillas de un tablero de m filas y n columnas, no más de

una piedra por casilla, de manera que todas las columnas tengan la misma cantidad de

piedras, y no existan dos filas con la misma cantidad de piedras.

Page 354: COMPENDIUM OMEFL - ToomatesXXXV Olimpiada Matemática Española Primera Fase Soluciones de la propuesta de problemas Problema 1 ¿Qué dígitos se han omitido en la siguiente multiplicación?

5

En el triángulo ABC con lado mayor BC, las bisectrices se cortan en I. Las rectas AI,

BI, CI cortan a BC, CA, AB en los puntos D, E, F, respectivamente. Se consideran

puntos G y H en los segmentos BD y CD, respectivamente, tales que ABCGID y

ACBHID . Probar que CGFBHE .

6

Al desarrollar nxx 21 en potencias de x, exactamente tres términos tienen

coeficiente impar. ¿Para qué valores de n es esto posible?

Page 355: COMPENDIUM OMEFL - ToomatesXXXV Olimpiada Matemática Española Primera Fase Soluciones de la propuesta de problemas Problema 1 ¿Qué dígitos se han omitido en la siguiente multiplicación?

Soluciones 2020 (versión propia)

1

En primer lugar vemos que los triángulos ACP y AMP son congruentes. En efecto, ambos

son triángulos rectángulos, y comparten el ángulo 2/A , y comparten la hipotenusa AP .

Luego MPCP y 2/ACMPPCM .

Esta última igualdad se deduce de

2/21802/90 APCMAPCMACPMAAPMAPC

De la misma manera vemos que los triángulos BCQ y BNQ son congruentes, y por tanto

QNQC y 2/BQNCQCN .

Así pues:

º45º45º90º452

º90

2

º180

22290

CBAAB

PCMQCN

2 21001001000 xdcban

Primer caso.

Vamos a suponer que las cifras iguales están en la segunda y tercera posición, como en el

ejemplo del enunciado. Entonces:

100111000101001000

110100100010001101001000)1(

2

2

xdcba

dcbadcbaym

Luego

xyxyxy 22100113117

Las posibilidades son:

50110022100113117

1

yy

xy

xy y no es aceptable porque 99992 y .

462468775

562575150713112

1311

7

2

2

xnx

ymyy

xy

xy

Page 356: COMPENDIUM OMEFL - ToomatesXXXV Olimpiada Matemática Española Primera Fase Soluciones de la propuesta de problemas Problema 1 ¿Qué dígitos se han omitido en la siguiente multiplicación?

1600401151

2601511022

91137

11

2

2

xnx

ymyy

xy

xy

1024321345

202545902

77117

13

2

2

xnx

ymyy

xy

xy

04513

202545902

13

77117 2

x

ymyy

xy

xy no es aceptable.

05111

2601511022

11

91137 2

x

ymyy

xy

xy no es aceptable.

0757

5625751502

7

1431311 2

x

ymyy

xy

xy no es aceptable.

501100221

100113117

yy

xy

xy no es aceptable.

Segundo caso.

Vamos a suponer que las cifras iguales están en la primera y tercera posición. Entonces:

1011100101001000

1101001001000110100)1(1000

2

2

xdcba

dcbadcbaym

Luego

xyxyxy 22101

Puesto que 101 es primo, solo hay dos posibilidades:

0511

2601511022

1

101 2

x

ymyy

xy

xy no es aceptable.

25005051101

2601511022

101

1

2

2

xnx

ymyy

xy

xy

Tercer caso.

Vamos a suponer que las cifras iguales están en la primera y segunda posición. Entonces:

111101001001000

110101001000110)1(1001000

2

2

xdcba

dcbadcbaym

Luego xyxyxy 2211

De nuevo, puesto que 101 es primo, solo hay dos posibilidades:

10003661221

112

ymyy

xy

xy no es aceptable.

100036612211

12

ymyy

xy

xy no es aceptable.

Cuarto caso.

Vamos a suponer que las cifras iguales están en la primera y cuarta posición. Entonces:

110101001001001000

10101001001000110)1(100)1(1000

2

2

xdcba

dcbadcbaym

Page 357: COMPENDIUM OMEFL - ToomatesXXXV Olimpiada Matemática Española Primera Fase Soluciones de la propuesta de problemas Problema 1 ¿Qué dígitos se han omitido en la siguiente multiplicación?

Luego xyxyxy 221152

Las posibilidades son:

2/11111121101152

1

yy

xy

xy no es aceptable.

2/5757255115

2

yy

xy

xy no es aceptable.

2/2727222112

5

yy

xy

xy no es aceptable.

2/212121052

11

yy

xy

xy no es aceptable.

2/2121211

1052

yy

xy

xy no es aceptable.

2/272725

22112

yy

xy

xy no es aceptable.

2/575722

55115

yy

xy

xy no es aceptable.

2/11111121

1101152

yy

xy

xy no es aceptable.

No hay ningún caso aceptable.

Quinto caso.

Vamos a suponer que las cifras iguales están en la tercera y cuarta posición. Entonces:

110010010001001001000

101001001000100010100)1(1000)1(

2

2

xdcba

dcbadcbaym

Luego

xyxyxy 2222 1152

Estudiando las posibles combinaciones, como en los casos anteriores, llegamos a las dos únicas

soluciones:

100019636

142

xn

y

x no es aceptable

3600

2500

60

50

2

2

ym

xn

y

x que es la única solución aceptable.

Sexto caso.

Vamos a suponer que las cifras iguales están en la segunda y cuarta posición. Entonces:

10101010001001001000

10101001000100010)1(1001000)1(

2

2

xdcba

dcbadcbaym

Y estudiando las posibles combinaciones observamos que no hay ninguna aceptable.

Page 358: COMPENDIUM OMEFL - ToomatesXXXV Olimpiada Matemática Española Primera Fase Soluciones de la propuesta de problemas Problema 1 ¿Qué dígitos se han omitido en la siguiente multiplicación?

Así pues, las soluciones para este problema son las cinco parejas siguientes:

(5625,4624), (2601,1600), (2025, 1024) , (2601, 2500), (2500, 3600)

Nota: Podríamos haber desechado muchos casos directamente teniendo en cuenta que yx y

yx tienen la misma paridad, y que 1003110000999 22 yxyx y por tanto

19963 yx .

3

Primera versión. Mediante aritmética modular.

Buscamos conjuntos ordenados ),,,( dcba , con 0a , tales que

dcbaabcd 101001000010000|70|7 (1)

dcbabcda 101000100010000|70|7 (2)

dcbacdab 100100100010000|70|7 (3)

dcbadabc 010100100010000|70|7 (4)

010100100010000|70|7 dcbaabcd (5)

Puesto que: )7(mod410000 , )7(mod61000 , )7(mod2100 y )7(mod310 ,

las condiciones del enunciado se pueden escribir como:

)7(mod0326)7(mod0101001000010000 dcbadcba

)7(mod0324)7(mod0101000100010000 dcbadcba

)7(mod0364)7(mod0100100100010000 dcbadcba

)7(mod0264)7(mod0010100100010000 dcbadcba

)7(mod03264)7(mod0010100100010000 dcbadcba

Restando la primera de la segunda llegamos a

7,0)7(mod02 aa , y solo puede ser 7a

Restando la tercera de la segunda llegamos a

7,0)7(mod02 bb

Restando la tercera de la cuarta llegamos a

7,0)7(mod0 cc

Restando la quinta de la cuarta llegamos a

7,0)7(mod02 dd

Así pues, las soluciones posibles son los todos los números de cuatro cifras que se forman con

los dígitos 0 y 7, es decir, los siete números: 7000, 7007, 7070, 7077, 7700, 7707 y 7777.

Segunda versión. Sin aritmética modular.

El tratamiento sería similar. Por ejemplo:

dcbaabcdx 1010010000100000

dcbabcday 1010001000100000

axyy

x9000|7

|7

|7

Y puesto que 7 no es divisor de 9000, se llega a a|7 , es decir, 7,0a . Y como no puede ser

cero puesto que estamos suponiendo un número de cuatro cifras, solo queda 7a .

Y con un argumento similar se deducen los otros tres dígitos.

Page 359: COMPENDIUM OMEFL - ToomatesXXXV Olimpiada Matemática Española Primera Fase Soluciones de la propuesta de problemas Problema 1 ¿Qué dígitos se han omitido en la siguiente multiplicación?

4

Estudiando un poco la situación vemos que para un número elevado de columnas siempre es

posible encontrar solución al problema.

Si el número de filas m es impar, rellenamos la fila del medio con piedras, y quitamos y

ponemos algunas en las primeras columnas siguiendo el siguiente esquema:

3m

X X X X X X X

X X X X X X X X

X

5m

X X X X X X

X X X X X X X

X X X X X X X X

X X

X

7m

X X X X X

X X X X X X

X X X X X X X

X X X X X X X X

X X X

X X

X

Está claro que con este procedimiento podemos siempre obtener tableros en los que todas las

columnas tengan el mismo número de piedras y en las filas tengamos

nnnn 123...321

piedras, es decir, un número diferente en todas y cada una de las filas.

Si m es par el procedimiento sería el mismo pero dejando la última fila vacía. Por ejemplo:

4m

X X X X X X X

X X X X X X X X

X

6m

X X X X X X

X X X X X X X

X X X X X X X X

X X

X

8m

X X X X X

X X X X X X

X X X X X X X

X X X X X X X X

X X X

X X

X

El caso 2m se soluciona fácilmente llenando una fila de piedras y dejando la otra vacía.

Este procedimiento falla cuando

Page 360: COMPENDIUM OMEFL - ToomatesXXXV Olimpiada Matemática Española Primera Fase Soluciones de la propuesta de problemas Problema 1 ¿Qué dígitos se han omitido en la siguiente multiplicación?

par 1

impar

mmn

mmn

Por ejemplo, con 7m y 6n nos encontramos con el tablero siguiente:

X X X

X X X X

X X X X X

X X X X X X

X X X

X X

X

En el que la primera fila y la quinta tienen el mismo número de piedras.

En las soluciones oficiales encontramos el siguiente argumento para el caso límite 1mn y

m impar:

Hay m filas, se pueden colocar un máximo de 1m piedras en cada fila, todas las filas con

diferente cantidad, luego forzosamente hay que poner 1...,,2,1,0 m piedras (no

necesariamente en este orden) pero entonces el total de piedras será

2

)1(1...210

mmm

que deberá ser de la forma

)1(2

)1(

mk

mm

para cierto k que es el número de piedras en cada columna. Luego

km

2

lo cual es imposible porque m es impar.

5

Sea 2/BACDAC

Resolveremos este problema mediante semejanza de triángulos. Por el criterio AA:

DACDHICDAIDH

ADCIDH

ACDDIH

De la misma manera, por el criterio AA:

BADIGDBADIGD

ADBIDG

ABDGID

Así pues IGHIHG y por tanto el triángulo IGH es isósceles en I, luego HIGI .

Page 361: COMPENDIUM OMEFL - ToomatesXXXV Olimpiada Matemática Española Primera Fase Soluciones de la propuesta de problemas Problema 1 ¿Qué dígitos se han omitido en la siguiente multiplicación?

HBIABIABIIBH

BAIIHB

Pero como además comparten un lado común IB, los dos triángulos serán congruentes, y por

tanto IAIH .

Luego el triángulo HIA es isósceles en I, luego IHAIAH .

De lo anterior se deduce que AHBBAH , es decir, el triángulo ABH es isósceles en B.

Sea J el punto de corte entre BE y AH.

En un triángulo isósceles coinciden bisectriz, mediatriz y mediana, luego AJE y HJE son

dos triángulos rectángulos congruentes, de donde JHEJAE , y por tanto

IAEIHE .

Con razonamientos análogos llegamos a FAIFGI , con lo que, finalmente:

GHEFGHBAC 2 .

6

(Ver la solución oficial)

Page 362: COMPENDIUM OMEFL - ToomatesXXXV Olimpiada Matemática Española Primera Fase Soluciones de la propuesta de problemas Problema 1 ¿Qué dígitos se han omitido en la siguiente multiplicación?

Fase aragonesa de laLVII Olimpiada Matemática Española

Primera etapa (8 de enero de 2021)

1. Sea ABC un triángulo rectángulo con el ángulo recto en el vértice C. Sea P el puntode corte de la bisectriz del ángulo ∠BAC con el segmento BC, y Q el punto de cortede la bisectriz del ángulo ∠ABC con el segmento AC. Sean M y N los puntos de cortecon el segmento AB de las rectas perpendiculares al mismo y que pasan por P y Qrespectivamente.

¿Cuánto vale el ángulo ∠MCN?

Solución: La situación del problema queda reflejada en el siguiente gráfico:

?

A B

C

P

Q

MN F

Por ser un punto de la bisectriz de ∠BAC, la distancia de P a la recta AC es igual ala distancia a la recta AB. Por tanto los segmentos PC y PM tienen igual longitud yel triángulo PCM es isósceles. Deducimos que ∠PCM = ∠PMC = ∠MCF . De aquíobtenemos 2∠MCF = ∠FCB y este es complementario de ∠FBC = ∠ABC, luego2∠MCF = ∠BAC.

El mismo razonamiento da 2∠NCF = ∠ABC. Concluimos que

∠MCN = ∠MCF + ∠NCF = 12(∠BAC + ∠ABC) = 45◦.

De otro modo: Los triángulos APC y APM son congruentes, pues ambos son rectán-gulos, tienen la misma hipotenusa y el ángulo en A igual. Por tanto el triángulo CAMes isósceles y obtenemos

∠AMC = ∠ACM = 180◦ − ∠BAC

2 .

Del mismo modo obtenemos ∠BNC = 180◦ − ∠ABC

2 . Ahora, usando el triángulo CMN ,concluimos que:

∠MCN = 180◦ − 180◦ − ∠BAC

2 − 180◦ − ∠ABC

2 = ∠BAC + ∠ABC

2 = 45◦.

1

Page 363: COMPENDIUM OMEFL - ToomatesXXXV Olimpiada Matemática Española Primera Fase Soluciones de la propuesta de problemas Problema 1 ¿Qué dígitos se han omitido en la siguiente multiplicación?

2. Considera el siguiente par de números naturales de 4 cifras:(m, n) = (2601, 1600) (es decir, m = 2601, n = 1600).

Fíjate que m y n verifican las siguientes propiedades:Son números de 4 cifras (esto es, entre 1000 y 9999).Son cuadrados perfectos.Tienen las mismas cifras en exactamente dos de las cuatro posiciones. (En nuestroejemplo, en la segunda y tercera posiciones.)En las dos posiciones en las que las cifras son distintas, la cifra que aparece en mes igual a la que aparece en n más 1.

Encuentra todos los pares de números naturales que verifican estas cuatro propiedades.

Solución: Buscamos números naturales de la forma m = a2, n = b2, con 32 ≤ b < a ≤99 y tales que a2 − b2 es uno de los siguientes números: 1100, 1010, 1001, 110, 101, 11.

Como a2 − b2 = (a + b)(a − b), a + b y a − b tienen la misma paridad, 65 ≤ a + b ≤ 197y a − b ≥ 0, teniendo en cuenta las factorizaciones como producto de primos: 1100 =22 × 52 × 11, 1010 = 2 × 5 × 101, 1001 = 7 × 11 × 13, 110 = 2 × 5 × 11, las únicasposibilidades son:

a2 − b2 = 1100, a + b = 2 × 5 × 11 = 110, a − b = 2 × 5 = 10. En este caso a = 60,b = 50 y obtenemos el par (m, n) = (3600, 2500).a2 − b2 = 1001, a + b = 11 × 13 = 143, a − b = 7. En este caso a = 75, b = 69 yobtenemos el par (m, n) = (5625, 4624).a2 − b2 = 1001, a + b = 7 × 13 = 91, a − b = 11. En este caso a = 51, b = 40 yobtenemos el par (m, n) = (2601, 1600).a2 − b2 = 1001, a + b = 7 × 11 = 77, a − b = 13. En este caso a = 45, b = 32 yobtenemos el par (m, n) = (2025, 1024).a2 − b2 = 101, a + b = 101, a − b = 1. En este caso a = 51, b = 50 y obtenemos elpar (m, n) = (2601, 2500).

Así pues, existen exactamente 5 posibilidades.

2

Page 364: COMPENDIUM OMEFL - ToomatesXXXV Olimpiada Matemática Española Primera Fase Soluciones de la propuesta de problemas Problema 1 ¿Qué dígitos se han omitido en la siguiente multiplicación?

Soluciones, tarde del jueves 21 de enero

1. Determinar todos los numeros de cuatro cifras n = abcd tales que alinsertar un dıgito 0 en cualquier posicion se obtiene un multiplo de 7.

Solucion. Comenzamos observando que el numero que resulta de in-sertar un 0 al final de n es 10n, que al ser multiplo de 7 obliga a quen tambien lo sea. De hecho, son multiplos de 7 los siguientes cinconumeros:

n = abcd = 1000a+ 100b+ 10c+ d

x = a0bcd = 10000a+ 100b+ 10c+ d

y = ab0cd = 10000a+ 1000b+ 10c+ d

z = abc0d = 10000a+ 1000b+ 100c+ d

w = abcd0 = 10000a+ 1000b+ 100c+ 10d

Como n, x son multiplos de 7, tambien lo es su diferencia x−n = 9000a,y puesto que 9000 no es multiplo de 7, debe serlo a. Al ser n un numerode 4 cifras, se tiene que a 6= 0 y necesariamente debe ser a = 7 .

De forma similar, y − n = 9000a + 900b es multiplo de 7, y sabemosque a es multiplo de 7, luego 900b es multiplo de 7. Y como 900 no esmultiplo de 7, debe serlo b, y se deduce que b = 0 o b = 7 .

Analogamente, razonando con z − n = 9000a + 900b + 90c se obtieneque c = 0 o c = 7 , e insertando las condiciones de ser a, b, c multiplos

de 7 en el numero de partida n deducimos que tambien d es 0 o 7 .

Ası pues, los numeros buscados son todos los que empiezan por 7 y lasrestantes cifras son 0 o 7:

7000, 7007, 7070, 7077, 7700, 7707, 7770, 7777.

1

Page 365: COMPENDIUM OMEFL - ToomatesXXXV Olimpiada Matemática Española Primera Fase Soluciones de la propuesta de problemas Problema 1 ¿Qué dígitos se han omitido en la siguiente multiplicación?

2. Determinar todas las parejas de enteros positivos (m,n) para los cualeses posible colocar algunas piedras en las casillas de un tablero de m filasy n columnas, no mas de una piedra por casilla, de manera que todaslas columnas tengan la misma cantidad de piedras, y no existan dosfilas con la misma cantidad de piedras.

Solucion 1. Veremos que las soluciones son todas las parejas (m,n)con n ≥ m si m es impar, o n ≥ m− 1 si m es par.

Para m = 1, es inmediato que cualquier tablero (1, n) es posible, porejemplo llenandolo completamente de piedras.

Notese la condicion necesaria n ≥ m− 1: para m filas son necesarias almenos m− 1 columnas, ya que m− 1 es la menor cantidad de piedrasque puede contener la fila que tenga mas piedras.

Sea m ≥ 3 impar. Veamos que no es posible alcanzar el caso lımiten = m − 1. En efecto, para llenar un tablero (m,m − 1) con distintascantidades de piedras en cada fila, es necesario que las filas tengan0, 1, . . . ,m − 1 piedras, en algun orden. El numero total de piedras es(m−1)m

2, y debe ser igual a t(m − 1), siendo t la cantidad de piedras

de cada columna. Como la igualdad m2

= t es imposible cuando m esimpar, en este caso el numero de columnas debe ser n ≥ m.

Una solucion valida para el tablero (m,m), para m = 2k−1, se obtienesiguiendo el esquema de la siguiente figura:

Las piedras se colocan en dos triangulos rectangulos isosceles de catetosk−1, y en un cuadrado de lado k. Las cantidades de piedras en las filasson 1, 2, . . . , k− 1, k, k+ 1, . . . , 2k− 1, y cada columna tiene k piedras.

Para m ≥ 2 par, el tablero (m,m − 1) se resuelve partiendo de unasolucion del tablero (m− 1,m− 1) y anadiendo una fila vacıa.

Finalmente, toda solucion para un tablero (m,n) puede extenderse aun tablero (m,n + 1), de esta manera: si todas las columnas tienent piedras, colocamos t piedras en la nueva columna, en las posicionescorrespondientes a las t filas que mas piedras tenıan. Ası, las columnassiguen teniendo t piedras cada una, y sigue sin haber dos filas conel mismo numero de piedras. Repitiendo las veces que haga falta la

2

Page 366: COMPENDIUM OMEFL - ToomatesXXXV Olimpiada Matemática Española Primera Fase Soluciones de la propuesta de problemas Problema 1 ¿Qué dígitos se han omitido en la siguiente multiplicación?

operacion de paso de (m,n) a (m,n+1), se resuelven todos los tableros(m,n) con n ≥ m si m es impar, o n ≥ m− 1 si m es par.

Solucion 2. Similar a la anterior, utilizando el siguiente procedimientopara rellenar las casillas en el tablero (m,m), cuando m es impar.

Vamos a colocar k piedras en la fila k ∈ [1, n]. Empezamos por la es-quina superior izquierda colocando una piedra. Si la fila tiene todas laspiedras que vamos a colocar, seguimos por la casilla inmediatamenteabajo a la derecha (si estamos a la derecha del todo, seguimos por laizquierda del todo). Si la fila aun no tiene todas las piedras, entoncesseguimos colocando piedras inmediatamente a la derecha de la anterior(la misma convencion aplica en el borde). Evidentemente cuando haya-mos terminado la ultima fila, todas las filas tendran el numero deseadode piedras. Ademas, como el numero total de piedras es divisible porel numero de columnas, habra el mismo numero de piedras en cadacolumna.

3

Page 367: COMPENDIUM OMEFL - ToomatesXXXV Olimpiada Matemática Española Primera Fase Soluciones de la propuesta de problemas Problema 1 ¿Qué dígitos se han omitido en la siguiente multiplicación?

3. En el triangulo ABC con lado mayor BC, las bisectrices se cortan enI. Las rectas AI,BI, CI cortan a BC,CA,AB en los puntos D,E, F ,respectivamente. Se consideran puntos G y H en los segmentos BD yCD, respectivamente, tales que ∠GID = ∠ABC y ∠HID = ∠ACB.Probar que ∠BHE = ∠CGF .

Solucion Comenzamos con una figura, donde se senalan algunas igual-dades de angulos que inmediatamente justificaremos.

Notese que los triangulos ABD y GID tienen un angulo comun en Dy angulos iguales en B y en I, por lo que sus terceros angulos debencoincidir, es decir ∠DGI = ∠DAB. De forma analoga, razonando conlos triangulos ACD y HID se obtiene ∠DHI = ∠DAC.

Los triangulos BIA y BIH resultan ser congruentes por tener dosangulos iguales y un lado comun, esto revela que los puntos A y H sonsimetricos con respecto a la recta BI, y de forma similar A y G sonsimetricos respecto de CI.

Las simetrıas que acabamos de establecer prueban que:

∠BHE = BAE y ∠CGF = ∠CAF,

y queda demostrada la igualdad de angulos que se pedıa.

Nota: otras soluciones pueden probar y utilizar que los cuadrilaterosABGI y ACHI son inscriptibles (cıclicos), y/o que G,H son puntosde la circunferencia de centro I que pasa por A.

4

Page 368: COMPENDIUM OMEFL - ToomatesXXXV Olimpiada Matemática Española Primera Fase Soluciones de la propuesta de problemas Problema 1 ¿Qué dígitos se han omitido en la siguiente multiplicación?

4. Al desarrollar (1+x+x2)n en potencias de x, exactamente tres terminostienen coeficiente impar. ¿Para que valores de n es esto posible?

Solucion Empezamos estudiando que efecto tiene sobre los coeficientesde un polinomio multiplicar por (1 + x+ x2):

(1 + x+ x2)n+1 = (1 + x+ x2)n(1 + x+ x2)

= (ao + a1x+ a2x2 + a3x

3 + · · · )(1 + x+ x2)

= ao + (ao + a1)x+ (ao + a1 + a2)x2 +

+ (a1 + a2 + a3)x3 + · · ·

Si llamamos {ai}2ni=0 a los coeficientes de Pn(x) = (1+x+x2)n y {bi}2n+2i=0

a los de Pn+1(x), entonces para todo i se cumple que bi = ai−2+ai−1+ai,identificando con 0 los coeficientes que no estan definidos. Esto sugiereordenar los coeficientes de los polinomios Pn(x) en forma de triangulo,empezando con 1 en la cuspide, y donde los restantes coeficientes sonla suma de los tres que se situan por encima de el. Algo ası:

1

1 1 1

1 2 3 2 1

1 3 6 7 6 3 1

1 4 10 16 19 16 10 4 1

...............................

Cada fila es simetrica con extremos 1, y el coeficiente central siemprees impar por ser suma de un impar mas dos numeros iguales.

Tambien vemos que para n = 1, 2, 4, Pn(x) tiene 3 coeficientes impa-res. Veamos que esta situacion ocurre para todas las potencias de 2. Essencillo comprobar que si Pn(x) tiene 3 coeficientes impares (necesaria-mente los de grado 0, n y 2n), lo mismo ocurre para P2n(x):

(1 + x+ x2)2n =((1 + x+ x2)n

)2≡ (1 + xn + x2n)2 ≡ 1 + x2n + x4n (mod 2),

donde hemos despreciado los “dobles productos”, al trabajar modulo2. Partiendo de que P1(x) tiene 3 terminos impares, el procedimientoanterior de paso de n a 2n prueba que Pn(x) tiene 3 terminos imparespara todo n potencia de 2. Veamos que no existen mas valores de n conesta propiedad.

Si n no es potencia de 2, existen una potencia de 2 dada por a = 2k yun numero b, con 0 < b < a, tales que n = a + b. Por lo visto antes,podemos asumir que Pa(x) ≡ 1 + xa + x2a (mod 2). Entonces:

Pn(x) = Pb(x)Pa(x) ≡ Pb(x)(1 + xa + x2a) (mod 2)

≡ Pb(x) + (terminos de grado ≥ a > b)

Ademas, sabemos que Pb(x) tiene su termino central xb impar, y estetermino “sobrevive” sin que nadie lo cancele, por lo que Pn(x) tiene almenos 4 terminos impares: los de grado 0, b, n y 2n.

5